Download as pdf or txt
Download as pdf or txt
You are on page 1of 284

10

R
ADDITIONAL
PR ACTICE

MATHEMATICS

Update Answer Key

DNA education
New Delhi-110002
Chapter

1 Real Numbers

Multiple choice Questions

1. (a) Here, a = Dividend, b = Divisor, q = \ HCF = 13.


Quotient and r = Remainder The largest number which divides 70
Using Euclid's Division Lemma, and 125 leaving remainders 5 and 8
respectively is 13.
a = bq + r, 0 < r < b
a = 3q + r WoRKSHeet - 1
Here b = 3;
Section-A
So, possible values of r = 0, 1, 2.
1. Here, a is a dividend.
\ 0 < r < 3.
2. 13233343563715 is a composite number as it
2. (c) LCM of 23 and 33 = 23 × 33
is also divisible by 5 besides 1 and the number
3. (b) Given A = 2n + 13 and B = n + 7 and itself.
since n is a natural number A > B. Since
3. Dividend = Divisor × Quotient + Remainder
A/B is not an integer we can conclude
that A and B cannot have any integer in = 53 × 34 + 21
common. Therefore HCF of A and B is 1.
= 1823
4. (d) Prime factorization of 102 = 2 × 3 × 17
4. y = 5 × 13 = 65
Prime factorization of 85 = 5 × 17 =
(2 + 3) × 17 x = 3 × 195 = 585

Two numbers are: 5. HCF (k, 2k, 3k, 4k, 5k) = k

(1) 3 × 17 = 51 and (2) 2 × 17 = 34. 6. Smallest composite number = 4


\ Numbers are 51 and 34. Smallest prime number = 2
5. (a) Since 5 and 8 are remainders, So we \ HCF (2, 4) = 2
subtract these remainders from 70 and
125 respectively. 7. 6n = (2 × 3)n

We get; We know that a number ends with digit 0 only


if it has both 2 and 5 as factors. As 6n does not
(i) 70 – 5 = 65 (ii) 125 – 8 = 117
have 5 as a prime factor, so, 6n does not end
Now, taking HCF of 65 and 117; with digit 0.
117 = 65 × 1 + 52 8. LCM (p, q) = LCM (ab, a2b) = a2b
65 = 52 × 1 + 13 9. HCF (a, b) = HCF (x3y2, xy3)
52 = 13 × 4 + 0 = xy2
2 ADDITIONAL
R

PR ACTICE MATHEMATICS - 10
a×b 17. Let x and x + 1 be two consecutive positive
10. LCM (a, b) = integers.
HCF(a,b)
1800 If x is even, x + 1 is odd, so, x (x + 1) is even
= = 150 If x is odd, x + 1 is even, so, x (x + 1) is even.
12
Section-B Therefore, the product of two consecutive
positive integers is always divisible by 2.
11. Using Euclid's Division lemma,
18. 3 × 5 × 13 × 46 + 23
a = 4q + r, 0 < r < 4
= 23 × (3 × 5 × 13 × 2 + 1)
r = 0, a = 4q
So, 23 is a factor of 3 × 15 × 13 × 46 + 23
r = 1, a = 4q + 1
besides 1 and the number itself.
r = 2, a = 4q + 2
Therefore, 3 × 5 × 13 × 46 + 23 is a composite
r = 3, a = 4q + 3 number.
Hence, every positive integer can not be of
form 4q + 2, it can also be of the form 4q, 4q 19. As least prime factor of a is 3, a is an odd
+ 1 or 4q + 3. number (because if a is even then it's least
prime factor must be 2). Also, as least prime
12. Using Euclid's Algorithm, factor of b is 5, b is an odd number.
240 = 228 × 1 + 12 Therefore, a + b is even such that it's least
prime factor is 2.
228 = 12 × 19 + 0
Here, remainder = 0, Divisor = 12 20. No, two numbers can not have 15 as their
HCF and 175 as their LCM because 15 is not
So, HCF (240, 228) = 12 a factor of 175.
253 × 440 (HCF of two numbers is always the factor of
13. LCM (253, 440) =
HCF (253, 440) their LCM)
253 × 440
\  LCM (253, 440) = = 253 × R Section-C
1
So, R = 440
21. Using Euclid's Division lemma.
14. 3 × 12 × 101 + 4
   a = 6q + r; 0 < r < 6
= 4 × (3 × 3 × 101 + 1)
r = 0, a = 6q = 2 (3q), even
So, 4 is also a factor of 3 × 12 × 101 + 4
r = 1, a = 6q + 1 = 2 (3q) + 1, odd
besides 1 and the number itself.
r = 2, a = 6q + 2 = 2 (3q + 1), even
So, 3 × 12 × 101 + 4 is a composite number.
r = 3, a = 6q + 3 = 2 (3q + 1) + 1, odd
15. 1200 =
= 2 4 × 3 × 52 2 2 × 5 3
r = 4, a = 6q + 4 = 2 (3q + 2), even
So, square root of product is a rational number
if we multiply 1200 by 3 such that r = 5, a = 6q + 5 = 2 (3q + 2) + 1, odd

1200 × 3 = 2 4 × 32 × 52 = 60 So, any positive even integer can be written in


the form of 6q, 6q + 2 or 6q + 4.
16. The least number that is divisible by all the
numbers from 1 to 10 is basically equal to LCM 22. We know that any positive odd integer (say a)
(1, 2, 3, 4, ..., 10) = 2520 is of form 4q + 1 or 4q + 3
ADDITIONAL
R

PR ACTICE MATHEMATICS - 10 3
Case 1 = 5 (5q2 + 8q + 3) + 1
a = 4q + 1 = 5m + 1 (m = 5q2 + 8q + 3)

a2 = (4q + 1)2 = 16q2 + 1 + 8q = 8 (2q2 + q) + 1 So, square of positive integer cannot be of
form 5m + 2 or 5m + 3.
  = 8m + 1 (m = 2q2 + q)
25. Minimum distance each should walk so that
Case 2
each can cover the same distance.
a = 4q + 3 2 40, 42, 45
= LCM (40, 42, 45)
a = (4q + 3) = 16q + 9 + 24q
2 2 2
2 20, 21, 45
= 2520 cm
  = 8 (2q + 3q + 1) + 1
2
2 10, 21, 45
  = 8m + 1 (m = 2q + 3q + 1) 2
3 5, 21, 45
so, square of an odd positive integer is of form 3 5, 7, 15
8m + 1.
5 5, 7, 5
23. First, find HCF (180, 252)
7 1, 7, 1
252 = 180 × 1 + 72
1, 1, 1
180 = 72 × 2 + 36
26. 7 × 19 × 11 + 11
72 = 36 × 2 + 0
= 11 (7 × 19 × 1 + 1)
Remainder = 0, Divisor = 36, So, HCF = 36
So, 11 is also a factor of 7 × 19 × 11 + 11
Now, find HCF (36, 324)
besides 1 and number itself.
324 = 36 × 9 + 0
So, 7 × 19 × 11 + 11) is a composite number.
So, HCF (36, 324) = 36
7 × 6 × 4 × 3 × 2 × 1 + 3 = 3 (7 × 6 × 4 × 2
\ HCF (180, 252, 324) = 36
× 1 + 1)
24. Using Euclid's Division lemma, So, 3 is also a factor of 7 × 6 × 4 × 3 × 2 × 1
+ 3 besides 1 and number itself.
a = 5q + r; 0 < r < 5
So, 7 × 6 × 4 × 3 × 2 × 1 + 3 is a composite
r = 0, a = 5q, a2 = 25q2 = 5m (m = 5q2)
number.
r = 1, a = 5q + 1, a2 = 25q2 + 1 + 10q
27. H
 ere, we have to find LCM (12, 15, 18) which
= 5 (5q + 2q) + 1 2
indicates after how long they all again toll
together.
= 5m + 1 (m = 5q2 + 2q) 2 12, 15, 18
LCM (12, 15, 18)
r = 2, a = 5q + 2, a2 = 25q2 + 4 + 20q 2 6, 15, 9
= 180
= 5 (5q2 + 4q) + 4 3 3, 15, 9
So, three bells
= 5m + 4 (m = 5q2 + 4q) will toll together 3 1, 5, 3
after 180 minutes
r = 3, a = 5q + 3, a2 = 25q2 + 9 + 30q 5 1, 5, 1
i.e. 3 hours.
= 5 (5q2 + 6q + 1) + 4 1, 1, 1
28. Using Euclid's
= 5m + 4 (m = 5q2 + 6q + 1) division
algorithm,
r = 4, a = 5q + 4, a2 = 25q2 + 16 + 40q
4 ADDITIONAL
R

PR ACTICE MATHEMATICS - 10
1170 = 650 × 1 + 520 Section-D
650 = 520 × 1 + 130 31. Using Euclid's Division Algorithm, we get
520 = 130 × 4 + 0 256 = 36 × 7 + 4
So, HCF (650, 1170) = 130 36 = 4 × 9 + 0
Therefore, the largest number which divides
Here, remainder = 0, divisor = 4
650 and 1170 exactly is 130.
So, HCF (256, 36) = 4
1 1 3 -2 2
29. Consider = × Now, 2 256, 36
3+ 2 2 3+ 2 2 3 -2 2
2 128, 18
3−2 2 3−2 2
 = = =3–2 2 2 64, 9
( )
2
32 − 2 2 1
2 32, 9
Let if possible 3 – 2 2 is rational
2 16, 9
p 2 8, 9
3 – 2 2 = , p and q are integers and q 0
q
1 p 2 4, 9
 3 −  = 2
2 q 2 2, 9
1 p 3 1, 9
Here,  3 −  is rational but 2 is irrational
2 q 3 1, 3
which is not possible.
1, 1
So, we get a contradiction.
Now, LCM (256, 36) = 28 × 32 = 2304
Therefore, 3 – 2 2 is irrational .
HCF × LCM = 4 × 2304
1
i.e. is irrational. = 9216
3+2 2
Product of numbers = 256 × 36
30. Using Euclid's division lemma, = 9216
117 = 65 × 1 + 52 So, HCF × LCM = Product of numbers
65 = 52 × 1 + 13 32. We know that every positive even integer is
52 = 13 × 4 + 0 of form 2q and every positive odd integer is of
form 2q + 1.
Here, remainder = 0, divisor = 13
So, HCF (117, 65) = 13 Case 1
n = 2q
To find : m, n Consider n2 – n = 4q2 – 2q = 2 (2q2 – q)
13 = 65 – 52(1) \
n2 – n is divisible by 2
= 65 – (117 – 65 (1)) Case 2
n = 2q + 1
= 65(2) + 117(–1) Consider n2 – n = (2q + 1)2 – (2q + 1)
= 65m + 117n = 4q2 + 1 + 4q – 2q – 1
So, m = 2,  n = –1 = 4q2 + 4q – 2q

ADDITIONAL
R

PR ACTICE MATHEMATICS - 10 5
= 2 (2q2 + 2q – q) 35. In order to find the number of fruits to be
put in each basket in order to have minimum
= 2 (2q2 + q)
number of baskets, we will find HCF (990,
\
n2 – n is divisible by 2. 945)
From Case 1, Case 2, we get n2 – n is divisible 990 = 2 × 32 × 5 × 11
by 2 for every positive integer n.
945 = 33 × 5 × 7
33. According to Euclid's Division lemma,
HCF (990, 945) = 32 × 5 = 45
a = 3q + r, 0 < r < 3
Therefore, 45 fruits should be put in each
For r = 0 basket.
a = 3q a3 = 27q3 a3 = 9 (3q3) 990
Number of baskets containing apples =
45
    = 9m (m = 3q3) = 22
For r = 1 945
Number of baskets containing oranges =
a = 3q + 1 a3 = 27q3 + 1 + 27q2 + 9q 45
= 21
= 9 (3q3 + 3q2 + q) + 1
So, total number of baskets = 22 + 21
= 9m + 1 (m = 3q3 + 3q2 + 2)
    = 43.
For r = 2
36. Let the three consecutive positive integers be
a = 3q + 2 a3 = 27q3 + 8 + 54q2 + 36q n, n + 1 and n +2.
= 9 (3q3 + 6q2 + 4q) + 8 If number is divided by 3, remainder can be 0,
= 9m + 8 (m = 3q3 + 6q2 1 or 2. i.e. n = 3q + r, 0 < r < 3
+ 4q) If r = 0, n = 3q  divisible by 3
Therefore, cube of any positive integer is of
If r = 1, n + 2 = 3q + 1 + 2
form 9m, 9m + 1 or 9m + 8 for some integer m.
= 3q + 3
34. (a) Greatest possible length of each plank
= 3 (q + 1) divisible by 3
= HCF (42, 49, 56)
= HCF (2 × 3 × 7, 72, 23 × 7) If r = 2, n + 1 = 3q + 2 + 1 = 3 (q + 1) divisible
by 3
= 7
So, one of numbers n, n + 1, and n + 2 must be
 So, greatest possible divisible by 3 i.e. n (n + 1) (n + 2) is divisible
length of each plank is by 3
7m.
Now, if a number is divided by 2, remainder is
(b) HCF (182, 169) 0 or 1
= HCF (2 × 7 × 13, 13 ) 2
i.e. n = 2q + r ;  0 < r < 2
= 13 If r = 1 n + 1 = 2q + 1 + 1 = 2q + 2
2 182
13 169 = 2 (q + 1) divisible by 2
7 91
13 13 If r = 2 n + 2 = 2q + 2 + 2 = 2q + 4
13 13
1
= 2 (q + 2) divisible by 2.
1
6 ADDITIONAL
R

PR ACTICE MATHEMATICS - 10
So, one of n, n + 1 or n + 2 is divisible by 2 i.e. p
2 3
n (n + 1) (n + 2) is divisible by 2. = ; p and q are
5 q
integers, q 0
Since, n (n + 1) (n + 2) is divisible by 2 and 3
implies n (n + 1) (n + 2) is divisible by 6. 5p
3 = 2q
37. (a) 2 420 2 180 2 378 5p
Here, is rational but 3 is irrational which
2 210 2 90 3 189 2q
is not possible, so we get a contradiction.
5 105 3 45 3 63
2 3
3 21 3 15 3 21 \ is irrational.
5
7 7 5 5 7 7 (b) 3 rational numbers between 1.12 and
1.13 are 1.1210, 1.1211, 1.1213.
1 1 1
So, HCF (420, 180, 378) 3 irrational numbers between 1.12 and
1.13 are 1.121121112111..., 1.1221222...,
= HCF (22 × 3 × 5 × 7, 22 × 32 × 5, 1.123123312333...
2 × 33 × 7)
2 378, 180, 420
=2×3=6
WoRKSHeet - 2
LCM (378, 180, 420) 2 189, 90, 210
= 22 × 33 × 5 × 7 3 189, 45, 105 Section-A
= 3780 3 63, 15, 35 1. Here, denominator = 22 57 72. As
denominator is not of the form 2m × 5n,
3 21, 5, 35 so, the given rational number has a non-
5 7, 5, 35 terminating repeating decimal expansion.

Now HCF × LCM 7 7, 1, 7 2 45 + 2 20 6 5 + 4 5 10 5


2. = = =5
= 6 × 3780 = 22680 1, 1, 1 2 5 2 5 2 5

Product of numbers = 378 × 180 × 420 which is rational.

= 28576800 3. HCF (a, b) × LCM (a, b) = a × b

So, HCF × LCM Product of numbers. 15 × LCM = 45 × 105


3
(b) Let if possible 2 2 is rational. 45 ×105
LCM = = 315
p 15
2 2 = , p and q are 4. Decimal expansion will terminate after 4
q
integers, q 0 places of decimal.
p
2 = 5. HCF × LCM = 100 × 170 = 17000.
2q
p 6. Here, denominator = 1500 = 22 × 3 × 53
Here, is rational but 2 is irrational.
2q
So, we get a contradiction. As denominator is not of the form 2m × 5n,
so, it has non-terminating repeating decimal
\2 2 is irrational. expansion.

2 3 7. HCF (a, b) × LCM (a, b) = a × b


38. (a) Let if possible is rational.
5 9 × 360 = a × 45

ADDITIONAL
R

PR ACTICE MATHEMATICS - 10 7
9 × 360 14. Maximum capacity = HCF (850, 680)
= a
45 = HCF (2 × 52 × 17, 23 × 5 × 17)
72 = a
= 2 × 5 × 17
7
8. = 0.0112 = 170 l.
625
95 15 95 + 150 245 15. (a) (–1) + (–1)2n + (–1)2n + 1 + (–1)4n+1
9. + = = = 6.125
40 4 40 40
= (–1) + (1) + (–1) + (–1)
10. Decimal expansion will terminate after 5 = –2
places of decimal.
−5 -5 1
( )

(b) 23 3 = 2 3 = 2–5 =
25
Section-B
1
0 . 3 7 5 =
11. 32
8 3 13 13
16. = 6
–0 64 2
Here, Denominator = 26 = 26 × 50
3 0
– 24 i.e. of form 2m × 5n, so, it has terminating
decimal expansion.
6 0
Here, highest power in 26 × 50 is 6, so it's
– 56
decimal expansion has 6 decimal places.
4 0
17. Using Euclid's Algorithm.
– 4 0
0 4052 = 420 × 9 + 272

420 = 272 × 1 + 148
3
\ = 0.375 272 = 148 × 1 + 124
8
12. Let if possible 5 6 is rational. 148 = 124 × 1 + 24
p 124 = 24 × 5 + 4
5 6 = ; p , q are integers,
q 24 = 4 × 6 + 0
q 0
p Here remainder = 0, Divisor = 4
6 =
5q So, HCF (4052, 420) = 4
p
Here, is rational but 6 is irrational which 3
5q 18. Let if possible is rational
is not possible. So, we get a contradiction i.e. 5
3 p
5 6 is irrational. = , p, q are integers, q 0
5 q
13. Let x = 1.41... .....(i) 3q
5 =
x × 100 = 1.41 × 100 p
100x = 141. 41 .....(ii) 3q
Here, is rational but 5 is irrational which
On subtracting (i) from (ii), we get p
99x = 140 is not possible, so we get a contradiction.
140 3
x = \  is irrational.

99 5
8 ADDITIONAL
R

PR ACTICE MATHEMATICS - 10
19. Using Euclid's Division Algorithm,
22. Let if possible 2 3 + 7 is rational number.
180 = 144 × 1 + 36 p
2 3 + 7 = , p, q are integers, q 0
144 = 36 × 4 + 0 q
p
Here, remainder = 0, divisor = 36 7 = –2 3
q
So, HCF (144, 180) = 36 On squaring both sides, we get
We can write p2 4p
7 =2
+ 12 - 3
36 = 180 – 144 (1) q q
= 36 4p p2
3 = 2 +5
q q
= 39 – 3
q p2
3 = +5
= 13 (3) – 3 4p Ł q2 ł
= 13 m – 3 q p2
Here, + 5 is rational but 3 is
4p Ł q2 ł
m = 3
irrational which is not possible. So, we get a
contradiction
20. 9n = (3 × 3)n
Since, prime factorization does not contain 2 \
2 3 + 7 is irrational number.
and 5, so, it cannot end with digit 0.
( )( ) ( ) −( 7)
2 2
2 3 + 7 2 3 − 7 = 2 3
Section-C = 12 – 7
= 5 which is rational number.
21. Let if possible 3 + 5 is rational number.
p 23. 5 × 7 × 13 × 17 + 289 = 17 (5 × 7 × 13 × 1
3 + 5 = ,  p and q are
q + 17)
integers and q 0
p Here, 17 is also a factor of 5 × 7 × 13 × 17
3 = – 5 + 289 besides 1 and number itself. So, it is a
q
2
composite number.
p 
( )
2
3 = q − 5 Also, 7 × 11 × 13 × 15 + 225 = 15 (7 × 11 ×
  13 × 1 + 15)
p2 2p Here, 15 is also a factor of 7 × 11 × 13 × 15 +
3 = 2 +5− 5
q q 225 besides 1 and number itself. So, it is also
2p p2 a composite number.
= 2 +2
q
24. LCM (20, 30, 40) = 120 2 20, 30, 40
q  p2 
5 =  2 + 2 2 10, 15, 20
2p  q  So, all the three bells
will toll together
q p2 2 5, 15, 10
Here, + 2 is rational but 5 is after 120 minutes
2p Ł q2 ł i.e. 2 hours. 3 5, 15, 5
irrational, which is not possible.
5 5, 5, 5
Therefore, 3 + 5 is irrational number. 1, 1, 1
ADDITIONAL
R

PR ACTICE MATHEMATICS - 10 9
25. Using Euclid's Division algorithm.
which is not possible.
2058 = 378 × 5 + 168
So, we get a contradiction
378 = 168 × 2 + 42
\ 7 – 2 3 is irrational number.

168 = 42 × 4 + 0
Here, remainder = 0, divisor = 42 Section-D
1
So, HCF (2058, 378) = 42 29. (a) Let if possible is rational number.
2
Therefore, the largest number which divides 1 p
2058 and 378 is their HCF (i.e. 42). =
, p and q are
2 q
integers, q 0
26. Let HCF = x
q
\
LCM = 14x 2 = p
q
LCM + HCF = 600 Here, is rational but 2 is irrational
p
14x + x = 600 which is not possible. So, we get a
15x = 600 x = 40 contraction.


We know that HCF (a, b) × LCM (a, b) = a × b 1
\   is irrational number.
40 × 14 × 40 = a × 280 2
(b) Let if possible 7 5 is rational number.
40 ×14 × 40
a = p
280 7 5 = , p and q are

= 80 q
integers, q 0
27. Using Euclid's division lemma, p
5 = 7q
a = 4q + r, 0 < r < 4
p
r = 0, a = 4q = 2 (2q)  even Here, is rational but 5 is irrational
7q
r = 1, a = 4q + 1 = 2 (2q) + 1 odd which is not possible. So, we get a
r = 2, a = 4q + 2 = 2 (2q + 1) even contraction.
r = 3, a = 4q + 3 = 2 (2q + 1) + 1 odd \
7 5 is irrational number.
So, any positive odd integer is of form 4q + 1 30. Using Euclid's division algorithm
or 4q + 3.
237 = 81 × 2 + 75
28. Let if possible 7 – 2 3 is rational number.
81 = 75 × 1 + 6
p
7 – 2 3 = , p and q are integers and q 0 75 = 6 × 12 + 3
q
p 6 = 3×2+0
2 3 = 7 –
q So, HCF (237, 81) = 3
1  p Consider 3 = 75 – 6 (12)
3 =  7− 
2  q = (81 – 6) – 6 (12)
1 p = 81 – 13 (6)
Here,  7 −  is rational but 3 is irrational
2 q = 81 – 13 (81 – 75)
10 ADDITIONAL
R

PR ACTICE MATHEMATICS - 10
= 81 – 13 (81) +13 (75) 3q2 = p2
= 81 – 13 (81) +13 (237 – 162) 3 divides p2 3 divides p
= 81 – 81(13) + 13 (237 p = 3c
– 81(2))
p2 = 9c2 3q2 = 9c2
= 81 (1 – 13 – 26) + 237 (13)
q2 = 3c2
= 81 (–38) + 237 (13)
3 divides q2 3 divides q
= 81x + 237 y
where x = –38, y = 13 So, p and q have atleast 3 in common which is
a contradiction to the fact that HCF (p, q) = 1
31. HCF (96, 240, 336)
So, our supposition was wrong,
= HCF (25 × 3, 24 × 3 × 5, 24 × 3 × 7)
3 is irrational number.
= 24 × 3
34. According to Euclid's division lemma, for any
= 48 positive integer n, we have
96
So, number of stacks of English books = n = bq + r, 0 < r < b
48
= 2 Take b = 5
240 n = 5q + r, 0 < r < 5
Number of stacks of Hindi books = =5
48
For r = 0
Number of stacks of Mathematics books
336 n = 5q, divisible by 5
= =7
48 n + 4 = 5q + 4, not divisible by 5
32. (a) 5 rational numbers between 1.1 and 1.2
are 1.11, 1.12, 1.13, 1.14, 1.15 n + 8 = 5q + 8, not divisible by 5

5 irrational numbers are between 1.1 n + 12 = 5q + 12, not divisible by 5


and 1.2 are n + 16 = 5q + 16, not divisible by 5
1.1121121112111..., 1.1121122111222...,
1.131331333..., 1.141441444..., So, for r = 0, only n is divisible by 5
1.151551555...
For r=1
(b) HCF (70 – 5, 125 – 8)
n = 5q + 1, not divisible by 5
= HCF (65, 117)
n + 4 = 5q + 1 + 4
= HCF (5 × 13, 3 × 13) 2
= 5q + 5
= 13
= 5 (q + 1), divisible by 5
33. Let if possible 3 is rational number. n + 8 = 5q + 1 + 8
p
3 = q , p and q are integers q 0, = 5q + 9, not divisible by 5
HCF (p, q) = 1 n + 12 = 5q + 1 + 12

q 3 = p = 5q + 13, not divisible by 5


ADDITIONAL
R

PR ACTICE MATHEMATICS - 10 11
n + 16 = 5q + 1 + 16 p
\  m = q –n
= 5q + 17, not divisible by 5.
p
So, for r = 1, only n + 4 is divisible by 5 Here, – n is rational (as p, q are integers
q
For r = 2, and n is rational) but m is irrational.
n = 5q + 2, not divisible by 5 So, we get a contradiction.
n + 4 = 5q + 6, not divisible by 5 Therefore, n + m is irrational number.
n + 8 = 5q + 10
p + q is rational number.
36. Let if possible
= 5 (q + 2), divisible by 5 a
p + q = b , a , b are integers
n + 12 = 5q + 14, not divisible by 5 and b 0
a
n + 16 = 5q + 18, not divisible by 5 p = b – q
So, for r = 2, only n + 8 is divisible by 5 On squaring both sides, we get
For r = 3 a2 2a
p = 2
+ q- q
n = 5q + 3, not divisible by 5 b b
2a a2
n + 4 = 5q + 7, not divisible by 5 q = 2 + q-p
b b
n + 8 = 5q + 11, not divisible by 5
b a2
q = + q-p
n + 12 = 5q + 15 2a Ł b 2 ł
= 5 (q + 3), divisible by 5 b a2
Here + q - p is rational but q is
n + 16 = 5q + 19, not divisible by 5. 2a Ł b 2 ł
So, for r = 3, only n + 12 is divisible by 5. irrational (as square root of a prime number
is irrational) which is not possible.
For r = 4
So, we get a contradiction.
n = 5q + 4, not divisible by 5
Therefore, p + q is irrational number.
n + 4 = 5q + 8, not divisible by 5
n + 8 = 5q + 12, not divisible by 5 37. Circumference of circular field = 360 km
Distance traveled by 1st cyclist = 48 km
n + 12 = 5q + 16, not divisible by 5
360
n + 16 = 5q + 20 So, number of days takes by 1st cyclist =
48
= 5 (q + 4), divisible by 5 = 7.5 days

So, for r = 4, only n + 16 is divisible by 5. Similarly,


360
From 1, 2, 3, 4 It is clear that, one and only one Number of days takes by IInd cyclist =
60
out of n, n + 4, n + 12, n + 6 is divisible by 5. = 6 days
35. Let if possible n + m is rational number. 360
Number of days taken by IIIrd cyclist =
p 72
n + m = ; p , q are integers and q 0 = 5 days
q
12 ADDITIONAL
R

PR ACTICE MATHEMATICS - 10
Now, we need to find LCM (7.5, 6, 5) For r = 1
75 a = 6q + 1
7.5 =
10 = 2 (3q) + 1 which is odd
60
6 = For r = 2
10
50 a = 6q + 2
5 = = 2 (3q + 1) which is even
10
 75 60 50  For r = 3
So, LCM (7.5, 6, 5) = LCM  10 , 10 , 10 
  a = 6q + 3
LCM (75,60,50) = 6q + 2 + 1
=
HCF (10,10,10) = 2(3q + 1) + 1 which is odd
300 For r = 4
=
10
a = 6q + 4
=
30
= 2 (3q + 2) which is even
So, all the cyclists will meet at starting point
after 30 days. For r = 5

38. (a) In order to find the maximum number of a = 6q + 5


columns in which they can march, we will = 6q + 4 + 1
find HCF (32, 616).
= 2 (3q + 2) + 1 which is odd
32 = 2 5
Therefore, every positive integer is of form 6q
616 = 23 × 7 × 11 + 1 or 6q + 3 or 6q + 5.
So, HCF (32, 616) = 23 = 8 (b) LCM (x3 y3, x3 y5)
Hence, maximum number of columns = 8    =x3 y5
(b) W
 e know that for any two positive
40. (a) 135 and 225
integers a and b,
225 = 135 × 1 + 90
LCM (a, b) × HCF (a, b) = a × b
LCM (306, 657) × HCF (306, 657) 135 = 90 × 1 + 45
= 306 × 657 90 = 45 × 2 + 0
LCM (306, 657) × 9 = 306 × 657 So, HCF (135, 225) = 45
306 × 657 (b) 196 and 38220
LCM (306, 657) = = 22338
9
38220 = 196 × 195 + 0
39. (a) According to Euclid's Division lemma,
So, HCF (196, 38220) = 196
   a = bq + r; 0 < r < b
Take b = 6 (c) 867 and 255
a = 6q + r; 0 < r < 6 867 = 255 × 3 + 102
For r = 0 255 = 102 × 2 + 51
a = 6q 102 = 51 × 2 + 0
= 2 (3q) which is even So, HCF (867, 255) = 51
ADDITIONAL
R

PR ACTICE MATHEMATICS - 10 13
cASe StUDY-1 (iv) (b) 2 480, 130
2 240, 65
(i) (c) The product of non-zero rational and
2 120, 65
irrational number is always irrational.
2 60, 65
7 2 30, 65
(ii) (d) =1
7 3 15, 65
(iii) (a) 3 = 2 × 1 + 1 5 5, 65
5=2×2+1 13 1, 13
7=2×3+1 11, 1
Hence every odd integer is in form of LCM of 480 and 130 = 2 × 2 × 2 × 2 × 2
(2p + 1). × 3 × 5 × 13 = 6240.
(iv) (b) The number 2 and 3 are consecutive
natural numbers and their product, 6 is an (v) (c) The number of stacks required for english
even number. Total number of books
books =
(v) (c) The number 60 is divisible by 1, 2, 3, 4, 5. Number of book in each stack

480
cASe StUDY-2 =
10
= 48

(i) (a) The numbers which have no factor other The number of stacks required for hindi
than itself and 1 are called prime numbers. books
(ii) (b) The problem can be solved using Euclid's
Total number of books
division lemma. =
Number of book in each stack
(iii) (a) The maximum number of books that can
be equally placed in a stack are 10. As 10 is 130
the only common factor of 980 and 130. = = 13
10

14 ADDITIONAL
R

PR ACTICE MATHEMATICS - 10
Chapter

2 Polynomials
Multiple choice Questions

1. (a) Let a, b be the zeroes of f(x) 2. A quadratic polynomial is of form p(x) =


c {x2 – (sum of zeroes) x + product of zeroes}
\ ab = = 3
a
k  −1 
= 3 = {x2 –   x + (–3)}
l  2 
k = 3 = ½ {2x2 + x – 6}
–b −3 c 7 3. Let p(x) = x4 + x3 – 2x2 + x + 1
2. (c) a + b = = , ab = =
a 4 a 4
-b -3 p(1) =(1)4 + (1)3 – 2(1)2 +1 +1
1 1 α+β -3 Remainder is p(1) = 1 + 1 – 2 + 1 + 1 = 2
So, + = = a = 4 =
α β αβ c 7 7
a 4 4. A binomial of degree 6 is x6 + 4x2.
3. (d)
5. 3x3 – x2 – 3x + 1
4. (b) Let p(x) = 2x2 + 2ax + 5x + 10
= x2 (3x – 1) – 1 (3x – 1)
As (x + a) is a factor of p(x),
= (x2 – 1) (3x – 1)
\ p(–a) = 0
= (x + 1) (x – 1) (3x – 1)
2(–a)2 + 2a(–a) + 5 (–a) + 10 = 0
2a2 – 2a2 – 5a + 10 = 0 –B C
6. a + b = = 11, ab = = 30
A A
5a = 10
a3 + b3 = (a + b) (a2 + b2 – ab)
a = 2
= (a + b) [(a + b)2 – 3ab]
5. (b) Discriminant (D) = b – 4ac 2
= 11 (121 – 90)
Here, a = 4, b = 10 and c = 2
= 11 (31)
= (10) – 4(4)(2) = 100 – 32 = 68
2
= 341

WoRKSHeet - 1 7. p(x) = 6x2 – 3 – 7x


= 6x2 – 7x – 3
Section-A
= 6x2 – 9x + 2x – 3
1. b2 – 4ac = 0 = 3x (2x – 3)+ 1 (2x – 3)
p(x) has two equal zeroes. = (2x – 3) (3x + 1)

ADDITIONAL
R

PR ACTICE MATHEMATICS - 10 15
3 -1 11. p(x) = 3x2 – 6x + 4
Now, p(x) = 0 x= , .
2 3 –b c 4
a+b=
= 2, ab = =
3 -1 a a 3
So, zeroes are x = , .
2 3
α β  1 1
+ + 2  +  + 3αβ
8. p(x) = 4x2 – 5x – 1 β α α β

–b 5 c -1 a 2 + b2 a+b
a+b=
= , ab = = = +2 + 3ab
a 4 a 4 ab Ł ab ł
-1  5  -5
a2b + ab2 = ab (a + b) =
=
 
(a + b )
2
4 4 16 − 2ab a+b
= +2 + 3ab
ab  ab 
Section-B
8  
4−
9. p(x) = 6x3 + 3x2 – 5x + 1 = 3 + 2  2  + 3 4 
4 4  3 
-3 -1  
a+b+g=
= 3 3
6 2
-1 = 1 + 3 + 4
abg=

6
= 8
So, a b g
–1 –1   –1

1
1 1 12. Let the zeroes be a, .
= = = -6 a
abg -1
6 1 - 13 1 6a
a+ = 2 , α× =2
So, a2bg + ab2g + abg2 a a +9 α a +9

6a
= abg (a + b + g) So, 1 = a2 – 6a + 9 = 0
a +9
2

−1  −1 1 a2 – 3a – 3a + 9 = 0
= =
6  2  12
a (a – 3) – 3 (a – 3) = 0
10. Dividend = Divisor × Quotient + Remainder (a – 3) (a – 3) = 0
x + 2x + 4x + b = (x + 1) (x + ax + 3)
3 2 2
a = 3
+ (2b – 3)
= (x3 + ax2 + 3x + x2 + ax + 3 + 2b – 3) 13. p(t) = kt2 + 2t + 3k

On comparing coefficients of x2 and constant Sum of zeroes = Product of zeroes


terms we get, –2 3k
=
a + 1 = 2 a = 1 k k
b = 3 + 2b – 3 b=0 −2
k =
3

16 ADDITIONAL
R

PR ACTICE MATHEMATICS - 10
14. 2x + 2x – 1 On putting (ii) in (i), we get
2

4x2 + 3x – 2 8x4 + 14x3 – 2x2 + 8x – 12 2 r


3p2 – p - = 3q
Ł pł
8x4 + 6x3 – 4x2 r
2p2 + = 3q
– – + p

8x3 + 2x2 + 8x – 12 Section-C


8x3 + 6x2 – 4x
17. p(x) = 2x2 – 5x + 7
– – + 5 7
a + b = , ab =
– 4x2 +12x – 12 2 2
Polynomial whose zeroes are 2a + 3b and
– 4x2 – 3x + 2 3a + 2b is,
+ + – k {x2 – (2a + 3b + 3a + 2b) x + (2a + 3b)
15x – 14 (3a + 2b)}
= k {x2 – 5(a + b) x + 6a2 + 4ab + 9ab
15. Cubic polynomial is of form + 6b2}
{x3 – (a + b + g) x2 + (ab + bg + ag) x – abg} = k {x2 – 5(a + b) x + [6 {(a + b)2 – 2ab}
= k {x3 – (5 + 6 – 1) x2 + (30 – 6 – 5) x – (–30)} + 13ab]}

= k {x3 – 10x2 + 19x + 30}  2 25   25  91 


= k  x − x + 6  − 7  +  
 2   4  2  
16. Let a – d, a, a + d be the zeroes of p(x).
2 25
a – d + a + a + d = – 3p = k x - x + 41
2
3a = – 3p k
=
2
{
2x 2 − 25x + 82 }
a = – p
18. Dividend = Divisor × Quotient + Remainder
a ( a – d) + a (a + d) + (a – d) ( a + d) = 3q
x4 + 2x3 – 2x2 + x – 1 = (x2 + 2x – 3) Quotient
–p (–p –d) – p(–p + d) + (p2 – d2) = 3q + Remainder
x2 + 1
p2 + pd + p2 – pd + p2 – d2 = 3q
x2 + 2x – 3 x4 + 2x3 – 2x2 + x – 1
3p2 – d2 = 3q ...(i) x4 + 2x3 – 3x2
– – +
(a – d) a (a + d) = –r
x2 + x – 1
a (a2 – d2) = –r x2 + 2x – 3
– – +
–p (p2 – d2) = –r
–x +2
r
p2 – d2 =
p So, x4 + 2x3 – 2x2 + x – 1 = (x2 + 2x – 3) (x2
r + 1) + (–x + 2)
d=
2
p2 − ...(ii)
p So, – (–x + 2) = x – 2 must be added to
the polynomial p(x).
ADDITIONAL
R

PR ACTICE MATHEMATICS - 10 17
19. 2x2 + 5 Now, we will find the quotient.
3x2 + 4x + 1 6x4 + 8x3 + 17x2 + 21x + 7 Dividend = x3 + 2x2 + kx + 3
= x3 + 2x2 – 9x + 3
6x4 + 8x3 + 2x2
– – – Divisor = x – 3
15x2 + 21x + 7
x2 + 5x + 6
15x + 20x + 5 2

– – – x – 3 x3 + 2x2 – 9x + 3
x +2 x3 – 3x2
– +
On comparing x + 2 with ax + b, we get
5x2 – 9x + 3
a = 1, b = 2
5x2 – 15x
20. Let the quotient be q(x) = ax + bx + c and
2
– +
remainder r(x) = px + q 6x + 3
Using division algorithm, 6x – 18
– +
p(x) = g (x) q(x) + r(x)
21
  3x4 + 5x3 – 7x2 + 2x + 2
So, quotient = x2 + 5x + 6
= (x2 + 3x + 1) (ax2 + bx + c) + px + q
= ax4 + bx3 + cx2 + 3ax3 + 3bx2 + 3cx
22. Zeroes are - 3 and 3
+ ax2 + bx + c + px + q
So, factors are (x + 3 ), (x – 3)
a = 3
5 = b + 3a b = 5 – 3a b = –4 i.e. (x + 3 ) (x – 3 ) is also a factor

–7 = c + 3b + a i.e. x2 – 3 is a factor of given polynomial.


–7 = c – 12 + 3 c=2
2x + 1
2 = 3c + b + p x2 – 3 2x3 + x2 – 6x – 3
2 = 6–4+p p=0 2x3 – 6x
– +
2 = q+c q=2–2=0
x2 – 3
So, Remainder = px + q = 0
x2 – 3
– +
As remainder is zero, g(x) is a factor of p(x).
0

21. Let p(x) = x3 + 2x2 + kx + 3
For the remaining zero,
Remainder = f(3) = 21
put 2x + 1 = 0
33 + 2(3)2 + 3k + 3 = 21
-1
27 + 18 + 3k + 3 = 21 x =
2
3k = 21 – 48 = – 27 23. As 2 is a zero of given polynomial, x – 2
k = –9 is a factor of the polynomial.
18 ADDITIONAL
PR ACTICE
R

MATHEMATICS - 10
6x2 + 7 2 x + 4 Section-D
x– 2 6x3 + 2 x2 – 10x – 4 2 25. p(x) = x2 – px + q
6x3 – 6 2 x2
– + a + b = p, ab = q

7 2 x2 – 10x Consider
7 2 x – 14x 2
LHS
– +
4x – 4 2 a 2 b2 α 4 + β4
+ =
4x – 4 2 b2 a 2 α 2β2
– +
( α ) + (β )
2 2
2 2
0
=
For other zeroes, α 2β2
6x2 + 7 2 x + 4 = 0  α 2 + β2  − α 2β2
 
=
6x2 + 3 2 x + 4 2 x + 4 = 0 ( αβ )
2
3x (2x + 2 ) + 4( 2 x + 4) = 0 (a + b )2 − 2ab  − 2(ab) 2
 
3 2 x ( 2 x + 1) + 4( 2 x + 1) = 0 = 2
(ab)
(3 2 x + 4) ( 2 x + 1) = 0
p 2 − 2q − 2 (q)
2 2

-4 -4 2 -2 2 =
( q)
2
x = = =
3 2 6 3
- 2-1 p 4 + 4q2 - 4p 2 q - 2q2
and x = = =
2 2 q2
24. According to division algorithm, p 4 4p 2
Dividend = Divisor × Quotient + Remainder = 2 - +2 = RHS
q q
x3 – 3x2 + x + 2 = g(x) (x – 2) + (–2x + 4)
3 2
26. Let p(x) = x3 – 2x2 + qx – r
x - 3x + x + 2 + 2x - 4
g(x) = a+b+g=2

x-2
x 3 - 3x 2 + 3x - 2 For a+b = 0 0+g=2
=
x-2
g=2
x2 – x +1
Also, abg = r
x – 2 x3 – 3x2 + 3x – 2
x3 – 2x2 2ab = r
– +
ab + bg + ag = q
– x2 + 3x – 2
– x2 + 2x
ab + g (a + b) = q

+ – ab + g (0) = q [As a + b = 0]
x – 2
ab = q
x – 2
– + r
0 = q
2
So, g(x) = x2 – x + 1 2q = r
ADDITIONAL
R

PR ACTICE MATHEMATICS - 10 19
2x2 – 3x + (–8 –2 k) = k {x2 – (a + 2 + b + 2)x + (a + 2) (b + 2)}
27.
x2 + 2x + k 2x4 + x3 – 14x2 + 5x + 6
= k {x2 – (a + b + 4)x + ab + 2 (a + b) + 4}
2x + 4x + 2kx 4 3 2

– – – = k {x2 – (2 + 4)x + 3 + 2 (2) + 4}


– 3x3 + x2(–14 –2k) + 5x + 6
= k {x2 – 6x + 11}
– 3x3 – 6x2 – 3kx
+ + +
x2 (–8 –2k) + x (5 + 3k) + 6 (b) Sum of zeroes
x2 (–8 –2k) + x (–16 – 4k) + k(–8–2k) a -1 b -1
– – – = +
x(5 + 3k + 16 + 4k) + 6 + 8k + 2k2 a +1 b +1

Remainder = (21 + 7k)x + 6 + 8k + 2k2


( a - 1)(b + 1) + ( a + 1)(b - 1)
=
( a + 1)(b + 1)
As x2 + 2x + k is a factor of
ab + a - b - 1 + ab - a + b - 1
2x4 + x3 – 14x2 + 5x + 6, =
ab + a + b + 1
So, Remainder should be zero 2ab - 2
=
(21 + 7k)x + 6 + 8k + 2k2 = 0 ab + a + b + 1
6−2 As
   = 0x + 0 =
3 + 2 +1
On comparing coefficient of x, we get a+b=2
4 2
21 + 7k = 0 = =
6 3 ab = 3
k = –3
Now, we will find zeroes of the two  α −1   β −1 
Product of zeroes =   
polynomials.  α + 1  β + 1

2x4 + x3 – 14x2 + 5x + 6 =
(α −1)(β −1)
= (x2 + 2x + k) [2x2 – 3x + (–8 –2 k)] (α +1)(β +1)
= (x2 + 2x – 3) (2x2 – 3x – 2) αβ − ( α + β ) +1
=
= (x + 3x – x – 3) (2x – 4x + x – 2)
2 2
αβ + ( α + β ) +1
= [x (x + 3) – 1 (x + 3)] [2x (x – 2) + 1
3 - 2 +1
(x – 2)] =
3 + 2 +1
= (x + 3) (x – 1) (2x + 1) (x – 2)
2
-1 =
So, zeroes are –3, 1, , 2. 6
2 1
=
28. p(x) = x2 – 2x + 3 3
A quadratic polynomial is of form
a+b = 2

k {x2 – (sum of zeroes) x + product of zeroes}
ab
= 3
(a) Roots are (a + 2, b + 2)  2 2 1
= k  x − x + 
 3 3
Polynomial is
k
k {x2 – (sum of zeroes) x + product of zeroes} =
3
{
3x 2 − 2x +1 }
20 ADDITIONAL
R

PR ACTICE MATHEMATICS - 10
29. x2 – 2 5 x + 3 -3 -3 3
So,  g =+3= +3=
a 2 2
x– 5 x3 – 3 5 x2 + 13x – 3 5
−b −b
Also, ab + bg + ag = =
x3 – 5 x2 a 2
– +
– 2 5 x2 + 13x – 3 5
2 + (–2) 3
2 ( )
+ (–1) 3
2
= ( )
–b
2
3 –b
2 – 3 – =
– 2 5 x2 + 10x 2 2
+ – 3 –b
–1 – =
2 2
3x – 3 5
–5 –b
=
3x – 3 5 2 2
– + b = 5
0
31. As zeroes of q(x) are also the zeroes of p(x),
For other zeroes, so, remainder should be zero. [As q(x) is a
Consider x2 – 2 5 x + 3 = 0 factor of p(x)].
x2 – 3x + 2
2 5 ± 20 −12
x = x3 + 2x2 + a x5 – x4 – 4x3 + 3x2 + 3x + b
2
x5 + 2x4 + ax2
2 5± 8 – – –
= – 3x – 4x3 + (3 – a)x2 + 3x + b
4
2
– 3x4 – 6x3 – 3ax
2 5±2 2 + + +
=
2 2x + (3 – a)x + (3 + 3a)x + b
3 2

= 5± 2 2x3 + 4x2 + 2a
– – –
30. Let p(x) = ax3 + 3x2 – bx – 6 (–a –1) x2 + (3 + 3a) x + (b – 2a)

Let   a, b, g be the zeroes such that Remainder = 0


a = –1 and b = –2 (–a –1) x2 + (3 + 3a) x + (b – 2a) = 0
–a –1 = 0, b – 2a = 0
−3
a+b+g = a = –1,   b + 2 = 0
a
a = –1,   b = –2
-3
–1 –2 + g = Now,
a
p(x) = (x3 + 2x2 + a) (x2 – 3x + 2) + 0
−3
g = +3 = (x3 + 2x2 – 1) (x2 – 3x + 2)
a
6 For other zeroes of p(x),
Also, abg =
a
Put x2 – 3x + 2 = 0
-3 6 x2 – 2x – x + 2 = 0
2 +3 =
Ł a ł a
x(x – 2) – 1(x – 2) = 0
−3 3
+3 = (x – 1) (x – 2) = 0
a a
x = 1, 2
6
3 = a=2 So, x = 1, 2 are zeroes of p(x) but not of q(x).
a
ADDITIONAL
R

PR ACTICE MATHEMATICS - 10 21
32. (a) f(x) = x3 – 5x2 – 16x + 80 −a
1 =
Let the two zeroes be a, –a and the third 5
zero be g. a = –5
a + (– a) + g = 5 4. p(x) has 2 distinct real zeroes.
g = 5 5. Let p(x) = x3 + ax2 + bx + c
Also a (– a) g = – 80 Let a, b, g be zeroes of p(x)
– a2 (5) = – 80 Such that a = –1
80 abg = –c
a2 = = 16
5 (–1) bg = –c
a = +4
bg = c
For a = – 4, – a = – (– 4) = 4
So, product of other two zeroes = c
For a = 4, – a = – 4.
6. Quadratic polynomial is of form p(x) =
So, zeroes are – 4, 4, 5
{x2 – (Sum of zeroes) x + product of zeroes}
(b) p(x) = x2 – p (x + 1) – c
= x2 – px – (p + c)  2 2 1 2  −1 
= x −  −  x +  
 3 4 3  4 
a + b = p, ab = – (p + c)
 2 5 1
Consider = x −   x − 
 12  6
(a + 1) (b + 1) = ab + (a + b) + 1
12x 2 − 5x − 2 
= – (p + c) + p + 1 =  
 12 
=1–c
= (12x – 5x – 2)
2

WoRKSHeet - 2 7. 2y2 + 7y + 5
–7
Section-A a+b =
2
5
1. p(x) has 2 real zeroes. ab =
2
2. x2 + 7x + 12 8. The sign of c is negative.
= x2 + 3x + 4x + 12 9. p(x) = (k2 + 4)x2 + 13x + 4k
= x2 + (x + 3) + 4(x + 3) 1
Let the two zeroes be a,
a
= (x + 3) (x + 4)
1 4k
For zeroes of polynomial, 1 = α  = 2
α
  k +4
x + 3 = 0, x + 4 = 0 k2 – 4k + 4 = 0
x = –3, x = –4 (k – 2)2 = 0
1 k = 2
3. Let a, be the zeroes of p(x).
a 10. x2 + 99x + 127
1 −a a + b = – 99, ab = 127
a =
a 5
22 ADDITIONAL
R

PR ACTICE MATHEMATICS - 10
a, b are either both positive or both negative
14. 3x2 – x
If a, b are both positive then a + b = – 99 is 3x2 + x – 1 9x4 – 4x2 + 4
not possible
So, a and b must be negative. 9x4 + 3x3 – 3x2
– – +
Section-B

– 3x3 – x2 + 4
– 3x3 – x2 + x
11. p(x) = x – px + q
2
+ + –
a + b = p,   ab = q
– x + 4
(a) Consider a2 + b2 = (a + b)2 – 2ab
Quotient = 3x2 – x

= p2 – 2q
Remainder = –x + 4
1 1 α+β p
(b) += = 15. p(x) = x2 – 1 = x2 + 0 x – 1
α β αβ q
a + b = 0, ab = –1

12. p(x) = x – 5x + k,
2
2a 2b
a + b = 5 Sum of zeroes = +
b a
a – b = 1 a 2 + b2
= 2
2a = 6 Ł ab ł
a = 3  ( α + β)2 − 2αβ 
So, b = 5 – a = 5 – 3 = 2 = 2  
 αβ 
As, 2 is a zero of p(x)  
 (0 )2 − 2x (−1) 
p(2) = 0 = 2  
 (−1) 
4 – 10 + k = 0
= 2 (–2) = –4
k = 6
2 α 2β
13. Quadratic polynomial is of form p(x) = Product of zeroes = × =4
β α
{x2 – (sum of zeroes)x + Product of zeroes} A quadratic polynomial is of form p(x) = {x2 –
(sum of zeroes) x + product of zeroes}
4+ 2 4- 2
+
Sum of zeroes =
2 2 = {x2 + 4x + 4}
= 4
16. p(x) = ax2 + bx + c
4 + 2  4− 2 
Product of zeroes =     -b c
 2   2  a+b = , ab =
a a
16 - 2 14 7 a 2
b 2
α +β
3 3

= = = Consider + =
4 4 2 b a αβ
So, quadratic polynomial is p(x) = =
(
(α + β ) α2 + β2 − αβ )
αβ
 2 7
 x − 4x +  (α + β ) ( α + β ) − 3αβ
2

 2
= {2x2 – 8x + 7} =
αβ
ADDITIONAL
R

PR ACTICE MATHEMATICS - 10 23
i.e. (x – 5) (x – 8) = 0
 −b    −b  c
2

 a    a  − 3× a 
  x = 5, 8
= c
19. 2x2 + 2x – 1
a
−b  b 2 − 3ac  4x2 + 3x – 2 8x4 + 14x3 – 2x2 + 7x – 8
= c 
 a 2 
8x4 + 6x3 – 4x2
– – +
17. As 1 is a zero of p(x),
8x3 + 2x2 + 7x – 8
so, (x – 1) is a factor of p(x).
8x3 + 6x2 – 4x
–x2 – x + 6 – – +
x – 1 – x3 + 7x – 6 – 4x2 + 11x – 8
– x3 + x2 – 4x2 – 3x + 2
+ – + + –
– x2 + 7x – 6 14x – 10
– x2 + x
+ – 
So, 14x – 10 must be subtracted from
6x – 6 8x4 + 14x3 –2x2 + 7x – 8. So, that the resultant
6x – 6 polynomial is exactly divisible by 4x2 + 3x – 2.
– + 20. p(t) = t2 – 4t + 3
0
a + b = 4,
ab = 3
For other zeroes of p(x), Consider
put –x2 – x + 6 = 0 a4 b3 + a3 b4 = a3 b3 (a + b)

x2 + x – 6 = 0 = (ab)3 (a + b)

x2 + 3x – 2x –6 = 0 = 27 (4) = 108

x (x + 3) –2 (x + 3) = 0 1 1 α+β 4
And + = =
a b αβ 3
(x – 2) (x + 3) = 0
x = 2, –3 Section-C
So, other zeroes are x = 2, –3 21. Let a – d, a and a + d be the zeroes of f(x).
18. p(x) = x2 – 13x + k a – d + a + a + d = 12
Let a, b be two zeroes of p(x), 3a = 12
ab = k = 40 a = 4
So, p(x) = x2 – 13x + 40 Also, (a – d) a (a + d) = 28
= x2 – 5x – 8x + 40 (4 – d) 4 (4 + d) = 28
= x (x – 5) – 8 (x – 5) 16 – d2 = 7
= (x – 5) (x – 8) d2 = 9
For zeroes of p(x), put p(x) = 0 d = +3
24 ADDITIONAL
R

PR ACTICE MATHEMATICS - 10
Case 1 Case 2 8 19 65 8 26
x + - + - +
a = 4, d = 3 a = 4, d = –3 Ł9 3 9 ł Ł 9 9 ł

So, zeroes are So, zeroes are 7, 4, 1
= 30x4 + 9x3 + x2 + 0x + 2
a – d, a, a + d = 1, 4, 7
= 30x4 + 9x3 + x2 + 2
Therefore, zeroes of polynomial are 1, 4 and 7.
= Dividend   Hence verified.

22. 19 8 23. p(x) = 4 3 x2 + 5x – 2 3


10x2 + x –
3 9
p(x) = 4 3 x2 + 8x – 3x – 2 3
3x – x + 1 30x + 9x
2 4 3
+ x + 2
2

= 4x ( 3 x + 2) – 3 ( 3 x + 2)
30x4 – 10x3 + 10x2
– + – = (4x – 3 ) ( 3 x + 2)
19x3 – 9x2 + 2
For zeroes of p(x), put p(x) = 0
19 2
19
19x3 –
+ x x
33 (4x – 3 ) ( 3 x + 2) = 0
– +

8
19 3 −2
– x2 –x + 2 x= ,
33 4 3
8
8 8 – Coefficient of x
– x2 +
x – Sum of zeroes =
3
9 9 Coefficient of x2
+ +–
65 26 3 2 5
– x+ = - = −
9 9 4 3 4 3
Dividend = 30x4 + 9x3 + x2 + 2 3−8 5
= = -
Divisor = 3x2 – x + 1 4 3 4 3
19 8 Coefficient of x
Quotient = 10x2 + x– \
Sum of zeroes =
3 9 Coefficient of x2
65 26  3   −2 
Remainder = – x+
9 9   Product of zeroes =    
 4  3
According to divisor algorithm,
1
Dividend = Divisor × Quotient + Remainder = -
2
Consider, Constant term

Divisor × Quotient + Remainder Coefficient of x2

 19 65  65 26 2 3
= (3x2 – x + 1) 10x 2 + x −  − + = -
 3 9  9 9 4 3
8 19 2 8 1
= 30x4 + 19x3 – x2 – 10x3 – x + x+ = -
3 3 9 2
19 8 65 26 Constant term
10x2 + x– – x+ \
Product of zeroes =
3 9 9 9 Coefficient of x2
 8 19  Hence, the relationship between zeroes and
= 30x 4 + x 3 (19 −10 ) + x 2  − − + 10  + its coefficient is verified.
 3 3 
ADDITIONAL
R

PR ACTICE MATHEMATICS - 10 25
24. p(x) = x2 – x – 2 16 - 9 28
–b c = 4 =
a+b= = 1, ab = = –2 Ł 9 ł 9
a a
Sum of zeroes = 2a + 1 + 2b + 1 a 2b 2 1
Product of zeroes = = ab =
ab 3
= 2(a + b) + 2
Quadratic polynomial is of the form p(x) =
= 2 (1) + 2 {x2 – (sum of zeroes) x + product of zeroes}
= 4 28 1
2
= k x - x+
Product of zeroes = (2a + 1) (2b + 1) 9 3
= 4ab + 2(a + b) + 1 k
=
9
{
9x 2 − 28x + 3 }
= – 8 + 2 + 1
= –5 26. x2 + x + 7
Quadratic polynomial is of form p(x) = x2 + 1 x4 + x3 + 8x2 + ax + b

{x2 – (sum of zeroes) x + Product of zeroes} x4 + x2


– –
= {x2 – 4x – 5} x3 + 7x2 + ax + b
Now, we need to find a3 + b3 x3 + x
– –
= (a + b) (a + b – ab) 2 2

7x2 + (a – 1) x + b
= (a + b) [(a + b)2 – 3ab]
7x2 +7
= 4(16 + 15) – –
= 4(31) (a – 1)x + (b – 7)

= 124 As x4 + x3 + 8x2 + ax + b is exactly divisible by


x2 + 1
25. p(x) = 3x2 – 4x + 1
\
Remainder = 0
–b 4 c 1
a+b= = , ab = =
a 3 a 3 (a – 1)x + (b – 7) = 0
α 2
β 2
a = 1, b = 7
Sum of zeroes = +
β α
a + b3
3
27. Let a, b be the zeroes of p(x)
=
ab
b c
(
(α + β ) α2 + β2 − αβ ) a + b = - , ab =
a a
=
αβ 1 1
Zeroes of the required polynomial are ,
a b
(a + b) (a + b) − 3ab 
2

 Quadratic polynomial is of form p(x) =


=
ab  2 1 1 1
4 16 
x −  +  x + 
−1  a b ab 
3  9 
= a+b 1
1 = x2 - x+
3 Ł ab ł ab

26 ADDITIONAL
R

PR ACTICE MATHEMATICS - 10
  b  30. x2 + 2x + 3
 2  − a  a 
= x −  x +  x2 + 5 x4 + 2x3 + 8x2 + 12x + 18
 c  c
  x4 + 5x2
  a   – –
 2 b a 2x3+ 3x2 + 12x + 18
= x + x + 
 c c 2x3 + 10x
= {cx + bx + a}
2 – –
3x + 2x + 18
2

28. p(x) = x3 – 4x2 – 3x + 12 3x2 + 15


– –
As 3 , – 3 are zeroes of p(x), so (x – 3) 2x + 3
and (x + 3 ) are factors of p(x). On comparing 2x + 3 with px + q,
i.e. (x – we get   p = 2, q = 3
3 ) (x + 3 ) = x – 3 is a factor of
2

p(x)
Section-D
x – 4
31. x – 4x + (8 – k)
2
x2 – 3 x3 – 4x2 – 3x + 12
x2 – 2x + k x4 – 6x3 + 16x2 – 25x + 10
x – 3x 3
x4 – 2x3 + kx2
– + – + –
– 4x2 + 12 – 4x3 + (16 – k)x2 – 25x + 10

– 4x2 + 12
– 4x3 + 8x2 – 4kx
+ – + – +
(8 – k) x2 + (–25 + 4k) x + 10
0
– (8 – k) x2 – 2(8 – k) x + k (8 – k)

For third zero, x – 4 = 0 (–9 + 2k)x + (10 – 8k + k2)


  x = 4
Remainder = (–9 + 2k)x + (10 – 8k + k2)
29. p(x) = 2x2 + 5x + k    =x+a
–b 5 c k –9 + 2k = 1     10 – 8k + k2 = a
a+b=
= – , ab = =
a 2 a 2 2k = 10      10 – 40 + 25 = a
21 k = 5   –5 = a
Given: a2 + b2 + ab =
4 32. p(x) = x4 – 6x3 – 26x2 + 138x – 35
21 Zeroes of p(x) are 2 + 3.
   (a + b)2 – ab =
4
So, [x – (2 + 3 )], [x – (2 – 3 )]
25 k 21
   – = are factors of p(x)
4 2 4
i.e. [(x – 2) – 3 ] [(x – 2) + 3 ] is a factor
k 25 21
   = – =1 of p(x).
2 4 4
i.e. (x – 2)2 – ( 3 )2 is a factor of p(x)
   k = 2
i.e. x2 + 4 – 4x – 3 is a factor of p(x)
ADDITIONAL
R

PR ACTICE MATHEMATICS - 10 27
i.e. x2 – 4x + 1 is a factor of p(x) If a = 3, b = 7 – a = 7 – 3 = 4

x2 – 2x – 35 If a = 4, b = 7 – a = 7 – 4 = 3
So, zeroes of the polynomial are 3, 4 and –2.
x2 – 4x + 1 x4 – 6x3 – 26x2 + 138x – 35
x4 – 4x3 + x2 34. p(x) = 2x4 – 2x3 – 7x2 + 3x + 6
– + –
3 3
– 2x – 27x2 + 138x
3 - and are zeroes of p(x).
2 2
– 2x3 + 8x2 – 2x 
+ – + 3  3
 x + 2  ,  x − 2  are factors of p(x).
– 35x2 + 140x – 35    

– 35x2 + 140x – 35  3  3
+ – +  x + 2   x − 2  is a factor of p(x).
   
0
 2 3
 x − 2  is a factor of p(x).
For other zeroes,
Put x2 – 2x – 35 = 0 (2x2 – 3) is a factor of p(x).

x2 – 7x + 5x – 35 = 0 x2 – x – 2
x (x – 7) + 5 (x – 7) = 0 2x2 – 3 2x4 – 2x3 – 7x2 + 3x + 6
(x + 5) (x – 7) = 0 2x4 – 3x2
– +
x = –5, 7 – 2x3 – 4x2 + 3x + 6
So, other zeroes are –5 and 7. – 2x3 + 3x
33. p(x) = x3 – 5x2 – 2x + 24 + –
– 4x 2
+6
Let a, b, g be the zeroes of p(x).
– 4x2 + 6
ab = 12 ...(i) +   –
–b 0
a + b + g = =5
a
–d For other zeroes of p(x),
abg =
= – 24 12g = –24
a
g=–2 Put x2 – x – 2 = 0
Also, a + b + g = 5 a + b –2 = 5 x2 – 2x + x – 2 = 0
a + b = 7 ...(iii) x (x – 2) + 1 (x – 2) = 0
On solving (i) and (ii), we get
(x + 1) (x – 2) = 0
a (7 – a) = 12
x = –1, 2
7a – a2 = 12
a2 – 7a + 12 = 0 35. p(x) = 6x2 + x – 2
a2 – 3a – 4a + 12 = 0 –b 1 c 1
a+b= =– , ab = =–
a(a – 3) – 4 (a – 3) = 0
a 6 a 3
α 2 + β2 ( α + β ) − 2αβ
2
(a – 3) (a – 4) = 0 α β
(a) + = =
a = 3, 4 β α αβ αβ
28 ADDITIONAL
R

PR ACTICE MATHEMATICS - 10
1 2 1 + 24 (b) Let p(x) = 15
+
= 36 3 = 36 g(x) = 4
1 1
− − q(x) = 2
3 3
25 3 25 r(x) = 7
= ×- = -
36 1 12 deg q(x) = deg r(x) = 0
(b) a3 + b3 (c) Let p(x) = 20
= (a + b) (a2 + b2 – ab) g(x) = 3
= (a + b) [(a + b)2 – 3ab) r(x) = 2

 −1  −1
2
 −1  q(x) = 6
 6   6  − 3×  3  

       Here, deg r(x) = 0

 −1  1  37. Let p(x) = 2x3 + x2 – 5x + 2
=    +1
 6   36 
1 3 2

p   = 2   +   − 5   + 2
−1 37 −37 1 1 1
= × =
6 36 216 2    
2 2 2

 1 1 α+β 1 1 5
= 2  + − + 2
(c) 2  +  = 2  
α β  αβ   8  4 2
 1 1 1 5
− = + - +2
 6 4 4 2
= 2 
1 1 5
−  = +2-
 3 2 2
1 5 5
= 2   = - = 0
2 2 2
= 1 1
So, is a zero of p(x).
2
(d) a3 b3 – a5 b5
p(1) = 2 + 1 –5 + 2 = 0
= a3 b3 (1 – a2 b2)
So, 1 is a zero of p(x).
3
 1  1
= −  1 − 9  p(–2) = 2(– 8) + 4 + 10 + 2
 3
= –16 +16
1  9 −1
= − = 0
27  9 
So, – 2 is a zero of p(x).
1 8 8
= −   =
− – Coefficient of x2
27  9  243 Sum of zeroes =
Coefficient of x3
36. (a) Let p(x) = 8 1 1
= +1–2 = –
g(x) = 3 2 2
q(x) = 2 1
= –
2
r(x) = 2 – Coefficient of x2
deg p(x) = deg q(x) = 0 So, Sum of zeroes =
Coefficient of x3
ADDITIONAL
R

PR ACTICE MATHEMATICS - 10 29
Sum of product of zeroes taken two at a For other zeroes of p(x),
Coefficient of x put x2 + 11x + 10 = 0
time =
Coefficient of x3
1 1 5 x2 + 10x + x + 10 = 0
= (1) + 1(–2) + (–2) = –
2 2 2 x(x + 10) + 1 (x + 10) = 0
1
= –2 –1 (x + 1) (x + 10) = 0
2
1 x = –1, –10
= –3
2 So, zeroes of p(x) are –2, –1, –10.
5
= – 39. p(x) = ax2 + bx + c
2
So, sum of product of zeroes taken two at a b c
a + b = – , ab =
Coefficient of x a a
time = (a) a b +ab = ab (a + b)
2 2
Coefficient of x3
– Constant term c  b bc
Product of zeroes = = a  −  =
− 2
Coefficient of x3  a a
1 2
= (1) (–2) = – a4 + b4 = (a2)2 + (b2)2
(b)
2 2
= –1 = –1 = [a2 + b2]2 – 2a2b2
– Constant term = [(a + b)2 – 2ab]2 – 2(ab)2
So, product of zeroes =
Coefficient of x3 b 2 2c 2c2
2

Hence, relationship between zeroes and the = 2


- -
Ła a ł a2
coefficients is verified.
1 2c2
( )
2
38. p(x) = x3 + 13x2 + 32x + 20 = 4 b 2 − 2ac − 2
a a
p(–2) = (–2)3 + 13(–2)2 + 32(–2) + 20 1 4 2c 2
= –8 + 52 – 64 + 20
=
a4
(b + 4a 2 2
c − 4ab 2
c − )
a2
= 12 – 12 b 4 4c 2 4b 2 c 2c 2
= 4 + 2 - 3 - 2
a a a a
= 0
b 4 4b 2 c 2c 2
x + 2 is a factor of p(x). = - 3 + 2
a4 a a
x2 + 11x + 10
40. p(x) = x3 – 6x2 + 3x + 10
x+2 x + 13x + 32x + 20
3 2
a + (a + b) + (a + 2b) = 6
x + 2x 3 2

– – 3a + 3b = 6
11x2 + 32x + 20 a+b= 2
11x + 22x 2
b = 2-a
– –
10x + 20 a(a + b) (a + 2b) = –10

10x + 20 a(2) (4 – a) = –10


–   – 2a (4 – a) = –10
0
8a – 2a2 = –10
30 ADDITIONAL
R

PR ACTICE MATHEMATICS - 10
2a2 – 8a – 10 = 0
−5 ± 25 − 24
a2 – 4a – 5 = 0 =
4
a2 – 5a + a – 5 = 0 −5 ±1
=
a (a – 5) + 1 (a – 5) = 0 4
(a + 1) (a – 5) = 0 −5 +1
x = = –1 and
a = –1, 5 4
−5 −1
For a = –1, b = 2 – a = 2 – (–1) = 3 x = = –1.5
4
For a = 5, b = 2 – a = 2 – 5 = –3
a = –1, b = 3 cASe StUDY-2
zeroes are a, a + b, a + 2b (i) (b) The curve BCD represents a parabola.
= –1, –1 + 3, –1 + 6 (ii) (c) x2 – 7x + 12 = 0
= –1, 2, 5 x2 – 4x – 3x + 12 = 0
for a = 5, b = –3
x (x – 4) –3 (x – 4) = 0
zeroes are a, a + b, a + 2b
(x – 3) (x – 4) = 0
= 5, 5 – 3, 5 – 6
x–3=0 x–4=0
= 5, 2, –1
So, zeroes of the given polynomial are –1, 2 x=3 x=4
and 5. (iii) (b) For the Quadratic equation ax2 + bx + c
= 0 the products of roots a, b is
cASe StUDY-1 c
a–b=
a
(i) (b) As the graph of y = p(x) does not touch
x axis hence there are no zeroes available For the polynormial 2x2 – 7x + k, the root
for the graph. are reciprocal therefore the product of
(ii) (a) The graph which does not intersect with k
roots will be 1 and is equal to
x axis has no zeroes. 2
k
(iii) (c) The graph 1 cut x axis at 3 points hence it \ =1
2
has maximum of 3 zeroes.
⇒k=2
(iv) (a) Those points where the graph of y = p(x)
(iv) (b) The zeroes of the given curve are –4, 0, 2
cuts the x axis are called zeroes of y =
and 4.
p(x).
The sum of –4, 0, 2 and 4 is 2.
(v) (b) p(x) = 2x2 + 5x + 3
(v) (a) For cubic equation ax3 + bx2 + cx + d = 0.
To find zero, equate p(x) to zero
−b
The sum of roots a, b, g is
p(x) = 0 a
2x2 + 5x + 3 = 0 −d
The product of roots abg is
a
(5) − 4 ( 2 )(3)
2
−5 ± The sum product of roots taken 2 at a
x = c
2 (2 ) time is .
a
ADDITIONAL
R

PR ACTICE MATHEMATICS - 10 31
ATQ: abg = –(–r)

Comparing the given polynomial with ab (2) = r
standard form of quadratic polynomial. r
ab = ... (ii)
a = 1, b = –2, c = q, d = –r 2
ab + bg + ag = q
\ a + b + g = –2
r
+ b2 + 2a = q [From (ii)]
As a+b= 0 2
r
\ g=2 ...(i) + 2b – 2b = q
2
As a + b = 0 \ a = –b r = 2q

32 ADDITIONAL
R

PR ACTICE MATHEMATICS - 10
Chapter Pair of Linear Equations
3 in Two Variables
Multiple choice Questions

1. (c) The system of equations has no solution if 15


=
a1 b1 c1 12
= ≠ = 7.5 sq. units
a 2 b2 c2
1 1 4 4. (c) Let number of coins of ` 1 = x
i.e. = ≠
2 k 3 number of coins of ` 2 = y
i.e. k = 2 x + y = 50
\
a1 b1 c1 x + 2y = 75
2. (d) For no solution, = ≠ – – –
a 2 b2 c2
– y = –25
k 5 2
i.e. =- ≠
6 2 7 y = 25
i.e. k = – 15 So, x = 50 – y

3. (a) Y = 50 – 25
B = 25
4

3
A
2

X1 X
0 2x
–2 –1 1 2 3 4 5 6 +3
y=
–1 12
C
1
=

–2
y

x

Y1

2x + 3y = 12 x 6 0 5. (b) Let x be the tens digit and y be the ones


digit.
x–y=1 y 0 4
\ x+y = 9 ...(i)
x=0 x 1 0
y 0 –1 and 10x + y + 27 = 10y + x

1 9x – 9y = –27
area of DABC = ×5×3 x – y = –3 ....(ii)
2

ADDITIONAL
R

PR ACTICE MATHEMATICS - 10 33
On solving (i) and (ii), we get Given pair of equations has infinitely many
solutions if
\ x + y = 9
a1 b c
x – y = –3 = 1 = 1
a2 b2 c2
2x = 6 2 3 7
= =
x = 3 8 a + b 28
From (i), y = 9–x=9–3=6
3 × 28 4
So, number is 10x + y a+b = = 12
7
= 10 (3) + 6 = 36 a + b = 12
WoRKSHeet - 1 6. The pair of linear equations has infinitely many
solutions if
Section-A a1 b c
= 1 = 1
a2 b2 c2
1. 3x – y + 8 = 0, 6x – ky + 16 = 0
10 5 k-5
The equations represent coincident lines if = =
20 10 k
a1 b1 c1
= = k −5
a 2 b2 c2 5 1
= =
10 2 k
3 -1 8
i.e. = = k = 2k – 10
6 -k 16
k = 10
k=2
2. Let number of girls be x and number of boys 7. 2x + 3y = 7
be y. (a + b)x + (2a – b)y = 21
x + y = 15 ...(i) System of equations has infinitely many
x=5+y x–y = 5 ...(ii) a1 b c
solutions if = 1 = 1
On solving equations (i) and (ii), we get a2 b2 c2
x + y = 15 2 3 7
i.e. = =
a + b 2a − b 21
x – y = 5
3 × 21
2x = 20 4a – 2b = 3a + 3b, 2a – b = =9
7
x = 10 a = 5b 2a – b = 9
y = 15 – x = 15 – 10 = 5 2 (5b) – b = 9
9b = 9
3. General form of a pair of linear equations in
two variables x and y is b=1
But, a = 5b
a1x + b1y + c1 = 0
a = 5(1)
a2x + b2y + c2 = 0
a=5
4. If a pair of linear equations in two variables 8. The given system of linear equation has unique
is consistent, then the lines are either solution if
intersecting or coincident.
a b
5. 2x + 3y = 7 ≠
l m
8x + (a + b)y = 28 i.e. am bl
34 ADDITIONAL
R

PR ACTICE MATHEMATICS - 10
Section-B Y
9. x + y = 3 3
3x – 2y = 4
2

4
y=
–2
x 3 0
1

3x
y 0 3
X1 X
x 0 2 –3 –2 –1 0 1 2 3 x
+
y=
y –2 1 –1 3

–2

–3
Y1
Y
10. 3x + 4y = 12
6
6x + 8y = 48
5

x 4 0 4
6x
+
y 0 3 3 8y
=
48

x 8 0 2

y 0 6 1

X1 X
–6 –5 –4 –3 –2 –1 0 1 2 3 4 5 6 7 8 9
3x
–1 +
4y
=
12
–2

–3

–4

–5

Y1

11. (a) x + 2y = –1 = –1 –2 (–2)


(b) 2x – 3y = 12 = –1 +4
= 3
From (a), x = –1 –2y
2x y
2 (–1 –2y) – 3y = 12 [Put in (b)] 12. + = 2 ...(i)
a b
–2 – 4y – 3y = 12 x y
- = 4 ...(ii)
a b
–7y = 14 x y
From (ii), we have = 4+
y = –2 a b
y
So, x = –1 – 2y x = a 4+ ...(iii)
Ł bł
ADDITIONAL
R

PR ACTICE MATHEMATICS - 10 35
Putting this value of x in (i), we get On solving equation 12p + 4q = 1 and 3p + 2q
= 0, we get
2a  y y
 4 +  + = 2 12p + 4q = 1
a  b b
y y 2 (3p + 2q = 0)
2 4 + + = 2 12p + 4q = 1
Ł bł b
2y y 6p + 4q = 0
8 + + = 2 – – –
b b
6p = 1
3y
= –6 1 1
b p = =
6 x
- 6b
y = = –2b 1
3 x = =6
p
 y From equation 3p + 2q = 0, we get
From (iii), x = a 4 + 
 b
1
 2b  3   + 2q = 0
= a  4 −  6
 b  1
2q = –
= a (4 – 2) 2
1
= 2a q = − y=–4
4
13. 28x + 5y = 9 ...(i) Now, we need to find a
3x + 2y = 4 ...(ii) y = ax – 4
On multiplying (i) by 2 and (ii) by 5, we get – 4 = 6a – 4
56x + 10y = 18 6a = 0
15x + 10y = 20 a = 0
– – –
41x = –2 15. 2x + y = 35 ...(i),
2 3x + 4y = 65 ...(ii)

x = –
41 On multiplying equation (i) by 3 and equation
 2 (ii) by 2, we get
From (i), 28  −  + 5y = 9
 41 6x + 3y = 105
56 6x + 8y = 130
- + 5y = 9 – – –
41
56 425 – 5y = –25
5y = 9 + =
41 41
y = 5
85
y = From (i), 2x + 5 = 35
41
1 1 2x = 30
14. Let = p, = q
x y
x = 15
2 1
2p + q = 12p + 4q = 1 a1 b1
3 6 16. For unique solution: ≠
Other equation becomes 3p + 2q = 0 a 2 b2
36 ADDITIONAL
R

PR ACTICE MATHEMATICS - 10
k 2 According to the given condition,

3 1 5x − 8 4
=
k 6 6x − 8 5
a1 b1 c1 25x – 40 = 24x – 32
For infinitely many solutions: = =
x = 8
a 2 b2 c2
k 2 5 So, numbers are 5x = 5(8) = 40
= =
3 1 2.5 and 6x = 6(8) = 48.
k=6
20. 3x – y = 2 2
17. 2x + ky = 11 x 0
3
5x – 7y = 5 y –2 0
a1 b1 c1
For no solution: = ≠
a 2 b2 c2 9x – 3y = 6 x 1 0
2 k 11 y 1 –2
= ≠
5 −7 5
As lines are coincident, so, system of equations
5k = – 14 has infinitely many solutions.
−14 Y
k=
5 2
a b

y=2

6
For unique solution: 1 ≠ 1

3y =
a 2 b2 1

3x –

9x –
2 k
≠ X1 X
5 −7 –4 –3 –2 –1 0 1 2 3 4
14 –1
k -
5
a1 b1 c1 –2
18. For infinitely many solutions: = =
a 2 b2 c2
8 5 9 Y1
= =
k 10 18
5 2 15 7
5k = 8 × 10 21. - = - 1,  + = 10
x+y x-y x+y x-y
k = 16
a b 1 1
For unique solution: 1 ≠ 1 Let =p and =q
a 2 b2 x+y x-y
8 5 5p – 2q = – 1, 15p + 7q = 10

k 10 Using elimination method, we get
5k 8 × 10; k 16 3 [(5p – 2q) = –1] 15p – 6q = –3
15p + 7q = 10
Section-C – – –
19. Let the numbers be 5x and 6x. – 13q = –13
If 8 is subtracted from each of the numbers, q = 1
they become
5x – 8 and 6x – 8 x–y=1

ADDITIONAL
R

PR ACTICE MATHEMATICS - 10 37
From equation 5p – 2q = –1, we get On solving equations (i) and (ii), we get
5p – 2(1) = –1 2(3x – y = 3) 6x – 2y = 6
5p = 1 – 3x + 2y = 6
1 3x = 12
p =
5
x= 4
x+y=5 From (i), y = 3x – 3

= 12 – 3
On solving equations x + y = 5 and
= 9
x – y = 1, we get
x – y = 1 So, Total number of students = xy

x +y = 5 = 4(9)
= 36
2x = 6
24. 3x + 2y = 5
x = 3 y=5–3=2
3x = 5 – 2y
22. Let p(x) = x3 + ax2 +2bx – 24
5 - 2y
x =
As (x – 4) is a factor of p(x), 3
To check : (1, 1) is a point on the 3x + 2y = 5
p(4) = 0
LHS = 3x + 2y
43 + a(4)2 + 2b(4) – 24 = 0
= 3(1) + 2(1)
64 + 16a + 8b –24 = 0 = 5
16a + 8b + 40 = 0 = RHS
2a + b + 5 = 0 So, (1, 1) is a point on the line 3x + 2y = 5.
From a – b = 8, b = a–8 x 1 3
So, 2a + (a – 8) + 5 = 0 y 1 –2
3a = 3
a = 1 Y
3
\
b = a – 8 = 1 – 8 = –7
23. 
Let number of rows be x and number of 2
students in each row be y. So, total number of
students = xy 1
According to question,
X1 X
(y + 3) (x – 1) = xy –3 –2 –1 0 1 2 3
3x

xy + 3x – y – 3 = xy –1
+
2y

3x – y = 3 ...(i)
=

–2
5

Again, (y – 3) (x + 2) = xy
xy + 2y – 3x – 6 = xy –3
–3x + 2y = 6 ...(ii) Y1
38 ADDITIONAL
R

PR ACTICE MATHEMATICS - 10
25. Let digit at ten's place be x and digit at unit's
Y
place be y
3
So, number = 10x + y
According to question, 2

x + y = 5 ...(i) 1
10y + x = 10x + y + 9
X1 X
0 = 9x – 9y + 9 –3 –2 –1 0 1 2 3
–1
x – y = –1 ...(ii)
=2
2y
On solving (i) and (ii), we get x– –2
x +y = 5
–3
x – y = –1 4x – 2y = 5
Y1
2x = 4
As the lines are intersecting, so the system
x = 2 of equations has unique solution and hence,
From (i), y = 5–2=3 consistent.

So, number = 10x + y (b) 3x – 4y = 7,   5x + 2y = 3

= 10(2) + 3       

= 23 x –3 1 x 1 2
26. Let the adjacent angle be x. y –4 –1 y –1 –3.5
4 Y
Other angle = x
5 4
As sum of adjacent angles of a parallelogram is
180°, 3
4 2
x + x = 180
5
1
9x
= 180
5 X1
0
X
20 –4 –3 –2 –1 1 2 3 4
180 × 5 –1
x = = 100°
9
4 –2
Angles are x, x
5x

5
+2

7 –3
y=
4
y=

4
= 100, (100) 3x–
3

–4
5
= 100°, 80° Y1

27. (a) x – 2y = 2,   4x – 2y = 5 28. In DABC,


       A + B + C = 180° (Angle sum
property)
x 0 2 x 0 1.25
y –1 0 y –2.5 0 x + 3x + y = 180°
ADDITIONAL
R

PR ACTICE MATHEMATICS - 10 39
4x + y = 180° ...(i) i.e. x + y = 12 ...(iii)
Given: 3y – 5x = 30° ...(ii) 1
From (i), 32p + 36 =7
From (i), y = 180 – 4x Ł12 ł
So, eqn (ii) becomes 3(180 – 4x) – 5x = 30 32p = 7 – 3 = 4
540 – 12x – 5x = 30 1
p =
17x = 510 8
x – y = 8 ...(iv)
x = 30
From (i), y = 180 – 4 (30) On solving (iii) and (iv), we get

= 180 – 120 x = 10
= 60° y = 2
So,  A = x = 30° Speed of boat in still water = 10 km/hr
B = 3x = 90°     Speed of stream = 2 km/hr
C = y = 60° 30. ax + by = 1 ...(i)
In DABC, B = 90°, so it is a right angled
(a + b )
2
triangle. −1
bx + ay = ...(ii)
a 2 + b2
Section-D a 2 + b 2 + 2ab - a 2 - b 2
=
a 2 + b2
29. Let speed of boat in still water be x km/hr and 2ab
that of stream be y km/hr. bx + ay = 2 ...(iii)
a + b2
So, speed of boat upstream = (x – y) km/hr On multiplying (i) by b and (iii) by a, we get
Speed of boat downstream = (x + y) km/hr abx + b2y = b
According to question, 2a 2b
abx + a2y = 2 2


32
+
36
= 7 – – –a +b
x-y x+y 2a 2b
40 48 y(b2 – a2) = b – 2
+ = 9   a + b2
x-y x+y
1 1 a 2b + b3 - 2a 2b
Let = p, =q y(b – a ) =
2 2

x-y x+y a 2 + b2
So, we get equations as b3 - a 2b
= 2
32p + 36q = 7 ...(i) a + b2
40p + 48q = 9 ...(ii)
b b2 − a 2 ( )
=
a 2 + b2
On multiplying (i) by 5 and (ii) by 4, we get
b
160p + 180q = 35 \ y = 2
a + b2
160p + 192q = 36
– – – b
From (i), ax + b 2 2 =1
– 12q = –1 Ła + b ł
1 b2
ax = 1 – 2

q = a + b2
12
40 ADDITIONAL
R

PR ACTICE MATHEMATICS - 10
a2 10p – 10q = 1
=
a 2 + b2 10p – 20q = –1
– + +
a2 10q = 2
ax =
a 2 + b2 1
a q = y=5
x = 5
a + b2
2
1
From (iii), we get 10 p – 10   = 1
31. Let speed of X be x km/hr and that of Y be y 5
km/hr. 10 p = 1 +2 = 3
Time taken by X to walk 30 km 3
p =
10
30
= hours 10
x x =
3
Time taken by Y to walk 30 km
10
So, Speed of X = km/hr
30 3
= hours
y Speed of Y = 5 km/hr
According to question,
32. a (x + y) + b (x – y) = a2 – ab + b2 ...(i)
30 30
= +3 a (x + y) – b (x – y) = a2 + ab + b2 ...(ii)
x y
Let x + y = p and x – y = q
30 30
- =3
x y So, equations (i) and (ii) becomes
1 1 1 ap + bq = a2 – ab + b2 ...(iii)
- = ...(i)
x y 10 ap – bq = a2 + ab + b2 ...(iv)
Also, 30 = 30 - 3 On adding (iii) and (iv) we get,
2x y 2 2ap = 2 (a2 + b2)
15 30 3 1
= - p = (a2 + b2)
x y 2 a
From equation (iii),
15 30 - 3
- = 1
x y 2 a (a2 + b2) + bq = a2 – ab + b2
a
1 2 1 a2 + b2 + bq = a2 – ab + b2
- = - ...(ii)
x y 10
bq = –ab
1 1 q = –a
Let = p, =q
x y 1
So, x + y = (a2 + b2)
So, equations (i) and (ii) become a
1 x – y = –a
p – q = 10 p – 10q = 1 ...(iii)
10 1 2
1 2x = (a + b2) – a
and p –2q = – 10 p – 20q = 1 ...(iv) a
10 b2 b2
On solving equations (iii) and (iv), we get 2x = a + –a=
a a
ADDITIONAL
R

PR ACTICE MATHEMATICS - 10 41
b2 k -1
x = ≠
2a 6 -2
k 3
b2 b 2 + 2a 2
So, y = x + a = +a=
2a 2a a1 b1 c1
(ii) For no solution : = ≠
1 1 a2 b2 c2
33. Let = p and =q
2x + 3y 3x - 2y k -1 2
= ≠
So, equations become 6 -2 3
1 12 1 k = 3
p+ q =
2 7 2 The system has infinitely many
and 7p + 4q = 2
a b c
solutions, if     1 = 1 = 1
i.e. 7p + 24q = 7 .......(i) a 2 b2 c2
and 7p + 4q = 2 ........ (ii) k -1 2
i.e. = =
On subtracting (ii) from (i), we get 6 -2 3
−1 2
1 Clearly, ≠ ,
20q = 5 q = −2 3
4
1 So, there is no value of k for which the given
From (i), 7p + 24   = 7 system of equations has infinitely many
4
solutions.
7p =1
35. Let the expenditures of U and V be 19x and
1 16x and income of U and V be 8y and 7y.
p =
7
So, we get 2x + 3y = 7 ...(iii) According to question,
3x – 2y = 4 ...(iv)
8y – 19x = 1250 ...(i)
On multiplying (iii) by 3 and (iv) by 2 and 7y – 16x = 1250 ...(ii)
subtracting, we get On multiplying (i) by 7 and (ii) by 8, we get
6x + 9y = 21 56y – 133x = 8750
6x – 4y = 8 56y – 128x = 10000
– + – – + –
13y = 13 – 5x = – 1250
y = 1    x = 250
From (iii), From (i), 8y – 19(250) = 1250
2x + 3(1) = 7 8y – 4750 = 1250
2x = 4 8y = 6000
x = 2 y = 750
Therefore,
34. kx – y = 2
Income of U = 8y = 8 (750)
6x – 2y = 3
= ` 6000
a1 b1 Income of V = 7y = 7 (750)
(i) For unique solution : ≠
a 2 b2 = ` 5250
42 ADDITIONAL
R

PR ACTICE MATHEMATICS - 10
36. The system of equations has infinite number 4p – 2q = 3p + 3q 3p + 3q + 3 = 6p – 3q
of solutions if
p = 5q ...(i) 0 = 3p – 6q – 3
a1 b1 c1
= = p – 2q = 1 ...(ii)
a 2 b2 c2
2 3 9 On solving (i) and (ii), we get
i.e. = =
p + q 2p - q 3 (p + q +1) 1
5q – 2q = 1 q =
2 3 3 3
i.e. = = 5
p + q 2p - q p + q +1 So, p = 5q =
3

WoRKSHeet - 2

Section-A
1. Y
7

5
Area of D = ½ × 6 × 6
4
= 18 sq. units
y
=

3
x

X1 X
–5 –4 –3 –2 –1 0 1 2 3 4 5 6
–1

–2

–3
Y1 x=6

a1 b1 c1
2. The system of equations has no solution if = ≠
a 2 b2 c2
1 2 5
i.e. =
3 k 15
k = 6
3. 3x + y = 1
(2k – 1)x + (k – 1)y = 2k + 1
a1 b1 c1
System of equations is inconsistent if = ≠
a 2 b2 c2
ADDITIONAL
PR ACTICE
R

MATHEMATICS - 10 43
3 1 1 1 ab
i.e. = ≠ Area of triangle = ×a×b= .
2k −1 k −1 2k +1 2 2
8. As (3, a) lies on line 2x – 3y = 5
3k – 3 = 2k – 1
k = 2 2(3) – 3(a) = 5
4. The system of equations represent intersecting 6 – 3a = 5
a b
lines if 1 ≠ 1 3a = 1
a 2 b2
2 1
5 a =
k 3
7
14 9. x + 2y = 8
k
5
2x + 4y = 16
5. The system of equations has a unique solution
a b a1 1
if 1 ≠ 1 Here, =
a 2 b2 a2 2
k –1 b1 2 1
6 –2 = =
b2 4 2
k 3 c1 8 1
= 16 =
6. x + ky = 0 c2 2
2x – y = 0 a1 b c
As = 1 = 1
System of equations has a unique solution if a2 b2 c2
a1 b1 So, the system of equations has infinitely many

a 2 b2 solutions.
1 k
i.e. 10. x + y = 14
2 –1
–1 x–y=4
i.e. k
2

7. Y

(0, b)

X1 (a, 0)
X

x y
+ =1
a b
Y1
44 ADDITIONAL
R

PR ACTICE MATHEMATICS - 10
a1 1 b1 1 c1 14 7 a1 b1
Here, = , = = –1, = = 14. For unique solution,
a2 1 b2 –1 c2 4 2 a2 b2
a1 b1 6 2
As , i.e.
a2 b2 k 1
The system has a unique solution i.e. the k 3
system is consistent.
a1 b1 c1
For infinitely many solutions, = =
a2 b2 c2
Section-B
11. – 4x + y = 1 ...(i) 6 2 3
= =
k 1 2
6x – 5y = 9 ...(ii) 2 3
Clearly . So, there does not exist any
On multiplying eqn (i) by 5 and adding both 1 2
the equations, we get values of k for which the system of equations

5(–4x + y) + 6x – 5y = 5 + 9 has infinitely many solutions.

–20x + 5y + 6x – 5y = 14 15. 99x + 101 y = 499 ...(i)


–14x = 14 101x + 99y = 501 ...(ii)
x = –1 On subtracting (i) from (ii), we get
From (i), y = 1 + 4x = 1 – 4 2x – 2y = 2
y = –3
x – y = 1 ...(iii)
12. The given pair of linear equations intersect if
On adding (i) and (ii), we get
a1 b1 Here, a1 = 2, b1 = –3 200 x + 200y = 1000

a2 b2 a2 = 4, b2 = – 5
x + y = 5 ...(iv)
2 –3
i.e. On adding (iii) and (iv), we get
4 –5
13. 2x = 5y + 4 ...(i) 2x = 6
3x – 2y + 16 = 0 ...(ii) x = 3
5y + 4 From (iv), y = 5–x
From (i), we get x =
2
= 5 – 3
5y + 4
From (ii), 3 – 2y + 16 = 0 = 2
Ł 2 ł
16. The system of equations has infinite solutions
15y + 12 – 4y + 32 = 0
if
11y + 44 = 0 a b c
44 1 = 1 = 1
a2 b2 c2
y = – 11 = – 4
1 k +1 5
5y + 4 5 ( -4 ) + 4 i.e. = =
So, x= = k +1 9 8k −1
2 2
- 20 + 4 1 k +1 k +1 5
= =–8 = ; =
2 k +1 9 9 8k −1
ADDITIONAL
R

PR ACTICE MATHEMATICS - 10 45
(k + 1)2 = 9 8k2 + 8k – k – 1 = 45 3 4 12
= =
k + 1 = +3 8k + 7k – 46 = 0
2 a+b 2 (a − b ) 5a - 1

−7 ± 49 +1472 3 4 4 12
= =
k = –4, 2 k =
16 a+b 2 (a − b ) 2 (a − b ) 5a - 1
−7 ± 1521 3 2 4 12
= =
=
16 a+b a-b 2 (a − b ) 5a - 1
- 7 – 39 3a – 3b = 2a + 2b 5a – 1 = 6 (a – b)
=
16
a = 5b   ...(i) 5a – 1 = 6a – 6b
−46 32
= ,
16 16 –a + 6b = 1 ...(ii)
- 23 From (i) and (ii),
= ,2
8
So, we get k = 2. a = 5b and –a + 6b = 1

17. Let the numerator be x and denominator be y. So, we get –5b + 6b = 1, b = 1


x \ a = 5b = 5(1) = 5
So, fraction =
y 19. 2x – 3y + 6 = 0 ...(i)
According to question,
4x – 5y + 2 = 0 ...(ii)
x +1 7
= a1 2 1
y +1 8 Here, = =
8x + 8 = 7y + 7 a 2
4 2
b1 - 3 3
8x – 7y = –1 ...(i) = =
b2 -5 5
x −1 6 c 6
Again, = 1 = = 3
y −1 7 c2 2
7x – 7 = 6y – 6
a1 b1
7x – 6y = 1 ...(ii) As „ , so, the system has a unique
a 2 b2
On multiplying (i) by 7 and (ii) by 8, we get, solution.
56x – 49y = –7 On multiplying (i) by 2 and subtracting (ii)
from (i), we get
56x – 48y = 8 4x – 6y + 12 = 0
– + –
     4x – 5y + 2 = 0
– y = –15 – + –
y = 15 –y = –10

From (i), 8x – 7(15) = –1 y = 10
From (i), 2x – 3(10) + 6 = 0
8x = –1 +105 = 104
2x – 24 = 0
104
x = = 13 x = 12
8
18. The system of equations has infinitely many x y
20. + + 1 = 15
solutions if 10 5
x y
a1 b1 c1 + = 14
= = 10 5
a2 b2 c2 2x + y = 140 ...(i)
46 ADDITIONAL
R

PR ACTICE MATHEMATICS - 10
x y x 1 , y = 1
Again, + = 15 =
8 6 - 14 - 5 9 -5
3x + 4y -9
= 15 14
24 x= , y =
5 5
3x + 4y = 360 ...(ii)
From (i), y = 140 – 2x 23. (i) 5x + 6y = 15
On putting this value of y in (ii), we get
4 −5  a1 b1 
As ≠ ≠
3x + 4 (140 – 2x) = 360 5 6  a 2 b 2 
3x + 560 – 8x = 360
(ii) 8x – 10y = 30
–5x = –200
x = 40 4 - 5 10 a1 b1 c1
As = „ = „
So, y = 140 – 2x 8 - 10 30 Ł a 2 b 2 c 2 ł

= 140 – 2(40) (iii) 8x – 10y = 20


= 140 – 80
4 - 5 10 a1 b1 c1
= 60 As = = = =
8 - 10 20 Ł a 2 b 2 c 2 ł

Section-C
24. 2x – (a – 4)y – (2b + 1) = 0
21. Let the fixed charge be ` x and cost of food 4x – (a – 1)y – (5b – 1) = 0
per day be ` y.
The system of equations has infinite solutions
According to question, if
x + 20y = 3000 ...(i) a1 b1 c1
= =
x + 25y = 3500 ...(ii) a2 b2 c2
– – –
2 a-4 2b + 1
– 5y = –500 i.e. = =
y = 100 4 a -1 5b - 1
1 a-4 1 2b + 1
From (i), we get x = 3000 – 20 (100) = , =
2 a -1 2 5b - 1
= 3000 – 2000
a – 1 = 2a – 8, 5b – 1 = 4b + 2
= 1000
a = 7, b=3
So, fixed charge = `1000
So, a = 7, b = 3
Cost of food per day= `100
22. x + y = 1 1 1
25. Let =p and =q
x -1 y-2
2x – 3y = 11
So, equations become
According to cross multiplication method,
5p + q = 2 ...(i)
x y 1
= = 6p – 3q = 1 ...(ii)
- 11 - 3 - 2 + 11 - 3 - 2
x y 1 On multiplying (i) by 3 and adding equation (i)
= =
- 14 9 -5 and (ii), we get
ADDITIONAL
R

PR ACTICE MATHEMATICS - 10 47
15p + 3q = 6 y = 15
6p – 3q = 1 So, x = 70 – 2y

21p = 7 = 70 – 2(15)
1 = 70 – 30
p =
3 = 40
\
x–1=3 x=4
So, age of father = 40 years
From eqn (i), q = 2 – 5p
age of son = 15 years
1
= 2 – 5 28. Let speed of train be x km/hr and speed of car
Ł3 ł be y km/hr.
1
= According to question,
3
\
y–2=3 y=5 160 600
+ = 8
x y
1 1
26. Let = p and = q.
x y 240 520 41
+ =
So, equations become x y 5
p – 4q = 2 (as 8 hours +12 min = 8 + (12/60) = 41/5)
p + 3q = 9 1 1
Let = p,   =q
– – – x y
So, we get equations as
– 7q = –7
   q = 1 160 p + 600q = 8 ...(i)

From eqn, p – 4q = 2, we get 1200p + 2600q = 41 ...(ii)

p = 2 + 4(1) On multiplying (i) by 30 and (ii) by 4, we get


4800p + 18000q = 240
= 6
1 4800p + 10400q = 164
So, x = , y = 1 –   – –
6    7600q = 76
27. Let father's age be x years and son's age be y 76 1
years. q = =
7600 100
According to question, i.e. y = 100
From (i), we get
2y + x = 70 ...(i)
1
2x + y = 95 ...(ii) 160p + 600 = 8
Ł100 ł
From (i), x = 70 – 2y
160p + 6 = 8
On putting value of x in (ii), we get
160p = 2
2 (70 – 2y) + y = 95
1
140 – 4y + y = 95 p =
80
3y = 45 i.e. x = 80
48 ADDITIONAL
R

PR ACTICE MATHEMATICS - 10
So, Speed of train = 80 km/hr 30. Let father's age = x years
Speed of car = 100 km/hr Sum of ages of 2 children = y years
According to question,
29. Let time taken by one man alone be x days.
x = 2y ...(i)
Let time taken by one boy alone be y days.
According to question, and
8 12 1
+ = x + 20 = y + 20 + 20
x y 10
x – y = 20 ...(ii)
6 8 1
+ = On putting (i) in (ii), we get
x y 14
1 1 2y – y = 20
Let = p, and =q
x y y = 20
So, we get equations as \
x = 2y = 40
1 So, father's age = 40 years
8p + 12q =
10
80p + 120q = 1 ...(i)
Section-D
Another equation becomes,
31. Let, Speed of car A = x km/hr
1
6p + 8q = Speed of car B = y km/hr
14
84p + 112q = 1 ...(ii) According to question,
On multiplying (i) by 21 and (ii) by 20, we get
A B
1680p + 2520q = 21
4 4
1680p + 2240q = 20 x + y = 80
3 3
–   – –
   280q = 1 [1 hour 20 minutes = 1 + (20/60) = 4/3]

1 x + y = 60 ...(i)
q =
280
So, y = 280 A B

From (i), 8x – 8y = 80
 1  x – y = 10 ...(ii)
80p + 120   = 1
 280  On adding (i) and (ii), we get
3
80p + = 1 2x = 70
7
3 4 x = 35
80p = 1 – =
7 7 From (i), y = 60 – x
1
p =
140 = 60 – 35
So, x = 140 = 25
\ A man can complete the work in 140 days
So, Speed of car A = 35 km/hr
and a boy can complete the work in 280
days. Speed of car B = 25 km/hr
ADDITIONAL
R

PR ACTICE MATHEMATICS - 10 49
32. Let cost of one chair be ` x and cost of one x – 12y = – 110
table be ` y.
x – 2y = 10
According to question, – +   –

4x + 3y = 2100 ...(i) – 10y = –120


y = 12
5x + 2y = 1750 ...(ii)
From (ii), x = 10 + 2y
On multiplying eqn (i) by 5 and (ii) by 4, we
get = 10 + 24
20x + 15y = 10500 = 34
20x + 8y = 7000 So, Father's age = 34 years
– –   –
Son's age = 12 years
    7y = 3500
3500 34. Perimeter of ABCDE = 21 cm
y = = 500
7 i.e. AB + BC + CD + DE + AE = 21
From (i), 4x + 3 (500) = 2100
3 + x – y + x + y + x – y + 3 = 21
4x = 2100 – 1500
3x – y = 15 ...(i)
4x = 600
As BE || CD and BC || DE,
x = 150
BCDE is a parallelogram
Cost of one chair = ` 150
\
BE = CD (opposite sides of
Cost of one table = ` 500 parallelogram)
Therefore, i.e. x + y = 5 ...(ii)
Cost of five chairs = 5 × 150 On adding equations (i) and (ii), we get
= ` 750 4x = 20

Cost of eight tables = 8 × 500 x = 5

= ` 4000 from (i), 3(5) – y = 15


y = 0
33. Let father's age = x years
So, BC = x – y = 5 – 0 = 5 cm
Son's age = y years
CD = x + y = 5 + 0 = 5 cm
According to question,
DE = x – y = 5 – 0 = 5 cm
x – 10 = 12 (y – 10)
BE = 5 cm
i.e. x – 12y = –110 ...(i) So, perimeter of quadrilateral BCDE
For another eqn, = 4 × 5 (perimeter = 4 × side)
x + 10 = 2 (y + 10) = 20 cm
1 1
x – 2y = 10 ...(ii) 35. Let = p and = q.
x y
On subtracting eqn (ii) from (i), we get So, equations become
50 ADDITIONAL
R

PR ACTICE MATHEMATICS - 10
ap – bq = 0 q 1
2 =
ab p + a bq = a + b
2 2 2 2
a a +b 2
( 2
)
ab a + b 2 ( )
p q 1 1 1
= 3 p =    q=
2 = a b
3
a b+b -0 0 + a + ab
3 2
a b + ab3
\ x=a y=b
p q 1
2 = = 36. 2x + y = 6
b a +b 2
( a a + b2
2
) ( ) (
ab a + b 2
2
) x
y
3
0
0
6
p 1
2 = , 2x – y = 2
b a +b 2
( ab a + b 2
2
) ( ) x
y
0
–2
1
0

Y
6

2 (2, 2)

X1 X
–6 –5 –4 –3 –2 –1 0 1 2 3 4 5 6
–1
2x
+y

–2
=6

–3
=2

–4
–y


2x

–5

–6
Y1
As the equations intersect at point (2, 2), so, (2, 2) is a solution of given set of equations.
1
Area of triangle formed by lines representing these equations with the x – axis = × 2 × 2
2
= 2 sq units.
1
Area of triangle formed by lines representing these equations with the y – axis = × 8 × 2
2
= 8 sq units.
2 1
So, Ratio = = .
8 4
ADDITIONAL
PR ACTICE
R

MATHEMATICS - 10 51
37. 2x + y = 2 x 0 1 2x + y = 6 x 0 3
y 2 0 y 6 0

Y
6 B (0, 6)

4
3

A (0, 2) 2

1
C (3, 0)
X1 X
–6 –5 –4 –3 –2 –1 0 1 2 3 4 5 6
–1 D (1, 0)

2x
+y
–2

=6
2x
–3 +y
=2
–4
Y1

Vertices of trapezium are A(0, 2), B(0, 6), C(3, 0) and D(1, 0).
Area of trapezium ABCD = area of DBOC – area of DAOD
1 1
= ×3×6– ×1×2
2 2
= 9–1
= 8 sq. units

38. Let the numerator be x and denominator be y. From (i), y = 5 + 2x


According to question,
On putting this value of y in (ii), we get
y = 5 + 2x
8x – 3 (5 + 2x) = 5
–2x + y = 5 ...(i)
8x – 15 – 6x = 5
For the other equation,
2x = 20
x −1 3
= x = 10
y −1 8
8x – 8 = 3y – 3 So, y = 5 + 2(10)
8x – 3y = 5 ...(ii) = 25
52 ADDITIONAL
R

PR ACTICE MATHEMATICS - 10
x 10 x y 1
So, Fraction = = = 2 =
y 25 (m + n) m+n
2 2
m -n
39. mx – ny = m2 + n2 x 1 y 1
= 2 2 =
(m + n) m -n m+n
2
m+n
x + y = 2m
(m + n )
2

x y 1 m 2 - n2
= = x= y=
2 2
2mn + m2 + n2 -m - n + 2m
2
m+n m+n m+n
=m+n =m–n

40. (a – b)x + (a + b)y = a2 – 2ab – b2


(a + b)x + (a + b)y = a2 + b2
x y 1
= =
( a + b ) ( −a 2
) − ( a + b ) ( a + b ) ( −a
−b 2 2
+ 2ab + b 2
) − ( a - b )( a + b ) - ( a + b ) 2

( −a 2
+ 2ab + b ) 2
( a − b ) ( −a 2
− b2 )
x y 1
= = 2
−a − ab − a 2b − b3
3 2 3 2 2
2 2
- a + 2a b + ab - a b + 2ab + b 3
a − b − a − b 2 − 2ab
2 2

+a3 − 2a 2b − ab 2 + a 2b + a3 + ab 2 - a 2b - b3
−2ab 2 − b3
x y 1
2 =
=
−2b − 2a b − 4ab
3 2
4ab 2
−2b − 2ab
2

−2b3 − 2a 2b − 4ab 2
x =
−2b 2 − 2ab
(
−2b b 2 + a 2 + 2ab )
=
−2b (b + a )
= a+b
y 1
Also, 2 =
4ab −2b ( a + b )
- 2ab
y =
a+b

(iii) (d) x + y = 10
cASe StUDY-1
x–y = 4
(i) (b) The sum of number of students who took
+ + +
__________
part in Quiz is b. The number of boys are
represented by x and number of girls are 2x = 14
represented by y. x = 7
x + y = 10 7 + y = 10
(ii) (c) The difference between the number of y = 3
girls and number of boys is 4. \ The solution of given pair of equation is
x–y=4 (7, 3).
ADDITIONAL
R

PR ACTICE MATHEMATICS - 10 53
(iv) (a) Linear equations have unique solutions. x = 120
(v) (c) Area of DABC = ½ (base (height) x – y = 20
The base is AB which is having lengths of 6 120 – y = 20
units.
y = 100
The height is measured from x axis to point
(iv) (c) x – y = 20
C. The ordinate of point C represents the
height of DABC. x + y = 220
Height = 3 units + + +
__________
Area of DABC = ½ (6) (3) = 9 sq units. 2x = 240
x = 120
cASe StUDY-2
y = 100
(i) (b) x – 10 = y + 10
The solution are x = 0 and y = b
x – y = 20
a = 120
(ii) (b) y + 20 = x
b = 100
(iii) (c) x – y = 20
\a>b
x + y = 220
(v) (d) The line x = 120 lies parallel to y axis and
+ + +
__________ line x = 100 lies parallel to x axis.
2x = 240 Thus both lines are intersecting.

54 ADDITIONAL
R

PR ACTICE MATHEMATICS - 10
Chapter

4 Quadratic Equations
Multiple choice Questions

1
1. (b) As x = − is a solution of 3x2 + 2kx – 3 Also, q is a root
=0 2
\ q2 – pq + q = 0
2
 1  1 q (q – p + 1) = 0
\ 3  −  + 2k  −  − 3 = 0
 2  2
q = 0 or q = p – 1
3
-k -3 = 0
4 \ q = p–1
3
k = -3 p = q+1
4
3 - 12 = 0+1
=
4 = 1
−9 So, p = 1, q = 0
=
4
2. (b) Equation has no real roots if D < 0
WoRKSHeet - 1
i.e. b2 – 4ac < 0
i.e. b2 – 4 (1) (1) < 0 Section-A
i.e. b2 – 4 < 0
1. x2 – 7x + 12
i.e. (b + 2) (b – 2) < 0
x2 – 3x – 4x + 12
i.e. –2 < b < 2
x (x – 3) – 4 (x – 3)
3. (d) let α, β be the roots then αβ = 3
αβ = 3 (x – 3) ( x – 4)

(1) β = 3 (... α = 1) 2. 2x2 + 3x – 4 = 0


β = 3 b2 – 4ac = 9 – 4(2) (–4)
4. (a) 3 3x 2 +10x + 3 =0 = 9 + 32
D = b2 – 4ac = 41 > 0
= (10)2 – 4 (3 3 × 3 ) As b2 – 4ac > 0
= 100 – 36 The equation has real and distinct roots.
= 64
3. 3x2 + 13x + 14 = 0
5. (a) x – px + q = 0
2
LHS = 3x2 + 13x + 14
As p is the root
\ p2 – p2 + q = 0 = 3(–2)2 + 13 (–2) + 14 (Put x = –2)
q = 0 = 12 – 26 + 14
ADDITIONAL
R

PR ACTICE MATHEMATICS - 10 55
= 0 k2 = 456
= RHS k = ±2 114
So, x = –2 is a root of 3x2 + 13x + 4 = 0
8. bx2 – 2 ac x + b = 0
4. x2 – 3x – 1 = 0
The equation has equal roots if discriminant
LHS = x2 – 3x – 1 =0
= 12 – 3(1) – 1 (Put x = 1)
( )
2
i.e. 2 ac – 4(b)(b) = 0
= 1–3–1
4ac – 4b2 = 0
= –3 RHS (= 0)
b2 = ac
So, x = 1 is not a solution of equation
x2 – 3x – 1 = 0 Section-B
5. x2 – 3x – 10 = 0 9. 16x2 – 24x – 1 = 0
  D = b2 – 4ac
−b ± b 2 − 4ac
= (–3)2 – 4(1) (–10) x =
2a
= 9 + 40
− (−24 ) ± (−24 ) − 4 (16)(−1)
2

= 49 =
2 (16 )

6. Let 6 + 6 + 6 + ... = x 24 ± 576 + 64


=
32
6+ x = x
24 ± 640
On squaring both sides, we get =
32
6 + x = x2 24 ± 8 10
=
x –x–6 = 0
2
32
x2 – 3x + 2x – 6 = 0 3 ± 10
=
4
x (x – 3) + 2 (x – 3) = 0
1 2 6
(x – 3) (x + 2) = 0 10. + =
x − 2 x −1 x
x = 3, x = – 2 x - 1 + 2x - 4 6
=
As value of 6 + 6 + 6 + ... cannot be
( x - 1)( x - 2 ) x
negative, so, x = 3 x (3x – 5) = 6 (x – 1) (x – 2)
7. 3x2 – kx + 38 = 0 3x2 – 5x = 6(x2 – 3x + 2)
The quadratic equation has equal roots 3x2 – 5x = 6x2 – 18x + 12
if D = 0 0 = 3x2 – 13x + 12
i.e. b2 – 4ac = 0 0 = 3x2 – 9x – 4x + 12
i.e. k2 – 4(3) (38) = 0 0 = 3x (x – 3) – 4(x – 3)
k2 – 456 = 0 0 = (3x – 4) (x – 3)
56 ADDITIONAL
R

PR ACTICE MATHEMATICS - 10
3x – 4 = 0, or   x – 3 = 0 = 81a4 + 64 b4 + 144a2b2
4 = (9a2 + 8b2)2
x= ,   or  x = 3
3
−b ± D
11. x2 – 2ax + a2 – b2 = 0 x =
2a

x2 + [(–a –b) + (– a + b)] x + (a + b) (a – b) = 0
=
( ) (
9a 2 − 8b 2 ± 9a 2 + 8b 2 )
x2 – (a + b) x – (a – b) x + (a + b) (a – b) = 0 24ab
2
18a - 16 b 2
x [x – (a + b)] – (a – b) [x – (a + b)] = 0 x = or 
x=
24ab 24ab
[x – (a – b)] [x – (a + b)] = 0 3a - 2b
x = or  x=
x – (a – b) = 0 or x – (a + b) = 0 4b 3a
x = a – b or x = a + b 15. Let the two numbers be x and 16 – x.
According to question,
12. 4x2 – 4a2x + (a4 – b4) = 0
1 1 1
4x2 + [–2 (a2 – b2) – 2 (a2 + b2)] x + (a4 – b4) + =
x 16 − x 3
=0
16 − x + x 1
4x2 – 2 (a2 – b2) x – 2(a2 +b2) x + (a2 – b2) =
x (16 − x ) 3
(a2 + b2) = 0
48 = 16x – x2
2x [2x – a2 – b2] – (a2 + b2) (2x – a2 – b2) = 0
x2 – 16x + 48 = 0
[2x – (a – b )] [2x – (a + b )] = 0
2 2 2 2

x2 – 12x – 4x + 48 = 0
2x – (a – b ) = 0 or 2x – (a + b ) = 0
2 2 2 2

x (x – 12) – 4 (x – 12) = 0
2 2 2 2
-b + a a +b
x =    or x = (x – 4) (x – 12) = 0
2 2
x = 4, 12
13. (k – 12) x2 + 2 (k – 12) x + 2 = 0
The equation has equal roots if discriminant If x = 4,   Other number = 16 – 4 = 12
D=0 if x = 12, Other number = 16 – 12 = 4
i.e. b – 4ac = 0
2
1 1
16. x+ = 11
4 (k – 12) – 4 (k – 12) (2) = 0
2
x 11
(k – 12) [4(k – 12) – (4) (2)] = 0 x2 +1 122
=
x 11
(k – 12) (4k – 48 – 8) = 0 11(x + 1) = 122 x
2

(k – 12) (4k – 56) = 0


11x2 – 122x + 11 = 0
(k – 12) 4(k – 14) = 0
11x2 – x – 121x + 11 = 0
k = 12, 14
x (11x – 1) – 11 (11x – 1) = 0
14. 12abx2 – (9a2 – 8b2)x – 6ab = 0
(11x – 1) (x – 11) = 0
D = (9a2 – 8b2)2 – 4 (12ab) (–6ab)
11x – 1 = 0 or x – 11 = 0
= (9a2 – 8b2)2 + 288a2b2 1
x= or x = 11
= 81a4 + 64 b4 – 144a2b2 + 288a2b2 11
ADDITIONAL
R

PR ACTICE MATHEMATICS - 10 57
Section-C If smaller side is tripled and the larger side is
doubled,
17. Let D1 and D2 be the discriminants of
equations x2 + 2cx + ab = 0 and x2 – 2(a + b) (3x)2 + (2y)2 = (15)2
x + a + b + 2c = 0 respectively.
2 2 2
9x2 + 4y2 = 225 ...(ii)
x + 2cx + ab = 0
2
From (i), x2 = 45 – y2
  D1 = (2c) – 4 (1) (ab)
2
So, we get 9(45 – y2) + 4y2 = 225
= 4c – 4ab
2

405 – 9y2 + 4y2 = 225


= 4 (c – ab)
2

5y2 = 180
As roots are real and unequal,
180
so D1 > 0 y2 = = 36
5
c2 – ab > 0 ...(i)
y = +6
x2 – 2(a + b) x + a2 + b2 + 2c2 = 0
For y = – 6, x2 = 45 – 36 = 9
  D2 = 4 (a + b) – 4(1) (a + b + 2c )
2 2 2 2
x=+3
= 8ab – 8c2
For y = 6, x2 = 45 – 36 = 9
= – 8 (c2 – ab) < 0 [From (i)]
x=+3
So, the given equation has no real roots.
As length cannot be negative,
1 1 11
18. - = So, y = – 6, x = – 3 rejected
x+4 x-7 30
x −7−x −4 11 \ x = 3, y = 6
=
( x + 4 )( x − 7 ) 30 Length of smaller side = 3 cm
- 11 11 Length of larger side = 6 cm
2 =
x - 3x - 28 30
20. As x = – 2 is a root of equation
−1 1
2 = 3x2 + 7x + p = 0, we have
x − 3x − 28 30
x2 – 3x – 28 + 30 = 0 3(–2)2 + 7 (–2) + p = 0
x2 – 3x + 2 = 0 12 – 14 + p = 0
x2 – 2x – x + 2 = 0 p = 2
x (x – 2) – 1 (x – 2) = 0 x2 + k (4x + k – 1) + p = 0
(x –1) (x – 2) = 0 x2 + k (4x + k – 1) + 2 = 0 (Put p = 2)
x = 1, 2 x2 + (4k)x + k2 – k + 2 = 0
19. Let the smaller side and larger side be x cm As roots are equal,
and y cm respectively.
Discriminant (D) = 0
Hypotenuse = 3 5 cm
(4k)2 – 4(k2 – k + 2) = 0
(3 5 )
2
So, x2 + y2 = 16k2 – 4k2 + 4k – 8 = 0
x2 + y2 = 45 ...(i) 12k2 + 4k – 8 = 0
58 ADDITIONAL
R

PR ACTICE MATHEMATICS - 10
3k2 + k – 2 = 0 23. (x – a) (x – b) + (x – b) (x – c) + (x – c)
(x – a) = 0
3k2 + 3k – 2k – 2 = 0
x2 – bx – ax + ab + x2 – cx – bx + bc +
3k (k + 1) – 2 (k + 1) = 0
x2 – ax – cx + ac = 0
(3k – 2) (k + 1) = 0
3x2 – 2bx – 2ax – 2cx + ab + bc + ca = 0
3k – 2 = 0 or k + 1 = 0
3x2 – 2 (a + b + c) x + (ab + bc + ca) = 0
2
    k = or k = –1 Discriminant (D)
3
= 4 (a + b + c)2 – 12 (ab + bc + ca)
21. x2 (a2 + b2) + 2 (ac + bd) x + (c2 + d2) = 0
= 4 (a2 + b2 + c2 + 2ab + 2bc + 2ac – 3ab
Consider, Discriminant (D) – 3bc – 3ca)
= 4 (ac + bd)2 – 4 (a2 + b2) (c2 + d2) = 4 (a2 + b2 + c2 – ab – bc – ca)
= 4 (a2c2 + b2d2 + 2abcd) – 4 (a2c2 + a2d2 = 2 (2a2 + 2b2 + 2c2 – 2ab – 2bc – 2ac)
+ b2c2 + b2d2)
= 2 [(a – b)2 + (b – c)2 + (a – c)2]
= 8 abcd – 4a d – 4b c
2 2 2 2

D = 2 [(a – b)2 + (b – c)2 + (a – c)2] > 0


= –4 [(ad)2 +(bc)2 – 2 abcd)
As D > 0, so roots are real.
= –4 (ad – bc)2
Roots are equal if D = 0
<0
i.e. 2[(a – b)2 + (b – c)2 + (a – c)2] = 0
For no real roots, D < 0
i.e. a – b = 0, b – c = 0, a – c = 0,
i.e. D 0 i.e. ad bc a = b, b = c, a = c

22. As 2 is a root of the quadratic equation i.e. a = b = c.


3x2 + px – 8 = 0, 24. Let the two numbers be x and y such that
3(2)2 + p(2) – 8 = 0 x > y.

12 + 2p – 8 = 0 x – y = 3 ...(i)

2p = –4 1 1 3
Also, - = ...(ii)
y x 28
p = –2
From (i), x = 3+y
\Other equation becomes

Putting in (ii), we get
4x – 2(–2) x + k = 0
2
1 1 3
4x + 4x + k = 0
2 - =
y 3+ y 28
As roots are equal, 3+ y - y 3
=
Discriminant (D) = 0 y (3 + y ) 28
3 3
=
i.e. 16 – 4(4) (k) = 0 y (3 + y ) 28
16 – 16k = 0 28 = y2 + 3y
k = 1 y2 + 3y – 28 = 0
ADDITIONAL
R

PR ACTICE MATHEMATICS - 10 59
y2 + 7y – 4y – 28 = 0 According to equation,
y (y + 7) – 4 (y + 7) = 0 24 24
= +1 (... up = D + 1)
(y – 4) (y + 7) = 0 18 − x 18 + x
24 24
y = 4, –7 - = 1
18 - x 18 + x
As y is a natural number,
18 + x −18 + x 1
=
y = – 7 is rejected (18 − x )(18 + x ) 24
So, y = 4 2x 1
2 =
\
x=3+y = 7 324 - x 24
324 – x2 = 48x
Section-D x2 + 48x – 324 = 0

x-2 x-4 10 x2 + 54x – 6x – 324 = 0


25. + =
x-3 x-5 3 x (x + 54) – 6 (x + 54) = 0

( x − 2 )( x − 5) + ( x − 3)( x − 4 ) = 10 (x – 6) (x + 54) = 0
( x − 3)( x − 5) 3 x = 6, – 54
x 2 - 7x + 10 + x 2 - 7x + 12 10 As speed cannot be negative,
2 =
x - 8x + 15 3
x = – 54 is rejected.
2x 2 −14x + 22 10
= So, x = 6
x 2 − 8x +15 3

5 \
Speed of stream = 6 km/hr
x 2 - 7x + 11
=
2
x - 8x + 15 3  3x −1   2x + 3 
27. 3  −2 = 5
3x2 – 21x + 33 = 5x2 – 40x + 75  2x + 3   3x −1 
3x - 1
0 = 2x2 – 19x + 42 Let =y
2x + 3
0 = 2x2 – 12x – 7x + 42 So, equation becomes
0 = 2x (x – 6) – 7 (x – 6) 2
3y – = 5
y
0 = (2x – 7) (x – 6)
3y2 – 2 = 5y
(2x – 7) (x – 6) = 0
3y2 – 5y – 2 = 0
2x – 7 = 0 or x–6=0 3y2 – 6y + y – 2 = 0
7
x = or x=6 3y (y – 2) + 1 (y – 2) = 0
2
(3y + 1) (y – 2) = 0
26. Let speed of stream be x km/hr
  3y + 1 = 0  or y – 2 = 0
Speed of boat in still water = 18 km/hr
1
y=– or y = 2
So, Speed of boat downstream = (18 + x) 3
km/hr 1
y=– y=2
Speed of boat upstream = (18 – x) km/hr 3
60 ADDITIONAL
R

PR ACTICE MATHEMATICS - 10
3x - 1 1 3x - 1 29. Let the usual speed of plane be x km / hr
=– =2
2x + 3 3 2x + 3 1500
\
Time taken = hours
9x – 3 = – 2x – 3 3x – 1 = 4x + 6 x
New speed = x + 250 km / hr
11x = 0 x = –7
1500
x=0 \
Time taken = hours
x + 250
According to question,
28. Let original speed of the aircraft be x km/hr.
1500 1500 1
\
New speed = (x – 200) km/hr. = –
x + 250 x 2
Duration of flight at original speed 1500 1500 1
– =
x x + 250 2
600 1 1
= hours 1
x – =
x x + 250 3000
Duration of flight at reduced speed
x + 250 − x 1
=
600 x ( x + 250 ) 3000
= hours
x - 200 x2 + 250 x = 750000
According to question,
x2 + 250 x – 750000 = 0
600 1 600
= + x2 + 1000x – 750x – 750000 = 0
x - 200 2 x
x (x +1000) – 750 (x + 1000) = 0
600 600 1
– =
x - 200 x 2 (x – 750) (x + 1000) = 0
1 1 1 x = 750 or x = –1000
− =
x − 200 x 1200 Now, x being the speed of plane cannot be
x - x + 200 1 negative,
=
x(x - 200) 1200 x = –1000 is rejected

x2 – 200x = 240000 So, x = 750

x2 – 200x – 240000 = 0 \   Speed of plane = 750 km/hr


x2 – 600x + 400x – 240000 = 0 30. Let total number of camels be x.

x (x – 600) + 400 (x – 600) = 0 According to question,


1
(x + 400) (x – 600) = 0 x + 2 x + 15 = x
4
x
x = – 400 or x = 600 2 x + 15 = x –
4
As x, being speed of aircraft can't be negative. 3x
2 x + 15 =
4
So, x = 600
8 x + 60 = 3x
\ Original speed of aircraft = 600 km/hr
3x – 8 x – 60 = 0
600
Duration of flight = = 1 hour
( )
2
600 3 x – 8 x – 60 = 0

ADDITIONAL
R

PR ACTICE MATHEMATICS - 10 61
Let x = y 29 ± 841 − 8 ( 99 )
y =
3y2 – 8y – 60 = 0 4
29 ± 49
3y2 – 18y + 10y – 60 = 0 y =
4
3y (y – 6) + 10 (y – 6) = 0 29 ± 7
y =
(3y + 10) (y – 6) = 0 4
29 + 7 29 − 7
10 y= , y=
y=– or y = 6 4 4
3 11
y = 9, y =
10 2
Now, y = - is rejected as number of 11
3 Now, y = is rejected
2
camels can not be negative. So, y = 9
So, y = 6 ∴ Nihal's age = 9 years
i.e. x = 6 9 + 5y
Varun's age =
∴ x = 36 2
9 + 45
So, total number of camels = 36 =
2
= 27 years
31. Let Varun's age be x years and Nihal's age be
y years. 1 1 1 1
32. - = +
a+b+ x x a b
According to question,
x − (a + b + x ) a+b
x – 7 = 5 (y – 7)2 =
x (a + b + x ) ab
x – 7 = 5 (y – 7)2 ...(i)
− (a + b ) a+b
=
For second equation, x (a + b + x ) ab
2
y+3 = (x + 3) −1 1
5 =
x (a + b + x ) ab
5y + 15 = 2x + 6
x (a + b + x) + ab = 0
2x – 5y = 9 ...(ii) xa + xb + x2 + ab = 0
9 + 5y x2 + xa + xb + ab = 0
From (ii), x =
2
x (x + a) +b (x + a) = 0
Putting in (i), we get
(x + a) (x + b) = 0
9 + 5y
– 7 = 5 (y – 7)2 x = –a or x = –b
2
9 + 5y – 14 = 10 (y2 + 49 – 14y)
WORKSHEET - 2
5y – 5 = 10 (y + 49 – 14y)
2

y – 1 = 2 (y2 + 49 – 14y)
SECTION-A
1. LHS = x2 – 3 3 x + 6
y – 1 = 2y2 + 98 – 28y
2y2 – 29y + 99 = 0 = (–2 3 )2 – 3 3 (–2 3 ) + 6

62 ADDITIONAL
R

PR ACTICE MATHEMATICS - 10
= 12 + 18 + 6 = 8 + 24
= 36 = 32
RHS ( = 0) 7. 2x2 + 5 3 x + 6 = 0
So, x= –2 3 is not a solution of the given Discriminant (D) = (5 3 )2 – 4(2) (6)
equation.
= 75 – 48
1
2. As x = - is a solution of 3x2 + 2kx – 3 = 0, = 27 > 0
2
2
1  1 So, the given equation has real roots.
3 - + 2k  −  – 3 = 0
Ł 2ł  2 
8. abx2 + (b2 – ac)x – bc = 0
3
-k -3 = 0 abx2 + b2x – acx – bc = 0
4
3 −9 bx (ax + b) – c (ax + b) = 0
k= − 3 =
4 4 (bx – c) (ax + b) = 0
3. Let the two consecutive positive integers be bx – c = 0 or ax + b = 0
x and x + 1. c b
  x = or x = –
According to question, b a
x (x + 1) = 240 9. 2x2 – kx + 1 = 0

x2 + x – 240 = 0 As the equation has real and equal roots,

4. x2 + 6x + 5 = 0 Discriminant (D) = 0

x2 + 5x + x + 5 = 0 k2 – 4(2) (1) = 0

x (x + 5) + 1 (x + 5) = 0 k2 = 8

(x + 1) (x + 5) = 0 k = +2 2

x + 1 = 0 or x + 5 = 0  1
10. x2 +  a +  x + 1 = 0
 a
x = – 1   or x = –5
ax2 + (a2 + 1) x + a = 0
2
5. x+ = 3 ax2 + a2x + x + a = 0
x
x + 2 = 3x
2
ax (x + a) + 1 (x + a) = 0
x – 3x + 2 = 0
2
(ax + 1) (x + a) = 0
x2 – 2x – x + 2 = 0 ax + 1 = 0 or x + a = 0
x (x – 2) –1 (x – 2) = 0 1
  x=– or x = – a
a
(x – 1) (x – 2) = 0
x = 1, 2 Section-B
6. 3 x2 – 2 2 x – 2 3 = 0 2
11. As x = is a root of equation
3
Discriminant = (–2 2 )2 – 4( 3 ) (–2 3 ) ax2 + 7x + b = 0
ADDITIONAL
R

PR ACTICE MATHEMATICS - 10 63
2 k = 0
2 2
a   + 7   + b = 0
3 3 if p = 4k, 20 (4k) + k = 0
4 14 k = 0
a+ +b = 0
9 3
4a + 42 + 9b
= 0 13. 2x + 9 + x = 13
9
4a + 9b = – 42 ...(i) 2x + 9 = 13 – x
As x = –3 is a root of equation Squaring both sides,
ax2 + 7x + b = 0 2x + 9 = 169 + x2 – 26x
9a – 21 + b = 0 x2 – 28x + 160 = 0
9a + b = 21 ...(ii) x2 – 20x – 8x + 160 = 0
From (ii), b = 21 – 9a x (x – 20) – 8 (x – 20) = 0
Putting in (i), we get (x – 8) (x – 20) = 0
4a + 9 (21 – 9a) = – 42 x=8 or x = 20
4a + 189 – 81a = – 42 If x = 20
189 + 42 = 81a – 4a LHS = 40 + 9 + 20 = 27 RHS (= 13)
231 = 77a So, x = 20 is rejected
a = 3 If x = 8,
So, b = 21 – 9 (3)
LHS = 16 + 9 + 8
= 21 – 27 = 5 + 8
= – 6 = 13
12. As –5 is a root of equation = RHS
px2 + px + k = 0 Therefore, x = 8
p(–5)2 + p (–5) + k = 0
14. 9x2 – 6b2x – (a4 – b4) = 0
25p – 5p + k = 0
9x2 + [–3 (b2 – a2) – 3 (b2 + a2)]x + (–a4 + b4)
20p + k = 0 ...(i) =0
Also, as equation has equal roots, 9x2 – 3 (b2 – a2) x – 3 (b2 + a2) x + (a2 + b2)
(–a2 + b2) = 0
Discriminant = 0
3x [3x – (b2 – a2)] – (a2 + b2) [3x – (b2 – a2)]
p2 – 4pk = 0
=0
p (p – 4k) = 0
[3x – (a2 + b2)] [3x – (b2 – a2)] = 0
p = 0 or p = 4k
a 2 + b2 b2 - a 2
if p = 0, 20(0) + k = 0 x = or x=
3 3
64 ADDITIONAL
R

PR ACTICE MATHEMATICS - 10
4 5 2x ( 3 x – 1) – 3 ( 3 x – 1) = 0
15. –3 =
x 2x + 3
4 - 3x 5 (2x – 3 ) ( 3 x – 1) = 0
=
x 2x + 3 2x – 3 =0 or 3x–1 = 0
(4 – 3x) (2x + 3) = 5x
8x + 12 – 6x2 – 9x = 5x 3 1
x= or x =
2 3
6x2 + 6x – 12 = 0 19. Let x be the side of square.
x2 + x – 2 = 0 So, area of square = x2
x2 + 2x – x – 2 = 0
Number of students = x2 + 24
x (x + 2) – 1 (x + 2) = 0
If side of a square is increased by one
(x – 1) (x + 2) = 0 student, side = x + 1
x = 1, – 2 So, number of students = (x + 1)2 – 25

16. 2 y2 + 7y + 5 2 = 0 According to question,

2 y2 + 2y + 5y + 5 2 = 0 x2 + 24 = (x + 1)2 – 25
x2 + 24 = x2 + 1 + 2x – 25
2 y (y + 2 ) + 5 (y + 2 ) = 0
48 = 2x
(y + 2 ) ( 2 y + 5) = 0
x = 24
y + =0 or 2y + 5 = 0
\
Number of students = x2 + 24
5
y = – 2 or y=– = (24)2 + 24
2
17. Roots of the equation are equal if Discriminant = 576 + 24
(D) = 0 = 600
mx (6x + 10) + 25 = 0 20. 4x2 + 3x + 5 = 0
6mx2 + 10mx + 25 = 0
3
(2x)2 + 2   x + 5 = 0
D = 0 2
(10m)2 – 4 (6m) (25) = 0 3
(2x)2 + 2   2x + 5 = 0
100 m2 – 600 m = 0 4
2 2
100 m (m – 6) = 0 3 3 3
(2x) + 2   2x +
2
4 – 4 + 5 = 0
4   Ł ł
m = 0, 6
2
 3 9
For m = 0, equation will become 25 = 0,  2x + 4  –
16
+5=0
which is not possible.
2
So, m = 6  3 9
 2x + 4  =
16
-5

18. 2 3 x2 – 5x + 3 = 0 2
 3 71
 2x + 4  = -
2 3 x2 – 2x – 3x + 3 = 0 16
ADDITIONAL
R

PR ACTICE MATHEMATICS - 10 65
As square of a number can't be negative. (x + 5) (x – 12) = 0
So, the given equation has no real roots. x = –5, 12
Since, side cannot be negative,
Section-C
So, x = – 5 is rejected
25
21. (x – 5) (x – 6) = \
x = 12
( 24 )
2

25 BC = x = 12 cm
x – 11x + 30 =
2
AB = x – 7 = 12 – 7 = 5 cm
576
25 23. (a – b) x2 + (b – c) x + (c – a) = 0
x2 – 11x + 30 – = 0
576 As roots of equation are equal,
17255 Discriminant (D) = 0
x2 – 11x + = 0
576 (b – c)2 – 4 (a – b) (c – a) = 0
576 x2 – 6336 x + 17255 = 0
(b2 + c2 – 2bc) – 4 (ac – a2 – bc + ab) = 0
576 x2 – 2856 x – 3480 x + 17255 = 0 4a2 + b2 + c2 – 4ac + 2bc – 4ab = 0
24x (24x – 119) – 145 (24x – 119) = 0 (2a)2 + b2 + c2 – 4ac + 2bc – 4ab = 0
(24x – 145) (24x – 119) = 0 (–2a + b + c)2 = 0

24x – 145 = 0 or 24x – 119 = 0 –2a + b + c = 0


2a = b + c
145 119
x= or x =
24 24 24. Let the sides of two squares be x and y
22. A Area of square with side x = x2
13 Area of square with side y = y2
x–7

cm
Perimeter of square with side x = 4x
Perimeter of square with side y = 4y
B C
x According to question,
Let the base of D ABC = x cm x2 + y2 = 468 ...(i)

\ Altitude of D ABC = (x – 7) cm
4x – 4y = 24
i.e. x – y = 6 ...(ii)
We know that,
From (ii), x = 6 + y
(Hypotenuse)2 = (Base)2 + (Perpendicular)2
On putting in (i), we get
AC = AB + BC
2 2 2
(6 + y)2 + y2 = 468
(13)2 = (x – 7)2 + x2 36 + y2 + 12y + y2 = 468
169 = x2 + 49 – 14x + x2 2y2 + 12y – 432 = 0
y2 + 6y – 216 = 0
2x2 – 14x – 120 = 0
y2 – 12y + 18y – 216 = 0
x – 7x – 60 = 0
2
y(y – 12) + 18(y – 12) = 0
x – 12x + 5x – 60 = 0
2
(y – 12) (y + 18) = 0
x(x – 12) + 5 (x – 12) = 0 y = 12, –18
66 ADDITIONAL
R

PR ACTICE MATHEMATICS - 10
As side cannot be negative, 4x + 6 = 0 or x + 1 = 0
y = –18 is rejected 3
x = –   or x = –1
\ y = 12 2
So, x = 6+y 27. Let the three consecutive natural numbers
= 6 + 12 be x – 1, x and x + 1.

= 18 According to question,
So, sides of two squares are 12m and 18m x2 = [(x + 1)2 – (x – 1)2 ] + 60
respectively.
x2 = x2 + 1 + 2x – x2 – 1 + 2x + 60
25. a2 x2 – 3abx + 2b2 = 0
x2 = 4x + 60
3
(ax)2 – 2   abx + 2b2 = 0 x2 – 4x – 60 = 0
2
3 b  x2 – 10x + 6x – 60 = 0
(ax) – 2ax   + 2b = 0
2 2
 2  x (x – 10) + 6 (x – 10) = 0
2 2
3 b  3 b  3 b 
(ax)2 – 2ax   +   + 2b2 –   = 0 (x + 6) ( x – 10) = 0
 2   2   2 
2 x = – 6 or 10
 3b 9 2
 ax − 2  + 2b − 4 b = 0
2
As x is a natural number,
2
 3b b2 x = – 6 is rejected
 ax − − = 0
2  4 So, x = 10
2
 3b b2 \   The three numbers 9, 10, 11.

 ax − 2  = 4
28. Let the time taken by smaller tap to fill tank
3b b
ax - = – completely = x hours
2 2
3b b 3b b Time taken by larger tap to fill tank completely
ax = ax - =– = x – 8 hours
2 2 2
4b b 3b According to question,
ax = = 2b ax = - + =b
2 2 2 1 1 5
2b b + =
x = x = x x–8 48
a a
2x 1 3x + 9 x-8+ x 5
26. + + = 0 =
x - 3 2x + 3 ( x - 3)( 2x + 3) x(x - 8) 48
2x − 8 5
2x ( 2x + 3) + ( x − 3) + 3x + 9 =
= 0 x(x − 8) 48
( x − 3)(2x + 3) 48 (2x – 8) = 5x (x – 8)
2x (2x + 3) + (x – 3) + 3x + 9 = 0
96x – 384 = 5x2 – 40x
4x2 + 6x + x – 3 + 3x + 9 = 0
4x2 + 10x + 6 = 0 5x2 – 136x + 384 = 0
4x2 + 4x + 6x + 6 = 0 5x2 – 16x – 120x + 384 = 0
4x (x + 1) + 6 (x + 1) = 0 x (5x – 16) – 24 (5x – 16) = 0
(4x + 6) (x + 1) = 0 (x – 24) (5x – 16) = 0
ADDITIONAL
R

PR ACTICE MATHEMATICS - 10 67
16 x2 + 42x – 10x – 420 = 0
x = 24 or
5 x (x + 42) – 10 (x + 42) = 0
For x = 24, (x – 10) (x + 42) = 0
Time taken by smaller tap = 24 hours x = 10 or – 42
Time taken by larger tap = x – 8    x = – 42 is rejected as x < 0.
= 24 – 8 \
x = 10
= 16 hours So, the required parts are 10 and 6.
16
For x = ,
5 Section-D
Time taken by larger pipe = x – 8
1 1 1 1
31. = + +
16 2a + b + 2x 2a b 2x
= –8
5 1 1 1 1
24 - = +
= – 2a + b + 2x 2x b 2a
5
Since time cannot be negative, 2x - 2a - b - 2x 2a + b
=
16 2x (2a + b + 2x) 2ab
x =is rejected.
5 −2a − b 2a + b
\ Time taken by smaller tap = 24 hours
=
2x (2a + b + 2x) 2ab
Time taken by larger tap = 16 hours −1 1
=
2x (2a + b + 2x) 2ab
29. 9x2 – 63x – 162 = 0
-1 1
Discriminant (D) = (–63)2 – 4(9) (– 162) =
x (2a + b + 2x) ab
= 3969 + 5832
x (2a + b + 2x) + ab = 0
= 9801
2x2 + 2ax + bx + ab = 0
−b ± D
x = 2x (x + a) + b (x + a)= 0
2a
63 ± 9801 (2x + b) (x + a) = 0
=
18 -b
x= , –a
63 ± 99 2
=
18 32. Let number of books = x
63 + 99 63 − 99
x = or x = 80
18 18 \ Cost of each book =

x
x = 9   or x = –2 According to question,
30. Let the larger part be x. 80 80
= -1
\ Smaller part = 16 – x
x+4 x
According to question, 80 80
- = 1
2(x)2 = (16 – x)2 + 164 x x+4
2x2 = 256 + x2 – 32x + 164 1 1 1
- =
x2 + 32x – 420 = 0 x x+4 80
68 ADDITIONAL
R

PR ACTICE MATHEMATICS - 10
x+4-x 1 2x2 + 8x – 7x – 28 = 0
=
x (x + 4) 80 2x (x + 4) – 7 (x + 4) = 0
1 1 (x + 4) (2x – 7) = 0
=
x(x + 4) 320 7
x = –4,
x + 4x – 320 = 0
2
2
Since, time cannot be negative,
x2 – 16x + 20x – 320 = 0
7
x (x – 16) + 20 (x – 16) = 0 x = = 3.5 hours
2
(x – 16) (x + 20) = 0 34. Let speed of stream = x km/hr

x = 16 or x = –20 Speed of boat in still water = 20 km/hr

Since, number of books cannot be negative, Speed of boat upstream = (20 – x) km/hr
x = 16 Speed of boat downstream = (20 + x) km/hr
So, number of books = 16 According to question,
33. Let original duration of flight = x hours 48 48
= +1
2800 20 - x 20 + x
Speed of an aircraft = km/hr
x 1 1 1
If time increased by 30 minutes - =
20 - x 20 + x 48
1 2800 20 + x - 20 + x 1
i.e. hour, speed = =
2 1 (20 - x) (20 + x)
x+ 48
2
According to question, 2x 1
=
2800 2800 (20 - x) (20 + x) 48
= – 100
1 x 96x = 400 – x2
x+
2 x2 + 96x – 400 = 0
2800 2800
– = 100 x2 + 100x – 4x – 400 = 0
x 2x + 1
2 x (x + 100) –4 (x + 100) = 0
2800 5600 (x – 4) (x + 100) = 0
– = 100
x 2x + 1
x = 4, –100
28 56
- = 1 Being the speed, x can not be negative.
x 2x + 1
So, x = –100 is rejected
1 2 1
- =
x 2x + 1 28 \
x=4
2x + 1 - 2x 1 Speed of stream = 4 km/hr
=
x (2x + 1) 28 1 1
35. + = 1
1 1 2x - 3 x - 5
=
x (2x +1) 28 x − 5 + 2x − 3
= 1
2x + x – 28 = 0
2
(2x − 3) (x − 5)

ADDITIONAL
R

PR ACTICE MATHEMATICS - 10 69
3x − 8 37. Let number of articles be x
2 = 1
2x −10x − 3x +15 \ Cost of production of each article

2x2 – 13x + 15 = 3x – 8 = 2x + 3
2x2 – 16x + 23 = 0 According to question,
Discriminant (D) = (–16)2 – 4 (2) (23) x (2x + 3) = 90
= 256 – 184 2x2 + 3x – 90 = 0
= 72 2x2 – 12x + 15x – 90 = 0

−b ± D 2x (x – 6) + 15 (x – 6) = 0
x =
2a (2x + 15) (x – 6) = 0
16 ± 72 - 15
= x = or x = 6
4 2
16 ± 6 2 Being number of articles, x cannot be
= negative.
4
\
x = 6
8±3 2
x = Number of articles = 6
2
8+3 2 8-3 2 Cost of production of each article
Roots are and .
2 2 = 2x + 3
36. Let present age of sister be x years. = 12 + 3
\
Age of girl = 2x years = ` 15
According to question,
38. Let Shefali's marks in English be x.
(x + 4) (2x + 4) = 160
\
Shefali's marks in Mathematics = 30 – x
2x2 + 12x + 16 – 160 = 0
According to question,
2x2 + 12x – 144 = 0
x2 + 6x – 72 = 0 (30 – x + 2) (x – 3) = 210

x2 + 12x – 6x – 72 = 0 (32 – x) (x – 3) = 210


x (x + 12) – 6 (x + 12) = 0 32x – 96 – x2 + 3x = 210
(x – 6) (x + 12) = 0 x2 – 35x + 306 = 0
x = 6, –12 x2 – 17x – 18x + 306 = 0
As age cannot be negative, x (x – 17) – 18 (x – 17) = 0
x = –12 is rejected
(x – 17) (x – 18) = 0
\
x = 6
x = 17 or x = 18
Age of sister = 6 years
If x = 17
Age of girl = 2x
= 2 (6) Shefali's marks in English = 17

= 12 years Shefali's marks in Mathematics = 30 – 17 = 13

70 ADDITIONAL
R

PR ACTICE MATHEMATICS - 10
If x = 18
cASe StUDY-1
Shefali's marks in English = 18
(i) (b) Area of rectangle = xy
Shefali's marks in Mathematics = 30 – 18
100 – xy
= 12
100
39. Let speed of train = x km/hr x =
y
(ii) (d) x + y + y = 30
Distance covered = 360 km
x + 2y = 30
360
So, time taken = hours 100
x (iii) (d) x = ...(i)
According to question, y

360 360 x + 2y = 30 ...(ii)


= –1
x+5 x Put equation (i) in equation (ii)
360 360 100
– = 1 + 2y = 30
x x+5 y
x + 5- x 1
= 100 + 2y2 = 30y
x (x + 5) 360
5 1 2y2 – 30y + 100 = 0
=
x(x + 5) 360 y2 – 15y + 50 = 0
x2 + 5x – 1800 = 0
100
x2 – 40x + 45x – 1800 = 0 (iv) (a) x=
y
x (x – 40) + 45 (x – 40) = 0 100
y=
(x – 40) (x + 45) = 0 x
100
x = 40, – 45 Substitute y = in equation y2 – 15y +
x
50 = 0
Being speed of train, x = – 45 is rejected. 2
100 100
\ Speed of train = 40 km/hr - 15 + 50 = 0
Ł x ł Ł x ł
40. Let breadth of rectangular mango grove = x m 10000 1500
− + 50 = 0
\ Length = 2x m x2 x
According to question, 200 30
− +1 = 0
2x (x) = 800 x2 x
200 – 30x + x2 = 0
2x2 = 800
x2 – 30x + 200 = 0
x = 400
2

x = + 20 (v) (c) x2 – 30x + 200 = 0

(30 ) − 4 (1)( 200 )


2
Being a dimension, x cannot be negative. 30 ±
x=
\ x = 20 2

(30 ) − 4 (1)( 200 )


2
So, Breadth = 20 m 30 ±
=
Length = 2x = 40 m 2

ADDITIONAL
R

PR ACTICE MATHEMATICS - 10 71
x × (cost of 1 articles) = ` 90
30 – 900 - 800
= 90
2 cost of 1 article = `
30 ±10 x
= (iii) (a) Cost of production of 1 article = 2x + 3
2
90
30 + 10 30 - 10 2x + 3 =
x= x= x
2 2
2x2 + 3x = 90
x = 20 x = 10
2x2 + 3x – 90 = 0
when x = 10
(iv) (d) 2x2 + 3x – 90 = 0
100 100
y = =
(3 ) − 4 ( 2 )( −90 )
2
x 10 −3 ±
x =
= 10 m 2 (2 )
When x = 20
−3 ± 9 + 720
100 =
y = 4
20
- 3 – 27
= 5m =
4
As x > y
- 3 + 27 −3 − 27
∴ x = 20 x, y = 5 m is the correct x = x=
4 4
dimension.
24 - 30
= x=
CASE STUDY-2 4 4
=6 = –7.5
(i) (c) Twice the number of articles produced is As the number of articles cannot be
represented by 2x. negative 4 m fractions hence discarding
ATQ the value x = –7.5
Three more than twice the number of 90
(v) (a) Cost of per article =
articles produced is 2x + 3. x
(ii) (b) The articles produced are represented by 90
x. = = `15
6

72 ADDITIONAL
R

PR ACTICE MATHEMATICS - 10
Chapter

5 Arithmetic Progressions
Multiple choice Questions

1. (b) an = 3n + 7 2a – b = 18 ...(i)

an + 1 = 3 (n + 1) + 7 = 3n + 10 b – a = –3 –b

So, d = an + 1 – an a – 2b = 3 ...(ii)

= 3n + 10 – 3n – 7 Solving (i) and (ii), we get

= 3 a = 11, b = 4

2. (c) a = 1, an = 11 So, a + b = 11 + 4

Sn = 36 = 15

n
We know that Sn =(a + an) WoRKSHeet - 1
2
n
36 = (1 + 11) Section-A
2
36 × 2 1. k + 9, 2k – 1 and 2k + 7 are in A.P. if
n =
12
(2k –1) – (k + 9) = (2k + 7) – (2k – 1)
= 6
k – 10 = 8
3. (b) Sn = 2n2 + 5n
k = 18
an = Sn – (Sn – 1)
2. Sn = 3n2 + 5n
= (2n2 + 5n) – [2 (n – 1)2 + 5(n – 1)]
S20 = 3 (20)2 + 5(20)
= 2n2 + 5n – 2n2 – 2 + 4n – 5n + 5
= 3 (400) + 100
= 4n + 3
= 1200 + 100
4. (d) We can write reverse AP as
= 1300
185, ..., 13, 9, 5
3. Consider A.P. : 2, 4, 6, 8, ..., n
Such that a = 185, d = –4
Here a = 2, d = 2
So, a9 = 185 + (9 – 1) (– 4)
= 185 – 32 n
Sn = [2a + (n – 1) d]
2
= 153 n
= [4 + (n – 1) × 2]
5. (a) 18, a, b, – 3 are in AP. 2
n
\ a – 18 = b – a = –3 – b = [2n + 2]
2
a – 18 = b – a = n (n + 1)
ADDITIONAL
R

PR ACTICE MATHEMATICS - 10 73
5 5 So, sequence is 1, 4, 7, ...
4. A.P.: –5, – , 0, , ... .
2 2 a2 – a1 = 4 – 1 = 3
5 5
an = a + (n – 1)d, d = – + 5 = a3 – a2 = 7 – 4 = 3
2 2
5 As difference between the terms is same, so,
\ a25 =
–5 + (25 – 1)
2 the given sequence is in A.P.
5
= –5 + 24 a10 = 30 – 2 = 28
Ł2 ł
= –5 + 60
Section-B
= 55
5. Sp = ap2 + bp 9. A.P : 18, 16, 14, ...

ap = Sp –Sp – 1 Sn = 0

= (ap2 + bp) – [a (p – 1)2 + b (p – 1)] n


[2a + (n – 1) d] = 0
2
= ap2 + bp – [ap2 + a – 2ap + bp – b] n
[36 + (n – 1) (–2)] = 0
= ap2 + bp – ap2 – a + 2ap – bp + b 2
n (36 – 2n + 2) = 0
= 2ap – a + b
n (38 – 2n) = 0
\
ap + 1 = 2a (p + 1) – a + b
38
= 2ap + 2a – a + b n = = 19
2
= 2ap + a + b 10. a4 = 0 a + 3d = 0 a = – 3d

So, d = ap + 1 – ap To prove : a25 = 3a11

= 2ap + a + b – 2ap + a – b Consider, a25 = a + (25 – 1)d

= 2a = a + 24d

6. an = n2 + 1 = –3 d + 24 d

a1 = 1 +1 = 2 = 21 d

a2 = 22 + 1 = 5 a11 = a + 10 d

3n - 2 = – 3d + 10d
7. an =
4n + 5 = 7d
3- 2 1 6-2 4 So, a25 = 3a11
a1 = = , a2 = =
4+5 9 8 + 5 13
11. A. P: 6, 13, 20, ..., 216
9-2 7
a3 = = an = a + (n – 1) d
17 17
1 4 7 216 = 6 + (n – 1) 7
So, sequence is , , , ...
9 13 17
8. an = 3n – 2 210 = 7 (n – 1)
a1 = 3 – 2 = 1 30 = n – 1
a2 = 6 – 2 = 4 n = 31
a3 = 9 – 2 = 7 so on So, 216 is 31st term of an A.P.
74 ADDITIONAL
R

PR ACTICE MATHEMATICS - 10
So, 16th term is the middle term 115
d = =5
a16 = 6 + (16 – 1) 7 23
From (ii), 2a + 9(5) = 47
= 6 + 7 (15)
= 6 + 105 2a + 45 = 47

= 111 a = 1

12. Consider 9, 12, 15, 18, .... So, A.P. is a, a + d, a + 2d, ...

a2 – a1 = 12 – 9 = 3 i.e. 1, 6, 11, ...

a3 – a2 = 15 – 12 = 3 14. A.P. : 5, 15, 25, ...

a4 – a3 = 18 – 15 = 3 Let nth term of an AP be 130 more than its 31st


term
As difference between the terms is same, i.e. an = 130 + a31
So, the terms are in A.P.
5 + (n – 1) 10 = 130 + 5 + (31 – 1) 10
a16 = a + 15d 5 + 10n – 10 = 135 + 300
= 9 + 15 (3) 10n = 435 + 5
= 9 + 45 10n = 440
= 54 n = 44
an = a + (n – 1) d So, 44th term of an AP is 130 more than its 31st
term.
= 9 + (n – 1) 3
15. a5 + a9 = 72
= 9 + 3n – 3
a + 4d + a + 8d = 72
= 3n + 6
2a + 12d = 72
13. S5 + S7 = 167
a + 6d = 36 ...(i)
5 7
[2a + (5 – 1) d] + [2a + (7 – 1) d] = 167
2 2 a7 + a12 = 97
5 7 a + 6d + a + 11d = 97
5a + (4d) + 7a + (6d) = 167
2 2 2a + 17d = 97 ...(ii)
5a + 10d + 7a + 21d = 167
On multiplying (i) by 2 and subtracting (i) from
12a + 31d = 167 ...(i) (ii), we get
S10 = 235 (2a + 17d) – (2a + 12d) = 97 – 72
10 5d = 25
[2a + (10 – 1) d] = 235
2 d = 5
2a + 9d = 47 ...(ii)
From (i), a = 36 – 6 (5)
Multiplying equation (ii) by 6 and subtracting
(i) from (ii), we get = 36 – 30 = 6
(12a + 54d) – (12a + 31d) a = 6
= 282 – 167 So, A.P. is a, a + d, a + 2d, ...
23d = 115 i.e. 6, 11, 16, ...
ADDITIONAL
R

PR ACTICE MATHEMATICS - 10 75
16. Consider AP : 7, 14, 21, ..., 497 n
2a + (n −1) d 7n + 1
an = a + (n – 1)d 2 =
n 4n + 27
497 = 7 + (n – 1)7 2a1 + (n −1) d1 
2
497 – 7 = 7 (n – 1) 2a + (n −1) d 7n + 1
=

490
= 70 = n – 1 2a + (n −1) d
1 1
4n + 27
7
n = 71  n −1
a+ d
 2  7n + 1
=
Section-C  n −1 1 4n + 27
a1 +   d
 2 
17. S7 = 49
a + (m - 1)d 7 (2m −1) +1
7 1 1 =
[2a + 6d] = 49 a + (m - 1)d 4 (2m −1) + 27
2
2a + 6d = 14 am 14m − 6
=
1
am 8m + 23
a + 3d = 7 ...(i)
19. Let the digits be a – d, a, a + d
Also, S17 = 289 a – d + a + a + d = 15
17
[2a + 16d] = 289 3a = 15
2
2a + 16d = 34 a = 5

a + 8d = 17 ...(ii) Also, 100 (a – d) + 10a + a + d

On subtracting (ii) from (i), we get     = [100 (a + d) + 10a + a – d] – 594


\
100 a – 100 d + 11a + d
(a + 3d) – (a + 8d) = 7 – 17
    = 100 a + 100 d + 11a – d – 594
–5d = –10
0 = 200 d – 2d – 594
d = 2
198 d = 594
From (i), a = 7 – 3d
d = 3
= 7 – 6
So, number = 100 (a + d) + 10a + a – d
= 1
= 100 (8) + 50 + 2
n
So, Sn = [2a + (n – 1)d] = 852
2
n 20. ap = q a + (p – 1) d = q ...(i)
= [2 + (n – 1)2]
2
aq = p a + (q – 1) d = p ...(ii)
= n [1 + (n – 1)]
On subtracting (ii) from (i), we get
= n2
(p – 1) d – (q – 1) d = q – p
18. Let Sn and S1n be sum of a terms of two A.P. d [p – 1 – q + 1] = q – p
Sn 7n +1 q-p
1 = d = = –1
Sn 4n + 27 p-q
76 ADDITIONAL
R

PR ACTICE MATHEMATICS - 10
From (i), a + (p – 1) (–1) = q 22. For S1, a = 1, d = 1
a = p–1+q n
So, S1 = [2 + (n – 1) (1)]
2
So, an = a + (n – 1)d n
= (n + 1)
= (p – 1 + q) + (n – 1) ( –1) 2
For S2, a = 1, d = 2
= p – 1 + q – n + 1 n
So, S2 = [2 + (n – 1) (2)]
= p + q – n 2
1 = n [1 + n – 1]
21. Here, a2 – a1 = 19 – 20
4 = n2
77
= – 20 For S3, a = 1, d = 3
4
n
77 - 80 3 S3 = [2 + (n – 1)3]
=- = 2
4 4 n
= [3n – 1]
1 1 2
a3 – a2 = 18 – 19 n n
2 4 Consider, S1 + S3 = (n + 1) + (3n – 1)
37 77 2 2
= – n 2 2
n 3n n
2 4 = + + -
2 2 2 2
74 − 77 4n2
= =
4 2
-3 = 2n2
=
4 = 2S2
as a3 – a2 = a2 – a1
23. A.P.: a, 7, b, 23, c
i.e. difference between the terms is same, so,
the given sequence forms an A.P. As the terms are in A.P.,
7 – a = b – 7 = 23 – b = c – 23
-3
Here, a = 20, d = As 7–a = b–7
4
an < 0 a + b = 14 ...(i)
a + (n – 1) d < 0 As b – 7 = 23 – b
 −3  2b = 30
20 + (n – 1)   < 0
 4  b = 15
−3
(n – 1) < – 20 From (i), a = 14 – b = 14 – 15
4
 −4  = –1
n – 1 > –20  
 3  As 23 – b = c – 23
80
n–1 > 23 – 15 = c – 23
3
80 83 c = 31
n > +1= = 27.67 24. Let the four parts be
3 3
So, n = 28 a – 3d, a – d, a + d, a + 3d such that
So, a28 is the first negative term. a – 3d + a – d + a + d + a + 3d = 32
ADDITIONAL
R

PR ACTICE MATHEMATICS - 10 77
4a = 32 100 m/min, 110 m/min, 120 m/min, ...
a = 8 So, distance travelled by policeman

Also,
( a - 3d)( a + 3d) =
7 = Sn
( a - d)( a + d) 15 n
= [2a + (n – 1) d]
2
i.e.
(8 − 3d)( 8 + 3d) =
7 n
= [200 + (n – 1) 10]
(8 − d)(8 + d) 15 2
= n [100 + 5n – 5]
64 - 9d2 7
=
64 - d2
15 = n (95 + 5n)
960 – 135d2 = 448 – 7d2 Distance travelled by thief

512 = 128 d2 = Distance travelled by


policeman
d2 = 4
100 (n + 1) = n (95 + 5n)
d = ±2
100n + 100 = 95n + 5n2
For a = 8, d = 2
5n2 – 5n – 100 = 0
Four parts are
n2 – n – 20 = 0
a – 3d, a – d, a + d, a + 3d
n2 – 5n + 4n – 20 = 0
i.e. 8 – 6, 8 – 2, 8 + 2, 8 + 6
n (n – 5) + 4 (n – 5) = 0
i.e. 2, 6, 10, 14
(n + 4) ( n – 5) = 0
For a = 8, d = – 2
n = – 4 or n = 5
Four parts are As n cannot be negative, n = 5
a – 3d, a – d, a + d, a + 3d 26. Consider the sequence formed by all three
i.e. 8 + 6, 8 + 2, 8 – 2, 8 – 6 digit numbers which leaves a remainder 3,
when divided by 4: 103, 107, 111, 115, ..., 999.
i.e. 14, 10, 6 and 2.
The above sequence forms an A.P. with a =
103 and common difference d = 4
Section-D
an = a + (n – 1)d
25. Let the time in which policeman catches the 999 = 103 + (n – 1) 4
thief is n minutes.
4 (n – 1) = 999 – 103
Uniform speed of thief = 100 m/min
4 (n – 1) = 896
As after one minute a policeman runs after
the thief to catch him. n – 1 = 224
So, distance travelled by thief n = 225
th
= 100 (n + 1) minutes The middle term is n + 1 term
Given that speed of policeman increases by Ł 2 ł
225 + 1
10m/min. i.e. = 113th term
Ł 2 ł
speed of policeman forms an AP: a113 = 103 + (113 – 1) 4
78 ADDITIONAL
R

PR ACTICE MATHEMATICS - 10
= 103 + 112 (4) As m n, 2a + (m + n – 1) d = 0
= 103 + 448 Consider
= 551 m+n
Sm + n =
[2a + (m + n – 1) d]
Sum of all terms before middle term 2
m+n
= S112 = (0)
Ł 2 ł
112 = 0
= [2 (103) + (112 – 1) 4]
2 So, sum of its (m + n) terms is zero.
= 56 [ 206 + 444]
28. AP = –12, –9, –6, ... , 21
= 56 (650)
If 1 is added to each term,
= 36,400

A.P. becomes –12 + 1, –9 + 1, –6 + 1, ..., 21 + 1
225
S225 = [2 (103) + (225 – 1) 4] i.e. – 11, –8, –5, ..., 22
2
225 We know that
= [206 + 896]
2 an = a + (n – 1) d
225
= (1102) 22 = –11 + (n – 1) (3)
2
= 123975 33
= n–1
So, sum of terms after the middle term 3
n = 12
= 123975 – (S112 + 551)
12
= 123975 – 36400 – 551 S12 = [2 (–11) + (12 – 1) 3]
2
= 87024 = 6 [–22 + 33]

27. Given: Sm = Sn = 6 (11)

To prove: Sm + n = 0 = 66
m 29. Let the prizes be a, a – 20, a – 40, ....
Sm = [2a + (m – 1)d]
2 S10 = 1600
n
Sn = [2a + (n – 1)d] 10
2 [2a + (10 – 1) (– 20)] = 1600
2
As Sm = Sn
5 (2a – 180) = 1600
m n
\ [2a + (m – 1)d] = [2a + (n – 1)d] 2a – 180 = 320
2 2
m [2a + (m – 1)d] = n [2a + (n – 1)d] 2a = 500
2am + md (m – 1)] = 2na + nd (n – 1) a = 250
2am + m2d – md = 2an + n2d – nd So, the prize are 250, 230, 210, 190, 170, 150,
2am – 2an + m d – n d – md + nd = 0
2 2 130, 110, 90.

2a (m – n) +d (m2 – n2) – d (m – n) = 0 30. First term = a


(m – n) [2a + (m + n) d – d) = 0 Second term = b
(m – n) [2a + (m + n – 1) d] = 0 last term (an) = c
ADDITIONAL
R

PR ACTICE MATHEMATICS - 10 79
(a + c )(b + c − 2a ) So, amount paid for installments:
To prove: Sn =
2 (b − a ) 12200, 11600, ... forms an AP
Here,
d = b–a First term (a) = 12200
n Common difference (d) = 11600 – 12200
Sn = [2a + (n – 1) d]
2
= –600
or

n = 12
n
= [a + an] n
2 Sn = [2a + (n – 1) d]
2
n
= [a + c] ...(i) 12
2 = [2 (12200) + (12 – 1) (–600)]
2
We know that an = c = 6 [24400 + 11 (–600)]
i.e. a + (n – 1) (b – a) = c = 6 (24400 – 6600)
c-a = 6 (17800)
(n – 1) =
b-a
= ` 106,800
c-a
n= +1
b-a 32. a4 + a8 = 24
c + b - 2a a + 3d + a + 7d = 24
= ...(ii)
b-a 2a + 10d = 24
On putting (ii) in (i), we get
a + 5d = 12 ...(i)
1 c + b - 2a
Sn = (a + c) Again,
2 Ł b-a ł
a6 + a10 = 44
=
(a + c )(b + c − 2a ) a + 5d + a + 9d = 44
2 (b − a )
2a + 14d = 44
31. Amount paid in cash = ` 60,000
a + 7d = 22 ...(ii)
Amount pending = ` 120000 – ` 60000
On subtracting eqn (i) from (ii), we get
= ` 60000
a + 7d – a – 5d = 22 – 12
Amount of the first installment
2d = 10
12
= 5000 +
100
(60,000 ) d = 5

= 5000 + 7200 From (i), a = 12 – 5d


= 12 – 25
= 12200
= –13
Amount of second installment
10
12 S10 = [2 (–13) + (10 – 1) 5]
= 5000 + (60000 – 5000) 2
100
= 5000 + 6600 = 5 (–26 + 45)

= 11600 = 5 (19) = 95

80 ADDITIONAL
R

PR ACTICE MATHEMATICS - 10
5. First term = a
WoRKSHeet - 2
Second term = b
Section-A
Last term (an) = 2a
1. Consider an AP: 12, 18, 24, ..., 96 Common difference (d) = b – a
an = a + (n – 1) d n n
Sn = [2a + (n – 1)d] or [a + an]
96 = 12 + (n – 1) 6 2 2
n
96 – 12 = 6 (n – 1) Sn = [a + 2a]
2
84 3a
n–1 = = n ...(i)
6 2
n –1 = 14 As an = 2a
n = 15 a + (n – 1) d = 2a
2. Sq = 2q + 3q 2
a + (n – 1) ( b – a) = 2a
Sq – 1 = 2 (q – 1) + 3 (q – 1) 2
(n – 1) (b – a) = a
= 2q – 2 + 3q2 + 3 – 6q a
n–1 =
= 3q – 4q + 1
2
b−a
a
aq = sq – sq – 1 n = +1
b−a
= 2q + 3q2 – 3q2 + 4q – 1 b
n = ...(ii)
= 6q – 1 b−a
On putting (ii) in (i), we get
aq + 1 = 6q + 6 – 1 = 6q + 5
3a b
\ d = aq + 1 – aq = 6q + 5 – 6q + 1 Sn =
2 (b - a )
=6 3ab
=
3. Consider AP: 1, 3, 5, 7, ..., n 2 (b − a )
1 1 + m 1 + 2m
with a = 1, d = 2 6. , , , ...
m m m
n 1
Sn = [2a + (n – 1) d] Here, a =
2 m
n 1+ m 1 1 1
= [2 + (n – 1) 2] d = – = +1– =1
2 m m m m
= n [1 + n – 1)
an = a + (n – 1) d
=n 2
1
= + (n – 1) 1
4. As terms are in AP, m
13 – (2p + 1) = (5p – 3) –13 1
= +n–1
m
13 – 2p – 1 = 5p – 3 – 13 7. Let 184 be the nth term of
12 + 16 = 7p AP = 3, 7, 11, ... where
7p = 28 a = 3
p = 4 d = 7–3=4
ADDITIONAL
R

PR ACTICE MATHEMATICS - 10 81
an = a + (n – 1) d Section-B
184 = 3 + (n – 1) 4 11. A.P : 27, 24, 21, ...
184 – 3 = 4 (n – 1) Let sum of n terms of the A.P. be 0.
181 Here, first term (a) = 27
n–1 =
4 Common difference (d) = 24 – 27
185
n =  hich is not a natural =
w –3
4
number. Sn = 0
So, 184 is not a term of AP: 3, 7, 11, ... n
[2a + (n – 1) d] = 0
2
n
8. Consider AP: 254, ..., 14, 9, 4 [54 + (n – 1) (–3)] = 0
2
where a = 254 n (54 – 3n + 3) = 0
d = 9 – 14 = –5 n (18 – n + 1) = 0
18 – n + 1 = 0
So, a10 = 254 + (10 – 1) (–5)
n = 19
= 254 – 45
So, sum of 19 terms is 0.
= 209
Sm m2
12. =
9. a1 = 4 Sn n2
am 2m - 1
an = 4an – 1 + 3, n > 1 To prove : =
an 2n - 1
a2 = 4a1 + 3 = 16 + 3 = 19 Sm m2
a3 = 4a2 + 3 = 4 (19) + 3 = 79 As = 2
Sn n
m
a4 = 4a3 + 3 = 4 (79) + 3 = 319 2a + (m −1) d m2
a5 = 4a4 + 3 = 4 (319) + 3 = 1279 \ 2 = 2
n n
a6 = 4a5 + 3 = 4 (1279) + 3 2a + (n − 1) d
2
= 5119.
2a + (m −1) d m
=
10. an = 4n + 5 2a + (n −1) d n
a1 = 4 + 5 = 9 (m −1)
a+ d m
2 =
a2 = 4 (2) + 5 = 13 (n −1) n
a+ d
a3 = 4 (3) + 5 = 17 2
On replacing m by 2m – 1 and n by 2n – 1 on
a4 = 4 (4) + 5 = 21
both sides of equation, we get
a2 – a1 = 13 – 9 = 4
a + (m −1) d 2m - 1
=
a3 – a2 = 17 – 13 = 4 a + (n −1) d 2n - 1
a4 – a3 = 21 – 17 = 4 3n2 5n
13. Sn = +
2 2
As difference between the terms is same, the
We know that an = Sn – Sn – 1
sequence defined by an = 4n + 5 is an A.P. such
that d = 4. So, a25 = S25 – S24
82 ADDITIONAL
R

PR ACTICE MATHEMATICS - 10
3 5  3 5  17. a9 = –32
=  (625) + (25) −  (576) + (24)
2 2  2 2  a + 8d = –32 ...(i)
1875 125 1728 120 Also, a11 + a13 = –94
= + - -
2 2 2 2 a + 10d + a + 12d = –94
1875 +125 −1728 −120
=
2 2a + 22d = –94
= 76
a + 11d = –47 ...(ii)
14. Number of terms = 111
On subtracting (i) from (ii), we get
So, middle term must be the 56th term
a + 11d – a – 8d = –47 + 32
a56 = 30
3d = –15
a + 55d = 30
n d = –5
Consider Sn = [2a + (n – 1) d]
2 18. To prove: S30 = 3 (S20 – S10)
111
\ S111 = [2a + 110 d] = 111 [a + 55d] Consider, 3 (S20 – S10)
2
= 111 (30) = 3330  20 10 
= 3  2a + (20 −1) d − 2a + (10 −1) d 
15. First term (a) = – 7 2 2 

Common difference (d) = 5 { }


= 3 10 2a + (19 ) d − 5 2a + 9d

We know that an = a + (n –1) d = 30 (2a + 19d) – 15 (2a + 9d)


= –7 + (n – 1) 5 = 60a + 570d – 30a – 135d
= –7 + 5n – 5 = 30a + 435d
= 5n – 12 30
Also, S30 =
[2a + (30 – 1) d)
So, a18 = 5 (18) – 12 2
= 15 [2a + 29d)
= 90 – 12
= 30a + 435d
= 78
\
S30 = 3 [S20 – S10]
16. a10 = 52
19. a14 = 2a8
\
a + 9d = 52 ...(i)
a + 13d = 2 [a + 7d]
a17 = 20 + a13
a + 13d = 2a + 14d
a + 16d = 20 + a + 12d
–d = a
4d = 20
a6 = – 8
d = 5
From (i), a + 9 (5) = 52 a + 5d = – 8

a + 45 = 52 – d + 5d = – 8 (As a = – d)

a = 7 4d = – 8
So, AP is a, a + d, a + 2d, ... d = –2
i.e. 7, 12, 17, ... So, a = –d = 2
ADDITIONAL
R

PR ACTICE MATHEMATICS - 10 83
n So, A.P. is a, a + d, a + 2d, ...
We know that Sn = [2a + (n – 1) d]
2 i.e. 1, 5, 9, ....
20 1 1
\
S20 = [2 (2) + (20 – 1) (–2)] 22. AP : 18, 15 , 13, ..., –49
2 2 2
= 10 [4 – 38] 31 99
i.e.  18, , 13, ..., –
= 10 (–34) 2 2
= –340 Here, first term (a) = 18
31
20. First term (a) = 7 Common difference (d) = – 18
2
Last term (an) = 49 31 - 36 5
= = –
Sn = 420 2 2
n 99
We know that Sn = [a + an] Last term (an) = –
2 2
n 99
420 = [7 + 49] a + (n – 1) d = –
2 2
420  5 99
n = = 15 18 + (n – 1)  −  = –
28  2 2
Now, an = 49 5 99
– (n – 1) = – – 18
a + (n – 1)d = 49 2 2
5 - 99 - 36
7 + (15 – 1)d = 49 – (n – 1) =
2 2
14d = 42
5 135
d = 3 – (n – 1) = –
2 2
135 2
Section-C n–1 = – × = 27
2 –5
21. a2 + a7 = 30 n = 28
a + d + a + 6d = 30 So, number of terms (n) = 28
2a + 7d = 30 ...(i) n
We know that Sn = [2a + (n – 1) d]
Also, a15 = 2a8 – 1 2
  −5  
28
a + 14d = 2 [a + 7d] – 1 36 + (28 −1)   
S28 =
 2
2 
a + 14d = 2a + 14d – 1
 135 
= 14 36 −
0 = a–1
 2 
a = 1  72 −135 
= 14  
From (i), 2 (1) + 7d = 30  2
7d = 28 = 7 (–63)
d = 4 = –441

84 ADDITIONAL
R

PR ACTICE MATHEMATICS - 10
23. an = –4n + 15 From (ii), a + 9(4) = 41
a1 = –4 + 15 = 11 a = 41 – 36
a2 = –4 (2) + 15 = –8 + 15 = 7
= 5
a3 = –12 + 15 = 3.
We know that an = a + (n – 1)d
So, First term (a) = 11
Common difference (d) = 7 – 11 = – 4 = 5 + (n – 1) 4
We know that = 4n + 1
n
Sn = [2a + (n – 1) d] 26. Consider an AP = 504, 511, 518, ..., 896
2
20 Here, first term (a) = 504
S20 = [2(11) + (20 – 1) ( – 4)]
2 Common difference (d) = 511 – 504 = 7
= 10 [22 – 76]
Last term (an) = 896
= 10 (–54)
As an = 896
= – 540
a + (n – 1)d = 896
24. a8 = 31
504 + (n – 1) 7 = 896
a + 7d = 31 ...(i)
7 (n – 1) = 392
a15 = a11 + 16
n – 1 = 56
a + 14d = a + 10d + 16
n = 57
4d = 16 n
We know that Sn =
[2a + (n – 1) d]
d = 4 ...(ii) 2
From (i), a + 28 = 31 57
\ S57 = [2 × 504 + (57 – 1)7]
2
a = 3
57
= [1008 + 392]
So, A.P. is a, a + d, a + 2d, ... 2
= 39900
i.e. 3, 7, 11, ...
27. First term (a) = 5
25. a15 = 3 + 2a7
Let d be the common difference.
a + 14d = 3 + 2 (a + 6d) 1
S4 = [S8 – S4]
a + 14d = 3 + 2a + 12d 2
0 = a – 2d + 3 ...(i) 4
i.e. [2a + (4 – 1) d]
2
Also, a10 = 41
1 8 4 
a + 9d = 41 ...(ii) =  2a + ( 8 −1) d − 2a + ( 4 –1) d 
2 2 2 
On subtracting (i) from (ii), we get
i.e. 2 (2a + 3d) = 2 (2a + 7d) – (2a + 3d)
a + 9d – a + 2d = 41 + 3
4a + 6d = 4a + 14d – 2a – 3d
11d = 44
4a + 6d = 2a + 11d
d = 4
2a = 5d
ADDITIONAL
R

PR ACTICE MATHEMATICS - 10 85
2a 2 39
= (5) = 2
d = = [16 + 342]
5 5 2
So, common difference (d) = 2 = 6981
28. A.P: 3, 9, 15, ..., 99 30. Let the first term of an AP be ‘a’ and common
Here, first term (a) = 3 difference be ‘d’.
Common difference (d) = 9 – 3 S10 = –150
= 6 10
[2a + (10 – 1) d] = –150
Last term (an) = 99 2
5 (2a + 9d) = –150
a + (n – 1)d = 99
2a+ 9d = –30 ...(i)
3 + (n – 1)6 = 99
Also, S20 – S10 = –550
6 (n – 1) = 96
20 10
n – 1 = 16 (2a + 19d) – (2a + 9d) = –550
2 2
n = 17 10 (2a + 19d) – 5 (2a + 9d) = –550
n
We know that Sn = [2a + (n – 1)d] 2 (2a + 19d) – (2a + 9d) = –110
2
17 4a + 38d – 2a – 9d = –110
S17 = [6 + (17 – 1) 6]
2 2a + 29d = –110   ...(ii)
17 On subtracting (ii) from (i), we get
= [6 + 96]
2
2a + 9d – 2a – 29d = –30 + 110
= 867
– 20d = 80
29. First term (a) = 8
d = –4
Last term (an) = 350
From (i), 2a + 9( –4) = –30
Common difference (d) = 9
2a = –30 + 36
As an = 350
2a = 6
a + (n – 1)d = 350
a = 3
8 + (n – 1) 9 = 350
So, A.P. is a, a + d, a + 2d, ...
9 (n – 1) = 342
i.e. 3, 3 – 4, 3 – 8, ...
342 114
n–1 = = = 38 i.e. 3, –1, –5, ...
9 3
n – 1 = 38
Section D
n = 39
We know that 31. Let A, D be first term and common difference
respectively.
n
Sn = [2a + (n – 1)d] p
2 Sp = a [2A + (p – 1) D ] = a
39 2
S39 = [16 + (39 – 1) 9] q
2 Sq = b [2A + (q – 1) D ] = b
2
86 ADDITIONAL
R

PR ACTICE MATHEMATICS - 10
r 1
Sr = c [2A + (r – 1) D ] = c d =
2 mn
Consider, 1
1
a b c From (i), a + (m – 1) =
(q – r) + (r – p) + (p – q) mn n
p q r 1
1
1 p 1 q a + (m – 1) =
=  [2A + (p – 1) D] (q – r) + [2A + mn n
p 2 q 2
1 r 1 1 1
(q – 1) D] (r – p) + [2A + (r – 1)D] (p – q) a+ – =
r 2 n mn n
1 1 1
=  [2A + (p – 1) D] (q – r) + [2A + (q – 1) D] a =
2 2 mn
1
(r – p) + [2A + (r – 1) D] (p – q) Consider,
2
D mn  2 1 
= [ A (q – r) + A (r – p) + A (p – q)] + [(p – 1) Smn =  + (mn −1) 
2 2  mn mn 
(q – r) + (q – 1) (r – p) + (r – 1) (p – q)]
mn  2 1 
D =  +1−
= A (q – r + r – p + p – r) + [(p – 1) (q – r) 2  mn mn 
2
+ (q – 1) (r – p) + (r – 1) (p – q)] mn  1 
=  +1
D 2  mn 
= 0 + [pq – pr – q + r + qr – qp – r + p +
2 mn  mn +1
=
rp – rq – p + q] 2  mn 
= 0 + 0 1
= (mn + 1)
= 0 2
32. Let a and d be the first term and common 33. Length of each step = 50 m
term of an A.P.
1 1
am = Width of each step = m
n 2
1 1
a + (m – 1) d = ...(i) Height of first step = m
n 4
1 1 1 1
Also, an = Height of second step = + = m
m 4 4 2
1 3
a + (n – 1) d = ...(ii) Height of third step = m so on.
m 4

On subtracting (i) from (ii), we get Volume of concrete required to build the first
1 1
step (V1) = 50 × × m3
1 1 2 4
a + (n – 1)d – a – (m – 1)d = –
m n Volume of concrete required to build the
n-m 1  1
d (n – 1 – m + 1) = second step (V2) = 50 × ×  2 × 
mn 2  4
n-m 1 3 3
d (n – m) = (V3) = 50 × × m and so on.
mn 2 4
ADDITIONAL
R

PR ACTICE MATHEMATICS - 10 87
Total volume of concrete n
Sn = [2a + (n – 1) d]
= V1 + V2 + V3 + ... + V15 2
1000
 1 1  1  1  S1000 = [2 + (1000 – 1) 1]
=  50 × ×  + 50 × ×  2 ×   2
2 4  2 4 
= 500 (2 + 999)
 1 1  1  1 
+  50 × × 3 ×  + .... + 50 × × 15 ×   = 500 (1001)
 2 4  2  4 
= 500500
 1  1 2 3 15  Now consider list of numbers divisible by 2:
=  50 ×   + + + .... +  m3
 2  4 4 4 4 2, 4, 6, 8, ..., 1000

1 2 3 15  This sequence also forms an AP with a = 2,


= 25  + + + .... +  m
3
1000
4 4 4 4 d = 2, n = = 500
2
25 500
= (1 + 2 + .... + 15) m3 S500 = [2 (2) + (500 – 1) 2]
4 2
25 15 = 250 (4 + 499 (2)]
= × (1 + 15) = 750 m3 = 250500
4 2
Again, consider list of numbers divisible by 5:
34. Let the first term and common difference of
5, 10, 15, ..., 1000
an A.P. be a and d respectively.
Let S and S1 be the sum of odd terms and even 1000
Here, a = 5, d = 5, n = = 200
terms of A.P. 5
S = a1 + a3 + a5 + .... + a2n + 1 200
S200 = [10 + (200 – 1) 5]
2
n +1
= (a1 + a2n + 1) = 100 [10 + 5 (199)]
2
n +1 = 100500
= [a + a + (2n + 1 – 1) d]
2 Now, we will consider list of numbers divisible
= (n + 1) (a + nd) by both 2 and 5 i.e. 2 × 5 = 10
S1 = a2 + a4 + a6 + .... + a2n 10, 20, 30, ..., 1000
n This list of numbers form an AP with
S1 = [2a + 2nd]
2
1000
= n (a + nd) a = 10, d = 10, n = = 100
10
S (n +1)(a + nd) S100 =
100
[20 + (100 – 1) 10]
Consider =
S1 n ( a + nd) 2
= 50 (20 + 990)
n +1
= = 50500
n
Therefore, sum of numbers which are either
35. Consider 1, 2, 3, ..., 999, 1000
divisible by 2 or 5
This sequence forms an AP with first term = S200 + S500 – S100
(a) = 1 and common difference (d) = 1 = 100500 + 250500 – 50500
We know that = 300500
88 ADDITIONAL
R

PR ACTICE MATHEMATICS - 10
So, sum of numbers from 1 to 1000 that are We know that
neither divisible by 2 nor by 5 n
Sn = [2a + (n – 1) d]
= S1000 – 300500 2
= 500500 – 300500 30
S30 = [2 (200) + (30 – 1) 50]
= 200000 2

36. Suppose the work is completed in n days. = 15 [400 + 1450]


Consider an AP: 150, 146, 142, ... = 27,750
Here, First term (a) = 150
\ The contractor has to pay ` 27,750 as
Common difference (d) = – 4 penalty, if he has delayed the work by 30
Total number of workers who worked all the days.
n days = Sn 38. Consider AP: 20, 19, 18, ....
n
= [2 (150) + (n – 1) (– 4)] Here, First term (a) = 20
2
n Common difference (d) = – 1
= (300 – 4n + 4)
2
Let 200 logs be placed in n rows
n
= [304 – 4n]
2 \ Sn = 200
= n (152 – 2n) n
[2(20) + (n – 1) (–1)] = 200
If the workers did not drop, work would have 2
been finished in (n – 8) days such that 150 n
[40 – n + 1] = 200
workers work on each day. 2
n (41 – n) = 400
\ Total number of workers who worked all
the n days = 150 (n – 8) – n2 + 41n – 400 = 0
\
n(152 – 2n) = 150 (n – 8) n2 – 41n + 400 = 0
152 n – 2n2 = 150n – 1200 n2 – 16n – 25n + 400 = 0
152n – 150n = 2n2 – 1200 n (n – 16) – 25 (n – 16) = 0
2n – 2n – 1200 = 0
2
(n – 16) (n – 25) = 0
n2 – n – 600 = 0 n = 16 or 25
n2 – 25n + 24n – 600 = 0
If n = 25,
n (n – 25) + 24 (n – 25) = 0
a25 = 20 + (25 – 1) (–1)
(n + 24) (n – 25) = 0
= 20 – 24
n = –24, n = 25
= – 4 not possible
Being the number of days, n cannot be negative,
so, n = 25 So, n = 16

\
Work was completed in 25 days. So, 200 logs are placed in 16 rows.
a16 = 20 + (16 – 1) (– 1)
37. Consider the sequence: 200, 250, 300, ...
= 20 – 15 = 5
This sequence form an AP with first term
(a) = 200 and common difference (d) = 50 So, there are 5 logs in the top row.
ADDITIONAL
R

PR ACTICE MATHEMATICS - 10 89
39. Given : a2, b2, c2 are in A.P. 25
= [2 (20) + (25 – 1) 10]
1 1 1 2
To prove : , , are in A.P. 25
b+c c+a a+b = [40 + 240]
2
1 1 1 25
, , are in AP if = × 280
b+c c+a a+b 2
1 1 1 1 = 25 × 140
− = −
c+a b+c a+b c+a = 3500 m
\
(b + c ) − ( c + a ) (a + c ) − (a + b ) Total distance covered by the gardener
to water all trees = 3500 m
i.e. =
(b + c )(a + c ) (a + b )(a + c )
b+c−c−a a + c −a −b cASe StUDY-1
i.e. =
(b + c )(a + c ) (a + b )(a + c )
(i) (a) A16 = a + 15l
b−a c-b
i.e. = A9 = a + d
(b + c )(a + c ) ( a + b )( a + c ) A16 = A9 + 7
b−a c−b A16 – A9 = 7
i.e. =
b+c a+b
a + 15d – (a + 8d) = 7
i.e. (b – a) (a + b) = (c – b) (b + c) 15d – 8d = 7
i.e. ab + b2 – a2 – ab = bc + c2 – b2 – bc d=1

i.e. b2 – a2 = c2 – b2 (ii) (c) A4 = a + 3d


\ a2, b2, c2 are in A.P. A8 = a + 7d
A4 + A8 = 30
40. 10m 5m 5m a + 3d + a + 7d = 30
Well
2a + 10d = 30
Distance covered by gardener to water 1st a + 5d = 15
tree and return to the initial position . .
a = 0 . the origin is occupied by the teacher.
= 10 m + 10 m = 20 m
\ 5d = 15
Distance covered by gardener to water 2nd
d = 3
tree and return to initial position
(iii) (c) A20 – A6 = 84
= 15 m + 15 m = 30 m
A20 = a + 19d
Distance covered by gardener to water 3rd
tree and return to initial position A6 = a + 5d
a + 19d – (a + 5d) = 84
= 20 m + 20 m = 40 m
14d = 84
So, we get an AP: 20, 30, 40, ...
d = 6
Where, first term (a) = 20
(iv) (d) There are total 21 persons standing in the
difference (d) = 10 queue the 10th person is the middle one.
Total distance covered by the gardener (v) (d) The positions of the teacher and students
= S25 are in AP hence the distance. A9 A10 is
90 ADDITIONAL
R

PR ACTICE MATHEMATICS - 10
equal to the distance between any two
− ( −41) ± ( −41) − 4 (1)( 400 )
2
consecutive persons.
n =
\ Distance A9 A10 = Distance A19 A20 2
−41 ± 1681 −1600
=
cASe StUDY-2 2
41 ± 9
=
(i) (b) The 20th row is the bottom row. Hence an 2
is 20. 41 + 9 41 - 9
n = n =
2 2
(ii) (b) a = Number of logs in first row = 25 = 16
n = total number of logs For n = 16, the number of logs in the 16th
a=1 row is:
n = 20 a16 = a + (n – 1) d
a + n = 21 a = 20, d = –1, n = 16
a16 = 20 + 15 (–1)
(iii) (c) The sum of logs is 200.
n =5
using the formula for the sum of AP For n = 25, the number of logs in 25th row is:
2
whose common difference is –1 and first a25 = 20 + (25 – 1) (–1)
term is 20. = 20 – 24
n
Sn = [2a + (n–1) d] = –4
2
n As the number of logs cannot be negative
200 = [2 (20) + (n – 1) (–1)] hence n = 16.
2
400 = n [40 + 1 – n] (v) (b) The 16th row from the bottom is the top
n – 414 + 400 = 0
2
row. Number of logs in the 16th row is
a16 = a + (16 – 1) d
(iv) (c) The number of rows can be calculated by
solving the below given linear equation. = 20 + 15 (–1)
n2 – 41n + 400 = 0 = 20 + 5
= 5

ADDITIONAL
R

PR ACTICE MATHEMATICS - 10 91
Chapter

6 Triangles
Multiple choice Questions

1. (b) ∆ ABC ~ ∆ PQR AX + 1 = AY +1


2
BX YC
∴ ar D ABC =  BC 
ar D PQR  QR  AB
= AC
BX CY
( 4.5)
2
9
⇒ =
16 (QR )
2 4 BX
= AC ( ... AB = 4 BX)
BX 2
⇒ (QR)2 = 4.5 × 4.5 × 16 4 = AC
9
2
⇒ QR = 4.5 × 4 AC = 8 cm
3
= 1.5 x 4 So, AY = AC – CY
= 6 cm = 8–2
2. (a) We know that ratio of area of two similar = 6 cm
triangles is equal to square of ratio of
their corresponding sides (say x and y) 4. (c)
2
A
9 x
⇒ =  
16 y

x 3
⇒ =
y 4

3. (d) A
B C

AB2 = 2 AC2
X
4B

X Y = AC2 + AC2
2 cm
= AC2 + BC2

B C [ ... AC = BC]
AS XY ll BC, so by basic proportionality ∴ AB2 = AC2 + BC2
theorem
∴ ∆ABC is a triangle right angled at C
AX = AY
BX YC i.e. ∠C = 90o
92 ADDITIONAL
R

PR ACTICE MATHEMATICS - 10
5.(b) A 1
OA = OC = AC = 15 cm
2
1
m OB = OD = BD = 20 cm

5c
2
6c

m
In ∆ AOB, ∠AOB = 90O
(Diagonals of rhombus bisect each other at
90o)
B 3 cm D C
AB2 = AO2 + OB2 (Pythagoras theorem)
As AD bisects ∠BAC
= (152) + (202)
AB AC = 225 + 400
∴ =
BD CD
= 625
[By internal angle bisector theorem]
AB = 25 cm
6
⇒ = 5 ∴ AB = BC = CD = AD = 25 cm
3 CD
3×5 (All sides of rhombus are equal)
⇒ CD = = 2.5 cm
6
3. A
WoRKSHeet - 1
6 cm
Section-A
1. ∆ ABC ~ ∆ DEF
In ∆ ABC, ∠A + ∠B + ∠C = 180o C B
57o + ∠B + 73o = 180o In ∆ ABC,
∠B + 130 = 180
o o
AC = BC = 6 cm (As ∆ ABC is isosceles)
∠B = 108o – 130o Also, ∠C = 90o
= 50o ∴ AB2 = AC2 + BC2 (Pythagoras theorem)
∴ ∠E = ∠B = 50o = 62 + 62
[Corresponding angles of similar triangles
= 36 + 36
are equal.]
AB2 = 72
2. A B AB = 6 2 cm

4. As, ∆DEF ~ ∆ABC


O
DE EF DF
= =
AB BC AC
D C
DE 4 DF
AC = 30 cm = =
3 2 2.5
BD = 40 cm
ADDITIONAL
R

PR ACTICE MATHEMATICS - 10 93
DE 4 4 DF (Ratio of areas of two similar triangles is equal
= = to square of their corresponding sides)
3 2 2 2.5
12 4 × 2.5 54 12
DE = DF = = 2
2 2 ar D PQR 3
= 6 cm = 5 cm 54 1
=
ar ∆PQR 9
Perimeter of ∆ DEF = DE + EF + DF
54 × 9
= 6 + 4 + 5 = 15 cm ar DPQR = = 486 cm2
1

5.
A 8. B N

D E 8m

W E
A 15 m O
B C
Let AE = x cm
∴ CE = AC – AE = 5.6 – x cm
As DE II BC, S
AD AE In ∆BAO, ∠BAO = 90o
=
DB CE OB2 = AB² + AO2 (Pythagoras theorem)
(By Basic proportionality theorem) = 82 + 152
= 64 + 225
3 x = 289
=
5 5.6 - x ∴ OB = 17 m
5x = 3 (5.6 – x)
5x = 16.8 – 3x Section-B
8x = 16.8 9. ∆ABC ~ ∆DEF,
x = 2.1 cm
ar D ABC = BC 2
∴ AE = x =2.1 cm ar D DEF EF 2
6. 
We know that ratio of the areas of two
(In two similar triangles, the ratio of their ar-
similar triangles is equal to the square of their
eas is the square of ratio of their sides)
altitudes.
2
2 64 BC
2
∴ Ratio of areas =   = 4:9 =
121 (15.4 )
2
3

7. DABC ~ DPQR 64
2 (BC)2 = x 15.4 x 15.4
ar D ABC  BC  121
= 
ar D PQR  QR  ∴ BC = 11.2 cm
94 ADDITIONAL
R

PR ACTICE MATHEMATICS - 10
10. In ∆ABC, AE bisects ∠ BAC
A
  
CE = AC ...(ii)
BE AB
From (i) and (ii), we get
E F CF CE
   =
DF BE
∴ EF II BD
(By converse of Basic proportionaly theorem)
B D C
In ∆ADB, DE is bisector of ∠ ADB 12. In ∆AOB ~ ∆COD
∠AOB = ∠COD (Vertically opposite angles)
BD AD
= AO BO
BE AE = (Given)
BD BE OC DO
i.e. = (i)
AD AE ∴ ∆AOB ~ ∆AOB (SAS)

In ∆ADC, DF is bisector of ∠ADC AO BO AB
So, = =
CD AD OC OD DC
= (Corresponding sides of similar triangles are
CF AF
proportional)
CD CF
i.e. =
AD AF 1 5
=
BD CF 2 CD
= (ii)
AD AF CD = 10 cm
(As AD is median ∴ BD = CD) 13. In ∆ KPN and ∆ KLM,
From (i) and (ii), we get ∠K = ∠ K (Common)
BE CF ∠ KNP = ∠ KML = 46o (Given)
=
AE AF ∴ ∆KPN ~ ∆KLM (AA similarity criterion)
AE AF KP PN KN
= = =
BE CF KL LM KM
So, by converse of Basic proportionality theorem
EF II BC x c
=
a b+c
11. A D
ac
x =
b+c
F
14. A B

B E C F E
In ∆ ADC, AF bisects ∠ DAC
CF AC D C
= ∴
DF AD
In ∆AFD and ∆BEF
AC
= (As AB = AD) ...(i) ∠DAF = ∠FEB (Alternate interior
AB angles)
ADDITIONAL
R

PR ACTICE MATHEMATICS - 10 95
∠AFD = ∠BFE (Vertically opposite ∴ x = 10
angles)
So, AB = 10 m
∴ ∆AFD ~ ∆EFB
  BC = 2x + 4 = 24 m
EF FB
So, =   AC = 2x + 6 = 26 m
FA DF
(Corresponding sides of similar triangles are 17. We know that diagonals of rhombus bisect
proportional) each other at 90o.
DF x EF = FB x FA Let AC = 24 cm
15. As DE II AC, so in ∆ABC, BD = 10 cm
BD BE 1
= (i) AO = OC = AC = 12 cm
AD EC 2
(Basic proportionality theorem)
1
BO = OD = BD = 5 cm
2
BE BC In ∆AOB,
Also, = (ii) (Given)
EC CP AB2 = BO2 + AO2
BD BC
From (i) and (ii), we have = = 52 + 122
AD CP
∴ DC II AP = 25 + 144
(By converse of Basic proportionality theorem.) = 169

16. A ∴ AB = 13 cm

2x+6 As all sides of rhombus are equal,


x
AB = BC = CD = AD = 13 cm

2x+ 4 C 18. In ∆ABC, DE II BC


B
AD AE
Let the shorter side be x m =
DB EC
∴ Hypotenuse = 2x + 6 m [ Basic proportionality theorem ]
Also, Third side = 2x + 6 – 2
x x+2
= 2x + 4 m =
x-2 x −1
In ∆ABC, AC2 = AB2 + BC2 x (x – 1) = (x – 2) (x + 2)
(2x + 6)2 = x2 + (2x + 4)2 x2 – x = x2 – 4
4x2 + 36 + 24x = x2 + 4x2 + 16 +16x   x = 4

0 = x2 – 20 – 8x
Section-C
x2 – 8x – 20 = 0
19. A
x2 – 10x + 2 (x2 – 10) = 0
p cm
(x2 – 10) (x – 2) = 0
x = 10, – 2
Being a side, x = – 2 is rejected B q cm C
96 ADDITIONAL
R

PR ACTICE MATHEMATICS - 10
In ∆ ABC, AC2 = AB2 + BC2 (BQ and RC are perpendicular)
p2 = AB2 + q2 ∴ DABQ ~ DACR (AA Similarity criterion)
AB = p – q
2 2 2
AB BQ AQ
= =
= (p – q) (p + q) AC CR AR
AB z
= 1 (p + q) = ...(ii)
AC y
AB² = p + q From (i),
AB = p +q BC z
1– =1–
AC x
QT QR AC – BC x–z
20. = (Given) =
PR QS AC x
In DPQR, ∠1 = ∠2 AB x–z
= ...(iii)
AC x
∴ PQ = PR
From (ii) and (iii)
[ sides opposite to equal angles are equal]
AB z x–z
QT QR = =
So, = AC y x
PR QS
z z
Also, ∠Q = ∠Q (common) =1 –
y x
∴ DPQS ~ DTQR z z
+ =1
x y
[ By SAS Similarity criterion]
1 1 1
+ =
x y z
21. In DCBQ and DCAP,
A
22.
∠BCQ = ∠ACP (common)
∠QBC = ∠PAC = 90°
(PA and QB are perpendicular) 2a 2a

∴ DCBQ ~ DCAP (AA Similarity criterion)


BC BQ CQ
= =
AC AP CP B D C
2a
[Corresponding sides of similar triangles are
Draw AD ⊥ BC
proportional]
z In DADB and DADC
BC
= ...(i)
AC x AB = AC = 2a (Given)
In DABQ and DACR, AD = AD (Common)

∠BAQ = ∠CAR (common) ∠ADB = ∠ADC


∠ABQ = ∠ACR = 90° = 90° (By Construction)
ADDITIONAL
R

PR ACTICE MATHEMATICS - 10 97
∴ DADB ≅ DADC (RHS) BC = 9 m
1 In DCDE, CE² = DE² + CD²
∴ BD = DC = BC
2
= a (CPCT) 15² = 9² + CD²

In DADC, right angled at D 225 – 81 = CD²

AC² = AD² + DC² 144 = CD²

(2a)2 = AD² + a² 12 = CD

AD² = 4a² – a² = 3a² So, BD = BC + CD


= 9 + 12 = 21 m
AD = 3 a
So, length of the altitude of an equilateral
triangle = 3 a cm Section-D
23. In DAOB, XY || AB 25. In DXPQ and DXYZ,
OX OY XP XQ

= ...(i) = = 3 (Given)
AX BY PY QZ
[ Basic proportionality theorem ] ∠X = ∠X (Common)
In DAOC, XZ || AC ∴ DXPQ ~ DXYZ
OZ OX (SAS Similarity criterion)

= ...(ii)
ZC AX
[ Basic proportionality theorem ] ar ∆XPQ  XP 
2
PQ
2
 XQ 
2

So, ar ∆XYZ =  = = 
OY OZ  XY  Ł YZ ł  XZ 
By (i) and (ii), =
BY ZC [ Ratio of area of two similar triangles is equal
∴ YZ || BC to square of their corresponding sides]
[By Converse of Basic proportionality theorem] ar ∆XPQ  XQ 
2
3
2

=  =  
A
32  XZ  4
24. E 9
ar DXPQ = × 32  XP = 3 
16  PY 
15  
m
15
m  PY = 1 
12 m 9m  XP 3 
 PY 1 
 +1= +1
 XP 3 
B C D  XY 4 
 XP = 3 
Let AC = CE denotes the ladder
= 18 cm²
In DABC, AC² = AB² + BC²
area of quadrilateral PYZQ
15² = 12² + BC² = ar DXYZ – ar DXPQ
225 – 144 = BC² = 32 – 18 cm²
BC² = 81 = 14 cm²
98 ADDITIONAL
R

PR ACTICE MATHEMATICS - 10
26. ∠BYX = ∠C (Corresponding angles)
A ∴ DABC ~ DXBY (AA Similarity criterion)
2
D ar ∆ABC  AB 
So, ar ∆XBY =  XB  ...(i)
[ Ratio of areas of two similar triangles is equal to
square of ratio of their corresponding sides]
B C Also, ar DABC = 2 ar DXBY
In DABC, right angled at B, 2
ar ∆ABC
i.e. = ...(ii)
We need to prove AC² = AB² + BC² ar ∆XBY 1
Draw BD⊥ AC From (i) and (ii),

We know that if a perpendicular drawn from  AB 


2
2
the vertex of the right angle of a right triangle  XB  = 1
to the hypotenuse then triangles on both sides AB
of the perpendicular are similar to the whole = 2
XB 1
triangle and to each other.
AX 1
So, DCBA and DCDB =
AB 2
CB CA [ Corresponding sides of similar AX 1
= ∴ 1– = 1–
CD CB triangles are proportional] AB 2
CB² = CA x CD ...(i) AB - XB 2 -1
=
AB 2
Also, DABC and DADB
AX 2 −1
AB BC AC =
= = AB 2
AD BD AB
2- 2
AB AC =
= 2
AD AB
AB² = AC × AD ...(ii) 28.
A
From (i) and (ii),

AB² + BC² = AC × AD + AC × CD c
b D
= AC (AD + CD) p

= AC × AC C a B
= AC²
In DACB, right angled at C such that CD ⊥ AB.
∴ AB² + BC² = AC²
We know that if a perpendicular is drawn from
the vertex of the right angle of a right triangle to
27. As XY || AC the hypotenuse then triangles on both sides of the
perpendicular are similar to the whole triangle and to
∠BXY = ∠A (Corresponding angles) each other.
ADDITIONAL
R

PR ACTICE MATHEMATICS - 10 99
So, DBDC ~ DBCA ∠BDE = ∠BAC (Corresponding angles)

∴ BD = DC = BC ∠BED = ∠BCA (Corresponding angles)


BC CA BA
p a ∴ DBDE ~ DBAC (AA Similarity criterion)
i.e.   =
b c Also, E and F are midpoints of sides BC and AC
pc = ab respectively.
ab ∴ EF || AB (Midpoint Theorem)
⇒ p =
c
a2 b2 In DCFE and DCAB,
⇒ p² =
c2 ∠CFE = ∠CAB (Corresponding angles)
1 c2
⇒ 2 = 2 2 ∠CEF = ∠CBA (Corresponding angles)
p ab
In DACB, AC² + BC² = AB² ∴ DCFE ~ DCAB (AA similarity criterion)
Similarly, we can prove DAFD ~ DACB
b² + a² = c²
So, the line segment joining the midpoints of the
1 a2 + b2 sides of a triangle form four triangles, each of
= 2 2
p2 a b which is similar to the original triangle.
1 a2 b2
= 2 2 + 2 2 30. E A B
p2 a b a b
1
1 1
= 1 + 2
p2 a 2 b
3
29. D C
A 2

D F F
Consider DEDA and DEFB
∠1 = ∠1 (Common)
∠3 = ∠4
B E C
[ Corresponding angles as AD || BF ]
Given : In DABC D, E and F are midpoints of sides
AB, BC and AC respectively. ∴ DEDA ~ DEFB (AA Similarity criterion)
DA
As D and E are midpoints of sides AB and BC ∴ = EA
FB EB
respectively.
[ Corresponding sides of similar triangles are
∴ DE || AC (Midpoint Theorem) proportional ]

In DBDE and DBAC, ⇒


DA
=
FB
...(i)
AE BE
100 ADDITIONAL
R

PR ACTICE MATHEMATICS - 10
Consider DEDA and DDFC From (i), (ii), (iii), we get
∠1 = ∠2 (Corresponding angles as BE || CD) AB
= PQ ⇒ AB
= AC
AC PR PQ PR
∠3 = ∠4 (Corresponding angles as AD || BF)
Also, ∠A = ∠P (Given)
∴ DEDA ~ DDFC   (AA Similarity criterion) ∴ DABC ~ DPQR

ED DA EA
∴ = = 32. A
DF FC DC
[Corresponding sides of similar triangles are
proportional]
E
DA EA
i.e. =
FC DC
DA FC
⇒ = ...(ii) B D C
AE CD
From (i) and (ii), DABC is a right triangle, right-angled at B.
DA FB FC ∴ AD² = AB² + BD²
= =
AE BE CD
(By Pythagoras theorem)
31. A P
2
 BC 
1 2 3 4 ⇒ AD² = AB² +   [ ∵ BD = DC ]
 2 
1
⇒ AD² = AB² + BC² ...(i)
4
Also, DBCE is a right triangle, right angled at B.
B D C Q S R
∴ CE² = BC² + BE²
Given: AD and PS are bisectors of ∠A and ∠P 2
respectively. Such that ⇒ CE² = BC² +  AB  [ ∵ BE = EA]
 2 
BD QS
= 1
DC SR ⇒ CE² = BC² + AB² ...(ii)
4
To prove = DABC ~DPQR
On adding (i) and (ii), we get
Proof: In DABC, AD is bisector of ∠A
5
AB AC AD² + CE² = (AB² + BC²)
∴ = 4
BD CD
5
i.e. AB
= BD ...(i) ⇒ AD² + CE² = AC²
4
AC CD
In DPQR, PS is bisector of ∠P [ As DABC is right triangle ∴ AC² = AB² + BC² ]
2
PQ PR 3 5  5
∴ = ⇒   + CE² = (25)
QS RS 2 4
 
PQ QS
i.e. = ...(ii) ⇒ CE² =
125

45
= 20
PR RS
4
BD
Also, = QS ...(iii) ∴ CE = 20 cm = 2 5 cm
DC SR

ADDITIONAL
R

PR ACTICE MATHEMATICS - 10 101


∴ AD ⊥ BC
WORKSHEET - 2
1
Also, DC = AC
SECTION-A 2
[ As AD is the Median ]
1. ∆ABC ~ ∆RPQ
AC² = AD² + CD²
AB BC AC
∴ = = 2
RP PQ RQ
( 3) 1 
2
AC² = +  AC 
[ Corresponding sides of similar triangles are 2 
proportional] 1
AC² = 3 + AC²
4
3 5 6
∴ =
6 10
=
RQ
3
AC² = 3
4
6 ×10 AC² = 4
RQ = = 12 cm
5
AC = 2 cm
2. ∆ABC ~ ∆DEF
ar ∆ABC
2 D C
 AB  4.
∴ =   1 3
ar ∆DEF  DE  x– x+
2
[Ratio of areas of similar triangles is proportional O x–
to the square of ratio of their corresponding 1
x +5
sides] 2
A B
(26 )
2
169
⇒ = In ∆COD and ∆AOB,
121 (DE )
2

∠1 = ∠2
( 26 )
2
×121
⇒ DE² = [ Alternate interior angles as AB || CD]
169
26 ×11 ∠AOB = ∠COD
⇒ DE = = 22 cm
13 [Vertically opposite angles]
3. A
∴ ∆COD ~ ∆AOB
CO OD CD
∴ = =
AO OB AB

3 cm [Corresponding sides of similar triangles are


proportional]
x +3 x-2
=
B D C x +5 x -1
∆ABC is equilateral and AD is the median such ⇒ (x + 3) (x – 1) = (x – 2) (x + 5)
that AD = 3 cm.
⇒ x² + 2x – 3 = x² + 3x – 10
In an equilateral triangle, median and altitude
are same. ⇒ 7 = x
102 ADDITIONAL
R

PR ACTICE MATHEMATICS - 10
5. In DSPT and DQPR, AD DE
⇒ =
AB BC
∠PST = ∠PQR
1.5 DE
[ Corresponding angles as ST || QR ] ⇒ =
6 8
∠PTS = ∠PRQ 1.5 × 8
⇒ DE = = 2 cm
6
∴ DSPT ~ DQPR
7. As MN || AB,
[AA Similarity criterion]
CM CN
ar ∆PST  PT 
2 =
∴ =   AM BN
ar ∆PQR  PR  [Basic proportionality theorem]
[Ratio of areas of two similar triangles is equal to 2 BC - BN
square of ratio of their corresponding sides] =
2 4 BN
 PT 
=  1 7.5 - BN
 PT + TR  =
2 BN
 2 
2
∴ BN = 15 – 2BN
= 
 2 + 4  ⇒ 3BN = 15
2   BN = 5 cm
2
=  
6 8. We know that ratio of area of two similar
1 triangles is equal to square of ratio of their
= corresponding sides.
9
So, Ratio of corresponding sides
6. DE || BC
25 5
∴ AD = AC =
64
=
8
BD CE
(Basic proportionality theorem) 9. DE || BC
BD CE
⇒ = AD AE
AD AE ⇒ =
DB CE
BD CE
⇒ +1 = +1 [ Basic proportionality theorem ]
AD AE
BD + AD CE + AE DB CE
⇒ = ⇒ +1 = +1
AD AE AD AE
AB AC AB AC
= ⇒   =
AD AE
AD AE
Also, ∠A = ∠A (Common)
Also, ∠A = ∠A (Common)
∴ DADE ~ DABC ∴ DADE ~ DABC
AD DE AE (SAS Similarity criterion)
⇒ = =
AB BC AC 2
[ Corresponding sides of similar triangles are ar D ADE  DE 
∴ = 
proportional] ar D ABC  BC 

ADDITIONAL
R

PR ACTICE MATHEMATICS - 10 103


2 12. A
2 
 3 BC 
ar D ADE
=

5 3m
81 BC 2
ar ∆ADE 4
=
81 9
4
ar DADE = × 81 = 36 cm² B 5m C
9
In DABC, right angled at B
10. Consider AC² + BC²
AC² = AB² + BC2
= AC² + AC² (∵ AC = BC)
( )
2
+ (5)
2
= 2AC² = 5 3

= AB² = 75 + 25

∴ DABC is right angled triangle = 100


∴ AC = 10 m
[ As we know that in a triangle, if square of one
side is equal to sum of the squares of other two
13. A
sides then the angle opposite the first side is a
right angle. ]

Section-B D E

11. D
cm
2.5

R B C

As DE || BC,
m
5c

AD AE
E 1.5 cm S 3.5 cm F =
DB CE
ER 5 BD CE
Consider, = =
RD 2.5 AD AE
= 2 BD CE
⇒ +1= +1
ES 1.5 3 AD AE
Also, = = AB AC
SF 3.5 7 ⇒ =
ER ES AD AE
As, ≠
RD SF AD AE
⇒ = ...(i)
AB AC
∴ RS is not parallel to DF
Also, ∠A = ∠A (Common)
[ As we know that if a line divides any two sides of a
triangle in the same ratio then the line is parallel to ∴ DADE ~ DABC (SAS Similarity criterion)
the third side ]

104 ADDITIONAL
R

PR ACTICE MATHEMATICS - 10
AD DE AE ∴ DABC is a right angled triangle
∴ = =
AB BC AC [ As we know that in a triangle if square of one side is
[ Corresponding sides of similar triangles are equal to the sum of squares of other two sides, then
proportional ] the angle opposite the first side is a right angle i.e.
triangle is right angled ]
AD AE
⇒ = A
AD + BD AC 15.

AD 4.5
⇒ =
AD + 3 AD AC

AD 4.5
⇒ =
4 AD AC

⇒   AC = 4.5 × 4
B D C
= 18 cm Draw AD ⊥ BC
AD AE In DADB and DADC
Also, = ( From (i) )
AB AC
AD = AD (Common)
AD AE
= AB = AC (DABC is equilateral)
AD + BD 18
AD AE ∠ADB = ∠ADC = 90° (By Construction)
=
AD + 3 AD 18
∴ DADB ≅ DADC (RHS)
1 AE
   =
4 18 1 1
⇒ CD = BC = 3 3 cm [CPCT]
18 9 2 2
 AE = = = 4.5 cm
4 2 In DADC,
14. Consider DABC with sides as AC² = AD² + CD²
AB = (a – 1) cm 2

(3 3 ) 3 3
2

BC = (2 a ) cm = AD² +
Ł 2 ł
AC = (a + 1) cm
27
AD² = 27 –
Consider AB² + BC² 4

( ) 108 − 27
2
( a −1)
2
= + 2 a =
4
= a² + 1 – 2a + 4a
81
=
= a² + 2a + 1 4
( a + 1)
2
= 9
∴ AC = = 4.5 cm
= AC² 2

ADDITIONAL
R

PR ACTICE MATHEMATICS - 10 105


16. F ∠BAP = ∠CDP = 90° (Given)
A ∠APB = ∠DPC (Vertically
opposite angles)
∴ DAPB ~ DDPC (AA similarity criterion)
AP PB AB
∴ = =
DP PC DC
(Corresponding sides of similar triangles are
B D C E proportional)
⇒ AP × PC = BP × PD
BD CD
To prove = =
BE CE
18. Consider DQPM and DRSM
As AD bisects ∠BAC,
∠QPM = ∠RSM = 90°

AB AC
= [Interior angle bisector theorem] ∠QMP = ∠RMS (Vertically opposite angles)

BD CD
CD AC ∴ DQPM ~ DRSM (AA similarity criterion)
∴ = ...(i)
BD AB QP PM QM
∴ = =
Also, AE bisects ∠CAF RS SM RM
[ Corresponding sides of similar triangles are
BE CE
∴ = proportional ]
AB AC
BE AB PM QM
⇒ = i.e.
=
CE AC SM RM
CE AC 3 QM
⇒ = ...(ii)   =
BE AB 4 6
From (i) and (ii) 3×6 3×3
    QM = = = 4.5 cm
4 2
CD CE
  =
BD BE A
BD CD 19.
⇒ =
BE CE

17. A D

P
B D C

B C AD bisects ∠A. So, by Interior angle bisector


theorem,
To prove: AP × PC = BP × PD AB BD
=
Consider, DAPB and DDPC AC DC

106 ADDITIONAL
R

PR ACTICE MATHEMATICS - 10
AB BD [Corresponding sides of similar triangles are
= =1 proportional]
AC DC
[ ∵ BD = DC as D is a midpoint of BC] DE AD
∴   =
  AB = AC BC AB
∴ DABC is isosceles. DE 7.6
⇒ =
AE + BE
8.4
20. A B DE 7.6
⇒ =
7.2 + 4.2
8.4
3 DE7.6
⇒  
=
1 O 11.4
8.4
7.6 × 8.4
⇒   DE =
11.4
D C
= 5.6 cm
Here, AC divides the diagonal BD in the ratio 1 : 3.
Consider DAOB and DCOD
22. In DABC, LM || CB,
∠BAO = ∠DCO AM AL
∴ = ...(i)
(Alternate interior angles as AB || CD) BM CL
[ Basic proportionality theorem ]
∠AOB = ∠COD (Vertically opposite
angles) In DADC, LN || CD.
∴ DAOB ~ DCOD (AA similarity criterion) AN AL
∴ = ...(ii)
AO OB AB DN CL
∴ = = [ Basic proportionality theorem ]
CO OD CD
[Corresponding sides of similar triangles are
AM AN
proportional] From (i) and (ii), =
BM DN
OB AB BM DN
⇒ = ⇒ =
OD CD AM AN
3 AB BM DN
⇒   = ⇒ +1 = +1
1 CD AM AN
⇒ AB = 3CD AB AD
⇒ =
AM AN
Section-C ⇒ AM × AD = AB × AN

21. In DADE and DABC, 23.


∠A = ∠A (Common) A P

∠ADE = ∠ABC (Given)

∴ DADE ~ DABC (AA similarity criterion)

AD DE AE
⇒ = =
AB BC AC B D C Q M R
ADDITIONAL
R

PR ACTICE MATHEMATICS - 10 107


In DABD and DPQM, AD² = DE² + AE² ...(i)
Now, DE = BE – BD
AB AD BC
  = = (Given)
PQ PM QR a a 1 a
= – [ ∵ BD = = ]
2 4 4 4
AB AD BD
⇒ = = a
PQ PM QM = ...(ii)
4
1
BC In DAEC,
AB AD 2
⇒ = =
PQ PM 1 AC² = AE² + CE²
QR
2 2
a
(As AD and PM are the medians)   a² = AE² +
Ł2 ł
∴ DABD ~ DPQM a 2
3a 2
AE² = a² – = ...(iii)
(SSS similarity criterion) 4 4
On putting (ii), (iii) in (i), we get
∴ ∠B = ∠Q
2
a 3a 2
[ Corresponding angles of similar triangles are  AD² = +
equal] Ł4 ł 4
2 2
a 3a
Now, in DABC and DPQR, = +
16 4
AB BC a + 12a 2
2

= (Given) =
PQ QR 16
∠B = ∠Q (Proved) 13a 2
=
16
∴ DABC ~ DPQR (SSS similarity criterion) 16 AD² = 13 a²
24. 16AD² = 13 BC²
A
25. As DABC is isosceles,
AB = AC
∴ ∠B = ∠C

B D E C (Angles opposite to equal sides are equal)

Let AB = BC = AC = a In DADB and DEFC,

∠ADB = ∠EFC
BC a
∴ BD = =
4 4 (As EF ⊥ AC and AD ⊥ CB)
Draw AE ⊥ BC
∠B = ∠C (Proved)
a
∴ BE = EC = ∴ DADB ~ DEFC (AA similarity criterion)
2
[ In equilateral triangle altitude is same as median] AD BD AB
∴ = =
In right angled triangle DAED, EF FC EC
108 ADDITIONAL
R

PR ACTICE MATHEMATICS - 10
AD AB 27. As DNSQ ≅ DMTR,
i.e. =
EF EC ∠NQS = ∠MRT (CPCT)
⇒ AD × EC = AB × EF
⇒ PQ = PR ...(i)
26. In DABC and DADE, (Sides opposite to equal angles are equal)
∠A = ∠A (Common) Also, as ∠1 = ∠2
∠ACB = ∠AED = 90° ∴ PS = PT ...(ii)
(As DE ⊥ AB and DABC is right angled at C) (Sides opposite to equal angles are equal)
∴ DABC ~ DADE On subtracting (ii) from (i), we get
(By AA similarity criterion) PQ – PS = PR – PT
AB BC AC QS = TR ...(iii)
⇒ = =
AD DE AE
[ Corresponding sides of similar triangles are From (ii) and (iii),
proportional ] PS PT PS PT
= ⇒ =
In DABC, ∠C = 90° QS TR PQ PR
Also, ∠P = ∠P (Common)
∴ AB² = AC² + BC² [By Pythagoras theorem]
∴ DPST ~ DPQR (SAS similarity criterion)
= (3 + 2)2 + 122
= 25 + 144
Section-D

= 169 28.
C
∴ AB = 13 cm E
AB BC
As = F
AD DE
13 12 D
∴ =
3 DE
12 × 3 36 B G A
∴ DE = = cm In DAFG and DDBG,
13 13
BC AC ∠AGF = ∠DBG
Also, =
DE AE
(Corresponding angles as GF || BC)
12 5
⇒ 36 = ∠GAF = ∠BDG = 90° (∵ DEFG is a square)
AE
13
∴ DAFG ~ DDBG ...(i)
12 ×13 5
⇒ = (AA similarity criterion)
36 AE
5 × 36 In DAGF and DEFC,
⇒ AE =
12 ×13
∠FAG = ∠CEF = 90°
15
= cm
13 ∠AFG = ∠ECF
ADDITIONAL
R

PR ACTICE MATHEMATICS - 10 109


(Corresponding angles as GF || BC) B
30.
∴ DAGF ~ DEFC ...(ii)
(AA similarity criterion) M
From (i) and (ii), we get,
DDBG ~ DEFC C L A

BD DG In DABC, BC² = AB² + AC²


⇒ =
EF EC (By Pythagoras theorem)
In DABL, BL² = AB² + AL²
[
Corresponding sides of similar triangles are
2
proportional] 1
  = AB² + AC
BD DE Ł2 ł
⇒ = 1
DE EC [As L is a midpoint of AC ∴ AL = AC ]
2
[ As DEFG is a square, EF = DE and DG = DE ] AC 2
  BL² = AB² +
⇒ DE² = BD × EC 4
4BL² = 4AB² + AC² ...(i)
In DCMA, CM² = AC² + AM²
29. In DAOD, MO bisects ∠AOD, 2
1 
So, by interior angle bisector theorem, = AC² +  AB 
2 
AO AM AB 2
= ...(i) = AC + 2
OD DM 4
1
In DBOC, NO bisects ∠BOC, [As M is a midpoint of AB ∴ AM = AB ]
2
⇒ 4 CM² = 4AC² + AB² ...(ii)
So, by interior angle bisector theorem,
  From (i) and (ii), we get
BO BN 4(BL² + CM²) = 5 AB² + 5 AC² = 5 BC²
=
CO CN A
31.
CO CN
⇒ = ...(ii)
BO BN
AO D
We know that AO = OD ⇒ =1
OD
CO
and CO = BO ⇒ =1
BO
(Radii of same circle) C E B

So, from (i) and (ii), we get To prove : AE² + BD² = AB² + DE²
Proof: In DACE, AE² = AC² + CE² ...(i)
AM CN
= (By Pythagoras theorem)
DM BN

110 ADDITIONAL
R

PR ACTICE MATHEMATICS - 10
In ∆DCB, BD² = DC² + BC² ...(ii) 6
= 3
(By Pythagoras theorem) BP
6
In ∆ABC, AB² = AC² + BC² ...(iii) BP = cm
3
(By Pythagoras theorem)
(ii) (a) In ∆RQC
In ∆DCE, DE² = DC² + CE² ...(iv) RQ
= tan 60
(By Pythagoras theorem) QC
Consider AE² + BD² 6
= tan 60
QC
= AC² + CE² + DC² + BC² [from (i) and (ii)]
6
= (AC² + BC²) + (CE² + DC²) QC = cm
3
= AB² + DE² [from (iii) and (iv)] BC = BP + PQ + QC
6 6
= +6+
3 3
cASe StUDY-1
 12 
=  + 6  cm
1 th  3 
(i) (d) ∆ABC is time of ∆PQR as the scale
10 12 × 3
factor of ∆ABC to ∆PQR is 1:10. = +6
3
∴ The sides PQ is 10(AB) or 30 cm
= 4 3 + 6 cm
Similarly QR is 40 cm and PR is 50 cm.
3 2
(ii) (d) PQ2 + QR2 = PR2 [Pythagores theoram] (iii) (b) Area of equilateral triangle is a , where
a is the side of triangle. 4
(iii) (d) Perimeter of ∆PQR is the sum of all sides.
∴ Perimeter = 30 cm + 40 cm to 50 cm 3
( )
2
Area = 4 3+6
= 120 cm 4

(iv) (b) Area of ∆PQR is


1
2
(PQ) (QR) =
4
3
(4 + 36 + 48 3 )
1 1
2
(PQ) (QR) = (40 (30)
2 = (
4
3
)
84 + 48 3 cm2

= 600 cm2 = 3 ( 21 +12 3 ) cm 2

(v) (d) ∆ABC and ∆PQR are similar,


QC QR
= (36 + 21 3 ) cm 2

=
AC RP (iv) (c) Area of square = (side)2
BC AC Side = 6 cm
= Area of square = (6)2
QR PR
= 36 cm2
cASe StUDY-2 Area of ∆ABC 36 + 21 3
(v) (c) =
(i) (d) In ∆BSP, ∠SBP = 60, SP = 6 cm Area of square PQRS 36
SP 12 + 7 3
= tan 60 =
BP 12
ADDITIONAL
R

PR ACTICE MATHEMATICS - 10 111


Chapter

7 Coordinate Geometry

Multiple choice Questions

1. (b) A 4. (d)
(4, 9)
A
D (0, 1)
(x, y)

(0, 0) O B (1, 0)

B (2, 3) (6, 5) C In ∆AOB,


AB² = AO² + OB²
 4 + 2 9 + 3
D(x, y) =  , = (3, 6) = 1² + 1²
 2 2 
= 2
AB =
( 6 - 3) + ( 5 - 6 )
2 2
So, CD = 2
Perimeter = AO + OB + AB
= 9 +1 = 1+1+ 2

= 10 units WoRKSHeet - 1

Section-A
2. (c) As A, B and C are collinear  x1 + x 2 + x3 y1 + y 2 + y 3 
1. Centroid =  , 
∴ x (– 4 + 5) – 3 (– 5 – 2) + 7 (2 + 4) = 0  3 3
4 - 9 + x3 - 3 + 7 + y3
x + 21 + 42 = 0 (1, 4) = ,
Ł 3 3 ł
x = – 63  −5 + x3 4 + y 3 
(1, 4) =  ,
 3 3 
−5 + x3 4 + y3
 6 − 2 −5 +11 =1 =4
3. (b) (2, p) =  , 3 3
 2 2 
x3 – 5 = 3 y3 + 4 = 12
= (2, 3) x3 = 8 y3 = 8
⇒ p = 3 So, third vertex is (8, 8).

112 ADDITIONAL
R

PR ACTICE MATHEMATICS - 10
2. k B (0, y) 1 ( 3 - 0 )2 + (P - 2 )2 = (P - 0 )2 + ( 5 - 2 )2
X Y 9 + (P - 2 )2 =
(a1, b1) (a2, b2) P2 + 9
(P – 2)2 = P2
Let the ratio be k : 1
P2 + 4 – 4P = P2
 ka2 + a1 kb2 + b1 
So, (0, y) =  ,
 k +1 k +1    4P = 4
ka2 + a1    P = 1
=0
k +1
 ka2 + a1 = 0 6. ( 4 - 1)2 + (K - 0 )2 = 5
 ka2 = – a1 32 + K 2 =5
−a1 On squaring both sides, we get
 k =
a2 9 + K² = 25
K² = 25 – 9 = 16
2
2 8
 5 + 5  + (2 − 2 )
3. Distance =
2 K² = 16
  K =±4
2
2 8 7.
=  5 + 5  + 0
A (1,2) B (4,3)
2
2 + 8
=  5 

= 2 sq. units
D (x,y) C (6,6)
4. Let point on y - axis be (0, y). We know that diagonals of a parallelogram
bisect each other
(6 − 0 ) + (5 − y ) =
2 2

Ê 1+ 6 2 + 6 ˆ Ê 4 + x 3 + y ˆ
( −4 − 0 ) + (3 − y )
2 2
∴Á , = ,
Ë 2 2 ˜¯ ÁË 2 2 ˜¯
36 + 25 + y 2 - 10 y = 16 + 9 + y 2 - 6 y Ê7 ˆ Ê 4 +x 3+y ˆ
Á 2 , 4 ˜   = Á 2 , 2 ˜
61 + y 2 - 10 y = 25 + y 2 − 6 y Ë ¯ Ë ¯
7 4+x 3+y
61 + y² – 10y = 25 + y² – 6y ∴ = and 4 =
2 2 2
36 = 4y
  7 = 4 + x and 8 = 3 + y
y =9
x = 3 and y = 5
So, point on y - axis which is equidistant
So, coordinates of fourth vertex
from point A (6, 5) and B (– 4, 3) is (0, 9).
= (x, y)
5. As point A (0, 2) is equidistant from the
= (3, 5)
points B (3, P) and C (P, 5). So,
ADDITIONAL
R

PR ACTICE MATHEMATICS - 10 113


8. 10.

P Q

A C B
Ê3 5ˆ A (u, v) (x, y) B
(x, 4) Á 2 ,2˜ (x +1, y + 2)
Ë ¯ (2, – 2) (– 7, 4)
As C is a midpoint of AB,
Point P divides AB in ratio 1: 2
Ê x + x +1 4 + y + 2 ˆ Ê 3 5 ˆ
Á 2
,
2 ˜ = Á 2 ,2˜ Ê 1( - 7 ) + 2 ( 2 ) 1( 4 ) + 2 ( - 2 ) ˆ
Ë ¯ Ë ¯
So, P (u,v) = ÁÁ , ˜
˜
Ê 2 x +1 y + 6 ˆ Ê 3 5 ˆ Ë 3 3 ¯
Á 2 , 2 ˜ = Á 2 ,2˜
Ë ¯ Ë ¯ Ê - 7 +4 4 - 4 ˆ
= Á ,
Ë 3 3 ˜¯
2 x +1 3 y +6 5
∴ = and = = (– 1, 0)
2 2 2 2
2x + 1 = 3 and y + 6 = 5 Point Q divides AB in ratio 2 : 1
2x = 2 and y = 5 – 6
Ê 2 ( - 7 ) +1( 2 ) 2 ( 4 ) +1( - 2 ) ˆ
x = 1 and y – 1 So, Q (x, y) = ÁÁ , ˜
˜
Ë 3 3 ¯
Ê - 14 + 2 8 - 2 ˆ
Section-B =Á ,
Ë 3 3 ˜¯
9. Let y – coordinate be v.
Ê - 12 6 ˆ
=Á , ˜
∴ x – coordinate = 2v Ë 3 3¯
So, point P is (2v, v). = (– 4, 2)

  PQ = PR 11.
A (3, 0)
( 2 - 2v )2 + ( - 5 - v )2 = ( - 3 - 2v )2 + ( 6 - v )2
On squaring both sides, we get
  (2 – 2v)² + (– 5 – v)² = (–3 – 2 v)2 + (6 – v)²
∴ 4 + 4 v² – 8v + 25 + v² + 10v
B (6,4) C (–1,3)
  = 9 + 4v² + 12v + 36 + v² –12v
⇒ 5v² + 2v + 29 = 5v² + 45 AB = ( 6 - 3 )2 + ( 4 - 0 )2
⇒ 2v = 45 –29 = 9 +16
2v = 16 = 25
v=8 =5
So, point P is (2v, v) i.e. (16, 8).
BC = ( - 1 - 6 )2 + ( 3 - 4 )2

114 ADDITIONAL
PR ACTICE
R

MATHEMATICS - 10
= 13.
49 +1
= 50 = 5 2 A (a,a)

AC = ( - 1 - 3 )2 + ( 3 - 0 )2
= 16 + 9
= 25
=5
B C
As AB = AC, ∆ABC is isosceles (– a, – a)
(- 3a, 3a )
Also,
AB² + AC² = 5² + 5²
AB = ( - a - a )2 + ( - a - a )2
= 25 + 25
= 4a2 + 4a2
= 50
= BC² = 8a 2

∴ ∠A = 90o = 2 2a units
∴ ∆ABC is an isosceles triangle right angled
at A. BC = ( - 3a + a )2 + ( 3a + a )2

12. = 3a 2 + a 2 - 2 3a 2 + 3a 2 + 2 3a 2 + a 2

= 8a 2
A P B
Ê1 3ˆ Ê3 5 ˆ (2, -5) = 2 2a units
Á 2 ,2˜ Á 4 ,12 ˜
Ë ¯ Ë ¯ AC = ( - 3a - a )2 + ( 3a - a )2
Let point P divides AB in ratio k : 1 = 3a 2 + a 2 + 2 3a 2 + 3a 2 + a 2 - 2 3a 2
Ê 1 3ˆ
Ê 3 5 ˆ Á 2 k + 2 - 5k + 2 ˜ = 3a 2 + a 2 + 3a 2 + a 2
So, Á , ˜ = Á , ˜
Ë 4 12 ¯ ÁÁ k +1 k +1 ˜
˜ = 8a 2
Ë ¯
1 3 = 2 2a units
3 2k + 5 - 5k + As AB = BC = CA, ∆ABC is an equilateral
2 2
= = triangle.
4 k +1 12 k +1

⇒ 3k + 3 = 8k + 2 5k + 5 = – 60k + 18 14.

⇒1 = 5k 65k = 13 k 1
1 1
k = k =
5 5 A (1, –3) C (x, 0) B (4, 5)
So, point P divides AB in ratio 1 : 5 Let point C (x, 0) divides AB in ratio k : 1
ADDITIONAL
R

PR ACTICE MATHEMATICS - 10 115


So,
PQ = ( 7 - 2 )2 + ( 0 - 4 )2
Ê 4 k +1 5k - 3 ˆ
(x, 0) =Á , ˜
Ë k +1 k +1 ¯ = 52 + ( - 4 )2
5k - 3
∴ =0
k +1 = 25 +16
5k – 3 = 0 = 41
3
 k =
5 Section-C
So, x – axis divides the line segment joining the
points (1, – 3) and (4, 5) in ratio 3 : 5. 17. Let y – axis divides the line segment joining
the points (– 4, – 6) and (10, 12) in ratio k : 1
15. ( 9 - x )2 + (10 - 4 )2 = 10
Point on y – axis must be of form (0, y)
81 + x² – 18x + 36 = 100
k 1
x² – 18x + 17 = 0
x² – 17x – x + 17 = 0 A C B
(– 4, – 6) (0, y) (10, 12)
x (x – 17) – 1 (x –17) = 0
Ê 10 k + ( - 4 ) 12 k - 6 ˆ
(x – 1) (x – 17) = 0 (0, y) = Á , ˜
Á k +1 k +1 ˜¯
x = 1, 17 Ë
Ê 10 k - 4 12 k - 6 ˆ
(0, y) = Á ,
16.
Ë k +1 k +1 ˜¯
10 k - 4
⇒ =0
k +1
⇒ 10k =4
Q P R 2
(7, 0) (2, 4) (x, 9) k =
5
PQ = PR So, ratio is 2 : 5.

⇒ ( 2 - 7 )2 + ( 4 - 0 )2 = ( x - 2 )2 + ( 9 - 4 )2 18. A (0, – 1) B (–2, 3)


⇒ 25 +16 = x 2 + 4 - 4 x + 25

⇒ 41 = x 2 - 4 x + 29
On squaring both sides, we get
41 = x² – 4x + 29 D (8, 3) C (6, 7)

0 = x² – 4x – 12 AB = ( - 2 - 0 )2 + ( 3 +1)2
0 = x² – 6x + 2x – 12
= 4 +16
0 = x (x – 6) + 2 (x – 6)
= 20
0 = (x + 2) (x – 6)
x = – 2 or 6 = 2 5 units
116 ADDITIONAL
R

PR ACTICE MATHEMATICS - 10
As point P is equidistant from A and B,
CD = ( 6 - 8 )2 + ( 7 - 3 )2
AP = BP
= 4 +16
( x + 3 )2 + ( y - 2 )2 = ( 4 - x )2 + ( - 5 - y )2
= 20
On squaring both sides, we get
= 2 5 units (x + 3)² + (y – 2)² = (4 – x)² + (– 5 – y)²
∴ AB = CD x² + 9 + 6x + y² + 4 – 4y
AD = ( - 8 - 0 )2 + ( 3 +1)2 = 16 + x² – 8x + 25 + y² + 10y
14x – 14y + 13 = 41
= 64 +16 = 80 = 4 5 units
14x – 14y – 28 = 0
BC = ( 6 + 2 )2 + ( 7 - 3 )2
x–y=2
= 64 +16
∴ y=x–2
= 80 20.
= 4 5 units A P Q B
(3, – 4) (p, – 2) Ê5 ˆ (1, 2)
∴ AD = BC Á 3 ,q ˜
Ë ¯
As AB = CD and AD = BC,
Point P divides AB in ratio 1 : 2.
So, ABCD is a parallelogram
So,
AC = ( 6 - 0 )2 + ( 7 +1)2 Ê 1(1) + 2 ( 3) 1( 2 ) + 2 ( - 4 ) ˆ
P (p, – 2) = ÁÁ , ˜
˜
Ë 3 3 ¯
= 36 + 64
Ê7 ˆ
= 100 P (p, – 2) = Á , - 2 ˜
Ë3 ¯
= 10 units 7
∴p=
( 8 + 2 )2 + ( 3 - 3 )2 3
BD = Point Q divides AB in ratio 2 : 1.
= 100
So,
= 10 units
Ê 5 ˆ Ê 2 (1) +1( 3) 2 ( 2 ) +1( - 4 ) ˆ
So, AC = BD Q Á ,q ˜ = ÁÁ , ˜
˜
Ë3 ¯ Ë 3 3 ¯
∴ ABCD is a parallelogram in which both
Ê5 ˆ Ê5 ˆ
diagonals are equal. Q Á ,q ˜ = Á , 0 ˜
Ë3 ¯ Ë3 ¯
So, ABCD is a rectangle.
∴  q =0
19.
21. As the points A (3p + 1, p), B (p + 2, p – 5)
A B and C (p + 1, – p) are collinear,
P
(– 3, 2) (x, y) (4, – 5) area of ∆ABC = 0
ADDITIONAL
R

PR ACTICE MATHEMATICS - 10 117


1 1
i. e. [(3p + 1) (p – 5 + p) + (p + 2) (– p – p) = [–3 (– 5) + 20]
2 2
+ (p + 1) (p – p + 5)] = 0 1
= [15 + 20]
⇒ [(3p + 1) (2p – 5) – 2p (p + 2) + 5 (p + 1)] = 0 2
35
⇒ [6p² – 15p + 2p – 5 – 2p² – 4p + 5p + 5] = 0 = sq. units
2
⇒ [4p² – 12p] = 0 Area of ∆ABC
1
\ p = 0, 3 = [(–3) (– 4 + 1) – 2 (– 1 + 1) + 4 (–1+4)]
2
22. 1
= [–3 (–3) – 2 (0) + 4 (3)]
2
A P B 1 21
= [9 + 12] = sq. units
(–2, – 2) (2, – 4) 2 2
So, area of quadrilateral ABCD
3
AP =AB = area of ∆ACD + area of ∆ABC
7
3 35 21
⇒ AP = (AP + BP) = +
7 2 2
56
⇒ 7 AP = 3 AP + 3 BP =
2
⇒ 4 AP = 3 BP = 28 sq. units

AP 3 24. Let y – axis divides the line segment joining


⇒ =
PB 4 points A (5, – 6), B (– 1, – 4) in ratio k : 1.
Let point P be (x, y), using section formula, Point C on y – axis is of form (0, y).
Ê 3(2) + 4 ( - 2) 3( - 4 ) + 4 ( - 2) ˆ k 1
  (x, y) = ÁÁ , ˜
˜
Ë 7 7 ¯ A C B
Ê 6 - 8 - 12 - 8 ˆ (5, – 6) (0, y) (–1, – 4)
= Á ,
Ë 7 7 ˜¯ By section formula,
Ê - 2 - 20 ˆ Ê - k +5 - 4 k - 6 ˆ
= Á , ˜ ,
Ë 7 7 ¯ (0, y) = Á ˜
Ë k +1 k +1 ¯

23. A (– 3, – 1) B (– 2, – 4) - k +5
0 =
k +1
k =5
So, y – axis divides AB in ratio 5 : 1
- 4k - 6
D ( 3, 4) C (4, – 1) Also, y =
k +1
Join AC - 20 - 6
=
5 +1
Area of ∆ACD
1 - 26
= [–3 (– 1 – 4) + 4 (4 + 1) + 3 (–1+1)] =
2 6
118 ADDITIONAL
PR ACTICE
R

MATHEMATICS - 10
- 13 Ê 9 17 ˆ
= (x, y) = Á ˜
3 Ë3 3 ¯
- 13
So, C (0, y) = (0, ) Ê 17 ˆ
3 = Á 3, ˜
Ë 3¯
Section-D For coordinates of E
25. Consider points (x1,y1) = (t, t –2), 2
1
(x2,y2) = (t + 2, t – 2) and
A E B
(x3,y3) = (t + 3, t) (x, y) (7, 2)
(4, 6)
Area of triangle
By using section formula,
1
= [x (y – y ) + x2 (y3 – y1) + x3 (y1 – y2)]
2 1 2 3 Ê 1( 7 ) + 2 ( 4 ) 1( 2 ) + 2 ( 6 ) ˆ
1 (x, y) = Á , ˜
= [t (t – 2 – t) + (t + 2) (t – t + 2) + (t + 3) Á 3 3 ˜
Ë ¯
2
(t – 2 – t + 2)] Ê 7 + 8 2 +12 ˆ
1 =Á ,
= [t (– 2) + (t + 2) (2)] Ë 3 3 ˜¯
2
1 Ê 14 ˆ
= [– 2t + 2t + 4] = Á 5, ˜
2 Ë 3¯
1
= (4) A
2 (4, 6)
= 2 sq. units
So, area of triangle is independent of t.

26. AD AE 1
  = =
AB AC 3
D E
AB AC 3
⇒ = = Ê 17 ˆ Ê 14 ˆ
AD AE 1 Á 3, 3 ˜ Á 5, 3 ˜
AB AC Ë ¯ Ë ¯
⇒ –1= –1=3–1
AD AE
ar ∆ADE
BD CE
⇒ = =2
AD AE
1 Ê 17 14 ˆ Ê 14 ˆ Ê 17 ˆ
AE = 4Á - ˜ + 3 Á - 6˜ + 5 Á6 -
3 ˜¯
AD 1
∴ = = 2 Ë 3 3¯ Ë 3 ¯ Ë
BD CE 2
For coordinates of D.
1 2 1 Ê 3ˆ Ê 14 - 18 ˆ Ê 18 - 17 ˆ
= 4 Á ˜ +3 Á ˜ +5 Á ˜
2 Ë 3¯ Ë 3 ¯ Ë 3 ¯
A D B
(4, 6) (x, y) (1, 5)
1 5
= 4 + (– 4) +
By using section formula, 2 3
Ê 1(1) + 2 ( 4 ) 1( 5 ) + 2 ( 6 ) ˆ 5
(x, y) = ÁÁ , ˜
˜ = sq. units
Ë 3 3 ¯ 6
ADDITIONAL
R

PR ACTICE MATHEMATICS - 10 119


27. y x² = 27

B (0, 3) x = ±3 3
∴ Coordinates of point A are ( - 3 3 , 0 ) .
As BACD is a rhombus and diagonals of
D A (x, 0) rhombus bisect each other. So, OD = OA
x1 o x
= 3 3 units
(0,0)
∴ Point D is ( - 3 3 , 0 )
C (0, – 3)
28. Area of triangle = 5 sq. units

y1 As third vertex lies on y = x + 3, so, it must be


of form (x, x + 3).
Let coordinates of B be (0, y).
Let (x1,y1) = (2, 1)
As (0, 0) is a midpoint of BC,
(x2,y2) = (3, – 2)
Ê 0 +0 y - 3ˆ
∴ (0, 0) = Á ,
2 ˜¯
(x3,y3) = (x, x + 3)
Ë 2
Ê 0 y - 3ˆ Area of triangle
(0, 0) = Á , ˜
Ë2 2 ¯ 1
= [ x1(y2 – y3) + x2(y3 – y1) + x3 (y1 – y2)]
Ê y - 3ˆ 2
(0, 0) = Á 0 ,
Ë 2 ˜¯ 5=
1
[ 2 (– 2 – x – 3) + 3 (x + 3 – 1) + x (1 + 2)]
y- 3 2
=0
2 10 = [ 2 (– 5 – x) + 3 (x + 2) + 3x ]
y = 3
So, point B is (0, 3). 10 = [ – 10 – 2x + 3x + 6 + 3x ]

Let coordinates of point A be (x, 0). 10 = [ 4x – 4 ]


Using distance formula, ∴ ± 10 = 4x – 4
AB = ( x - 0 )² + ( 0 - 3 )²
4x – 4 = 10 4x – 4 = – 10
= x² + 9
4x = 14 4x = – 6
BC = ( 0 - 0 )² + ( - 3 - 3 )²
7 -3
x= x=
= 36 2 2
So, third vertex is So, third vertex is
=6
(x, x + 3) (x, x + 3)
As ∆ABC is equilateral,
Ê7 7 ˆ Ê –3 –3 ˆ
AB = BC = Á , +3˜ = Á , +3˜
Ë2 2 ¯ Ë 2 2 ¯
i.e. x² + 9 = 6
Ê 7 13 ˆ Ê –3 3 ˆ
x² + 9 = 36 =Á , ˜ =Á , ˜
Ë2 2 ¯ Ë 2 2¯

120 ADDITIONAL
R

PR ACTICE MATHEMATICS - 10
29. Let (x1, y1) = (a, a²) (x3, y3) = (– 4, – 3)
(x2, y2) = (b, b²) area of ∆BCD
(x3, y3) = (c, c²) 1
= [ x1 (y2 – y3 ) + x2 (y3 – y1) + x3 (y1 – y2) ]
2
Consider, area of triangle
1
= [ 2 (2 + 3) + (– 3) (– 3 – 3) – 4 (3 – 2) ]
1 2
= [ x1 (y2 – y3 ) + x2 (y3 – y1) + x3 (y1 – y2) ]
2
1
1 = [ 10 + 18 – 4 ]
= [ a (b² – c² ) + b (c² – a²) + c (a² – b² ) ] 2
2
1 = 12 sq. units
= [ ab² – ac² + bc² – a²b + a²c – b²c ]
2 Area of parallelogram ABCD
1
= [ ab (b – a) + ac (a – c) + bc (c – b)] = area of ∆ABC + area of ∆BCD
2
Here, it is clear that area of triangle is 0 if = 12 + 12
a = b = c but it is given that a ≠ b ≠ c.
= 24 sq. units
30. D (– 4, – 3) C (– 3, 2)
We know that area of parallelogram
= base x height
24 = AB x height
A E B
(1, – 2) (2, 3) By using distance formula,

AB = ( 2 – 1)² + ( 3 + 2 )²
Let DE be the height of parallelogram ABCD.
For ∆ABD, = 1 + 25

Let (x1, y1) = (1, – 2) = 26 units


(x2, y2) = (2, 3)
∴ 24 = 26 x height
(x3, y3) = (– 4, – 3)
24 12 26
height = units = units
area of ∆ABD 26 13
1
= [ x1 (y2 – y3 ) + x2 (y3 – y1) + x3 (y1 – y2) ] 31.
2
1
= [ 1 (3 + 3) + 2 (– 3 + 2) – 4 (– 2 – 3 ) ]
2
1 O (2, – 3y)
= [ 6 – 2 + 20 ]
2
= 12 sq. units
For ∆BCD, Let the center be O (2, – 3y).

Let (x1, y1) = (2, 3) As points A and B lie on a circle,


(x2, y2) = (– 3, 2)
AO = BO
ADDITIONAL
R

PR ACTICE MATHEMATICS - 10 121


( 2 +1)² + ( - 3y – y )² = ( 2 – 5 )² + ( - 3y – 7 )² BD = ( 2 – 5 )² + ( 6 +1)²

9 +16y² = 9 + 9 y² + 49 + 42 y = 9 + 49

On squaring both sides, we get = 58


9 + 16y² = 9y² + 42y + 58 So, AC = BD
Also, by using midpoint formula,
7y² – 42y – 49 = 0
Ê 2 + 5 –1 + 6 ˆ
y² – 6y – 7 = 0 Midpoint of AC = Á ,
Ë 2 2 ˜¯
y² – 7y + y – 7 = 0
Ê7 5ˆ
y (y – 7) + (y – 7) = 0 =Á , ˜
Ë2 2¯
(y + 1) (y – 7) = 0 Ê 5 + 2 –1 + 6 ˆ
Midpoint of BD = Á ,
Ë 2 2 ˜¯
y = – 1, 7
Ê7 5ˆ
When y = –1 When y = 7 =Á , ˜
Ë2 2¯
A = (– 1, y) = (– 1, – 1) A = (–1, y) So, Midpoint of AC = Midpoint of BD.
O = (2, 3) = (–1, 7) So, AC and BD bisect each other.
So, O = (2, – 3y)
radius = AO = (2, – 21) WoRKSHeet - 2
= ( 2 +1)² + ( 3 +1)² So, Section-A
= 9 +16 radius = AO 1. Let P (x, y) be the point equidistant from the
pointly A (5, 1), B (–3, – 7) and C (7, – 1)
= 25 = ( 2 +1)² + (–21 – 7 )²
∴ PA = PB = PC
= 5 units = 9 + 784
PA = PB
= 793 units
⇒ ( 5 – x )² + (1 – y )²
32.
= (– 3 – x )² + (– 7 – y )²
A (2, – 1) B (5, – 1) ⇒ 25 + x² – 10 x +1 + y² – 2 y
= 9 + x² + 6 x + 49 + y² +14 y
On squaring both sides, we get,
O
x² + y² – 10x – 2y + 26 = x² + y² + 6x + 14y
D (2, 6) + 58
C (5, 6)
0 = 16x + 16y + 32
By using distance formula,
AC = ( 5 – 2 )² + ( 6 +1)² x+y=–2 ...(i)
= 9 + 49 = 58 units PB = PC
122 ADDITIONAL
R

PR ACTICE MATHEMATICS - 10
⇒ 3a = 11
(– 3 – x )² + (– 7 – y )²
11
  = ( 7 – x )² + (–1 – y )² a=
3
⇒ 9 + x² + 6 x + 49 + y² +14 y
4. By Distance formula,
  = 49 + x² - 14 x +1 + y² + 2 y
Distance = ( 0 + 6 )² + ( 0 – 8 )²
On squaring both sides, we get
= 36 + 64
x² + y² + 6x + 14y + 58
= 100
  = x² + y² – 14x + 2y + 50
= 10 units
20x + 12y + 8 = 0
5x + 3y = – 2 ...(ii) 5. A (2, 3)
From (i), we get
x = –2 – y
2
On putting in (ii), we get G 1, 3
Ł ł
5 (–2 – y) + 3y = – 2
– 10 – 5y + 3y = – 2 B (–2, 1) C (x, y)

  – 2y = 8 Let (x1, y1) = (2, 3)


y = –4 (x2, y2) = (–2, 1)
So, x = –2 – y (x3, y3) = (x, y)
= – 2 + 4 Ê 2ˆ
Centroid ( G ) = Á1, ˜
= 2 Ë 3¯
We know that
So, point P (2, – 4) is equidistant from point A
(5, 1), B (–3, – 7) and C (7, – 1). Ê x + x 2 + x3 y1 + y 2 + y 3 ˆ
Centroid = Á 1 , ˜
2. Reflection of (–3, 4) in X – axis (Q) = (– 3, – 4) Ë 3 3 ¯
Ê 2 ˆ Ê 2 – 2 + x 3 +1 + y ˆ
Reflection of (– 3, 4) in Y – axis (R) = (3, 4) Á1, 3 ˜ = Á 3
,
3 ˜¯
Ë ¯ Ë
So, by using distance formula, Ê 2 ˆ Ê x 4 +y ˆ
Á1, 3 ˜ = Á 3 , 3 ˜
QR = ( 3 + 3 )² + ( 4 + 4 )² Ë ¯ Ë ¯
x 2 4 +y
= 36 + 64 ⇒ 1= and =
3 3 3
= 100 ⇒ x = 3 and y = – 2

= 10 units 6. Let (x1, y1) = (k, 2k)


3. As point (3, a) lies on line 2x – 3y + 5 = 0 (x2, y2) = (3k, 3k)
∴ 6 – 3a + 5 = 0 (x3, y3) = (3, 1)

ADDITIONAL
R

PR ACTICE MATHEMATICS - 10 123


Since the points are collinear, area of triangle –2
is zero.  k =
6
1 –1
[ x1 (y2 – y3 ) + x2 (y3 – y1) + x3 (y1 – y2) ] = 0 =
2 3
[ k (3k – 1) + 3k (1 – 2k) + 3 (2k – 3k) ] = 0
AB –1
∴ =
[ 3k² – k + 3k – 6k² – 3k ] = 0 PB 3
[ –3k² –k ] = 0 PB
=–3
AP
k (3k + 1) = 0
PB
–1 + 1 = – 3 + 1
k= ,0 AP
3
AP + PB
7. k 1 =–2
AP
AB
A C B =–2
(6, 4) (x, 0) (1, – 7) AP
AP –1
Let the ratio in which x – axis divides AB be =
AB 2
k : 1.
–1
Point on x – axis must be of form (x, 0), so, by AP = AB
using section formula, 2
9. By using distance formula,
Ê k + 6 - 7k + 4 ˆ
(x, 0) = Á , ˜
Ë k +1 k +1 ¯ (a sin a + a cos a )2 + (–b cos a – b sin a )2
∴ –7 k + 4 = 0 = a 2 (sin α + cos α )2 + b 2 (sin α + cos α )2
k +1
4
k = = (a 2 + b 2 ) (sin a + cos a )2
7
So, x – axis divides line AB in ratio 4 : 7. = (sina + cosa) (a 2 + b 2 )

8. k 1
10. Point A (x1, y1), B (x2, y2) and C (x3, y3) are
collinear if
A (4, 2) P (2, 1) B (8, 4)
(i) AB + BC = AC
Let AP : PB = k : 1 or (ii) AB + AC = BC
By section formula,
or (iii) BC + AC = AB
Ê 8k + 4 4 k + 2 ˆ
P (2, 1) =Á , ˜
Ë k +1 k +1 ¯ Section-B
8k + 4 4k + 2
∴ 2= , 1= 11. Let the vertices of triangle be (x1, y1) = (–3, 1),
k +1 k +1 (x2, y2) = (0, – 2) and (x3, y3).
8k + 4 = 2k + 2
Centroid of triangle (x, y) = (0, 0)
6k =–2 We know that, Centroid of triangle

124 ADDITIONAL
R

PR ACTICE MATHEMATICS - 10
Ê x + x 2 + x3 y1 + y 2 + y 3 ˆ We know that diagonals of parallelogram bisect
(x, y) =Á 1 , ˜ each other,
Ë 3 3 ¯
Ê –3 + 0 + x3 1 – 2 + y 3 ˆ Ê 1+ 4 1+ 8 ˆ Ê x + 4 y + 4 ˆ
i.e. (0, 0) =Á , ∴ Á 2 , 2 ˜ =Á 2 , 2 ˜
˜ Ë ¯ Ë ¯
Ë 3 3 ¯
–3 + x3 –1 + y 3 (5, 9) = (x + 4, y + 4)
⇒ = 0, =0
3 3
x + 4 = 5, y + 4 = 9
⇒ x3 = 3, y3 = 1
x = 1, y=5
So, third vertex is (x3, y3) = (3, 1)
So, fourth vertex is (1, 5).
12. Let y – axis divide the line segment joining the
point P (– 4, 5) and Q (3, – 7) in ratio k : 1. Ê 3 11ˆ
14. Let the point C Á , ˜ divide the line segment
Point on y – axis must be of form (0, y). Ë5 5¯
joining point A (3, 5) and B (–3, –2) in ratio k : 1.
By using section formula,

Ê 3k - 4 - 7 + 5 ˆ k 1
(0, y) = Á , ˜
Ë k +1 k +1 ¯ C
A B
Ê 3 11 ˆ
3k - 4 (3, 5) Á5, 5 ˜ (– 3, – 2)
  = 0 Ë ¯
k +1
By using section formula,
  k = 4
3 Ê 3 11ˆ Ê - 3k + 3 - 2 k + 5 ˆ
Á 5 , 5 ˜ = Á k +1 , k +1 ˜
- 7k + 5 Ë ¯ Ë ¯
∴ y =
k +1 - 3k + 3 3
∴ =
Ê4ˆ k +1 5
- 7 Á ˜ +5
Ë3¯ 5 (– 3k + 3) = 3 (k + 1)
=
4
+1 – 15k + 15 = 3k + 3
3
- 28 12 = 18k
+5
= 3 2
7 k=
3 3
- 13 A (– 2, –1) B (1, 0)
= 15.
7

13. A (1, 1) B (4, 4)

D (1, y) C (x, 3)
We know that diagonals of parallelogram bisect
D (x, y) C (4, 8) each other,
ADDITIONAL
R

PR ACTICE MATHEMATICS - 10 125


Ê - 2 + x - 1+ 3 ˆ Ê 1 +1 y + 0 ˆ By midpoint formula,
∴ Á 2 , 2 ˜ = Á 2 , 2 ˜
Ë ¯ Ë ¯ Ê 7 - 2 2- 5ˆ
M (x, y) = Á ,
Ê - 2+x ˆ Ê y ˆ Ë 2 2 ˜¯
⇒ Á 2 ,1˜ = Á1, 2 ˜
Ë ¯ Ë ¯ Ê 5 - 3ˆ
y =Á , ˜
∴ - 2+x = 1,1 = Ë2 2 ¯
2 2 By Distance formula,
x = 4, y = 2 2 2
AM = Ê5 ˆ Ê-3 ˆ
Á2- 3˜ + Á - 4˜
16. C (– 1, 4) Ë ¯ Ë 2 ¯
2 2
Ê - 1ˆ Ê - 3 - 8 ˆ
= Á ˜ +Á ˜
Ë 2¯ Ë 2 ¯
1 121
= +
A (0, 3) B (– 2, a) 4 4
122 61
∆ABC is a right triangle, right angled at A. = =
4 2
So, by Pythagoras theorem,
18. As point A (x, y) is equidistant from B (6, – 1)
BC² =AB² + AC² and C (2, 3)

( ) = ( (–2 - 0 ) + (a – 3) ) ∴ AB = AC
2 2
(–2 +1)2 + (a – 4 )2 2 2

+ ( (–1- 0 ) + ( 4 – 3 ) ) ( 6 - x ) + ( - 1- y ) ( 2 - x ) + (3 - y )
2 2 2 2
2 2
2
=

On squaring both sides, we get


⇒ ( 1+ (a - 4 ) ) = ( 4 + (a - 3 ) ) = ( 1+1)
2 2 2
2 2

(6 – x)² + (– 1 – y)² = (2 – x)² + (3 – y)²


⇒ 1 + (a – 4)² = 4 + (a – 3)² + 2
36 + x² – 12x + 1 + y² + 2y = 4 + x² – 4x + 9
⇒ 1 + a² + 16 – 8a = 4 + a² + 9 – 6a + 2 + y² – 6y

⇒ – 8a + 17 = – 6a + 15 ∴ –12x + 2y + 37=– 4x – 6y +13


⇒ 0 = 8x – 8y – 24
⇒ 2 = 2a
⇒ 8x – 8y = 24
⇒a=1
⇒ x–y = 3
17. A (3, 4) ⇒ x = y+3

19. As the points A (2, 1) and B (1, 2) are equidistant


from the point C (x, y),
BC = AC

( x - 1) + ( y - 2 ) ( x - 2 ) + ( y - 1)
2 2 2 2
=
B (7, 2) M (x, y) C (–2, – 5) On squaring both sides, we get

126 ADDITIONAL
PR ACTICE
R

MATHEMATICS - 10
(x –1)² + (y – 2)² = (x – 2)² + (y – 1)² On squaring both sides, we get
x² +1 – 2x + y² + 4 + 4y = x² + 4 – 4x + y² + ⇒ (a + b – x)² + (b – a – y)²
1 – 2y
= (a – b – x)² + (a + b – y)²
– 2x + 4y + 5 = – 4x – 2y + 5
⇒ a2 + b2 + x2 + 2ab – 2bx – 2ax + a2 + b2 + y2
2x +6y = 0 – 2ab + 2ay – 2by = a2 + b2 + x2 – 2ab + 2bx –
2ax + a2 + b2 + y2 + 2ab – 2by – 2ay
x +3y = 0
⇒ 2bx – 2ax + 2ay – 2by = 2bx – 2by – 2ax –
20. Let the vertices of triangle be 2ay
(x1, y1) = (5, 2) 4ay = 4bx
(x2, y2) = (4, 7) ay = bx
(x3, y3) = (7, –4) 22. Internal ratio :
Area of triangle Let x – axis divides line segment joining the
points A (3, –2) and B (– 7, – 1) in ratio k : 1.
1 Point on x – axis is of form (x, 0).
= [x1 (y2 – y3 ) + x2 (y3 – y1) + x3 (y1 – y2)]
2 k 1
1
= [5 (7 + 4) + 4 (– 4 – 2) + 7 (2 – 7)]
2 A (3, – 2) C (x, 0) B (– 7,– 1)
1
= [5 (11) + 4 (–6) + 7 (– 5)] By using section formula,
2
1 Ê - 7k + 3 - k - 2 ˆ
= [55 – 24 – 35] (x, 0) = Á ,
2 Ë k +1 k +1 ˜¯
1 -k- 2
= [55 – 59] ∴ 0 =
2 k +1
4   k = – 2
= = 2 sq. units
2
External ratio :
Section-C
By using section formula,
21. Given : AP = AQ
Ê - 7k - 3 - k + 2 ˆ
(x, 0) = Á ,
To prove : ay = bx Ë k-1 k - 1 ˜¯
Proof : By using distance formula, - k +2
∴ 0 =
k-1
(a + b – x ) +(b – a – y ) –k+2 =0
2 2
AP =
  k =2
(a - b – x ) +(a + b - y )
2 2
AQ =
23.
AP = AQ P (8, 4) R (5, 1) Q (x, y)

(a + b – x ) +(b – a – y )
2 2
By mid-point formula,
Ê 8 +x 4 +y ˆ
(5, 1) = Á ,
(a - b – x ) +(a + b - y ) 2 ˜¯
2 2

= Ë 2
ADDITIONAL
R

PR ACTICE MATHEMATICS - 10 127


8+x 4 +y ⇒ ab – a – b = 0
5= ,1 =
2 2 ⇒ ab = a + b
x + 8 = 10, y + 4 = 2
a b
x = 2, y = – 2 ⇒ 1= +
ab ab
So, Coordinates of Q = (x, y) 1 1
⇒ 1= +
a b
= (2, – 2)
26.
24. Let points be A (6, 1) B (8, 2)
(x1, y1) = (c, a + b)
(x2, y2) = (a, b + c)
(x3, y3) = (b, a + c)
Area of triangle D (x, y) E (u, v) C (9, 4)
1
= [x (y – y ) + x2 (y3 – y1) + x3 (y1 – y2)] We know that diagonals of parallelogram
2 1 2 3 bisect each other.
1
= [c (b + c – a – c ) + a (a + c – a – b) + ∴ Midpoint of AC = midpoint of BD
2
b (a + b – b – c)]
So, by midpoint formula,
1
= [c (b – a) + a (c – b) + b (a – c)] Ê 7 +9 3 +4 ˆ Ê x +8 y +2 ˆ
Á , = ,
2 ˜¯ ÁË 2 2 ˜¯
2
Ë 2
1
= [bc – ac + ac – ab + ab –bc] Ê 15 5 ˆ Ê x + 8 y + 2 ˆ
2 i.e. Á 2 ,2˜ = Á ,
=0 Ë ¯ Ë 2 2 ˜¯

As area of triangle = 0 i.e. x + 8 = 15, y + 2 = 5


So, points A, B and C are collinear. x = 7, y = 3
So, point D = (7, 3)
25. Let the point be
Again, by midpoint formula,
(x1, y1) = (a, 0) Ê 7 +9 3 +4 ˆ
E (u, v) = Á ,
(x2, y2) = (0, b) Ë 2 2 ˜¯

(x3, y3) = (1, 1) Ê 16 7 ˆ


= Á , ˜
Ë 2 2¯
Points are collinear, if area of triangle = 0
Ê 7ˆ
1 = Á 8, ˜
i.e. [x (y – y ) + x2 (y3 – y1) + x3 (y1 – y2)] = 0 Ë 2¯
2 1 2 3
1 For area of ∆ADE
⇒ [a (b –1) + 0 (1 – 0) + 1 (0 – b)] = 0
2
Let (x1, y1) = (6, 1)
1
⇒ [ab – a – b] = 0
2 (x2, y2) = (7, 3)
128 ADDITIONAL
R

PR ACTICE MATHEMATICS - 10
7 By using distance formula,
(x3, y3) = 8,
Ł 2ł AB = ( 3 - 2 )2 + ( 4 +1)2
area of ∆ADE
= 1 + 25 = 26 units
1
= [ x1 (y2, – y3 ) + x2 (y3, – y1) + x3 (y1, – y2) ]
2 BC = ( - 2 - 3 )2 + ( 3 - 4 )2
È Ê 7ˆ Ê7 ˆ ˘ = 25 +1 = 26 units
= Í6 Á 3 - ˜ + 7 Á - 1˜ + 8 (1- 3) ˙
Î Ë 2¯ Ë2 ¯ ˚
CD = ( - 3 + 2 )2 + ( - 2 - 3 )2
1   −1 5 
= 6   + 7   + 8 ( −2 ) = 1 + 25
2  2 2 
1  35  = 26 units
=  −3 + −16 
2  2  AD = ( - 3 - 2 )2 + ( - 2 +1)2
1  35 
=  −19  = 26 units
2 2 
As AB = BC = CD = AD,
1  35 − 38 
=
2  2  ABCD is a rhombus
3 Again, by distance formula,
= sq. units
4
AC = ( - 2 - 2 )2 + ( 3 +1)2
27. Let the points be
= 16 +16
(x1, y1) = (p + 1, 2p –2)
= 32
(x2, y2) = (p – 1, p)
(x3, y3) = (p – 6, 2p – 6) = 4 2 units
Points are collinear if area of triangle is zero. BD = ( - 3 - 3 )2 + ( - 2 - 4 )2
1
[x (y – y ) + x2 (y3 – y1) + x3 (y1 – y2)] = 0 ( - 6 )2 + ( - 6 )2
2 1 2 3 =
[(p + 1) (p – 2p + 6) + (p – 1) (2p – 6 – 2p + 2)
= 72
+ (p – 6) (2p – 2 – p)] = 0
[(p + 1) (6 – p) + (p – 1) (– 4) + (p – 6) (p – 2)] = 0 = 6 2 units

[6p – p2 + 6 – p – 4p + 4 + p2 – 2p – 6p + 12] = 0 ∴ AC ≠ BD
As diagonals are not equal, ABCD is a rhombus
[ 7p + 22 ] = 0
but not a square.
−22
p =
7 29.
28. A (2, – 1) B (3, 4) A (3, – 5) P (x, y) B (– 4, 8)

AP k
=
PB 1
Let point P be (x, y).
D (– 3, – 2) C (– 2, 3) By using section formula,
ADDITIONAL
R

PR ACTICE MATHEMATICS - 10 129


Ê - 4 k + 3 8k - 5 ˆ So, area of quadrilateral ABCD
(x, y) =Á ,
Ë k +1 k +1 ˜¯ = area of ∆ABC + area of ∆ACD
Ê - 4 k + 3 8k - 5 ˆ = 25 + 107
(x, y) =Á ,
Ë k +1 k +1 ˜¯
= 132 sq. units
- 4k + 3 8k - 5
∴  x = , y=
k +1 k +1
As point P lies on line x + y = 0 Section-D
Ê - 4 k + 3 ˆ Ê 8k - 5 ˆ 31. A (4, – 6)
∴  Á ˜ +Á ˜ =0
Ë k +1 ¯ Ë k +1 ¯
⇒ 4k – 2 = 0
1
⇒ k=
2 B M C
30. (3, – 2) (x, y) (5, 2)
A (1, 1) B (7, – 3)
Let AM be the median such that point M is (x, y).
Ê 3 +5 - 2 +2 ˆ
(x, y) =Á ,
Ë 2 2 ˜¯

(x, y) = (4, 0)
D (7, 21) C (12, 2)
So, point M (x, y) = (4, 0)
Area of ∆ABC
Area of ∆AMB
1
= [ 1 (– 3 – 2) + 7 (2 – 1) + 12 (1 + 3) ] 1
2 =  4 ( −2 − 0 ) + 3 ( 0 + 6 ) + 4 ( −6 + 2 )
1 2
= [ – 5 + 7 + 48 ]
2 1 −6
= −8 +18 −16  =
 = −3 = 3 sq. units
1 2 2
= [ 50 ]
2
Area of ∆AMC
= 25 sq. units
1
Area of DACD =  4 (0 − 2 ) + 4 (2 + 6 ) + 5 (−6 − 0 )
2
1 1 −6
= [ 1 (2 – 21) + 12 (21 – 1) + 7 (1 – 2) ] = −
 8 + 32 − 30  = = −3 = 3 sq. units
2 2 2
1 So, median divides the triangle into two triangle
= 2 [ – 19 + 12 ( 20 ) – 7 ] of equal area.
1
= [ – 26 + 240 ] 32.
2
1 A P B
= [ 214 ]
2 (3, 4) (x, y) (5, – 2)
= 107 sq. units PA = PB
130 ADDITIONAL
PR ACTICE
R

MATHEMATICS - 10
=1+0
( - 3 - x )2 + ( 4 - y )2 = ( 5 - x )2 + ( - 2 - y )2
=1
On squaring both sides, we get
So, P (x, y) = (1, 0)
  (3 – x)2 + (4 – y)2 = (5 – x)2 + (– 2 – y)2
⇒ 9 + x² – 6x + 16 + y² – 8y = 25 + x² – 33.
10x + 4 + y² + 4y A (3, 2)

⇒ – 6x – 8y + 25 = – 10x + 4y + 29

)1
,y
⇒ 4x – 12y – 4 = 0 F (x3, y3)

1
(x
D
⇒ x – 3y = 1 ...(i)
Also, area of ∆PAB = 10
B (5, 4) E C
1 (x2, y2) (3, 6)
∴ [ x (4 + 2) + 3 (– 2 – y) + 5 (y – 4) ] = 10
2
⇒ [ 6x – 6 – 3y + 5y – 20 ] = 20 By midpoint formula,

⇒ [ 6x + 2y – 26 ] = 20 Ê 3 +5 2 + 4 ˆ
D (x1, y1) =Á ,
Ë 2 2 ˜¯
⇒ [ 3x + y – 13 ] = 10 D (x1, y1) = (4, 3)
⇒ 3x + y – 13 = ± 10
Ê 5 +3 4 +6 ˆ
Again, E (x2, y2) = Á ,
⇒ 3x + y = 23 ...(ii) Ë 2 2 ˜¯
= (4, 5)
or 3x + y = 3 ...(iii)
Ê 3 +3 2 +6 ˆ
  From (i), x = 1 + 3y F (x3, y3) =Á ,
Ë 2 2 ˜¯
So, eq. (ii) becomes 3 + 9y + y = 23 = (3, 4)
10y = 20 Area of ∆DEF
y =2 1
= [ x1 (y2 – y3 ) + x2 (y3 – y1) + x3 (y1 – y2) ]
So, x = 1 + 3y 2
1
= 1 + 6 = [ 4 (5 – 4) + 4 (4 – 3) + 3 (3 – 5) ]
2
= 7 1
= [ 4 + 4 – 6 ] = 1 sq. unit
So, P (x, y) = (7, 2) 2

On putting x = 1 + 3y in (iii), we get 34. Let (x1, y1) = (– 2, 5)


3 (1 + 3y) + y = 3 (x2, y2) = (k, – 4)
3 + 9y + y = 3 (x3, y3) = (2k + 1, 10)
10y = 0 Area of triangle
y =0 1
= [x (y – y ) + x2 (y3 – y1) + x3 (y1 – y2)]
So, x = 1 + 3y 2 1 2 3
ADDITIONAL
R

PR ACTICE MATHEMATICS - 10 131


1 36. Let D (x, y) be the Circumcentre.
53 = [– 2 (– 4 – 10) + k (10 – 5) + (2k + 1) (5 + 4)] We know that Circumcentre of a triangle is
2
1 equidistant from each of the vertices.
53 = [ 28 + 5k + 18k + 9 ]
2 Let the vertices be A (x1, y1) = (8, 6), B (x2,
1 y2) = (8, – 2) and C (x3, y3) = (2, – 2).
53 = [ 23k + 37 ]
2 So, AD = BD
∴ 23k + 37 = ± 106
( 8 - x )2 + ( 6 - y )2 = ( 8 - x )2 + ( - 2 - y )2
if 23k + 37 = 106
On squaring both sides, we get
23k = 69
(8 – x)² + (6 – y)² = (8 – x)² + (– 2 – y)²
k = 3
(6 – y)² = (– 2 – y)²
if 23k + 37 = 106
36 + y² – 12y = 4 + y² + 4y
23k = – 143
32 = 16y
- 143
k = , rejected k < 0. y = 2
23
∴ k = 3. Also, BD = CD

( 8 - x )2 + ( - 2 - y )2 = ( 2 - x )2 + ( - 2 - y )2
35. A (–2, 3) B (6, 5)
(8 – x)² + (– 2 – y)² = (2 – x)² + (– 2 – y)²

64 + x² – 16x + 4 y² + 4y = 4 + x² – 4x + 4 + y² +4y
⇒ – 16x + 4y + 68 = – 4x + 4y + 8
D (–4, – 3) C (x, – 5) ⇒ 12x = 60
Area of quadrilateral ABCD = 80 sq. units ⇒   x = 5

i.e. area of ∆ABC + area of ∆ACD = 80 So, Circumcentre is (x, y) = (5, 2)


Circumradius = AD
1 1
i.e. [–2 (5 + 5) + 6 (–5 –3) + x (3 – 5)] +
2 2 = ( 8 - x )2 + ( 6 - y )2
[–2 (–5 + 3) + x (–3 – 3) + (–4) (3 + 5)] = 80
= ( 8 - 5 )2 + ( 6 - 2 )2
1
⇒ [– 20 – 48 – 2x + 4 – 6x – 32] = 80
2 = 9 +16
1
⇒ [– 80 – 96] = 80
2 = 25
⇒ – 8x – 96 = ±160
= 5
⇒ –x – 12 = ± 20
37. By using midpoint formula,
–x – 12 = 20 –x – 12 = –20
Ê 0 + 2a 2b + 0 ˆ
C (x, y) = Á ,
x = –32 x = 8
Ë 2 2 ˜¯
\ Positive value of x = 8. = (a, b)
132 ADDITIONAL
R

PR ACTICE MATHEMATICS - 10
Using distance formula, we have È Ê x1 + x 2 ˆ È Ê y 2 + y 3 ˆ Ê y1 + y 3 ˆ ˘ ˘
ÍÁ ˜ ÍÁ ˜- Á ˜˙ ˙
ÍË 2 ¯ ÎË 2 ¯ Ë 2 ¯˚ ˙
BC = (a - 0 )2 + (b - 2b )2 Í
1 È ˘˙
= Í + ÊÁ x 2 + x 3 ˆ˜ Í ÊÁ y1 + y 3 ˆ˜ - ÊÁ y1 + y 2 ˆ˜ ˙ ˙
2 Í Ë 2 ¯ ÎË 2 ¯ Ë 2 ¯ ˚ ˙
= a2 + b2 Í ˙
Í Ê x1 + x 3 ˆ È Ê y1 + y 2 ˆ Ê y 2 + y 3 ˆ ˘ ˙
Í +Á 2
˜ ÍÁ ˜- Á ˜˙ ˙
OC = Î Ë ¯ ÎË 2 ¯ Ë 2 ¯˚ ˚
(a - 0 )2 + (b - 0 )2
È( x1 + x 2 ) ( y 2 - y1 ) ˘
= a2 + b2 1 Í ˙
= Í + ( x 2 + x3 ) ( y3 - y 2 ) ˙
8 Í ˙
Î + ( x1 + x3 ) ( y1 - y 3 ) ˚
AC = (a - 2a )2 + (b - 0 )2

= a2 + b2 È x È( y - y ) + ( y1 - y 3 ) ˘ ˘
1 Í 1Î 2 1 ˚ ˙
So, BC = CO = AC = Í +x 2 ÈÎ( y 2 - y1 ) + ( y 3 - y 2 ) ˘˚ ˙
8 Í ˙
∴ Point C is equidistant from the vertices A, ÍÎ +x3 ÈÎ( y 3 - y 2 ) + ( y1 - y 3 ) ˘˚ ˙˚
O and B.

38. A (x1, y1) 1 È x1 ( y 2 - y 3 ) + x 2 ( y 3 - y1 ) ˘


= Í ˙
2 2 ˆ

8 ÍÎ +x3 ( y1 - y 2 )
˜

˙˚
y
¯
, 1+
2 2 y

Ê x1 + x3 y1 + y 3 ˆ
x

F E Á , ˜ Ê1ˆ Ê1ˆ
Á x1 +

Ë 2 2 ¯ = Á ˜ Á ˜ [x1 (y2 – y3) + x2 (y3 – y1) + x3 (y1 – y2)]


Ë4¯ Ë2¯
Ê
Ë

1
= area of ∆ABC
B D C 4
(x2, y2) Ê x 2 + x3 y 2 + y3 ˆ (x3, y3)
Á , ˜
Ë 2 2 ¯ 39. Using formula for area of triangle,
1
By midpoint formula, ar ∆DBC = [ x (5 + 2) – 3 (– 2 – 3x) + 4 (3x – 5) ]
2
 x + x3 y 2 + y3  1
D is  2 , = [ 7x + 6 + 9x + 12x – 20 ]
2
 2 2 
1
x +x y +y  = [ 28x – 14 ]
E is  1 3 , 1 3  2
 2 2  = [ 14 x – 7 ] ...(i)
x +x y +y  Using formula for area of triangle, or DABC
F is  1 2 , 1 2 
 2 2 
1
= [ 6 (5 + 2) – 3 (– 2 – 3) + 4 (3 – 5) ]
Area of ∆ABC 2
1
1 = [ 42 + 15 – 8 ]
[ x1 (y2 – y3) + x2 (y3 – y1) + x3 (y1 – y2) ] 2
2
1
Consider, ar ∆DEF = [ 49 ] sq. units
2

ADDITIONAL
R

PR ACTICE MATHEMATICS - 10 133


ar DDBC 1 Distance covered on line AB = 2 m
As =
ar DABC 2 ∴ coordinates of green flag are (2, 25)
14 x – 7 1
1 (ii) (c) Distance covered on line AD = × 100
⇒ = 5
49 2 = 20 m
2 Distance covered on line AB = 8 m
2 14 x – 7 1 Coordinates of red flag are (8, 20)
⇒ =
49 2 (iii) (d) Coordinates of red flag are (8, 20)
49
⇒ 14x – 7 = ± Coordinates of green flag are (2, 25)
4
49 Distance between red and green flag
If 14x – 7 =
4 ( 8 - 2 ) + ( 25)
2 2
=
49 49 + 28 77
14x = +7= =
4 4 4 = 36 + 25
11 = 61 m
⇒ x =
8 (iv) (a) Coordinates of mid point of red flag and
- 49
If 14x – 7 = green flag are
4
- 49 - 49 + 28 - 21 8 + 2 20 + 25
14x = +7 = = ,
4 4 4 Ł 2 2 ł
-3 = (5, 22.5)
⇒ x =
8 (v) (d) Let the distance covered by Rohini is x m.
40. As the point (x1, y1), (x2, y2) and (x3, y3) lie on 1
×100 = 22.5
the same line, area of triangle formed by these x
points is 0. 22.5
x =
100
1
i.e. [ x1 (y2 – y3 ) + x2 (y3 – y1) + x3 (y1 – y2) ] = 0 9
2 x =
40
[ x1 (y2 – y3 ) + x2 (y3 – y1) + x3 (y1 – y2) ] = 0
cASe StUDY-2
On dividing by x1 x2 x3, we get
È üüüüüüü
- 3 + 2 3- 1 + - ˘ (i) (b) Point D is the mid point of AB.
1 2 3 1 2
Í ˙ =0 ∴ The coordinates of D would lie on
Î x1 x 2 x3 ˚ halfway between A and B.
È y 2 - y 3 y 3 - y1 y1 - y 2 ˘ Let coordinates of B are (x, y) so that
Í + + ˙ =0
Î x 2 x3 x1 x3 x1 x 2 ˚ D+ x −1 + y
= 1 = 0
2 2
y 2 - y 3 y 3 - y1 y1 - y 2
∴ + + =0 x = 2 y = 1
x 2 x3 x1 x3 x1 x 2
(ii) (a) Let coordinates of C are x' and y'
x '+ 0 y '- 1
cASe StUDY-1 = 0 = 1
2 2
1 x' = 0 y'= 3
(i) (a) Distance covered on line AD = × 100
4
= 25 m ∴ coordinates of c are (0, 3)
134 ADDITIONAL
R

PR ACTICE MATHEMATICS - 10
(iii) (c) Let coordinates of F be (x11, y11). Using 1
midpoint formula as F is the mid point of (iv) (d) Area of DABC = 2 (base × height)
BC. The base BC is 2 units 8 height AD is 2
2+0 unit
x11 =
2 1
Area = × 4 × 2
= 1 2
= 4 sq. units
1+ 3
y11 = 1
2 (v) (d) Area of DDEF = (base × height)
2
= 2 Base DE is 1 units
Coordinates of F are (1, 2) Height is 2 units.
1
Area = × 2 × 1 = 1 sq. units
2

ADDITIONAL
R

PR ACTICE MATHEMATICS - 10 135


Chapter
Introduction to
8 Trigonometry
Multiple choice Questions

12 3 tan 30∞
1. (a) Cot x = = 4. (b)
16 4 tan 0∞ - cot 30∞
sin x - cos x 1 - cot x 1
=
sin x + cos x 1 + cot x 3
=
3 0- 3
1-
4
=
3 -1
1+ =
4 3
1 4
= ×
4 7 5. (b) Consider
1
= (a sinq + b cosq)²
7
= a² sin²q + b² cos²q + 2ab sinq cosq

( 2)
2
x (2)
2
2
= a² (1 – cos²q) + b² (1 – sin²q) + 2ab sinq cosq
( 3) Ê 1 ˆ
2
2. (a) 2 = - Á ˜
2
Ê 1 ˆ Ê 3ˆ Ë 3¯ = a² + b² – a² cos²q – b²sin²q + 2ab sinq cosq
8Á ˜ Á ˜
Ë 2 ¯ ÁË 2 ˜¯ = a² + b² – (a² cos²q + b² sin²q) + 2ab sinq
cosq (i)
8x 1 8
= 3– =
3 3 3 Also, a cosq – b sinq = c
x =1 ⇒ (a cosq – b sinq)² = c²

3. (b) A + B + C = 180° ⇒ a² cos²q + b² sin²q – 2ab sinq cosq = c²

⇒ B + C = 180° –A ⇒ a² cos²q + b² sin²q = c² + 2ab sinq cosq (ii)

Ê B +C ˆ Ê 180∞ - A ˆ So, (a sinq + b cosq)²


∴ = sin Á ˜ = sin Á ˜
Ë 2 ¯ Ë 2 ¯ = a² + b² + 2ab sinq cosq – c² – 2ab sinq cosq
Ê Aˆ [From (i) and (ii)]
= sin Á 90∞ - ˜
Ë 2¯
= a² + b² – c²
A
= cos ∴ a sin + b cos = ± a2 + b2 - c 2
2

136 ADDITIONAL
R

PR ACTICE MATHEMATICS - 10
4. cos (a + b) =0
WORKSHEET - 1
a + b = 90°
SECTION-A
∴ a = 90° – b
1. cos ( 90 - θ ) sec ( 90 - θ ) tanθ
Consider sin (a – b) = sin [90° – 2b] = cos2b
cosec ( 90 - θ ) sin ( 90 - θ ) cot ( 90 - θ )
The statement is true.
tan ( 90 - q )
+
cot q 5. tan2 − sec 2 θ
sinθ cosecθ tanθ cot 2 − cosec 2θ
=
sec θ cos θ tanθ sec 2 q - tan2 q 1
= = =1
cos q cosec q - cot q 1
2 2
=
cot q
1 1 1
= + =2 6. cosecθ = 3x ⇒ x = cosecθ
1 1 3
3 1 1
2. Consider cotθ = ⇒ = cotθ
x x 3
tan A tan B + tan A cot B sin2 B 1 1 1 1
− consider x² – 2 = cosec²θ – cot²θ =
SinA sec B cos2 A x 9 9 9
tan (90° − B ) tan B + tan A cot (90° − A ) 5
=
sin A sec (90° − A ) 7. tan A =
12
Consider (sin A + cos A) Sec A
sin2 B
− 1
cos2 ( 90° − B ) = (sin A + cos A)
cos A
cot B tan B + tan2 A sin2 B = tan A + 1
= - 2
sin AcosecA sin B 5 17
= +1=
12 12
1 + tan2 A
= - 1 = tan2 A
1 6
8. Consider 6 tan²θ –
cos2 q
3. 1 + sinq = 6 (tan²θ – sec² θ)
1 - sinq
=6 (1)
1+ sin q 1+ sin q =6
= ×
1-sin q 1+ sin q

(1+ sinq )
2
SECTION-B
=
1 − sin2 q 9. 2 sin²30° – 3 cos² 45° + tan²60°
2 2
(1+ sinq ) 1  1 
( )
2 2

= =2   − 3  + 3
cos2 q 2  2 

1 + sinq 1 3
= = − +3= 2
cos q 2 2
ADDITIONAL
R

PR ACTICE MATHEMATICS - 10 137


10. (a) We know that – 1 ≤ sinq ≤ 1 x 1
sin = = sin30°
∴ 0 ≤ sin²q ≤ 1 2 2
x
1 = 30°
If sinq = x + , 2
x
On squaring both sides, we get x = 60°

1 12. Sinq + sin²q = 1


sin²q = x² + +2
x2 ⇒ sinq = 1 – sin²q
1
Here, R H S = x² + 2 + 2 > 2 ⇒ sinq = cos²q (i)
x
but maximum value of sin²q is ⇒ tanq = cosq
1 Consider cos²q + cos4q
∴ sin²q is ≠ x +
x
(b) As (a – b)² ≥ 0 = tan²q + tan4q

⇒ a² + b² – 2ab ≥ 0 = tan²q (1 + tan²q)

⇒ a² + b² ≥ 2ab = tan²q sec²q

a 2 + b 2 2ab 1
∴ cosq= ≥ = 1 = tan²q By (i)
2ab 2ab sinq
⇒ cosq ≥ 1 sinq
=
cos2 q
if cos = 1 sinθ
= By (i)
a2 + b2 sinθ
=1 = 1
2ab
a² + b² = 2ab
(a – b)² = 0 13.
a=b A

but a and b are distinct numbers


∴ cosq > 1
but – 1 ≤ cosq ≤ 1

a2 + b2
So, cosq ≠
2ab
B C
11. (a) 2 sin3x = 3 1
tan A =
3 3
sin3x = BC 1
2
=
3x = 60° AB 3
Let BC = k, AB = k (3^0.5)
x
  = 20°
∴ AC² = BC² + AB² = 4k²
x
(b) 2 sin =1 AC = 2k
2
138 ADDITIONAL
R

PR ACTICE MATHEMATICS - 10
Consider 9 4
= + + 2 +3−3
sinA cosC + cosA sinC 4 3
 BC   BC   AB   AB  9 4
=  + = + +2
 AC   AC   AC   AC  4 3

BC 2 AB 2 27 +16 + 24 67
= + = =
AC 2 AC 2 12 12

BC 2 + AB 2 Section-C
=
AC 2 17. tanq + cotq =2
2
AC
= =1 1
AC 2 tanq + =2
tanq
14. Consider tan²q – 2 tanq + 1 = 0

4 cot²45° – Sec²60° + sin²60° – cos²90° (tanq – 1)² =0

2 tanq – 1 =0
 3
= 4(1)² – (2)² +   – (0)² tanq = 1 = tan 45°
 2 
3 q = 45°
=4–4+
4
Consider tan7q + cot7q
3
=
4 = tan7(45°) +cot7(45°)
= 17 + 17
15. Consider
2 = 1+1
 1 cos q 
(cosecq – cotq)² =  −
 sin q sin q  = 2
2
1 − cos θ 
=  18. Consider
 sin θ 
sin q 1 + cos q
(1− cos q )
2
+
= 1 + cos q sin q
sin2 q
sin θ (sin θ ) + (1+ cos θ )(1 + cos θ )
(1− cos θ )
2
=
= sin θ (1 + cos θ )
1 − cos2 θ
1 − cos θ sin2 q +1 + cos2 q + 2 cos q
= =
1 + cos θ sin q (1 + cos q )
16. Consider
2 + 2 cos θ
3 cos²30° + sec²30° + 2 cos²0° + 3 sin²90° – =
tan²60° sin θ (1 + cos θ )

2 + (1 + cos θ )
2 2
 3  2 
( 3)
2
= 3   +   + 2(1)2 + 3(1)2 − =
 2   3 sin θ (1 + cos θ )
ADDITIONAL
R

PR ACTICE MATHEMATICS - 10 139


2 On squarring both sides, we get
= = 2 cosec q
sinq a² cos² q + b² sin² q + 2ab sin q cos q = m²   (1)

5 AC a sin q – b cos q = n
19. Sec A = =
4 AB On squarring both sides, we get
LHS RHS
a² sin² q + b² cos² q – 2ab sin q cos q = n²   (2)
3sin A − 4 sin3 A 3 tan A - tan3 A
4 cos3 A − 3cos A On adding (1) and (2), we get
1-3 tan2 A
C 3 a² (sin² q + cos² q) + b² (sin² q + cos² q) = m² + n²
3 3
3  −  
5k 4 4 ⇒ a² + b² = m² + n²
3k = 2
3
1− 3  
4 21. x = a cos³ q
B 4k A
9 27 y = b sin³ q
BC² = AC² – AB² -
   = 25k² – 16k² = 4 64 2 2

27 Consider  x 3  y 3
   = 9k² 1-  a  +  b 
16
∴ BC = 3k 2 2
144 − 27
=  a cos q  +  b cos q 
3 3 3 3
3
3sin A - 4 sin A
So, 64  a   b 
4 cos3 A - 3cos A =
16 − 24
3 = cos² q + sin² q
3 3 16
3  − 4  
5 5 = 1
= 117
3 =
4 4 −44
4   − 3 
5 5 22. sin (A + B) = 1 = sin 90°
−177
= A + B = 90°   (1)
9 108 44
-
= 5 125 3
256 12 cos (A – B) = = cos 30°
- 2
125 5
A – B = 30°   (2)
225 - 108
125 177 On solving (1) and (2), we get
=
256 - 300 - 44 A + B = 90°
125
A - B = 30
So, LHS = RHS
2 A = 120

20. a cos q + b sin q =m A = 60°

a sin q – b cos q =n From (1), B = 90° – A


To prove : a² + b² = m² + n² = 90° – 60°
Proof a cos q + b sin q = m = 30°

140 ADDITIONAL
PR ACTICE
R

MATHEMATICS - 10
23. Consider So, ∠A = ∠1 + ∠2
  (1 – sin q + cos q)² = 45° + 60°
= [(1 – sin q) + cos q]² = 105°
= (1 – sin q)2 + cos2 q + 2cos q (1 – sin q) 26. Consider
= (1 – sin q)2 (1 – sin2 q) + 2cos q (1 – sin q) sinq
LHS =
1 - cos q
= (1 – sin q) [1 – sin q + 1 + sin q + 2cos q]
sinq (1 + cos q )
= (1 – sin q) (2 cos q + 2) =
(1− cosq )(1+ cosq )
= 2 (1 + cos q) (1 – sin q)
sinq (1 + cos q )
= RHS =
1 − cos2 q
sinq (1 + cos q )
24. Consider =
sin2 q
tanq + sinq 1
+1 1 + cos q
tanq - sinq = cos θ =
sinq
1
sinq −1
+ sinq    cos θ 1 cos q
= +
= cos q sinq sinq
sinq sec q + 1
- sinq = = cosec q + cot q
cos q    sec q - 1
= RHS
   = R H S
27. Consider
Section-D
sin (2q + 45°) = cos (30° – q)
25.
⇒ Cos [90° – (2q + 45°)]
C 10
3 cm = cos (30° – q)
D
10 ⇒ cos (45° – 2q) = cos (30° – q)
cm

cm
2
10

1 ⇒   45° – 2q = 30° – q


A B
⇒   15° =q
In DADB, ∴ tanq = tan 15°
BD 10 = tan (45° – 30°)
tan (∠1) = = =1
AD 10
tan 45° − tan30°
∴ ∠1 = 45° =
1 + 3 tan 45° tan30°
In DADC,
1
1-
10 3 = 3
tan (∠2) = = 3
10 3
1+
∠2 = 60° 3

ADDITIONAL
R

PR ACTICE MATHEMATICS - 10 141


3 -1 1 1
= −
(
3 1+ 3 ) = sin A cosec A + cot A

=
( 3 -1)( 3 -1) i.e. To prove

3 (1 + 3 )( 3 - 1) 1 1 2
+ =
cosec A − cot A cosec A + cot A sin A
3 + 1- 2 3
=
3 (3 - 1)
Consider
4−2 3 1 1
= +
2 3 cosec A - cot A cosec A + cot A
2− 3
= 1 1
3 = +
1 cos A 1 cos A
- +
28. Consider sin A sin A sin A sin A
tan A cot A sin A sin A
LHS = + = +
1 - cot A 1 - tan A 1 - cos A 1 + cos A
sin A cos A  1 + cos A +1 − cos A 
=   sin A
 (1 − cos A )(1 + cos A ) 
= cos A + sin A  
cos A sin A
1- 1-  2 
sin A cos A =   sin A
1 − cos A 
2
sin2 A cos2 A
= + 2
cos A ( sin A - cos A ) sin A ( cos A - sin A ) = sin A
sin2 A
sin2 A cos2 A 2
= −
cos A (sin A − cos A ) sin A (sin A − cos A ) =
sin
sin3 A − cos3 A
=
sin A cos A (sin A − cos A ) 30. sinq + cosq= p, secq + cosecq = q

( sin A - cos A ) (sin 2


A + cos2 A + sin A cos A ) Consider
=
sin A cos A ( sin A cos A )   q (p² – 1)
1 + sin A cos A = (secq + cosecq) [ sin²q + cosq²) – 2 ]
=
sin A cos A = (secq + cosecq) [ sin²q + cos²q + 2sinq cosq -1]
= 1 + cosec A sec A
1 1
= + (2 sinq cosq)
= RHS Ł cos q sinq ł
 sinq + cos q 
=  (2 sinq cosq)
29. To prove :  sinq cos q 
1 1 = 2 (sinq + cosq)
-
cosec A - cot A sin A = 2p

142 ADDITIONAL
R

PR ACTICE MATHEMATICS - 10
31. Secθ + tanθ = p (i) =1+1
We know that Sec²θ – tan²θ =1 =2
⇒ (secθ – tanθ) (secθ + tanθ) = 1
WORKSHEET - 2
⇒ (secθ – tanθ) p = 1
1 SECTION-A
⇒ secθ – tanθ = (ii)
p 1. Consider
On adding (i) and (ii), we get
(1 + cot²θ) sin²θ
1
2 secθ = p +  cos2 q  2
p
= 1 + sin q
1 1  sin2 q 
secθ = p +  = sin²θ + cos²θ
2 p

On subtracting (i) from (ii), we get =1


1
–2 tanθ = -p
p 2. Consider
1 1 cosec²θ (1 + cosθ) (1 – cosθ) = x
tanθ =  p− 
2 p
⇒ cosec²θ (1 – cos²θ) = x
Also,
1 1 ⇒ cosec²θ sin²θ =x
 p−  1
tan θ 2  p  p2 −1 ⇒ sin²θ =x
=
sinθ = = sin2 q
sec θ 1  1  p2 +1
p+  ⇒
2  p 1 =x

32. sinθ + cosθ = 2


3. cos1° cos2° cos3°... cos179° cos188°
1 1
⇒ sinθ + cos θ =
1 = cos1° cos2° cos3°... cos90°... cos179° cos188°
2 2

p p = 0
cos sin q + sin cos q =1
4 4
π π
⇒ sin + θ = 1 = sin 4.
Ł4 ł 2 A
p p
⇒ +q =
4 2 3k 5k
π π
⇒ θ= −
2 4
p θ
4 B 4k C
Consider cotθ = 4
tanθ + cotθ
4
p p cotθ =
= tan + cot 3
4 4
ADDITIONAL
R

PR ACTICE MATHEMATICS - 10 143


BC  5sin θ 
=  + 2  cos θ
AB 5sin θ + 2 cos θ  cos θ 
Let BC = 4k, AB = 3k =
5sin θ − 2 cos θ  5sin θ 
 cos θ − 2  cos θ
By Pythagoras theorem,  

  AC² = AB² + BC² 3


5× −1
5 tan q +1 5
= (3K)2 + (4K)2 =
5 tan q −1 3
5 × +1
= 9k² + 16k² 5

= 25k² 2 1
= =
4 2
∴ AC = 5k D A
7.
4 cos q - sinq
Consider
2 cos q + sinq
5 4
4  3
4  −  
 5 5 90
= q q
4 3 E B
2  + C 3
5 5
Base
16 3 cos q =
- hypotenuse
= 5 5
8 3 3
+ cos q = q ≈ 53°
5 5 5
13 5 90° + q + φ = 180
= ×
5 11
φ ≈ 37°
13
= cos 37° = 0.8
11
8. cos² 17° – sin²73°
5. Consider (secA + tanA) (1 – sinA)
= cos² (90° – 73°) – sin²73°
1 sin
= (1- sin A ) = sin²73° – sin²73°
Ł cos A cos A ł
(1+ sin A )(1- sin A ) = 0
=
cos A
1 - sin A cos2 A
2 2 tan30
= = 9.  
cos A cos A 1 + tan2 30
= cos A  1 
2 
 3
= 2
6. 3cosq = 5sinq  1 
1+  
3 sinθ  3
=
5 cos θ 2 3
3 = ×
tanθ = 3 4
5
144 ADDITIONAL
PR ACTICE
R

MATHEMATICS - 10
14. sin 75° = sin (45° + 30°)
3
=
2 = sin 45° cos 30° + cos 45° sin30°

Section-B 1  3 1 1
=  +  
2 2  2 2
10. tan 2q = cot (q + 6°)
3 1
⇒ cot (90° – 2q) = cot (q + 6°) = +
2 2 2 2
⇒ (90° – 2q) = (q + 6°)
3 +1
⇒ 90° – 6° = 3q =
2 2
84
⇒ =q
3 15. sinq = cosq
⇒ 28° =q
sinq
=1
cos q
2 cos67 tan 40
11.   - - cos 0 tanq = 1
sin23 cot 50
p
2 cos ( 90° − 23° ) tan ( 90° − 50° ) ∴  q =
= − − cos 0° 4
sin23° cot 50°
Consider 2 tan²q + sin²q – 1
2 sin23° cot 50°
= − − cos 0°
sin23° cot 50° p p
= 2 tan² + sin² –1
= 2 –1 –1 4 4
2
= 0  1 
= 2 (1)² +  –1
 2 
1
12. sec 4A = cosec (A – 20°) = 2– –1
2
⇒ cosec (90° – 4A) = cosec (A – 20°) 1
= 1–
2
⇒ 90° – 4A = A – 20° 1
=
⇒ 110° = 5A 2

⇒ A = 22° 16. a + b = 90°


To prove: =
cos70° cos59°
13. + − 8 sin2 30° cos a cosec b - cos a sin b = sin a
sin20° sin31°

cos ( 90° − 20° ) cos ( 90° − 31° )


2
1 Consider cos a sec a - cos a cos a
= + − 8 
sin20° sin31° 2
cos α cosec ( 90 - α ) - cos α sin ( 90 - α )
sin20° sin31° 8 =
= + −
sin20° sin31° 4 cos α secα - cos α cos α
=
= 1+1–2
= 0 = 1 − cos2 a
ADDITIONAL
R

PR ACTICE MATHEMATICS - 10 145


sin2 a Section-C
=
20. O
= sin a

17. Consider 7 cm x+1


 cos58°   cos38°cosec 52° 
2  − 3
   sin32°   tan15° tan60° tan75° 

 cos (90° − 32°)  P x = 24 Q


= 2 
 sin32°  OQ – PQ =1
 cos (90° − 52°) cosec 52°  Let PQ =x
 − 3 
 tan (90° − 75°) tan60° tan75°  ∴ OQ =x+1
 sin32°   sin52° cosec 52°  By Pythagoras theorem,
= 2  − 3
 sin32°   cot 75° tan60° tan75° 
 OQ = OP + PQ
 1 
= 2− 3  (x + 1)² = 7² + x²
 3
= 2–1 x² + 1 + 2x = 49 + x²
= 1  2x = 48

1   x = 24
18. tanq + =2
tanq ∴ PQ = 24 cm and OQ = 25 cm
On squarring both sides, we get
OP
1 1 sinQ =
tan²q + + 2 tanq =4 OQ
tan q
2
tanq 7
1 =
tan²q + +2=4 25
tan2 θ PQ
cosQ =
1 OQ
tan²q + =4–2
tan2 q 24
=
1 25
=2
tan2 θ
21. Consider
2 tan67° sin 40°
19. − − tan0° (secq – tanq)²
cot 23° cos50°
2

2 tan ( 90° − 23° ) sin ( 90° − 50° ) =  1 − sin q 


= − − tan0°  cos q cos q 
cot 23° cos50°
2
2 cot 23° cos50° 1 − sin q 
= − − tan0° = 
cot 23° cos50°  cos q 

(1− sinq )
2
=2–1=0
=
= 1 cos2 q
146 ADDITIONAL
R

PR ACTICE MATHEMATICS - 10
24. cosecq + cotq = p (i)
(1- sinq )
2

= Consider
1 - sin2 q
cosec²q – cot²q = 1
(1- sinθ )(1- sinθ ) (cosecq – cotq) (cosecq + cotq) = 1
=
(1- sinθ )(1+ sinθ )
1 − sinq 1
= (cosecq – cotq) = (ii)
1 + sinq p
On adding (i) and (ii), we get
22. Consider
1
2 cosecq = p +
sec q - tanq p

sec q + tanq 1 1
cosecq =  p − 
=
( sec θ - tanθ )( sec θ - tanθ ) 2 p
(sec θ + tanθ )(sec θ - tanθ ) On subtracting (i) and (ii), we get

(sec q − tanq )
2
1
= – 2 cotq = –p
sec 2 q − tan2 q p

sec 2 q + tan2 q - 2 sec q tanq 1 1


= cotq =p-
1 2Ł pł
= 1 + tan²q + tan²q – 2 secq tanq cot q
∴ cosq =
cosecq
= 1 – 2 secq tanq + 2 tan²q
1 1
p-
2Ł pł
cos2 20 + cos2 70 =
23.   + 2 cosec58° 1 1
sec 2 50 - cot 2 40 p+
2Ł pł
– 2 cot58° tan32° – (4 tan13° tan37° tan45°
tan53° tan77°) p2 - 1
= 2
p +1
cos2 ( 90 - 70 ) + cos2 70
= + 2 cosec58°
sec 2 ( 90 - 40 ) - cot 2 40 25. tanq =
1
7
– 2 tan32° cot (90° – 32°) – 4 tan (90° – 77°)
tan (90° – 53°) (1) tan53° tan77° cosec 2q - sec 2 q

cosec 2q + sec 2 q
sin2 70 + cos2 70
=
cosec 2 40 - cot 2 40
+ 2 cosec 58°
=
(1+ cot θ ) − (1+ tan θ )
2 2

– 2 tan32° cosec32° – 4 cot77° cot53° tan53°


(1+ cot θ ) + (1+ tan θ )
2 2

tan77° = cot 2 q − tan2 q


2 + cot 2 q − tan2 q
= 1 + 2 cosec58° – 2 sec 32° – 4 2

( 7)  1 
2
−
 7 
= 1 + 2 cosec (90° – 32°) – 2 sec 32° – 4
=
= 1 + 2 sec32° – 2 sec 32° – 4 2

( 7)  1 
2
2+ +
= –3  7 
ADDITIONAL
R

PR ACTICE MATHEMATICS - 10 147


1 27.
7- A
= 7
1
2+7+
7

a
2
48 7 48 ak

+
= × =

b
2
7 64 64

k
12
=
16
3 B bk C
=
4 1
sinq =
26. C a2 + b2
AB
=
AC
k
2k

Let AB = ak, AC = a2 + b2 k
By Pythagoras theorem,

AC² = AB² + BC²


B k A
(a² + b²) k² = a²k² + BC²
2 AC
cosec A = =
1 BC BC² = b²k²
Let AC = 2k
BC = bk
BC = k
BC
\ cosq =
By Pythagoras theorem, AC
AC² = AB² + BC² b
=
2k² = AB² + k² a2 + b2

AB² = 2k² – k² = k² AB a
tanq = =
BC b
AB = k

So, 2 sin2 A + 3cot 2 A 28. Consider = sin6A + cos6A


4 tan2 A - cos2 A

( ) ( 
)
3 3
=  sin A + cos A 
2 2
2
 1   
+ 3 (1)
2
2  = (sin²A + cos²A) (sin A + cos4A – sin²A cos²A)
4
=  2
4 (1) − ( 2)
2 2
= (sin4A + cos4A – sin²A cos²A)
1+ 3 
( ) 2 
2
=  sin A + cos A − 3sin A cos A 
2 2 2
=  
4-2
= 1 – 3 sin²A cos²A
= 2

148 ADDITIONAL
R

PR ACTICE MATHEMATICS - 10
29. 5 tan x = 4 Case 1 Case 2
4 1 1
 tan x = secq = x +
5 secq = x +
4x 4x
Consider 5sin x - 3cos x tanq = x – 1 
tanq = –  x −
1 
5sin x + 2 cos x 4x  4 x 
So, So,
5sin x - 3cos x
secq + tanq secq + tanq
= cos x
5sin x + 2 cos x 1 1
= 2x =x+ –x+
cos x 4x 4x
2
5 tan x - 3 =
= 4x
5 tan x + 2 1
=
2x
4
5  − 3
5 1
= \ secq + tanq = 2x or
4 2x
5  + 2
5 15
31. cotq =
8
4 −3 1
= = BC
4+2 6 =
AB
A
Section-D
1
30. secq = x +
4x
8k 17k
We know that sec²q – tan²q = 1
tan²q = sec²q – 1
q
2
B 15k C
=  x + 1  −1
 4x  Let BC = 15k
2
 1   1 
= x 2 +   + 2 x   −1 AB = 8k
 4x   4x 
2
By Pythagoras theorem,
= x +  1  − 1
2

 4x  2 AC² = AB² + BC²


2
 1  = 64k² + 225k²
= x −
 4 x 
= 289k²
 1 
So, tan²q =  x −
 4 x  \ AC = 17k²

 1 
(a) Consider
(2 + 2 sinθ )(1− sinθ )
\ tanq = ±  x −
 4 x  (1+ cosθ )(2 − 2 cosθ )
ADDITIONAL
R

PR ACTICE MATHEMATICS - 10 149


2 (1 + sinq )(1 − sinq ) cos A
= m=n
2 (1 + cos q )(1 − cos q ) cos B
n
1 − sin2 θ ⇒ cos B = cos A (iii)
= m
1 − cos2 θ
On putting value of sin B and cos B from (ii)
cos2 q and (iii) in cos² B + sin² B = 1, we get
=
sin2 q
= cot²q n2 1
2 cos² A + sin² A = 1
m m2
225
= n² cos² A + sin² A = m²
64
cosec 2q - cot 2 q n² cos² A + 1 – cos² A = m²
(b)
cosec 2q + cot 2 q
n² cos² A – cos² A = m² – 1
1
=
cosec q + cot 2 q
2
(n² – 1) cos² A = m² – 1
1
= m2 −1
2
17  15 
2
\ cos² A =
n2 − 1
 8  +  8 
33. cosec q – sinq = l
64
= secq – cosq   = m
289 + 225
64 Consider
=
514 l² – m² (l² + m² + 3)
32 = (cosecq – sinq)² (secq – cosq)²[(cosecq –
=
257 sinq)² (secq – cosq)²] + 3
(c) sec²q + tan²q
2 2
2 2  1   1 
17   8  =  − sin q   − cos q 
=   +    sin q   cos q 
15  15 
 1 2
  1  
2
289 64      − sin q  +  − cos q   + 3
= +
225 225   sin q   cos q  

2 2
353 1 − sin θ 
2
1 − cos θ 
2
= =   =
225  sin θ   cos θ 

32. tan A = n tan B  1 − sin2 θ  2 1 − cos2 θ  2 



    + + 3

sin A sin B
(i)   sin θ   cos θ  
=n
cos A cos B 2 2
 cos2 q   sin2 q 
Also, sinA = m sinB = 
 sin q   cos q 
sin A
⇒ =m (ii)   cos2 θ  2  sin2 θ  2 
sin B  + + 3
     sin θ   cos θ  
From (i), and (ii), we get 
150 ADDITIONAL
R

PR ACTICE MATHEMATICS - 10
we get
cos4 θ sin4 θ  cos4 θ sin4 θ 
=  2 + + 3
sin θ cos θ  sin θ cos θ
2 2 2
 (secA + tanA)² (secB + tanB)² (secC + tanC)²
 cos6 q + sin6 q + 3sin2 q cos2 q  = (sec²A – tan²A) (sec²B – tan²B) (sec²C – tan²C)
= sin²q cos²q  
 sin2 q cos2 q  = 1
= (sin²q)³ + (cos²q)³ + 3 sin²q cos²q
\ (secA + tanA) (secB + tanB) (secC + tanC) = ±1
= (sin²q + cos²q) (sin4q + cos4q – sin²q cos²q)
+ 3 sin²q cos²q 36. x sin³q + y cos³q = sinq cosq
= sin q + cos q + 2 sin²q cos²q
4 4
⇒ x sin³q + cos²q (y cosq) = sinq cosq
= (sin²q + cos²q)²
⇒ x sin³q + cos²q x sinq = sinq cosq
= 1²
= 1 [ ... y cosq = x sinq ]
⇒ x sinq + (sin²q + cos²q) = sinq cosq
cos a cos a
34. = m and =n
cos b sin b ⇒ x sinq = sinq cosq
Consider ⇒ x = cosq
  (m² + n²) cos²b x sinq cos q sinq
\ y= = = sinq
cos q cos q
 cos2 a cos2 a 
= cos²b Also, we know that
 cos2 b + sin2 b 
sin²q + cos²q = 1
 cos2 a sin2 b + cos2 b cos2 a 
= cos²b
 cos2 b sin2 b  ⇒  y² + x² = 1
cos2 a sin2 b + cos2 b cos2 a ⇒  x² + y² = 1
=
sin2 b
37. cosecq – sinq = m
2
 cos a 
= 
 sin b  ⇒
1
– sinq = m
= n² sinq
1 - sin2 q
35. (secA + tanA) (secB + tanB) (secC + tanC) ⇒   =m
sinq
= (secA – tanA) (secB – tanB) (secC – tanC) (i) cos2 q
⇒   =m
On multiplying both side of (i) by (secA – sinq
tanA) (secB – tanB) (secC – tanC), we get Also, secq – cosq = n
(sec²A – tan²A) (sec²B – tan²B) (sec²C – tan²C)
1
– cosq = n
  = (secA – tanA)² (secB – tanB)² (secC – tanC)² cos q
⇒ 1 = (secA – tanA)² (secB – tanB)² (secC – tanC)² 1 - cos2 q
=n
⇒ (secA – tanA) (secB – tanB) (secC – tanC) = ±1 cos q
sin2 q
Again, Multiplying both sides of (i) by =n
cos q
(secA + tanA) (secB + tanB) (secC + tanC) So,
ADDITIONAL
R

PR ACTICE MATHEMATICS - 10 151


LHS ( ) (
= m2 n
2
3
+ mn2 )
2
3 = (
2 − a2 + 1+1− a2 )
=  cos θ sin θ  +  cos θ sin θ 
4 2 3 2
2
4 3
2
= (
2 − a2 + 2 − a2 )
 sin2 θ cos θ   sin θ cos2 θ  3

2 2
= (2 − a )
2 2

= ( cos q ) + (sin q )
3 3 3 3
CASE STUDY-1
= cos²θ + sin²θ
= 1 (i) (b) cos (A + B) = 0
cos (A + B) = cos 90o
38. a secθ + b tanθ + c = 0
A + B = 90
p secθ + q tanθ + r = 0
(A = 90 – B) ...(i)
To prove: sin (A – B) = sin (90 – B – B)
(br – qc)² – (pc – ar)² = (aq – pb)² = sin (90 – 2B)
Consider As sin (90 – θ) = cosθ
(br – qc)² – (pc – ar)² ∴ sin (90 – 2B) = cos 2B
(ii) (c) Cos 9A = Sin A
= [ b (– p secθ – q tanθ) + q (a secθ + b
tanθ)]² – [p (– a secθ – b tanθ) + a (p secθ As cos (90 – 0) = sin θ
+ q tanθ)]² ∴ cos (90 – A) = sin A
= [ – bp secθ – bq tanθ + aq secθ + bq tanθ ]² – cos 9A = cos (90 – A)
[ – ap secθ – bp tanθ + ap secθ + aq tanθ]² 9A = 90 – A
= [ secθ (aq – bp)]² – [(aq – bp) tanθ]² A = 9
= (aq – bp)² (sec²θ – tan²θ) sin5 A
tan 5A =
cos5 A
= (aq – bp)²
sin5(9)
=
39. tan²θ = 1 – a² cos5(9)

Consider sin 45
tan 5A =
cos 45
(secθ + tan³θ cosecθ)
1
= 1 + tan2 q + tan²θ tanθ cosecθ 2
=
= 1
1 + tan2 q + tan²θ tanθ 1 + cot 2 q
2
1
= 1+1− a2 + 1− a2 ( ) 1− a2 1+
tan2 q
= 1
(iii) (b) sin (45 + A) – cos (45o – A)
o

1 ... cos (90 – θ) = sinθ


= (
2 - a2 + 1- a2 ) 1- a2 1+
1- a2 ∴ sin (45 + A) = cos [90 – (45 + A)]
1- a2 = cos [45 – A]
= (
2 - a2 + 1- a2 ) 2 - a2
Hence, sin (45 + A) – cos (45 – A)
1- a2
= (
2 − a2 + 1− a2 ) 2 − a2
= cos (45 – A) – cos (45 – A) = 0
152 ADDITIONAL
PR ACTICE
R

MATHEMATICS - 10
(iv) (d) A + B + C = 180 1 cos2 q
= -
A + B = 180 – C 3sin2 q 3sin2 q
A+B 180 - c
∴ cosec = cosec 1 1 − cos q 2 
Ł 2 ł Ł 2 ł =  
3  sin2 q 
 c
= cosec  90 −  1  sin2 q 
 2 =  2 
3  sin q 
c
= sec 1
2 =
3
(v) (d) tan10° tan75° tan15° tan80°
tan10° = cot (90 – 10) (iv) (c) sinA – cosA = 0
= cot80° sinA = cosA ...(i)
tan75° = cot (90 – 75) As sin (90 – q) = cosq
= cot15° ∴ sin A = sin (90 – A)
∴ tan10° tan75° tan15° tan80° A = 90 – A
= cot80° cot15° tan15° tan80° 2A = 90
As, tanq cotq = 1 A = 45°
∴ cot80° cot15° tan15° tan80° = 1 sin4A + cos4A = sin445 + cos445
4 4
 1   1 
=  +
cASe StUDY-2  2   2 
1 1
sinq = +
(i) (c) tanq = 4 4
cos q
1
sinq =
= 2
1 − sin2 q (v) (c) sinq + cosecq = 2
(ii) (b) tanA + cotA = 4 1
cosecq =
Squaring both sides sinq
1
tan2A + cot2A + 2tanA cot A = 16 sinq + = 2
sinq
tan2A + cot2A + 2 = 16
sin2q + 1 = 2sinq
tan2A + cot2A = 14
Squaring both sides Let sinq = x
tan4A + cot4A + 2tan4A cot4A = 196 x2 + 1 = 2x
tan4A + cot4A = 196 – 2 ⇒ x2 – 2x + 1 = 0
tan4A + cot4A = 194 ⇒ x2 – x – x + 1 = 0
(iii) (d) 9x2 = cosec2q ⇒ x (x – 1) –1 (x – 1) = 0
9 ⇒ (x – 1)2 = 0
= cot2q
x2 x = 1
 1  3 sinq = 1
3  x 2 − 2  =  3x 2 − 2 
 x   x  q = 90o
cosec 2q cot 2 q sin19q + cosec19q = (1)19 + (1)19
= -
3 3 = 1+1 = 2
ADDITIONAL
R

PR ACTICE MATHEMATICS - 10 153


Chapter Some Application of
9 Trigonometry
Multiple choice Questions

3. (d) A
1. (d) A

B 60° C θ
100 C
B 3x
AB AB x 1
tan 60° = = =
BC tan θ =
BC 3x 3
AB θ = 30°
3 =
100
4. (b) A
∴ AB = 100 3 m
2. (c)
A

60°
30 m
B 2.5 m C
θ
B E BC
1.5 m Cos 60° =
1.5 m AC
C 28. 5 m D 1 2.5
=
2 AC
AB = AC – BC
AC = 5 m
= 30 – 1. 5 A
5. (a)
= 28.5 m
BE = CD = 28.5 m
In ∆ABE,
30°
AB 28.5 B E
tanθ = = =1 1.7 m 1.7 m
BE 28.5
⇒ θ = 45° C 20 3
D

154 ADDITIONAL
PR ACTICE
R

MATHEMATICS - 10
In ∆ABE, BE = CD = 20 3 m 2. A E

AB
tan 30° =
BE x y
1 AB
= 30° 60°
3 20 3
B C D
∴ AB = 20 m
Let AB and DE denote two towers
So, AC = AB + BC = 20 + 1. 7
In ∆ABC,
= 21. 7 m
AB
tan 30° =
BC
WoRKSHeet - 1 1 x
=
3 BC
Section-A ⇒ BC = 3
1. A In ∆CDE,
DE
tan 60° =
CD
3 y
=
CD
D
y
h Metres CD =
60° 30° 3
B C As BC = CD
Let AB denotes the tower. y
∴ 3x =
3
In ∆BCD, 3x = y
CD ⇒ x :y = 1 :3
tan 60° =
BC
3 h 3. In ∆ABC,
=
BC AB 20 1
h tan C = - =
BC 20 3 3
BC = Metre
3 ∴ C = 30°
In ∆ABC,
AB 4. A
tan 30° =
BC
1 E
AB
=
3 h
1 1.5 m
3
= θ
3 h D
B 3m C 4.5 m
h
AB = Metre Let CE denotes the boy and AB denotes a
3
ADDITIONAL
R

PR ACTICE MATHEMATICS - 10 155


lamp-post.
5a 3
=
In ∆DCE, 5a
CE 1.5 1 = 3
tan θ = = =
CD 4.5 3
In ∆ABC, θ = 60°
AB
tan θ =
BD 8. A
1 AB
=
3 7.5
∴ AB = 2. 5 m

5. In ∆ABC,
BC 45° 30°
sin 45° =
AC B x D  20 m C
1 BC
= Let AB denotes the chimney.
2 150
150 Let BD = x metre
BC =
2
In ∆ABC,
= 75 2 m
AB
tan 30° =
6. A BC
1 AB
=
3 x + 20
1
AB = (x + 20) ...(i)
3
θ In ∆ABC,
B C
AB
tan 45° =
Let AB denotes the vertical pole and BC BD
denotes the shadow of the pole.
AB
1 =
Let AB = BC = x x
In ∆ABC, AB = x ...(ii)
AB x From (i) and (ii),
tan θ = = =1
BC x
∴ θ = 45° 1
AB = x = (x + 20)
3
7. In ∆BAC, 3 x – x = 20
AB 20
tan θ = x =
AC 3 -1

156 ADDITIONAL
R

PR ACTICE MATHEMATICS - 10
Section-B
=
20
2
( 3 + 1) 11. D
x = 10 ( 3 +1)
A
∴ AB = x = 10 ( 3 + 1) 12 m

9. A
60°
B C

Let BD denotes a tree broken by the wind at


point A. Such that its top touches the ground
at point C
45°
So, AD = AC
B 10 m C
Let AB = x metre
Let AB denotes the tower and BC denotes
the shadow AD = 12 m
In ∆ABC, AB + AD = 12
AB x + AD = 12
tan 45° =
BC
AD = 12 – x
AB
1 =
10 AC = 12 – x (... AD = AC)

AB = 10 m In ∆ABC,
AB
10. A sin 60° =
AC

3 x
21 m =
2 12 - x
30° 2x = 12 3 - 3x
B C
Let AC denotes the string of kite.
x ( 3+2 ) = 12 3

In ∆ABC, 12 3
x =
AB 3+2
sin 30° =
AC
1
=
21 =
12 3
−1
( 3−2 )
2 AC
AC = 42 m (
= 12 3 2 - 3 m )
ADDITIONAL
R

PR ACTICE MATHEMATICS - 10 157


12. Let the window be at point E and AC be the
A house.
30°
60° Let
1200 m To find : AC
In ∆CDE,
60° 30°
B D DE
C tan 45° =
CD
Let the aeroplane be at point A and C and D 15
denote two ships sailing towards the aeroplane. 1 =
CD
To find : CD ∴ CD = 15 m

In ∆ABC, ∴ BE = CD  = 15 m

AB In ∆ABE,
tan 60° =
BC AB
1200 tan 30° =
= BE
3 BC 1 AB
1200 =
3 15
BC =
3
15
= 400 ∴ AB = =5 3 m
3 m 3
In ∆ABD, Also, BC = DE =15 m
AB ∴   AC = AB = BC
tan 30° =
BD
AC = 5 3 + 15
1 1200
= = 5( 3 + 3) m
3 BC + CD

1 1200 = 5 (1. 732 + 3) m


=
3 400 3 + CD = 5 x 4. 732

CD = 1200 3 – 400 3 = 23. 66 m

= 800 3 14.

A
13. A

30°
E 45° B

15 m 30° 45°
45° P B Q
D C 100 m
158 ADDITIONAL
PR ACTICE
R

MATHEMATICS - 10
Let AB denotes the tree. Let AB denotes the tree.
Let BP = x metre such that To prove : AB = ab metres
BQ = PQ – BP In ∆ABC,
= 100 – x AB
tan θ =
To find AB BC
In ∆ABQ, AB
tan θ =
a
AB
tan 45° = ∴ AB = a tan θ (i)
BQ
AB In ∆ABD,
1 =
100 - x
∴ AB = 100 – x (i) AB
tan (90 – θ) =
BD
In ∆ABP,
AB
AB cot θ =
tan 30° = b
PB
∴ AB = b cot θ (ii)
1 AB
= From (i) and (ii)
3 x
x AB = a tan θ = b cot θ
AB = (ii)
3 b
From (i) and (ii), tan2θ =
a
x
AB = 100 – x = b
3 ∴ tan θ =
a
x
100 = x+ So, AB = a tan θ
3
100 3 = x ( 3 + 1) = a b
a
100 3
x = = ab metres
3 +1

=
100 3
2
( 3 −1 ) 16. A

= 50 3 ( 3 −1 ) D

15. A 50 m

30° 60°
B C
θ 90°– θ
B a C D Let AB denotes the hill and CD denotes the
b tower such that CD = 50 m
ADDITIONAL
R

PR ACTICE MATHEMATICS - 10 159


In ∆BCD, ⇒   PS + b = b tan a cot b
CD ⇒  PS = b tan a cot b – b
tan 30° =
BC
So, PQ = PS + SQ
1 50
3
= = b tan a cot b – b + b
BC
= b tan a cot b
∴ BC = 50 3 m
18. A
In ∆ABC,
AB 4000 m B
tan 60° =
BC 60°
AB
3 = 45°
50 3 C D
∴ AB = 150 m
Let the two planes be at points A and B
respectively
17. P
To find : AB

R S Let AB = x
b
∴  BC = AC – AB
b b
In ∆BCD,
a b BC
A Q tan 45° =
CD
Let PQ denotes the tower 4000 - x
1 =
CD
To prove : PQ = b tana cotb ∴   CD = 4000 – x (i)
QS = AR = b ft In ∆ACD,
In ∆RAQ, AC
tan 60° =
AR CD
tanb =
AQ 4000
b 3 = (From (i))
4000 − x
tanb =
AQ
4000 3 – 3 x = 4000
AQ = b cotb
In ∆PQA, 3x = 4000 3 – 4000

PQ 4000
tana = x =
( 3 – 1)
AQ 3
PS + QS 4000
tana = = 3 ( 3 – 1)
cot b 3
PS + b 4000
tana = ∴   AB = x = 3 ( 3 – 1) metres
b cot b 3
160 ADDITIONAL
R

PR ACTICE MATHEMATICS - 10
Section-C Let AD be the ladder such that when it's foot
19. is pulled away from the wall through a distance
B r 'a' such that it slides a distance 'b' down the
C wall making an angle 'b' with the horizontal,
r the ladder comes to position BE.

a A ∴ AD = BE
b a cos a - cos b
To prove : =
P Q b sin b - sin a

Let P be the eye of observer. Proof : In ∆ACD,


As PB is tangent to circle at point B CD
cosa =
∴ ∠CBP = 90° AD
AC
Let CA = CB = r sina =
AD
Let h be the height of the center C.
b + BC
Also, let ∠APB = a =
AD
a In ∆BCE,
⇒ ∠APC = ∠BPC =
2 BC
In ∆CBP, sinb =
BE
a BC r CE
sin = = cosb =
2 PC CP BE
a
⇒ CP = r cosec (i) CD + a
2 =
BE
In ∆CQP,
Consider
CQ
sin b = cos a - cos b
CP
sin b - sin a
⇒ CQ = CP sin b
a CD CD + a
= r cosec sin b (From (i)) -
2 = AD BE
∴ height of the centre BC b + BC
-
a BE AD
= r cosec sin b
2 CD - CD - a
=
20. A BC - b - BC
[ As AD = BE ]
b
B -a
=
-b
a b a
C D a E =
b

ADDITIONAL
R

PR ACTICE MATHEMATICS - 10 161


21. A In AB denotes the hill such that the two
stones are at points C and D
60°
E B Let BC = x km
45°
25 m ∴   BD = x + 1 km

45° In ∆ABC,
D C
AB
Let DE denotes the ship and AC denotes the tan 45° =
BC
lighthouse. AB
1 =
Man is standing at point E. x
In ∆EDC, ∴ AB = x ...(i)
DE In ∆ABD,
tan 45° =
CD AB
tan 30° =
25 BD
1 =
CD 1 x
= (From (i))
∴ CD = 25 m 3 x +1
⇒ BE = CD = 25 m ⇒ x+1 = 3x

In ∆ABE, ⇒ 1 = x ( 3 – 1)
AB 1
tan 60° = ⇒ x = km
BE 3 -1
AB 1
3 = So, AB = x = km
25 3 -1
⇒ AB = 25 3 m
3 +1
∴ Height of the lighthouse = km
2
= AC 1.732 + 1
= km
2
= AB + BC
2.732
= 25 3 + 25 = km
2
[ BC = DE = 25 m ] = 1. 366 km

= 25 ( 3 + 1) m
Section-D
22. A 30° 23. A 30°
45° 45°

100 m

45° 30° 45° 30°


B C D B C D
162 ADDITIONAL
R

PR ACTICE MATHEMATICS - 10
Let the light house be at point A. CD denotes PQ
the distance travelled by the ship during the tan 60° =
PX
period of observation.
PS + SQ
In ∆ABC, 3 =
PX
AB 40 + QS
tan 45° = ∴ 3 = [ As PX = SY ]
BC SY
100 40 + QS
1 = 3 = (From (i))
BC QS
∴ BC = 100 m 3 QS = 40 + QS

In ∆ABD, QS ( 3 – 1) = 40
AB 40
tan 30° = QS = –1
BD 3
1 100 = 20 ( 3 + 1) m
=
3 BC + CD So, PQ = PS + SQ
BC + CD = 100 3 = 40 + 20( 3 + 1)
100 + CD = 100 3 = 20 ( 3 + 3) m
CD = 100 ( 3 – 1) m In ∆QPX,

24. Q PQ
sin 60° =
XQ

3
=
20 ( 3 +3 )
45° 2 XQ
S Y
⇒ XQ =
2
(20 ) ( 3 +3 )
40 m

P
60°
X
=
40
3
3 ( 3 +1)
PQ denotes the tower = 40 ( 3 +1 m )
PS = XY = 40 m
25. A
In ∆QSY,
h
QS B
tan 45° =
SY
QS b
1 = a
SY C D
QS = SY (i)
Let BC denotes the tower and AB denotes
In ∆QPX, the flag.
ADDITIONAL
PR ACTICE
R

MATHEMATICS - 10 163
h tan a In ∆DOE,
To prove : BC = OE
tan b - tan a tan 60° =
In ∆BCD, OD
BC 60 + BE
tana = 3 = [ From (i) ]
CD 3 AO
∴ BC = CD tana (i)
⇒ 3 AO = 60 + BE
In ∆ACD,
∴   3 AO = 60 + AO + 60
AC
tanb = [ ... BE = AB = AO + 60 ]
CD
AB + BC 2 AO = 120
tanb =
CD
AO = 60 m
h + BC
=
CD So, height of cloud = AB
h + CD tan a
tanb = [ From (i)] = AO + OB
CD
CD tanb = h + CD tana = 60 + 60

h = 120 m
∴ CD =
tan b - tan a 27. A 30°
From (i), BC = CD tana
45°
h tan a
=
tan b - tan a

26. A 45° 30°


B C D

Let speed of car be x m/s


30°
D O So, CD = 12 x 60 x
60°
60 m 60 m = 720 x metre
C In ∆ABC,
B
AB
tan 45° =
BC
AB
1 =
BC
E
In ∆AOD, ∴ AB = BC (i)

AO In ∆ABD,
tan 30° =
OD AB
1 AO tan 30° =
= BD
3 OD 1 AB
⇒ OD = 3 AO ...(i) =
3 BC + CD

164 ADDITIONAL
R

PR ACTICE MATHEMATICS - 10
1 BC 1 1500 3
= [ From (i) ] =
3 BC + 720 x 3 AB + 15 x

⇒ 3 BC = BC + 720 x 1 1500 3
=
3 1500 + 15x
720 x
⇒ BC = 1500 + 15x = 1500 x 3
3 −1
= 360 ( 3 + 1) x metre 15x = 4500 – 1500]

New, time taken to travel distance BC 15x = 3000

720 x x = 200 m/s


=
( 3 −1 x) =
200
× 3600
1000
720
= = 720 km/hr
3 -1
= 360 ( 3 + 1) seconds
WoRKSHeet - 2
= 360 (1. 732 + 1)
= 360 (2. 732)
Section-A
1.
= 983. 52 seconds A

= 984 seconds

28. D E
30°
B 20 m C
1500 3 m Let AB denotes the tower
In ∆ABC,
60° 30°
A B 15x C AD
tan 30° =
Let the speed of the jet plane be x m/s BC
1 AB
Distance BC = 15 x metre =
3 20
In ∆ABD, 20
AB =
BD 3
tan 60° =
AB 20 3
=
1500 3 3
3 = A
AB 2.
∴ AB = 1500 m
In ∆ACE, 10 m
CE
tan 30° = 45°
AC B C
ADDITIONAL
PR ACTICE
R

MATHEMATICS - 10 165
Let AB and AC denote the vertical pole and Let AB denotes the wall and AC denotes the
wire respectively. ladder.
In ∆ABC, In ∆ABC,
AB
sin 45° = AB
AC tan 60° =
BC
1 10 AB
= 3 =
2 AC 1.5
∴ AB = 1.5 3 m
∴ AC = 10 2 m

3. BD = AB – AD 6. A

= 6 – 2. 54
= 3. 46 m 3k
In ∆CBD,
BD θ
sin 60° = B k C
CD
3 3.46 Let AB denotes the tower and BC denotes
= the shadow of tower.
2 CD
3.46 × 2 AB : BC = 3:1
∴ CD =
3
Let AB = 3k
3.46 × 2
= BC = k
1.73

=4m In ∆ABC,
AB
4. AB denotes the pole and BC denotes shadow tan θ =
BC
of the pole.
3k
In ∆ABC, =
k
AB
tan 60° = = 3
BC
AB ∴ θ = 60°
3 =
20 3
7.
∴ AB = 60 m
A
5. A
215 m

B 60° C
60° Let the ball be at point A and AC denotes the
B 1.5 m C cable.
166 ADDITIONAL
R

PR ACTICE MATHEMATICS - 10
To find : AB CD
tan 60° =
In ∆ABC, BC
100
AB 3 =
sin 60° = BC
AC
100
3 AB BC =
= 3
2 215
100
215 = 3 m
AB = 3 m 3
2 In ∆CBA,
AB
8. tan 30° =
A BC
1 AB
=
3 100 3
3
60° 1 3 AB
=
B 2m C 3 100 3
Let AB denotes the wall and AC denotes the 3 AB = 100
ladder.
100
To find : AC AB = m
3
In ∆ABC, 100
So, height of building = m
3
BC
cos 60° = 10. A
AC
1 2
= B
2 AC
∴ AC = 4m 10 m
C 60° 30° D

Section-B Let BC and AB denote the building and tower


respectively.
9.
D In ∆BCD
BC
tan 30° =
CD
A
100 m 1 10
=
3 CD
∴ CD = 10 3 m
60° 30°
B C In ∆ACD,
AC
Let AB and CD denote building and tower tan 60° =
CD
respectively.
10 + AB
3 =
In ∆BCD, 10 3
ADDITIONAL
PR ACTICE
R

MATHEMATICS - 10 167
30 = AB + 10 In ∆ABC,
AB = 30 – 10 AB
sin 30° =
AC
= 20 m
1 1.5
=
So, height of tower = AB 2 AC
= 20 m ∴ AC =3m
In ∆PQR,
11. A
30° 45° PQ
sin 60° =
PR
100 m 3 3
=
30° 45° 2 PR
B C D 6
∴ PR =
Let AC denotes the tower and the two buses 3
be at points B and D respectively. = 2 3 m
To find : BD
In ∆ABC, 13.
AC A
tan 30° =
BC
1 100 30°
= E B h
3 BC
60 m
∴ BC = 100 3
In ∆ACD,
D 140 m C
AC
tan 45° = Let AC and DE denote two towers.
CD
100 In ∆ABE,
1 =
CD
AB
CD = 100 m tan 30° =
BE
So, BD = BC + CD 1 AB
=
= 100 3 + 100 3 140
[ .. BE = CD = 140 m ]
.
= 100 ( 3 + 1) m
140
∴ AB = m
12. 3
A P
So, AC = BC + AB

1.5 m 140
3m = 60 +
3
30° 60° 140
∴ DE = BC = 60 + m
B C Q R 3
168 ADDITIONAL
R

PR ACTICE MATHEMATICS - 10
14. A Let AB = x
In DABE
x
AB
tan 30o =
60° 30° BE
B xm C D 1 x
(y+40) = ⇒ x = 20 m
3 20 3
Let AB is the height of tower = x The height of tower AC is AB + BC i.e. 20 m +
1.7 m = 21.7 m
In DABC
16. A
AB x
tan 60o = =
BC y

0m
x=y 3 ...(i)

10
In DABD C 45°
G
AB x 30° 20 m 20 m
tan 30o = =
BD y + 40 B D E
1 x
= Let the girl is standing at point G and the boy
3 y + 40
is standing at point B.
(y + 40) = x 3
In DADB
y + 40 = (y 3) 3

AD
= sin 30
y + 40 = 3y BA
AC CD 1 AC + 20
2y = 40 = ⇒ =
1
100 2 50
y = 20 ⇒ AC = 30 m

\ x = 20 3 m In DACG

= 20 × 1.73 m AC 30 1
sin 45 = = =
AG AG 2
= 24.6 m
⇒ AG = 30 2 m

A
15.
17. A φ
30° θ
E B

96 m
1.7 m 1.7 m
θ φ
B C D

D C Let CB and CD are the distance of cars from


20 3 the foot of tower respectively.

ADDITIONAL
PR ACTICE
R

MATHEMATICS - 10 169
3 1 In DABE
tanq = and tanφ =
4 3 AB
= tan 30
In DABC In DABD BE
AB AB AB
tanq = tanφ = = AB 1 15 15 × 3
BC BD BC + CD = = AB = m=
15 3 3 3
3 96 1 96
= and =
4 BC 3 128 + CD = 5 3 m
96 × 4 AB + BC = AC = 24 m
BC = ⇒ 128 + CD = 288
3
CD = 160 m 5 3 + BC = 24
The distance between BC = 24 – 5 3
cars is CD = 160 m
Let ED be the height of first pole.
18. A
From the figure, BC = ED
y
B 20. A
y+x

x
h
60°
45° D
C 120 m x 90 – x
B D C
In DBCD 6m 7.5 m
x 13.5 m
tan 45 = ⇒ x = 120 m
120 Let AB be the tower
In DACD
In DABD
x+y
tan 60 =
120 h
= tan x ⇒ h = 6 tan x ...(i)
x+y 6
= 3 ⇒ 120 + y = 120 3
120 In DABC
y = 120 ( 3 – 1) m
h
= tan (90 – x)
13.5
19. 30° A h
= cot x
13.5
h = 13.5 cot x ...(ii)
30°
E B 24 m
Multiply equation (i) and (ii)
h2 = 81

D 15 m C h=9m

170 ADDITIONAL
R

PR ACTICE MATHEMATICS - 10
21. A In DABC
BC h
tan 30 = =
AC x
h h 1
= h 3 = x ...(i)
x 3
60° 45° In DBDE
B C D BE h + 4
x =
(x + 20) tan 60 =
DE x
Let AB = h be the height of tower. h+ 4
3 = ...(ii)
In DABC x
AB From equation (i) and (ii)
= tan 60
BC
h+ 4
h 3 = ⇒ 3h = h + 4 ⇒ 2h = 4
= 3 h 3
x
...(i) h=2m
h= x 3m
\ The height of balloon above the ground
In DABD
= h + 6 = 2 + 6 = 8 m.
AB
= tan 45
BD 23. B
AB = BD
h = x + 20 ...(ii) h
from equation (i) and (ii) 60o C
A 30o
x 3 = x + 20

x( 3 – 1) = 20
15 m 15 m
x =
20
Ł 3 - 1ł 3 + 1
3 +1
= 10 ( 3 +1 m)
h = 10 3 ( )
3 +1 m
E D
x
h = 30 + 10 3 m
Let BD represents the cable tower.
22. B In DABC
h

BC
A 30° = tan 60
Window 2 AC
C
h
h+4

⇒ = 3 ⇒ h= x 3 ...(i)
x
4m
In DACD
6m

15 1 15
tan 30 = ⇒ =
Window 1 60° x 3 x
D E ...(ii)
2m x = 15 3
F x G From equation (i) and (ii)
ADDITIONAL
PR ACTICE
R

MATHEMATICS - 10 171
h = (15 3 ) 3 = 45 m AC = 3 m
AC = l1 = 3m
∴ The height of tower from the ground is
(ii) (c) In DPQR
45 + 15 = 60 m.
B PQ
24. = sin60o
PR
h 3 3
=
l2 2
60o C
A 30o b
l2 =
3
= 2 3m
12 m 12 m (iii) (b) In DABC
AB
= tan30
BC
D E 1.5 1
x
=
BC 3
Let BE represents the height of cliff and DE
BC = 1.5 3 m
represents the distance between cliff and ship.
(iv) (c) In DPQR
In DABC
PQ
BC = tan60
= tan 60 QR
AC 3
h = 3
= 3 ⇒ h=x 3 QR
x QR = 3m
In DACE
(v) (b) l1 = 3m
CE
= tan 30 l2 = 2
3
AC
l1 + l2 = 3 + 2 3 m
12 1
= ⇒ x = 12 3 m
x 3
cASe StUDY-2
∴ h = x 3
(i) (b) The figure representing the condition is
h = (12 3 ) 3 ⇒ 36
A
Therefore, the height of cliff is 36 + 12 = 48m
and distance between ship and cliff is 12 3 m.

cASe StUDY-1
30o
(i) (a) In DABC
B 8m C
AB
= sin30o Height of broken tree = AC
AC
1.5 1 BC
= = cos 30
AC 2 AC
172 ADDITIONAL
PR ACTICE
R

MATHEMATICS - 10
(iv) (c) Let the height of tree is AB and length of
8 3
= shadow is BC.
AC 2 A
16
AC = m
3
(ii) (d) Height of remaining tree = AB 150
AB
= tan30
BC 30o
AB = BC tan30 B C
 1  AB
= 8  m = tan30
AC
 3
150 1
8 =
= m AC 3
3
AC = 150 3m
(iii) (a) Total height of tree is AB + AC
(v) (b) A
8 16
AB + AC = +
3 3
24
= m
3
45o
24 3 C
= m B 20 m
3× 3
AB
= tan 45
24 3 BC
= m AB
3 =1
BC
= 8 3 m AB = BC = 20 m

ADDITIONAL
R

PR ACTICE MATHEMATICS - 10 173


Chapter

10 Circle
Multiple choice Questions

( 4 − 0 ) + ( 0 − 3)
2 2
44 r
1. (b) From 37 + r = Diagonal = BD =
7
44 r = 16 + 9
37 = –r
7
= 25
37 r
37 =
7 =5
r = 7 cm 18
5. (c) Radius = = 9 cm
c = 2pr 2
\ Perimeter = 2pr
22
= 2× ×7 = 2p (9)
7
= 44 cm = 18p cm

2. (b) pr12 + pr22 = pr2 WoRKSHeet - 1


r1 + r2 = r
2 2 2

52 + (12)2 = r2 Section-A
25 + 144 = r2 θ
1. Arc length = 2π r
360
r2 = 169
θ
r = 13 cm 3p = 2π × 6
360
\ Diameter = 2r = 26 cm qp
⇒ 3p =
30
3. (b) Distance covered in one revolution
3π × 30
= 2pr ⇒ q=
π
22 35 = 90o
= 2× ×
7 2
= 110 cm 2. Diameter = 14 cm
14
A B (4, 0) ⇒ radius = = 7 cm
4. (a) 2
Perimeter of semi – circle protractor
1
= 2r + (2pr)
2
D (0, 3) C (4, 3) = 2r + pr
174 ADDITIONAL
R

PR ACTICE MATHEMATICS - 10
22 22
= 2 (7 ) + ×7 = 2 × p
7 7
= 14 + 22 Number of revolutions in covering a distance
of x metres.
= 36 cm x
= 22
3. A 2× ×r
7
7x
=
44 r
r
7. Let the diameter and a side be x units.
r x
B O r units C So, radius of circle = units
2
1 2
Area of DBAC = × BC × AO x
2 \ Area of circle = π  
2
1
= × 2r × r p x2
2 =
4
= r2 sq. units Area of equilateral triangle
4. Perimeter of sector 3
q = (side)2
2p r 4
= 2r +
360 3 2
= x
pq 4
= 2r 1 + Area of circle
Ł 360 ł \
 22 60  Area of equilateral triangle
= 2 (10.5) 1 + ×
 7 360  p x2
= 4
11
= 21 1 + 3 2
Ł 21ł x
4
21 × 32 p
= =
21
3
= 32 cm
8. C
5. r = 10 cm A
q = 108 o
a
θ 90°
area of sector = π r2 B
360° O a
108
= p (100 )
360
= 3p10
= 30p cm2 Perimeter of segment ABC
6. Distance covered in one revolution = BC + length of arc BAC
= 2pr In DBOC
ADDITIONAL
R

PR ACTICE MATHEMATICS - 10 175


BC2 = OC2 + OB2 As all the angles of triangle ABC are 60°, hence
(By Pythagoras theorem) triangle ABC is an equilateral triangle and thus
all the sides are equal.
BC2 = a2 + a2
\ ∠PAB = ∠PBA = 60°
= 2a2
\ AB = PA = PB = 5 cm
BC = 2a
11. C
Also length of arc BAC
R
90 22
= ×2× ×a D
360 7
11a Q
=
7
So, Perimeter of segment ABC S
11a
= 2a +
7 A
Section-B P
B
9. We know that AD = AF As we know that the length of tangents drawn
BD = BE from an external point to a circle are equal.
CE = CF AP = AS ...(i)
Let AD = AF = x BP = BQ ...(ii)
BD = BE = y CR = CQ ...(ii)
CE = CF = z DR = DS ...(iv)
Then x + y = 12 On adding both sides of (i), (ii), (iii) and (iv), we
y + z= 8 get
x + z = 10 AP + BP + CR + DR = AS + BQ + CQ + DS
On solving above equation we get \ (AP + BP) + (CR + DR) = (AS + DS) +
(BQ + CQ)
x = 7, y = 5, z = 3
AB + CD = BC + AD
So AD = 7, BE = 5, CF = 3
10. BP = AP = 5 cm 12. T
(The lengths of tangents drawn from an external
r Q
point to a circle are equal.)
O P
\ ∠PAB = ∠PBA r 2r
(Angle opposide to equal sides are equal.)
In PAB, S
∠P + ∠PAB + ∠PBA = 180°
∠TOP = q
(Angle sum property) As we know that radius is perpendicular to the
60° + 2 ∠PAB = 180° tangent at the point of contact.
2 ∠PAB = 120° ∠OTP = 90°
∠PAB = 60° So in DOTP
176 ADDITIONAL
R

PR ACTICE MATHEMATICS - 10
OT r 1 1 PS
cosq = = = ⇒ =
OP 2r 2 2 OP
1 ⇒ OP = 2 PS
\ cosq = cos 60°
2 14. In DOAP
⇒ q = 60° OA = 6 cm
AP = 8 cm
\ ∠TOS = 60° + 60° = 120°
\ OP2 = OA2 + AP2
As OT = OS
(By Pythagoras theorem)
⇒ ∠OTS = ∠OST = 62 + 82
(Angles opposite to equal sides are equal)
= 36 + 64
In ∠OTS, = 100
∠TOS + ∠OTS + ∠OST = 180°
⇒ OP = 10 cm
120° + 2 ∠OTS = 180° Now, In DOBP,
2 ∠OTS = 60° OP2 = OB2 + BP2
102 = 42 + BP2
∠OTS = 30°
(By Pythagoras theorem)
\ ∠OTS = ∠OST = 30°
100 = 16 + BP2
13. In DOTP and DOSP BP2 = 100 – 16
OT = OS (Radii of same circle) = 84
OP = PO (Common) \ BP = 2 21cm
PT = PS
(The lengths of tangents drawn from an external 15. ∠OAC = 90° (as radius ⊥ tangent)
point to a circle are equal) ∠BOC = ∠OAC + ∠ACO
\ DOTP ≅ DOSP (SSS) (Exterior angle property)
\ ∠OPS = ∠OPT 130° = 90° + ∠ACO
1 ∠ACO = 130° – 90° = 40°
= ∠TPS (CPCT)
2
16. D R C
1
= (120°)
2
= 60°
In DOSP, S o Q
OS ⊥ PS
(Radius is perpendicular to the tangent at the
point of contact)
PS A P B
cos (∠OPS) =
OP A rhombus is a parallelogram with all equal
sides.
PS
⇒ cos 60° = In 11 gm ABCD
OP

ADDITIONAL
R

PR ACTICE MATHEMATICS - 10 177


AB = CD and AD = BC 18. P
Hence AP = AS; BP = BQ; CR = CQ; DR = DS z
z
Adding we get AP + BP + CR + DR = AS + BQ
N
+ CQ + DS
(AP + BP) + (CR + DR) = (AS + DS) + (BQ + L
y
CQ) x
AB + CD = AD + BC Q x y R
M
AB + AB = AD + AD
QL = QM
2AB = 2AD
RM = RN
So AB = AD and AB = CD and AD = BC
So AB = CD = AD = CD PL = PN
So ABCD is 11 gm with equal sides. We know that the tangents drawn to a circle
from an external point are equal in length.
\ ABCD is a rhombus.
Let QL = QM = x
\ Proved.
Let RM = RN = y
17. D R C
Let PL = PN = z
Consider PQ + QR + PR = 60
2 3 ⇒ x + z + x + y + z + y = 60
S 1o 4 Q
8 5 ⇒ 2x + 2y + 2z = 60
7 6
⇒ x + y + z = 30
PQ = 20
A P B x + z = 20
Const : join OP, OQ, OR and OS. \ RN = 10 cm
Proof : Since, the two tangents drawn from an
Also, QR = 16
external point to a circle subtend equal angles
at the centre. x + y = 16
\ ∠1 = ∠2, ∠3 = ∠4, ∠5 = ∠6, ∠7 = ∠8 \ z = 30 – (x + y)
Since sum of all the angles subtended at a point = 30 – 16
is 360°.
= 14 cm
∠1 + ∠2 + ∠3 + ∠4 + ∠5 + ∠6 + ∠7 + ∠8
\ PL = 14 cm
= 360°
Again, PR = 24
⇒ 2 ∠2 + 2 ∠3 + 2 ∠6 + 2 ∠7 = 360°
⇒ 2 (∠2 + ∠3 + ∠6 + ∠7) = 360° y+z = 24

⇒ ∠2 + ∠3 + ∠6 + ∠7 = 180° \ x = 30 – (y + z)
⇒ (∠6 + ∠7) + (∠2 + ∠3) = 180° = 30 – 24
⇒ ∠AOB + ∠COD = 180° = 6
Similarly, we can prove ∠AOD + ∠BOC = 180° \ QM = 6 cm
178 ADDITIONAL
R

PR ACTICE MATHEMATICS - 10
Section-C Proof: ∠OCB = ∠OCA = 90° (Radius is
perpendicular to the tangents at the point of
19. P contact).
Consider DOCB and OQB
O T OC = OQ (radius of same circle)
∠OQB = ∠OCB = 90° (Radius is perpen-
dicular to the tangent at the point
Q
of contact)
In DOPT and DOTQ
BQ = 
BC (Tangents from the same
OP = OQ (Both are radius of same circle) point are equal)
OT = OT (Common) \ DOCB ≅ DOQB by SAS rule.

PT = QT (Tangents drawn from same
Consider DAPO and DAOC
points are eqaual)
OP = OC [Radius of same circle]
\ DOPT ≅ DOTQ (SSS)
∠OPT = 90° (Tangent is perpendicular to OA = OA [Common]
radius through point of contact). AP = 
AC [Tangent from the same
From the figure points are equal]
∠OPT = ∠OPQ + ∠TPQ = 90°
\ DAPO ≅ DAOC by SSS rule

⇒ ∠TPQ = 90 – ∠OPQ ∠POA + ∠COA + ∠QOB + ∠BOC = 180°
In DPTQ As DOCB is congurent to DOQB
∠PTQ + ∠TPQ + ∠TQP = 180° (angle sun
\ ∠QOB = ∠BOC

property) Similarly ∠POA = ∠COA
∠PTQ + ∠TPQ + ∠TQP = 180
\ 2∠COA + 2∠COB = 180
∠PTQ + ∠TPQ + ∠TPQ = 180
∠COA + ∠COB = 90°
[∠TPQ = ∠TQP as TP and QT are equal]
From the figure it is clear that ∠COA + ∠COB
∠PTQ + 2∠TPQ = 180
= ∠AOB.
∠PTQ + 2 (90 – ∠OPQ) = 180

\ ∠AOB = 90°
∠PTQ = 2 ∠OPQ
21.
20. P A
x mo
y 5c 3cm

P L Q
O C
Let O be the common centre of the two
concentric circle.
Let PQ be a chord of the larger circle which
x1 Q B y1
touches the smaller circle at L .
Given: A circle with center O to which XY and Join OL and OP.
XY' are tangents.
Since, the tangent at any point of a circle is
To prove: ∠AOB = 90° perpendicular to the radius through the point
Construction: Join point O to Point C. of contact.
ADDITIONAL
PR ACTICE
R

MATHEMATICS - 10 179
Therefore, Since alternate angles are equal, the lines PQ
∠OLP = 90° and RS are parallel to each other.

Now, Hence, proved.

In DOLP, we have Section-D


OP = OL + PL2 2 2
P
23. A
[Using Pythagoras theorem]
⇒ (5)2 = (3)2 + PL2
O T
⇒ 25 = 9 + PL2 E
⇒ PL2 = 16
Q B
⇒ PL = 4 cm
Since, the perpendicular from the centre of a OP = OQ = 5 cm
circle to a chord bisects the chord.
OT = 13 cm
Therefore,
∠OPT = ∠OQT = 90°
PL = LQ = 4 cm
[ ... radius is perpendicular to tangent at the
\ PQ = 2 PL = 2 × 4 = 8 cm point of contact]
Hence, the required length = 8 cm. In DOPT
22. P A Q OT2 = OP2 + PT2 ⇒ PT2 = OT2 – OP2
= 169 – 25 = 144
PT = 12 cm
O
AP = AE [tangents drawn from same point are
equal in length]
Similarly, BE = BQ
R B S
∠AEO = ∠AET = 90° [Radius is
Let AB be the diameter of a circle, with centre
perpendicular to the tangent
O. The tangents PQ and RS are drawn at point
at the point of contact]
A and B, respectively.
\ DAET is a right angled triangle.
We know that a tangent at any point of a circle
is perpendicular to the radius through the point AE2 + ET2 + AT2 ⇒ AE2 = AT2 – ET2
of contact. AE2 = (PT – PA)2 – (OT – OE)2
\ OA ⊥ PQ and OB ⊥ RS AE2 = (12 – AE)2 – (B – 5)2
⇒ ∠OBR = 90° AE2 = 144 + AE2 – 24AE – 64
∠OBS = 90° = 24 AE
∠OAP = 90° 80 10
AE = = cm
∠OAQ = 90° 24 3
We can observe the following: 10
Similarly, BE = cm
∠OBR = ∠OAQ and ∠OBS = ∠OAP 3
Also, these are the pair of alternate interior 20
\ AB = AE + BE = cm
angles. 3
180 ADDITIONAL
R

PR ACTICE MATHEMATICS - 10
24. T To prove: OA ⊥ l
Construction: Take a point B, other than A. On
P the tangent l join OB. Suppose OB meets the
O
circle at C.

A Proof: We know that, among all line segment


N joining the point O to a point on l, the
B
perpendicular is shortest to l.
To prove: PA.PB = PN2 – AN2
OA = OC (Radius of the same circle)
Construction: Join OP
Now, OB = OC + BC
Proof: Consider DAON and DBON
\ OB > OC
OA + OB (radius of same circle)
⇒ OB > OA
ON = ON (Common)
⇒ OA > OB
∠ONB = ∠ONA = 90°
B is an arbitrary point on the tangent l. Thus,
[As ON is perpendicular to chord] OA is shorter than any other line segment
\ DAON ≅ DBON joining O to any point on l.
AN = BN (CPCT) Here, OA ⊥ l
PA = PN – AN ⇒ PN – AN
PB = PN + BN = PN + AN 26. We know that ∠ADO = 90° (Since O'D is per-
pendicular to AC)
\ PA.PB = (PN – AN) (PN + AN)
∠ACO = 90° (OC (radius) perpendicular to
= PN2 – AN2
AC (tangent))
PN2 – AN2 = OP2 – OT2
In triangles ADO' and ACO,
Consider DOPN
OP2 = PN2 + ON2 ∠ADO = ∠ACO (each 90°)
\ PN2 = OP2 – ON2 ∠DAO = ∠CAO (common)
OP2 – ON2 – AN2 = OP2 – OT2 by AA criterion, triangles ADO' and ACO are
⇒ ON2 + AN2 = OT2 similar to each other.
Also OA = OT [Radius] AO ' DO '
=
In DOAN AO CO
OA2 = AN2 + ON2 (corresponding sides of similar triangles)
\ ON2 + AN2 = OT2 AO = AO' + O'X + OX
Hence proved. = 3AO' (since AO' = O'X = OX because radii
of the two circles are equal)
25.
AO ' 1
O =
AO 3

C DO ' AO ' 1
= =
l CO AO 3
A B
Given: A circle C (0, r) and a tangent l at point DO ' 1
=
A. CO 3

ADDITIONAL
R

PR ACTICE MATHEMATICS - 10 181


27. A [By Alternate Segment Theorem]
Given, RS || PQ
O N
P \ ∠RQP = ∠SRQ = 75° [Alternate
angles]
B ∠RSQ = ∠SRQ = 75°
OA = 10 cm \ QRS is also an isosceles triangle.
ON ⊥ AB [Since sides opposite to equal angles of a
triangle are equal.]
16
AN = NB = = 8 cm ∠RSQ + ∠SRQ + ∠RQS = 180°

2
Pythagoras Theorem [Angle sum property of a triangle]
In ΔONA, 75° + 75° + ∠RQS = 180°
ON2 + NA2 = OA2 150° + ∠RQS = 180°
ON2 = OA2 – NA2 \ ∠RQS = 30°

ON2 = 102 – 82 C R D
29.
ON = 36 cm
2

ON = 6 cm
r
AN 8 4 Q S
tan∠AON = = = O
ON 6 3
DOAP
PA B P A
tan∠AON =
OA Given: ABCD is a quadrilateral such that ∠D
4 PA = 90°.
=
3 10 BC = 38 cm, CD = 25 cm and BP = 27 cm
40 BP = BQ = 27 cm
⇒ PA = cm [Tangents from an external point]
3
BC = 38
28. Given ∠RPQ = 30° and PR and PQ are tangents
drawn from P to the same circle. ⇒ BQ + QC = 38
Hence PR = PQ [Since tangents drawn from an ⇒ 27 + QC = 38
external point to a circle are equal in length] ⇒ QC = 38 – 27
\ ∠PRQ = ∠PQR [Angles opposite to equal ⇒ QC = 11 cm
sides are equal in a D] \ QC = 11 cm = CR
In DPQR [Tangents from an external point]
∠RQP + ∠QRP + ∠RPQ = 180°
CD = 25 cm
[Angle sum property of a D] CR + RD = 25
2∠RQP + 30° = 180° ⇒ 11 + RD = 25
2∠RQP = 150° ⇒ RD = 25 – 11
∠RQP = 75° ⇒ RD = 14 cm
So ∠RQP = ∠QRP = 75° Also,
∠RQP = ∠RSQ = 75° RD = DS = 14 cm
182 ADDITIONAL
R

PR ACTICE MATHEMATICS - 10
\ OR and OS are radii of the circle. \ OP ⊥ PQ
From tangents R and S, ∠ORD = ∠OSD = 90° i.e. ∠OPQ = 90°
Now, ORDS is a square.
In ΔOPQ,
\ OR = DS = 14 cm
Thus, radius , r = 14 cm OQ² = OP² + PQ²
25² = OP² + 24²
WoRKSHeet - 2 625 = OP² + 576

Section-A OP² = 625 – 576

1. = 49
Q
m \ OP = 7 cm
12 c
P O
13 cm
3. A
As we know that tangent to a circle is m
perpendicular to the radius through the point 10 c
of contact. P 60°

\ OQ ⊥ PQ
B
\ ∠OQP = 90°
PB = PB
So, In ΔOQP,
= 10 cm
OP² = OQ² + PQ² (Length of tangents drawn from an external
point to a circle are equal.)
13² = OQ² + 12²
⇒ ∠PAB = ∠PBA
169 = OQ² + 144
(Angles opposite to equal sides are equal)
OQ² = 169 – 144
In ΔPBA,
= 25
∠P + ∠PAB + ∠PBA = 180°
\ OQ = 5 cm (Angle sum property)

So, radius of circle = 5 cm 60° + ∠PAB + ∠PBA = 180°


∠PAB + ∠PBA = 180° – 60°
2.
P = 120°
m
24 c
⇒ ∠PAB = ∠PBA = 60°
Q
25 cm O So, ∠PAB = ∠PBA = ∠P = 60°
⇒ ΔAPB is equilateral
As we know that tangent to a circle is
perpendicular to the radius through the point ⇒ AB = AP = 10 cm
of contact. (Sides of equilateral triangle are equal.)

ADDITIONAL
R

PR ACTICE MATHEMATICS - 10 183


4. P 1
= (80°)
5 cm 2
Q = 40°
13 cm O
Also, OA ⊥ AP

As we know that tangent to a circle is i.e. ∠OAP = 90°


perpendicular to the radius through the point (As tangent is perpendicular to radius
of contact. through point of contact.)
∴ OP ⊥ PQ In ∆OAP,
∠OAP + ∠APO + ∠AOP = 180°

i.e. ∠OPQ = 90°
(Angle sum property)
So, In ∆OPQ,
∴ 90° + 40° + ∠AOP = 180°
OQ² = OP² + PQ²
⇒ 130° + ∠AOP = 180°
13² = 5² + PQ²
⇒ ∠AOP = 180° – 130°
169 = 25 + PQ² = 50°
∴ PQ² = 169 – 25
6. PA = PB (Tangents drawn from same point
= 144 are equal)
PA = PC + CA
⇒ PQ = 12 cm
12 = PC + CA

5. CA = 
CQ (Tangents from same point are
A
equal)
12 = PC + CQ
80° O
P PC = 12 – CQ = 12 – 3 = 9 cm

B 7.

In ∆POA and ∆POB,


h
PA = PB r
(Length of tangents drawn from an external
point to a circle are equal.)
OP = PO (Common) Radius of inner circle = r
OA = OB (Radii of same circle) Area of inner circle = πr²
Radius of outer circle = r + h
∴ ∆POA ≅ ∆POB
Area of outer circle = π (r + h)²
(SSS congruence criteria)
So, area of circular path
∴ ∠APO = ∠BPO (CPCT) = area of outer circle
1 – area of inner circle
= ∠APB
2 = π (r + h)² – πr²
184 ADDITIONAL
R

PR ACTICE MATHEMATICS - 10
= π (r² + h² +2rh – r²) Section-B
= π (h² + 2rh) 1 1
= πh (h + 2r) 11. ∠ABQ = ∠AOQ = (58°) = 29°
2 2
= πh (2r + h) ( ... Angle subtended by an arc at the centre
is double the angle subtended by it on the
q remaining part of the circle.)
8. Area of sector = pr2
360
∠OAT = 90° [Radius is perpendicular to
q
20p = pr 2 tangent at point of contact]
360
In DABT
θ
20 = r2 ...(i) ∠BAT + ∠ABT + ∠BTA = 180

360
q 90° + 29° + ∠BTA = 180
Length of arc is 2pr
360 ∠BTA = 61°
q ∠BTA = ∠ATQ = 61°
2pr = 5p
360
qr 12.
P
2 = 5
360
rθ 5
⇒ = ...(ii) O R
360 2
From (i) and (ii)
Q

20 = r
Ł 360 ł Construction: Join OP and OQ
5
20 = r ⇒ r = 8 cm PR and RQ are tangents to circle at points P
2 and Q respectively.
9. A circle can have infinitely many tangents.
⇒ OP ⊥ PR and OQ ⊥ QR
10. Remark: If AB and CD are two common
tangents to the two circles of unequal radii (As tangent is perpendicular to radius through
then they will always intersect each other. point of contact.)
Given:Two circles with centre's O1 and O2. AB In ∆OPR and ∆OQR
and CD are common tangents to the circles
OP = OQ
which intersect in P.
(Radii of same circle)
To prove: AB = CD
Proof: OR = OR (Common)
AP = PC ...(i) ∠OPR = ∠OQR = 90° (Proved above)
(Length of tangents drawn from an external ∴ ∆OPR ≅ ∆OQR
point to the circle are equal)
(RHS congruence criteria)
PB = PD ...(ii)
1
(Length of tangents drawn from an external ⇒ ∠ORP = ∠ORQ = ∠PRQ
point to the circle are equal) 2
Adding (i) and (ii), we get 1
= (120°)
AP + PB = PC + PD 2
⇒ AB = CD = 60°
ADDITIONAL
R

PR ACTICE MATHEMATICS - 10 185


In ∆PRO, ⇒ ∠OAB + 65° = 90°
PR
cos 60° = ⇒ ∠OAB = 90° – 65°
OR
= 25°
1 PR
=
2 OR 14.
P
1 A
⇒ PR = OR ...(i)
2
R X
In ∆QRO
1 B
RQ = OR ...(ii) Q
2
On adding (i) and (ii), we get As we know that lengths of tangents drawn
from an exterior point to a circle are equal.
1 1
PR + RQ = OR + OR = RO
2 2 ∴ XP = XQ, AP = AR and BQ = BR
⇒ XA + AP = XB + BQ
13. A
⇒ XP + AR = XB + BR
P 50° O
15. As we know that lengths of tangents drawn
B from an exterior point to a circle are equal.

⇒ PA = PB CE = CD = 9 cm

(Length of tangents drawn from an external BF = BD = 6 cm


point to a circle are equal.) AE = AF = x cm
⇒ ∠PAB = ∠PBA ...(i) Also, OE ⊥ AC, OD ⊥ BC and OF ⊥ AB
(Angles opposite to equal sides are equal.) (As tangent is perpendicular to radius through
In ∆APB, point of contact.)

∠P + ∠PAB + ∠PBA = 180° 1


Area of ∆BOC = × BC × OD
2
(Angle sum property)
1
50° + ∠PAB + ∠PAB = 180° (From (i)) = × (9 + 6) × 3
2
2 ∠PAB = 130° 1
= × 15 × 3
∴  ∠PAB = ∠PBA = 65° 2
45
OA ⊥ AP = cm²
2
(As tangent is perpendicular to radius
1
through point of contact.) Area of ∆AOC = × AC × OE
2
⇒ ∠OAP = 90° 1
= × (9 + x) × 3
⇒  ∠OAB + ∠PAB = 90° 2
186 ADDITIONAL
R

PR ACTICE MATHEMATICS - 10
3 (Angle sum property of quadrilateral.)
= (9 + x) cm²
2 ⇒ ∠TOQ + 90° + 70° + 90° = 360°
1 ⇒ ∠TOQ + 250° = 360°
Area of ∆AOB = × AB × OF
2
⇒ ∠TOQ = 360° – 250°
1
= × (x + 6) × 3
2 = 110°
3 1
= (x + 6) cm²   ∠TRQ = ∠TOQ
2 2
Area of ∆ABC = area of ∆BOC (Angle subtended an arc at the centre is
double the angle subtended by it on the
+ area of ∆AOC remaining part of the circle.)

+ area of ∆AOB 1
= (110)
2
45 1 3 = 55°
54 = + (9 + x ) × ( x + 6)
2 2 2
45 27 18 3 3 17. P
54 = + + + x+ x
2 2 2 2 2
54 = 45 + 3x O T
9 = 3x
Q
∴ = 3
Construction: Join OT
So, AB = x + 6 = 3 + 6 = 9 cm OP ⊥ PT
AC = x + 9 = 3 + 9 = 12 cm (As tangent is perpendicular to the radius
through point of contact)
16. T i.e. ∠OPT = 90°
In DOPT,
R O 70° P
OT² = OP² + PT²

Q = 5² + 8²

Construction: Join O to T and O to Q. = 25 + 64

As we know that tangent is perpendicular = 89


to the radius through the point of contact.
∴ OT = 89 cm
∴ OT ⊥ PT and OQ ⊥ PQ
18. T
i.e. ∠OTP = ∠OQP = 90°
m
10 c
In quadrilateral TOQP
P
26 cm O
∠TOQ + ∠OQP + ∠QPT + ∠PTO = 360°
ADDITIONAL
R

PR ACTICE MATHEMATICS - 10 187


OT ⊥ PT i.e. ∠OTP = 90° (Lenghts of tangents drawn from an exterior
point to a circle are equal.)
(Tangent is perpendicular to radius through
point of contact.) Also, AB = AC ...(ii)
In ∆OTP, (Given)
OP² = OT² + PT² On subtracting (i) from (ii), we get
(By Pythagoras theorem) AB – AF = AC – AE
26² = OT² + 10² BF = CE ...(iii)
676 = OT² + 100 But BF = BD and CE = CD
OT² = 576 (Lengths of tangents drawn from an exterior
point to a circle are equal.)
OT = 24 cm
∴ BD = CD
∴ Radius of the circle = 24 cm

19. AE and CE are tangents to the circle with


Section-C
center O,
21. B
∴ AE = CE (i)
( ... Lengths of tangents drawn from an A O
exterior point to a circle are equal.)
C
Also, DE and BE are tangents to the circle
with centre O2. AB and AC are tangents to a circle.
∴ BE = DE (ii) OB ⊥ AB and OC ⊥ AC
( ... Lengths of tangents drawn from an (Tangent is perpendicular to the radius
exterior point to a circle are equal.) through the point of contact.)
On adding (i) and (ii), we get i.e. ∠OBA = ∠OCA = 90° ...(i)
AE + BE = CE + DE In quadrilateral ABOC,
∴ AB = CD ∠A + ∠B + ∠O+ ∠C = 360°
(Angle sum property of quadrilateral)
20. A
⇒ ∠A + ∠O + ∠B + ∠C = 360°
⇒ ∠A + ∠O + 90° + 90° = 360°
F E
From (i)
⇒ ∠A + ∠O + 180° = 360°
B D C
⇒ ∠A + ∠O = 360° – 180°
AF = AE ...(i) = 180°
188 ADDITIONAL
R

PR ACTICE MATHEMATICS - 10
22. BP and BQ are tangents to the circle ∠APC = ∠BPC (Proved above)
∴ BP = BQ ...(i) ∴ ∆APC ≅ ∆BCP (SSS congruence criteria)
(Lenghts of tangents drawn from an exterior ⇒ AC = BC and ∠ACP = ∠BCP = 90°
point to a circle are equal.)
(CPCT)
Also, CP = CR ...(ii)
So, OP is the perpendicular bisector of AB.
(Lenghts of tangents drawn from an exterior
point to a circle are equal.) S
24. E D
Consider
T R
Perimeter of ∆ABC = AB + BC + AC
F C
= AB + (BP + CP) + AC O

= AB + (BQ + CR) + AC U Q
From (i) and (ii), A P B
= AQ + AR (As we know that lengths of tangents drawn
= AQ + AQ from an external point to a circle are equal.)

= 2AQ ∴ AP = AU
... AQ = AR as lengths of tangents drawn BP = BQ
from an exterior point to a circle are equal. CQ = CR
1 DS = DR
∴ AQ = (Perimeter of ∆ABC)
2
ES = ET

23. Consider ∆OAP and ∆OBP, FU = FT

AP = BP Consider

(Lenghts of tangents drawn from an exterior AB + CD + EF


point to a circle are equal.) = (AP + BP) + (CR + DR) + (ET + TF)
OP = OP (Common) = (AU + BQ) + (CQ + DS) + (ES + UF)
AO = BO (Radii of same circle) = (BQ + QC) + (DS + ES) + (AU +FU)
∴ ∆OAP ≅ ∆OBP = BC + DE + AF
(SSS congruence criteria)
⇒ ∠APO = ∠BPO (CPCT) 25. PR and CR are tangents to circle with centre
A
Now, Consider ∆ACP and ∆BCP,
∴ PR = CR ...(i)
AP = BP
(Lenghts of tangents drawn from an exterior
PC = CP (Common) point to a circle are equal.)
ADDITIONAL
R

PR ACTICE MATHEMATICS - 10 189


QR and CR are tangent to circle with center Now, In ΔOCA and ΔOCB
B.
∠OCA = ∠OCB = 90°
∴ QR = CR (ii)
OA = OB (Radii of the larger circle)
(Lenghts of tangents drawn from an exterior
point to a circle are equal.) OC = OC (Common)
From (i) and (ii), we get By RHS congruency
PR = QR ΔOCA ≅ ΔOCB
∴ RC bisects PQ
\ CA = CB (By CPCT)

26. l
m 28. A
B

O1 P O F
A
D r
o
In ∆OPA and ∆OBP,
OA = OB (Radii of circle) B E C
PA = PB
Construction: Join AO. Extend OC to OD,
(Lengths of tangents from an external point OP to BF.
to a circle are equal.)
In ∆ABC, right angles at B
OP = PO (Common)
AC² = AB² + BC²
∴ ∆AOP ≅ ∆OBP (SSS congruence criteria)
= 24² + 10²
⇒ ∠APO ≅ ∠BPO (CPCT)
= 576 + 100
⇒ OP is the bisector of ∠APB
∴ O lies on the bisector of the angle = 676
between l and m.
∴ AC = 26 cm

27. 1
Area of ∆ABC = × BC × AB
2
O 1
= × 10 × 24
2
A C B
= 120 cm²

We know that the radius and tangent are Also, OF ⊥ AC, OE ⊥ BC and OD ⊥ AB
perpendicular at their point of contact.
( ... Tangent is perpendicular to the radius
\ ∠OCA = ∠OCB = 90° through point of contact.)

190 ADDITIONAL
R

PR ACTICE MATHEMATICS - 10
1 ⇒ ∠OPA = 30°
Area of ∆BOC = × BC × OE
2 Similarly, ∠OPB = 30°
1
= × 10 × r ∴ ∠APB = 30° + 30°
2
= 5r = 60°
1 Also, AP = BP
Area of ∆AOC = × 26 × r
2
(Lengths of tangent drawn from an external
= 13r
point to a circle are equal.)
1
Area of ∆AOB = × AB × OD So, In ∆APB,
2
1 ∠PAB = ∠PBA ...(i)
= × 24 × r
2 (Angles opposite to equal sides are equal.)
= 12r
In ∆APB,
Area of ∆ABC = area of ∆BOC
⇒ ∠PAB + ∠PBA + ∠APB = 180°
+ area of ∆AOC
(Angle sum property)
+ area of ∆AOB
⇒ ∠PAB + ∠PAB + 60° = 180°
∴   120 = 5r + 13r + 12 r
⇒ 2 ∠PAB = 180° – 60°
30r = 120
= 120°
r = 4 cm 120
⇒ ∠PAB =
29. 2
A
= 60°
r
θ P
O Q So, ∠PAB = ∠PBA = ∠APB = 60°
B ⇒ ∆APB is equilateral.

AP is tangent to the circle 30. As we know that lengths of tangents drawn


from an external point to a circle are equal,
∴   OA ⊥ AP
∴ PD = PF, RF = RE, QD = QE
i.e. ∠OAP = 90°
Consider
(Tangent is perpendicular to radius through
point of contact.) Perimeter of ∆PQR

In ∆OAP, = PQ +QR + PR

OA r = (PD + DQ) + (QE + ER) + (PF + FR)


= sin θ =
OP 2r = (PD + PF) + (RF + RE) + (QD + QE)
1
= = (PF +PF) + (RE + RE) + (QD + QD)
2
(OP = Diameter  = 2r) = 2 PF + 2 RE + 2 QD

∴ θ = 30° = 2 (PF + ER + QD)


ADDITIONAL
R

PR ACTICE MATHEMATICS - 10 191


Section-D MK BK MB
⇒ = =
MP OP MO
31. H
P (Corresponding sides of similar triangles are
K proportional.)
r
r r
A O B M BK BM
⇒ =
PO OM
y z
PM is the tangent to circle. ⇒ =
r r+z
∴ ∠MPO = 90° ⇒ yr + yz = rz
(Tangent is perpendicular to radius through ry
point of contact) ⇒ z = ...(ii)
r-y
Let AH = x, BK = y, BM = z From (i) and (ii), we get

Let r be the radius of circle 2ry ry


=
In ∆MKB and ∆MHA x-y r-y

∠M = ∠M (Common) 2 1
⇒ =
x−y r-y
∠MKB = ∠MHA = 90°
⇒ 2r – 2y = x – y
∴ ∆MKB ≅ ∆MHA
⇒ x + y = 2r
(AA similarity criteria.)
⇒ AH + BK = AB ( ... AB = 2r)
MK KB MB
⇒ = = =
MH HA MA
32. Consider ∆OEA and ∆OEP
(Corresponding sides of similar triangles are
proportional) OA = OP (Radii of same circle)

BK MB OE = OE (Common)
⇒ =
AH MA AE = PE (OE bisects AP)
y z
⇒ = ∴ ∆OEA ≅ ∆OEP
x 2r + z
⇒ 2r y + y z + = x z (SSS congruence criteria)
2ry ⇒ ∠OEA = ∠OEP (CPCT)
⇒ z = ...(i)
x-y
∴ ∠OEA = ∆OEP = 90° ...(i)
Now, In ∆MKB and ∆MPO,
(∠OEA and ∠OEP are linear pair)
∠M = ∠M (Common)
Also, AB ⊥ BC as BC is a tangent to the circle
∠MKB = ∠MPO = 90°
(Tangent is perpendiculer to the radius
∴ ∆MKB and ∆MPO, through the point of contact.)
(AA similarity criteria.) ⇒ ∠ABC = 90° ...(ii)

192 ADDITIONAL
R

PR ACTICE MATHEMATICS - 10
Now, In ∆AEO and ∆ABC (Tangent is perpendicular to radius through
point of contact.)
∠EAO = ∠BAC (Common)
In ∆OQP,
∠AEO = ∠ABC OP² = OQ² + PQ²
= 90° (From (i) and (ii) )
(Pythagoras theorem)
⇒ ∆AEO ~ ∆ABC = (3k)² + (4k)²
(By SS Similarity criteria) = 9k² + 16k²
33. Given: d1, d2 (d2 > d1) be the diameters of
= 25k²
two concentric circles and C be the length
of a chord of a circle which is tangent to the ∴ OP = 5k
circle. Also, Perimeter of ∆POQ = 60 cm
To prove: d2 = d12 + c2
Now, ⇒  PO + OQ + PQ = 60

d2 d1 ⇒  5k + 3k + 4k = 60
OQ = , OR = and PQ = c
2 2 ⇒  12k = 60
Since PQ is tangent to the circle therefore 60
OR is perpendicular to PQ ⇒  k=
12
PQ c = 5
⇒ QR = =
2 2 So, PQ = 4k = 4 × 5 = 20 cm
Using pythagorus theorem in triangle OQR
QR = 2(OQ) = 2 (3k) = 6k
OQ2 = OR2 + QR2
= 6 × 5= 30 cm
2 2 2
 d2   d1   c 
⇒  2  =  2  +  2  OP = 5k = 5 × 5 = 25 cm

1 1 35.
( d 2 ) = ( d1 ) + ( c )
2 2 2
⇒ A
4 4
O
⇒ d2 = d1 + c
2 2 2

B D E
Hence Proved.
BE is tangent to circle
34. Q
P
∴ OD ⊥ BE
O
i.e. ∠ODB = 90°
R (Tangent is perpendicular to radius through
OQ : PQ = 3 : 4 point of contact)
Let OQ = 3k, PQ = 4k ⇒ BD = DE
PQ is tangent to the circle (As perpendicular from centre to the chord
∴ OQ ⊥ PQ bisects the chord)

i.e. ∠OQP = 90° ⇒ D is a midpoint of BE


ADDITIONAL
PR ACTICE
R

MATHEMATICS - 10 193
Also, O being the centre is a midpoint of AB ⇒ ∠OCA + ∠ACD = 90° ...(i)
So, By midpoint theorem, Now, OA = OC

1 (Being radii of same circle.)


OD || AE and OD = AE
2
\ In ΔAOC,
\ AE = 2 (OD)
∠OCA = ∠OAC
= 2 (8)
(Angles opposite to equal sides are equal.)
= 16 cm
⇒ ∠OCA = ∠BAC ...(ii)
In ΔODB, ∠ODB = 90°
From (i) and (ii), we get
\ OB² = OD² + BD²
∠BAC + ∠ACD = 90°
(By Pythagoras theorem)
cASe StUDY-1
⇒ 13² = 8² + BD²
(i) (c) ∠BAC + ∠COB + ∠ABU + ∠ACU = 360°
⇒ 169 = 64 + BD²
[sum of all angles of quadrilateral is 360°]
⇒ BD² = 169 – 64
40° + ∠BOC + 90° + 90° = 360°
⇒ BD² = 105
∠BOC = 140°
⇒ BD = 105 cm (ii) (b) ∠BAO + ∠BOA + ∠ABO = 180°
⇒ DE = 105 cm [sum of all angles of triangle is 180°]

( ... BD = DE) 40° + ∠BOA + 90° = 180°

In ΔAED, ∠AED = 90° ∠BOA = 50°


In DABO and DAOC
\ AD² = AE² + DE²
AC = AB [Tangents drawn from same
= (16)² + ( 105 )² point to a circle are equal in
length]
= 256 + 105
OC = OB [radius]
= 361
AO = AO [common]
\ AD = 19 cm
\ DABO ≅ DAOC by SSS rule.
36. BD is tangent to the circle Hence ∠BOA = ∠COA [CPCT]

\ OC ⊥ BD \ ∠COA = 50°

i.e. ∠OCD = 90° ∠BOC = ∠BOA + ∠COA


= 2 ∠BOA
(Tangent is perpendicular to radius through
point of contact.) = 100°
194 ADDITIONAL
R

PR ACTICE MATHEMATICS - 10
(iii) (d) ∠BAC + ∠COB + ∠ABO + ∠OCA = 360°
cASe StUDY-2
[sum of all angles of quadrilateral is 360°]
50° + ∠COB + 90° + 90° = 360 (i) (c) ∠PAO + ∠AOB + ∠OBP + ∠BPA = 360°
∠PAO = ∠OBP = 90°
∠COB = 130°
\ ∠AOB = 360° – (90° + 90° + 60°)
B
= 120°
In OAB
A o
OA = OB [radius of circle]

C \ ∠OAB = ∠OBA
[Angles opposite to equal sides are equal]
As OB = OC (radius)
∠OAB + ∠OBA + ∠AOB = 180°
\ ∠OBC = ∠CBO
2∠OAB = 180° – ∠AOB
[Angles opposite to equal sides are equal]
= 180° – 120°
2 ∠OBC + ∠COB = 180° ∠OAB = 30°
[Angle sum property of triangle] As ∠OAB = ∠OBA
2 ∠OBC + 130° = 180° \ ∠OBA = 30°
2 ∠OBC = 50° Draw a perpendicular PM on AB
A
∠OBC = 25°
4 cm
(iv) (a) In DAOB, ∠B = 90° P 60 M o

[Radius is ⊥ to tangent at the point of contact]


B
\ DAOB is right triangle.
∠MAP + ∠OAM = ∠PAO = 90° [tangent
AO2 = AB2 + OB2 is ⊥r to radius at point of contact]

252 = 242 + OB2 ∠OAM = ∠OAB = 30°

OB2 = 49 \ ∠MAP = 90° – 30° = 60°

∠MAP = ∠BAP = 60°


OB = 7 m
∠BAP + ∠APB + ∠PBA = 180°
AB
(v) (b) cos 30° = [sum of all angles of triangle is 180°]
OA
∠PBA + 60° + 60° = 180°
3 AB
= ∠PBA = 60°
2 4m
Hence, DPAB is an equilateral triangle and
AB = 2 3 m thus AB = BP = PA = 4 cm
ADDITIONAL
R

PR ACTICE MATHEMATICS - 10 195


(ii) (a) As ∠PAB = ∠BPA = ∠ABP = 60°, hence (v) (c) As ∠PAO = 90° (tangent is ⊥r to radius)
DPAB is an equilateral triangle.
and
(iii) (b) Perimeter of DPAB sum of all sides
∠PAB = 60° (DPAB is equilateral
= 4 cm + 4 cm + 4 cm triangle)
= 12 cm
Also
3 2
(iv) (a) Area of equilateral triangle is a , where
4 ∠PAO = ∠PAB + ∠OAB
a is side of triangle.
∠OAB = ∠PAO – ∠PAB
3
( 4)
2
Area = = 90° – 60°
4
= 4 3 = 30°

196 ADDITIONAL
R

PR ACTICE MATHEMATICS - 10
Chapter

11 Constructions
Multiple choice Questions

For 5 : 7
1. (c)
Given, ∠BAX ∠90°
o 5 cm \ A5M || BA12
A
\ Total equal arc lengths are (5 + 7) = 12
4 cm
4. (d)
M
P
\ DOMA ∠M = 90°
11
By Pythagoras Theorem,
o 60°
OM2 = OA2 – AM2 A
12
= 5 –4 2 2

Q
\ OM2 = 9 = 32
\ OM = r = 3 cm
Given, ∠PAQ = 60°
2. (c)
Where AP and AQ are two tangents of
o the circle with centre O.
I
\ ∠POQ = 360° – (90° + 90° + 60°)
A
M = 360° – (240°)
B
∠POQ = 120°
Line AB is a tangent of circle with centre
O. 5. (a) A
C
\ OM ⊥ AR
\ ∠OMA = 90° P Q
M
3. (d) A B D
B
A5 Given, PB = 12 cm, CQ = 3 cm
PA and PB are two tangents of the circle
with centre O and CD is a third tangent.
A12
We know that the two tangents draw to a
X circle from an external point are equal.
ADDITIONAL
R

PR ACTICE MATHEMATICS - 10 197


\ PA = PB = 12 cm (i) Draw AB = 6 cm.
Same, CQ = CA = 3 cm (ii) Draw a ray AX making an acute ∠BAX.
\ P – C – A, (iii) Along AX, mark point A1, A2, A3..., A7. Such
\ PA = PC + CA that AA1 = A1A2...= A6A7.
\ 12 = PC + 3 (iv) Join A7B.
\ 12 – 3 = PC = 9 cm (v) Through A3 draw a line A3C || A7B
intersecting AB at C.
WoRKSHeet - 1
Thus, points C so obtained is the required
point which divides internally in the ratio 3:4.
Section-A
1. T A
3.
cm
10 A1
A o
26 cm

Since the tangent to a circle is perpendicular 60° 6 cm


B C
to the radius through the point of contact. C1
B1
B2
\ ∠OTA = 90°
B3
In right DOTA, we have B4

OA2 = OT2 + AT2 B5


B6
⇒ OT = 676 – 100
2

B7 X
⇒ 576
⇒ (24)2
Given, BC = 6 cm,AB = 5 cm, ∠ABC = 60° and
⇒ OT = 24 5
of the corresponding sides of the triangle
Hence the radius of the circle is 24 cm. 7
ABC.

A C B Steps of construction:
2.
(i) BC = 6 cm is drawn
A1 (ii) At point B, AB = 5 cm is drawn angle
A2 ∠ABC = 60° with BC.
A3 (iii) AC is joined to form DABC.
A4
(iv) A ray BX is drawn making an acute angle
A5
with BC opposite to vertex A.
A6
(v) 7 Points B1, B2, B3 ..., B7 at equal distance
A7
are marked on BX.
X (vi) B5 joined with C1 to form B5C1.
198 ADDITIONAL
PR ACTICE
R

MATHEMATICS - 10
(vii) C1A1 is drawn parallel to CA. So, we take two parts out of three equal parts
on BX from point B2 draw B2C || B3C and
Thus A1BC1 is the required triangle.
meeting BC at C.
(ix) From C1 draw C1A1 || AC and meeting
4. Given that
AB at A1.
Construct triangle of given data, BC = 6 cm,
∠B = 50° and ∠C = 60° and then a triangle Thus, DABC is the required triangle, each of
similar to it whose sides are (2/3)rd of the whose side is two third of the corresponding
corresponding sides of DABC. side of DABC.

We follow the following steps to construct


5.
the given
A T

A1

P O
Q
50° 60°
B C
C1
T1
B1

B2
(i) Take a point O in the plane of the paper
B3 and draw a circle of radius 3 cm.
X
(ii) Mark a point P at a distance of 5.5 cm
Steps of construction from the centre O and join OP.
(iii) 
Draw the right bisector of OP,
(i) First of all we draw a line segment BC = intersection OP at Q.
60°.
(iv) Taking Q as centre and OQ = PQ as
(ii) With B as centre draw an angle ∠B = radius, draw a circle to intersect the
50°. given circle at T and T1.
(iii) With C as centre draw an angle ∠C = (v) 
Join PT and PT1 to get the required
60° which intersecting the line drawn in tangents.
step ii at A.
Q
(iv) Join AB and AC to obtain DABC. 6.
(v) Below BC, makes an acute angle ∠CBX
= 60°.
P 60° 120° O
(vi) Along BX, mark off three points B1, B2
and B3 such that BB1 = B1B2 = B2B3.
(vii) Join B3C.
R
(viii) 
Since we have to construct a triangle
each of whose sides is two-third of the Given angle between tangents is 60°
corresponding sides of DABC. i.e. ∠QPR = 60°
ADDITIONAL
R

PR ACTICE MATHEMATICS - 10 199


Since Angle at centre is double the angle 8. A
between tangents A
1

\ ∠QPR = 2 × 60° = 120°



So, we need to draw ∠QPR = 120°
B C
\ We draw a radius, then second radius at
L B1
120° from first.
B1
Section-B
B2
7.
B3

Q B4
(i) Make an line BC 5.5 cm
M
P O (ii) Take its bisector and now cut it 3 cm.
(iii) Join BA and CA.
R (iv) Now from B make an circle (∠60°) angle.
(v) Extend that line.
(vi) Cut 4 arcs.
(vii) From B3 make a parallel line to CB4.
(viii) Do same formation inside the triangle.
Steps of construction (ix) Your triangle is ready.
(i) Draw a concentric circle two with circles 9. A
radii are 3 cm and 5 cm.
(ii) Let P is a point on the circumference
of center circle. Joined O to P, then OP
formed. E
7
cm

(iii) Perpendicular bisector of OP. M is a mid


5 cm

point of the OP.


(iv) Draw a circle M in a centre and OM is
a radius. This circle is intersected with
three circles in Q and P points. B C
6 cm D
(v) Joined P to Q and P to R.
Thus, PQ and PR are required two tangents. B1
OQ = 3 cm and OP = 5 cm,
B2
In DOPQ,
PQ2 = OP2 – OQ2 B3
= (5) – (3) 2 2
B4
= 25 – 9
PQ2 = 16 – 4 B5
X
\ PQ = 4
...(i)
200 ADDITIONAL
R

PR ACTICE MATHEMATICS - 10
Steps of construction (iii) From B mark an arc on BX at a distance
of 6 cm. Let it is A.
(i) Draw a line segment BC = 6 cm
(iv) Join A to C.
(ii) With B as centre and radius equal to 5
cm, draw an arc. (v)  Making an acute angle draw a line
segment BY from B.
(iii) With C as centre and radius equal to 7
cm, draw an arc. (vi) Mark B1, B2, B3, B4 on BX such that BB1 =
B1B2 = B2B3 = B3B4.
(iv)  Mark the point where the two arcs
intersect as A. (vii) Join B4 to C.
Join AB and AC. (viii) Draw a line segment B3C1 || to B4C to
meet BC at C1.
Thus, DABC is obtained.
(ix) Draw line segment CA1 || to CA to meet
(v) Below BC, make an acute ∠CBX
AB at A1.
(vi) Along BX, mark off five points B1, B2, B3,
A1BC1 is the required triangle.
B4, B5 such that B B1 = B1 B2 = B2 B3 = B3
B4 = B4 B5.
(vii) Join B C. 11. Q
5
S
(viii) From B4, draw B4D || B5C, meeting BC at
D.
A B
(ix) From D, draw DE || CA, meeting AB at E. O
Then DEBD is the required triangle each of
4 T
whose sides is of the corresponding side R
of DABC. 5
Steps of construction
10. A (i) Take AB = 7 cm
(ii) With A as centre and 3 cm as radius,
A1
draw a circle.
(iii) Similarly, with B as centre and 2 cm as
radius, draw a circle.
B C (iv) Now, draw the perpendicular bisector of
C1
B1 AB and mark the point of intersection O.
B2
(v) 
With O as centre and OA as radius,
B3
B4 draw a circle. Mark the 2 points where
the circles with centre O and A meet as
Y Q and R. Similarly, mark the points where
Steps of construction the circles with centres O and B meet as
S and T respectively.
(i) Draw a line segment BC = 8 cm
(vi) Join BR and BQ as well as AS and AT.
(ii) Draw line segment BX making an angle Now, BR, BQ, AS and AT are the required
of 90° at the point B of BC. tangents.
ADDITIONAL
R

PR ACTICE MATHEMATICS - 10 201


12. WoRKSHeet - 2
A E
Section-A
D F
6 cm 1. B P

O
B C
8 cm
90°
A
O

Steps of construction:
• Draw BC = 8 cm Steps of construction
• Draw the perpendicular at B and cut BA (i) Draw a circle of 3.5 cm radius with O as
= 6 cm on it. Join AC and right DABC is centre.
obtained.
(ii) Take a point A on the circumference
• Draw BD perpendicular to AC. of the circle and join OA. Draw a
perpendicular to OA at point A.
• Since ∠BDC = 90° and the circle has to
pass through B, C and D, BC must be the (iii) Draw a radius OB, making an angle of
diameter of this circle. So, take O as the 90° with OA.
midpoint of BC and with O as centre and (iv) Draw a perpendicular to OB at point B.
OB as radius draw a circle that will pass Let both the perpendicular intersect at
through B, C and D. point P.
• To draw tangents from A to the circle with (v) Join OP.
centre O.
PA and PB are the required tangents, which
– Join OA, and draw its perpendicular make an angle of 45° with OP.
bisector to intersect OA at point E.
– With E as centre and EA as radius draw 2. D
a circle that intersects the previous
circle at B and F. T

– Join AF.
– Thus, AF and AB are the required C B
P A
tangents to the circle with centre O.
Proof:
T1
∠ABO = ∠AFO = 90° (Angle in a semi-circle)
\ AB ⊥ OB and AF ⊥ OF (We know that Steps of construction
the line joining the centre of a circle to the (i) Draw a circle of radius 4 cm.
tangent is always perpendicular)
(ii) Take a point P outside the circle and
Hence AB and AF are the tangents from A to draw a secant PAB, intersecting the circle
the circle with centre O. at A and B.
202 ADDITIONAL
R

PR ACTICE MATHEMATICS - 10
(iii) Produce AP to C such that AP = CP. (ii) Take any points A on the circle. Join OA.
Mark another point B on the circle such
(iv) Draw a semi-circle with CB as diameter.
that ∠AOB = 135°, supplementary to
(v) 
Draw PD and CB intersecting the the angle between the tangents.
semicircle at D.
Since the angle between the tangents to
(vi) With P as centre and PD as radius draw be constructed is 45°.
arcs to intersect the given circle at T and
\ ∠AOB = 180° – 45° = 135°
T1.
(iii) Construct angles of 90° at A and B extend
(vii) Join PT and PT1.Then PT and PT1 are the
the lines so as to intersect at point P.
required tangents.
(iv) Thus AP and BP are the required tangents
3. to the circle.
P
5.

30° B
Q
60° A Q
O

O1
O P

Steps of construction:
1. Draw a circle with centre O and radius R
6 cm.
2. 
Draw a radius OA of this circle and
produce it to B. Steps of construction
3. Construct an angle ∠AOP equal to the (i) Taking point O as a centre draw a circle
complement of 30° i.e. equal to 60° of radius 4 cm.
[\ ∠OPQ = 90°]
(ii)  Now taking O as centre draw a
4. Draw perpendicular to OP at P which
concentric circle of radius 6 cm.
intersects OA produced at Q.
5. PQ is the required tangent such that (iii) Taking any point P on the outer circle
∠OQP = 30°. join OP.
(iv) Draw a perpendicular bisector of OP.
4. B (v) Name the intersection of bisector and
OP as O1.
cm
4.5

P (vi) Now, draw a circle taking O1 as centre


o 135° 45°
and O1P as radius.
(vii) 
Name the intersection point of two
A circles as R and Q.
(i) Draw circle of radius 4.5 cm with centre (viii) Join PR and PQ. These are the required
O. tangents.

ADDITIONAL
R

PR ACTICE MATHEMATICS - 10 203


6. A Section-B
7.
Y
4 cm X
A

A1
B C
8 cm

Steps of construction 45° 60°


B C
(i) BC = 8 cm is drawn. C1
Z1
(ii) Perpendicular bisector of BC is drawn Z2
and it intersects BC at M.
Z3
(iii) At a distance of 4 cm a point A is marked Z4
on perpendicular bisector of BC.
Z5
(iv) AB and AC are joined to form DABC.
Z
(v) Ray BX is drawn making an acute angle
with BC apposite to vertex A. Steps of construction

(vi) 4 points B1, B2, B3 and B4 are marked on (i) Draw a line segment BC = 8 cm.
BX. (ii) At B, draw ∠XBC = 45°.
(vii) B4 is joined with C to form B4C.
(iii) At C, draw ∠YCB = 60°. Suppose BX
(viii) B4C is drawn parallet to B4C and C A is
1 1 1
and CY intersect at A.
drawn parallel to CA. Thus A1BC1 is the
Thus, DABC is the required triangle.
required triangle formed.
A (iv) Below BC, draw an acute angle ∠ZBC.

(v) Along BZ, mark five points Z1, Z2, Z3, Z4


A 1
and Z5 such that BZ1 = Z1Z2 = Z2Z3 =
Z3Z4 = Z4Z5
(vi) Join CZ5.
B C (vii) From Z3, draw Z3C1 || CZ5 meeting BC at
C1 C1.
B1
(viii) From C1, draw A1C1 || AC meeting AB in
B2
A1.
B3
Here, DA1BC1 is the required triangle whose
B4 3
sides are of the corresponding sides of
X 5
DABC.

204 ADDITIONAL
R

PR ACTICE MATHEMATICS - 10
8. D'
Z Y 14. We see that A’BC’D’ is a parallelogram
C' where A’D’ = 6.5 cm, A’B = 4 cm and
∠A’BD’ = 60°
D X
C 15. Divide it into triangles A’BD’ and BC’D’
by the diagonal BD’.
6.5 cm B4
5 cm B3
B2 9.
X' A
60° B1
A' A B
3 cm
4 cm
A1
Steps of Construction
1. Construct a line AB = 3 cm
3 cm
2. Construct a ray BY which makes an acute
angle ∠ABY = 60°
C B
3. 
With B as centre and 5 cm as radius, C1 4 cm
construct an arc which cuts the point C B1
on BY B2
4. Construct a ray AZ which makes ∠ZAX’
B3
= 60° as BY || AZ and ∠YBX’ = ∠ZAX’ = B4
60° B5
X
5.  With A as centre and 5 cm as radius,
construct an arc which cuts the point D Steps of construction
on AZ
(i) BC = 4 cm is drawn.
6. Join CD
7. So we get a parallelogram ABCD (ii) At B, a ray making an angle of 90° with
BC is drawn.
8.  Join BD which is the diagonal of
parallelogram ABCD (iii) With B is centre and radius equal to
4 cm, an arc is made on provision ray
9.  Construct a ray BX downwards which
intersecting it at point A.
makes an acute angle ∠CBX
10. Now locate 4 points B1, B2, B3, B4 on BX (iv) AC is joined to form ABC.
where BB1 = B1B2 = B2B3 = B3B4
(v) Ray BX is drawn making acute angle with
11. Join B4C and from B3C construct a line BC opposite to vertex A.
B4C’ || B3C which intersects the extended
line segment BC at C’ (vi) 5 points B1, B2, ... , B5 at equal distance are
marked on BX.
12. Construct C’D’||CD which intersects the
extended line segment BD at D’. ∆D’BC’ (vii) B5C is joined and B3C1 is made parallel to
is the required triangle whose sides are B5C.
4/3 of the corresponding sides of DDBC
(viii) A1C1 is joined together.
13. Construct a line segment D’A’||DA where
A’ lies on the extended side BA Thus, A1BC1 is the required triangle.

ADDITIONAL
R

PR ACTICE MATHEMATICS - 10 205


10. C Thus, DAB'C' is the required triangle, each of
whose sides is (4/5)th of the corresponding
C1 sides of DABC.

11. X

60°
A B D G
B1
6 cm P
A1

A2 B C
8 cm
O
A 3

A4

 A5 X Steps of construction
Step of construction (i) Draw a line BC of 8 cm length.
(i) First of all we draw a line segment AB = (ii) Draw BX perpendicular to BC.
4.6 cm
(iii) Mark an arc at the distance of 6 cm on
(ii) With A as centre draw an angle ∠A = BX. Mark it as A.
60°.
(iv) Join A and C. Thus DABC is the required
(iii) With B as centre and radius = BC = triangle.
5.1 cm, draw an arc, intersecting the arc
drawn in step I at C. (v) With B as the centre, draw an arc on AC.

(iv) Joins BC to obtain DABC. (vi) Draw the bisector of this arc and join it
with B. Thus, BD is perpendicular to AC.
(v) Below AB, makes acute angle ∠BAX =
60°. (vii) Now, draw the perpendicular bisector of
BD and CD.Take the point of intersection
(vi) Along AX, mark off five points A1, A2, A3,
as O.
A4 and A5 such that AA1 = A1A2 = A2A3 =
A3A4 = A4A5. (viii) With O as the centre and OB as the
radius, draw a circle passing through
(vii) Join A5B.
points B, C and D.
(viii) 
Since we have to construct a triangle
each of whose sides is (4/5)th of the (ix) Join A and O and bisect it. Let P be the
corresponding sides of DABC. mid point of AO.

So, we take four parts out of five equal parts (x)  Taking P as the centre and PO as its
on AX from point A4 draw A4B' || A5B, and radius. Draw a circle which will intersect
meeting AB at B'. the circle at point B and G. Join A and G.

Step IX– From B' draw B'C' || BC and meeting Here, AB and AG are the required tangents
AC at C'. to the circle from A.
206 ADDITIONAL
R

PR ACTICE MATHEMATICS - 10
12. cASe StUDY-1
A
X1 (i) (b) ∠CBX is an acute angle as it is less than
90°.
(ii) (a) The points are marked at equal distance.
6.2 cm 5
O P (iii) (d) DABC is of DA'BC' is the point B5
3.5 cm M 7
should be joined to C so that DABC ~
DA'BC'.
X2 (iv) (d) B7C' should be parallel to B5C.
(v) (a) From the figure, it is obvious that A'C' is
parallel to AC.
B

cASe StUDY-2
Steps of construction
(i) (c) ∠APB + ∠PAO + ∠PBO + ∠BOA = 360°
(i) Draw the circle with centre O and radius [sum of all angles of quadrilateral is 360°]
3.5 cm.
∠APB = 45°
(ii) Joint P from centre to outside the circle. ∠PAO = 90°
OP = 6.2 cm. ∠PBO = 90°
(iii) Construct mid point of OP, M is the \ ∠BOA = 360° – (45° + 90° + 90°) = 135°
mid point of OP. (ii) (c) Only one tangent can be drawn from a
point which lie on circle.
(iv) Draw a circle with centre M and radius
OM intersects the given circle at X1 and (iii) (d) Two tangents can be drawn from a point
X2. that lies outside the circle.
(iv) (c) No tangent can be drawn from a point
(v) Join PX1 and PX2. inside the circle.
Thus, PX1 and PX2 are required two tangents (v) (c) PA = PB [Length of tangents from external
from point P. point are equal]

ADDITIONAL
PR ACTICE
R

MATHEMATICS - 10 207
Chapter

12 Areas Related to Circles

Multiple choice Questions


4. (c) Perimeter of circle = 2 (perimeter of
1. (d) 2(p)(r) – r = 37 square)
22 ⇒ 2pr = 2 ( 4x )
r (2 x – 1) = 37
7 ⇒ pr = 4x
37 x 7
r = ⇒ r = 4x
44 - 7
p
= 7 cm p 2
Ratio of areas = r2
22 x
∴ Area = x 7x7 2
7 = p 16x x 1
p2 x2
= 154 cm²
= 16 : p

2. (b) h 5. (d) Area of Section 51


Area of Section 52
120
r πr 2
360
=
150
πr 2
360
= 4 :5
Area of circular path
= p (r + h)² – pr² WoRKSHeet - 1
= p (r² + h² + 2rh) – pr²
Section-A
= p (h² + 2rh) 1. arc length = 3.5 cm
= p h (h + 2rh) q 2pr = 3.5
360
θπr
3. (a) 2pr = 22 ⇒ = 3.5 ...(i)
360 2
22 q
2x x r = 22 Area of sector = pr²
7 360
22 x 7
r = = 3.5 pr r
2 x 22 2
= 3.5 cm 3.5
= r (From (i) )
2
Area = 22 x 3.5 x 3.5 = 3.5 x 5
7 2
= 38.5 cm² = 8.75 cm²
208 ADDITIONAL
R

PR ACTICE MATHEMATICS - 10
⇒ r² = r1 + r2
2. Length of arc = q 2pr
360
= 12² + 5²
45
= x2xpx5
360 = 144 + 25
45p
= = 169
36
5p r = 13 cm = 13 cm
= cm
4 Diameter = 2r

3. Area of section = q pr² = 26 cm


360
120 22 8. Circumference = 582 cm
= x x 21 x 21
360 7 ⇒ 2pr = 582
= 462 cm²
22
⇒ 2x r = 582
4. Area of circle = Circumference of circle 7
291 x 7
⇒ pr² = 2pr ⇒  r =
2 x 11
⇒   r =2 2037
= cm²
22
5. Circumference   = 2pr metres ∴ area of circle = pr²

Distance covered = 5 m 22 2037 2037


= x x
7 22 22
Distance Covered
So, no. of revolutions = = 36943.95 cm²
Circumference
5
=
2 pr Section-B
9. Let r1 = 8 cm, r2 = 6 cm
6. Let r1 = 19 cm and r2 = 9 cm.
Area of circle = Sum of area of 2 circles
Circumference of circle
pr² = pr1² + pr2²
= Sum of circumferences of the two circles
  r² = r1² + r2²
⇒ 2pr = 2pr1 + 2pr2
= 8² + 6²
⇒ r = r1 + r2
= 64 + 36
= 19 + 9
= 100
= 28 cm
∴ r = 10 cm
7. Let r1 = 12 cm, r2 = 5 cm ∴ Circumference of circle
Area of circle = Sum of areas of the two cir- = 2pr
cles
22
⇒ pr² = pr1² + pr2² =2x x 10
7
ADDITIONAL
R

PR ACTICE MATHEMATICS - 10 209


440 1 22
= cm = x x 3.5 x 3.5
7 4 7
= 9.625 cm²
10. Time = 10 minutes
= 10 x 60 12. Angle subtended in 60 minutes = 360o
= 600 seconds
∴ Angle subtended in 10 minutes =
Speed = 66 km/hr 360 ¥10
11 5
60
66 x 1000 = 60o
= θ
3600
  Area = pr²
63 360
55 60 22
= m/s = x x 16 x 16
3 360 7
∴ Total distance covered = speed x time = 134.095 cm²

55
= x 600 13.
3
= 11000 m O1
cm 0
  Distance covered in one revolution cm
10
= 2pr
A B
22 40 88
=2x x = m
7 100 35 P
∴ Number of revolutions 90 22
(a) area of sector OAPB = x x
(10)² 360 7
11000 x 3
=
88
550
= 4375 = cm²
7
1
11.  Circumference = 22 area of sector ∆AOB = x 10 x 10
2
⇒ 2pr = 22 = 50 cm²
∴ area of minor segment
22
⇒ 2x r = 22 550
7 = – 50
22 x 7 7
⇒  r = 550 - 350
2 x 22 =
7
7 200
= = cm²
2 7
= 3.5 cm = 28. 6 cm²

1 (b) Area of major segment


area of quadrant = pr²
4 = area of circle – area of minor segment
210 ADDITIONAL
R

PR ACTICE MATHEMATICS - 10
200 17.5
= p (10)² – r = 8.75 cm
7 2
= 22 x 100 – 200 width of path = 3.5 cm
7 7
⇒ R = 8.75 + 3.5
2000 200
= –
7 7 = 12.25 cm
2000
= cm² area of path = p (R² – r²)
7
= 285.7 cm² 22
= [ (12.25)² – (8.75)² ]
7
14.
Area cleaned at each sweep of the blades
22
q = [ 150. 0625 – 76.5625 ]
= pr² 7
360
= 231 cm²
115 22
= x x 2.5 x 2.5
360 7
Section-C
= 6. 27 cm²
17. Cost of fencing at the rate of ` 24 per me-
∴ Total area cleaned = 2 x 6. 27 tre
= ` 5280
= 12. 54 cm²
15. Area of each semi circle ⇒ Perimeter of circular field x 24 = 5280
2
1 22 Ê 14 ˆ 5280
= x xÁ ˜ ⇒ Perimeter of circular field =
2 7 Ë 2¯ 24
1 22 = 220 m
= x x 7x7
2 7
= 11 x 7 ⇒ 2pr = 220

= 77 cm² 22
⇒ x r = 220
2x
7
Area of square = (14)²
220 x 7
= 196 cm² ⇒  r =
2 x 22
∴ area of shaded region = 35 m
= area of square – 2x area of semi circle ∴ Area of field = pr²
= 196 – 2 (77) 22
= x 35 x 35
= 196 – 154 7
= 3850 m²
= 42 cm²
Cost of ploughing the field = 0.5 x 3850
16.
= ` 1925

R 18. ∠QPR = 90° (Angle in a semi-circle is right


r angle.)
In DQPR,
Qr² = PR² + PQ² (Pythagoras theorem)
ADDITIONAL
R

PR ACTICE MATHEMATICS - 10 211


= 7² + (24)² q
Area of sector OAB = pr²
= 49 + 576 360
q
= 625 = pr r
360
∴ QR = 25 cm = 7.9 r

QR 25 = 7.9 (5.7)
∴ radius (r) = = cm = 45.03 cm²
2 2
So, area of semi-circle
1 40
= pr² 20. area of sector OBD = p (7)²
2 360
1 22 25 25 40
= x x x area of sector OAC = p (14)²
2 7 2 2 360
= 245.536 cm² ∴ area of shaded region
1 40
area of ∆QPR = x PR x QP = p [ (14)² – 7² ]
2 360
1 p
= x 7 x 24 = (196 – 49)
2 9
= 84 cm² 1 22
So, area of shaded region = 245.536 – 84 = x 147
9 7
= 161. 536 cm² = 51.33 cm²

19. 21. Area of each quadrant

O 1
=p (1)²
B 4
π
=
A 4
area of circle of diameter 2 cm
r = 5.7 cm
= p (1)²
Perimeter of sector of circle = 27.2 cm
= p cm²
OA + OB + length of arc AB = 27.2
So, area of shaded region
5.7 + 5.7 + length of AB = 27.2
= Area of square
length of AB = 27.2 – 5.7 – 5.7 = 15.8
Ê πˆ
cm –4 Á ˜ –p
Ë4¯
θ = 4² – p – p
⇒ 2pr = 15.8
360
= 16 – 2p
q 15.8
⇒ pr =
360 2 22
= 16 – x2
= 7.9 cm 7

212 ADDITIONAL
R

PR ACTICE MATHEMATICS - 10
44 q
= 16 – 24. (a) length of arc = 2pr
7 360
68
= cm² 60 22
7 = x2x x 21
360 7
= 9. 71 cm²
= 22 cm
1
22. p (21)²
Area of quadrant OAB = 60 22
4 (b) area of sector = x x (21)²
360 7
441
= p cm² = 231 cm²
4
1
Area of quadrant ODC = p (14)²
4 Section-D
196 25.
= p cm²
4
∴ Area of shaded region A 10 m B

441 196 60 m
= p – p
4 4
245 D C
= p
4 106 m
245 22 (a) Distance around the track along its inner
= x
4 7 edge = AB + CD + 2 (semi-perimeter of
= 192.5 cm² inner circles ends)

23. Angle subtended by minute hand Ê 22 ˆ


= 106 + 106 + 2 Á ¥ 30 ˜
in 60 minutes = 360° Ë 7 ¯
1320
360 = 212 +
in 1 minute = 7
60
= 6° 1484 +1320
=
7
in 5 minutes = 5 x 6
2804
= 30° =
7
So, area swept by minutes hand = 400.57 cm

30 (b)
in 5 minutes = p (14)²
360 P Q
p A B
= x 196
12
196 22
= x
12 7
D C
= 51.3 cm² S R
ADDITIONAL
R

PR ACTICE MATHEMATICS - 10 213


Area of track So, area of sector OAPB
= area of rectangle PQRS = 90 x 22 x (28 2 )²
360 7
– area of rectangle ABCD
1 22
+ 2 [ Area of semi-circle with radius 40 cm = x x 784 x 2
4 7
]
= 1232 m²
  [ area of semicircle with radius 30 cm ]
= (106 x 80) – (106 x 60) Also, area of ∆OAB

Èp p ˘ 1
+ 2 Í ( 40 )² - ( 30 )² ˙ = x OA x OB
2
Î2 2 ˚
= 8480 – 6360 + p (1600 – 900) 1
= x 28 2 x 28 2
2
22 = 784 m²
= 8480 – 6360 + x 700
7
So, area of shaded part
= 8480 – 6360 + 2200
= 2 [ 1232 – 784 ]
= 4320 m²
= 896 m²
26. Also, area of square lawn + area of flower
O
bets
= 896 + 3136
90°
= 4032 m²

A B
27. Area of square = 8²
= 64 cm²
P area of 1 quadrant
ABCD is a square 1
= p (1.4)²
4
∴ AC and BD bisect each other and are equal
∴ AO = OC = DO = BO 1 22 14 14
= x x x
4 7 10 10
In ∆AOB,
= 1.54 cm
  AB² = OA² + OB²
(56)² = OA² + OA² [∴ OA = OB ] ∴ Area of the shaded portion of the square

3136 = 20A² = Area of square – area of circle –


2 (area of a quadrant)
1568 = OA²
= 64 – 55.44 – 2 (1.54)
OA = 1568 = 8.56 – 3.08
= 28 2 m = 5.48 cm²
214 ADDITIONAL
R

PR ACTICE MATHEMATICS - 10
28. Area of square OABC = (21)² Area of square ABCD = (14)²
= 411 cm² = 196 cm²
C B
Area of square PQRS = 4²
= 16 cm²
21 cm
Area of 4 semi-circles with radius 2 cm

Ê 1 ˆ 22
A =4 Á ˜ x2x2
O 21 cm Ë2¯ 7
In ∆OAB, right angled at A 176
= cm²
OB² = OA² + AB² 7
So, area of shaded part
= (21)² + (21)²
= 441 + 441 = 882 cm² Ê 176 ˆ
= 196 – Á16 +
Ë 7 ˜¯
∴ OB = 21 2
176
So, area of quadrant OPBQ with OB as = 196 – 16 –
7
radius
176
90 22 = 180 –
= x x (21 2 )² 7
360 7 1260 +176
1 22 =
= x x 441 x 2 7
4 7
1084
= 693 cm² =
7
∴ area of shaded part = 154.85 cm²
= area of quadrant OPBQ
30. Area of circle = pr²
– area of square OABC
= p (7)²
= 693 – 441
= 49p cm²
= 252 cm²
49 x 22
= cm²
29. 7
A B = 154 cm²
Area of sector
P Q
60 22
= x x 7x7
360 7
14 cm
3 2 cm 3 154
cm cm = cm²
4 cm 6
S R
77
= cm²
3
D 14 cm C = 25.67 cm²
ADDITIONAL
PR ACTICE
R

MATHEMATICS - 10 215
∴ area of circle = pr²
3
Area of ∆ABC = (14)²
4 22
= (2 3 )²
3 7
= x 196
4 22
= 84.87 cm² = x 12
7
So, area of shaded part = 37.71 cm²
= Area of circle + Area of triangle So, area of shaded part
– 2 Area of sector = area of DABC
= 154 + 84.87 – 2 (25.67) – area of circle
= 154 + 84.87 – 51.34
= 36 3 – 37.71
= 187.53 cm²
31. = 62.352 – 37.71
A
= 24.642 cm²

32. Area of square OABC


R Q
o = 7²
r = 49 cm²
B P C Area of sector OAPC
AP ⊥ BC, BQ ⊥ AC and CR ⊥ AB 90 22
= x x7 x7
[ As tangent is perpendicular to radius through 360 7
point of contact.] 154
= cm²
4
3 = 38.5 cm²
Area of DABC = (side)²
4
So, area of shaded region
3
= (12)² = 49 – 38.5
4
= 3 (36) = 10.5 cm²

= 36 3 cm²
WoRKSHeet - 2
Also, area of DABC
Section-A
= area of DAOC + area of DBOC
+ area of DAOB þ
1. Area of section = pr²
1 1 1 360
36 3 = (12) r + (12) r (12) r
2 2 2
36 3 = 6r + 6r + 6r 2. Perimeter of circle = Perimeter of square
(radius = r) (side = x)
36 3
= r ⇒ 2pr = 4x
18
r =2 3 cm ⇒ pr = 2x ...(i)
216 ADDITIONAL
R

PR ACTICE MATHEMATICS - 10
area of circle pr² 350
So, = ⇒   x =
area of square x² 11 2
2
Ê 2x ˆ 350
pÁ ˜ =
= Ë p ¯ 15.554
x² = 22.50 cm²
p 4x 2 1
= x
p² x² 5. Area of circle = 220
4
= ⇒ pr² = 220
π
4¥7 220
= = 14 : 11 ⇒ r =
22 p
⇒ Diameter = 2r
60 22
3. Area of sector = x x 10 x 10
360 7 220
=2 cm
= 52.38 cm² π

4. A B
A B

x
x

D x C
D x C
Circumference of circle = 100
In ∆ABC,
2pr = 100
22 BD² = BC² + CD²
2x
x r = 100 2
7 Ê 220 ˆ
100 ¥ 7 ÁÁ 2 ˜˜ = 2x²
 r = Ë p ¯
2 ¥ 22
175 Ê 220 ˆ
= cm 4Á ˜ = 2x²
11
Ë p ¯
= 2r
350 880
⇒ Diameter = cm  x² =
11 p (2 )
In ∆BCD,
440
BD² = BC² + CD² =
p
2
Ê 350 ˆ So, area of square = x²
Á 11 ˜ = x² + x²
Ë ¯ 440
Ê 350 ˆ
2 =
p
 2x² =Á ˜
Ë 11 ¯ 440
2 = x7
1 Ê 350 ˆ p
 x² =
2 ÁË 11 ˜¯ = 140 cm²
ADDITIONAL
R

PR ACTICE MATHEMATICS - 10 217


6.   r = 0.25 m So, area of circle = pr²
Distance covered in one revolution 22
= x7 3 x7 3
= 2pr 7
= 462 cm²
22 8. Area of sector
=2x x 0.25
7
So, number of revolutions 90 22
= x x2x2
360 7
11¥1000 ¥ 7 22
= = cm²
2 ¥ 22 ¥ 0.25 7
= 7000
9.
7.
A r

R Q
o
r 5p
arc length = 5p cm
0B P C
q
2pr = 5
3 360
Area of ∆ABC = (side)² 2π
4
=5
3 360
= x 42 x 42 rθ = 900 (i)
4
= 441 3 cm² Also, area of sector = 20p
q
Also, AP ⊥ BC, BQ ⊥ AC and CR ⊥ AB pr² = 20p
360
[ As tangent is perpendicular to radius θr²
through point of contact.] = 20
360
So, rθ² = 7200 (ii)
From (i) and (ii), we get
area of ∆ABC = area of ∆BOC
900
+ area of ∆AOC r² = 7200
r
+ area of ∆AOB 900r = 7200
r = 8 cm
1 1 1
⇒ 441 3 = x 42 x r + x 42 x r + 10.
2 2 2
x 42 x r
r3
⇒ 441 3 = 63 r cm²
r1
441 3 r2
⇒  r =
63
= 7 3 cm
218 ADDITIONAL
R

PR ACTICE MATHEMATICS - 10
According to question, 12. R–r =7 ...(i)
1 pR² – pr² = 286
pr² – pr32 = pr²
4
1 22 286 ¥ 7
(R² – r²) =
⇒ pr² – pr² = pr32 7 22
4
3 = 91
⇒ pr² = pr 3
2

4 ⇒ (R – r) (R + r) = 91
3 2
⇒ r² = r 3 ⇒ 7 (R + r) = 91
4
3 ⇒ R+r = 13 ...(ii)
⇒ r3 = r
2
On solving (i) and (ii), we get
3
= (20) R - r =7
2
= 10 3 R + r =13

2r = 20
r =10 cm
Section-B
∴ R = 13 – r
11. = 13 – 10
= 3 cm
r r So, sum of radii = R + r
7 cm
= 10 + 3
= 13 cm
14 cm
13.
Area of rectangle = 14 x 7 D C
= 98 m²
28 cm
2
Ê7ˆ
Area of circle =p Á ˜
Ë2¯ A 40 cm B
22 49
= x 1
7 7 Area of semi-circle = pr²
2
77 1
m² = = p (14)²
2 2
So, area of remaining portion
1 22
= x x 196
= Area of rectangle 2 7
– 2 (area of circle) = 308 cm²

Ê7ˆ Area of rectangle ABCD


= 98 – 2 Á ˜ = AB x BC
Ë2¯
= 98 – 77 = 40 x 28
= 21 cm² = 1120 cm²
ADDITIONAL
R

PR ACTICE MATHEMATICS - 10 219


So, area of remaining paper So, Total perimeter
= 1120 – 308 = 44 + 2 (22) + AB
= 812 cm² = 44 + 44 + 28

14. Perimeter of the top of the table = 116 cm


270 17. Area of shaded region = Area of trapezium
= OA + OB + (2p) 42
360 – Area of quadrant
= 42 + 42 + 63p 1
Area of trapezium = (AD + BC) × AB =
2
= 84 + 63p
24.5
= 84 + 198
(AD + BC) AB = 49 ⇒ 14 (AB)
= 84 + 198
= 49 ⇒ AB = 3.5 cm
= 282 cm
1 2
Area of quadrant = pr
4
15. Circumference of circle = 2pr
1 22
22 = × × (3.5 ) (3.5 )
44 =2x x r 4 7
7
11× 3.5
44 ¥ 7 = = 9.625 cm2
r = 4
2 ¥ 22
\ Area of shaded region = 24.5 – 9.625
= 7 cm
= 14.875 cm2
1
So, area of quadrant = pr²
4 18. Area of quadrant OACB
1 22 1 22
= x x 7² = x x7x7
4 7 4 7
= 38.5 cm² = 38.5 cm²
1
16. Perimeter of semi-circle with OA as radius area of ∆AOD = x7 x4
2
= p (14) = 14 cm²
∴ area of shaded part = 38.5 – 14
22
= x 14 = 24.5 cm²
7
= 44 cm
q
Perimeter of semi-circle with OA as 19. Length of arc = 2pr
360
diameter
q 22
8.5 = x2x xr
= p (7) 360 7
22 8.5 ¥ 360 ¥ 7
= x (7) ∴ r =
7 30 ¥ 2 ¥ 22
= 22 cm = 16.23 cm
220 ADDITIONAL
R

PR ACTICE MATHEMATICS - 10
20. Cost of fencing 1 metre So, sum of area of part I and II
Circular field = ` 12 = area of square ABCD
Total cost of fencing a circular field = ` 2640 – 2 (area of semi-circle)
∴ Circumference of circular field Ê 275 ˆ
= 100 – 2 Á ˜
2640 Ë 7 ¯
= 550
12 = 100 –
7
= 220 m
150
= cm²
⇒ 2pr = 220 7
150
22 Also, sum of areas of part III and IV = cm²
2x x r = 220 7
7
So, area of shaded part
220 ¥ 7
r =
2 ¥ 22 = area of square ABCD
= 35 m – Sum of areas of part I and II
22 – Sum of areas of part III and IV
Area of circular field = (35)²
7
= 3850 m² 150 150
= 100 – –
7 7
∴ cost of ploughing the field
300
= 100 –
= 3850 x 2 7
400
= ` 7700 =
7
= 57.14 cm²
Section-C
22. In ∆QPR,
21.
∠QPR = 90° (Angle in a semi circle is a
A B
right angle.)
I
∴ QR² = PQ² + PR² (Pythagoras theorem)
III IV
= (12)² + 5²
II
= 144 + 25
D C
= 169
Area of square ABCD =(10)²
⇒ QR = 13 cm
= 100 cm²
area of semi-circle with QR as diameter
Area of semi-circle with AD as diameter 2
1 22 Ê 13 ˆ
1 22 = x xÁ ˜
= x x5x5 2 7 Ë2¯
2 7
11
275 22 169
= cm² = x
7 14 42
ADDITIONAL
R

PR ACTICE MATHEMATICS - 10 221


1859 So, area of part PMQA
= cm²
28 = area of sector POQA
1
area of ∆QPR = x PR x PQ – area of ∆POQ
2
1 = 52.38 – 43.3
= x 5 x 12
2
= 9.08 cm²
= 30 cm²
area of shaded part
∴ area of shaded part
= area of semi-circle
1859 - 840
= – area of part PMQA
28
1019 = 39.29 – 9.08
= cm²
28
= 36.4 cm² = 30.21 cm²
24. Area of sector (with radius 14 cm)
23. Area of semi-circle PBQ 60 22
= x x 14 x 14
1 22 360 7
= x x r² area of sector (with radius 28 cm)
2 7
11 60 22
= x 5² = x x 28 x 28
7 360 7
275 11
= cm² = (784 – 196)
7 21
= 39.29 cm²
11
As OP = OQ = PQ = 10 cm = (588)
21
⇒ ∆POQ is an equilateral triangle = 308 cm²
⇒ ∠POQ = 60°
25. Let AO = OB = x
3 ⇒ AB = 2x
Also, area of ∆POQ = (side)²
4 Perimeter of semi-circle (with AO as diameter)
3 1
= (10)² 22 x
4 = x2x ×
2 7 2
= 25 3 cm² 11x
= cm
7
= 43.3 cm²
Perimeter of semi-circle (with AB as diameter)
area of sector POQA
1 22
= ×2× x
60 22 2 7
= x x 10 x 10
360 7 22 x
= cm
= 52.38 cm² 7
222 ADDITIONAL
R

PR ACTICE MATHEMATICS - 10
Given : Perimeter of figure = 40 cm ⇒ both the diagonals must be equal
11x 22 x area of circle = 1256 cm²
⇒ + + OB = 40 cm
7 7
3.14 r² = 1256
33 x
⇒ +x = 40 1256
7 ⇒ r² =
40 x 3.14
⇒   = 40 = 400
7
⇒    x = 7 cm ⇒ r = 20 cm
⇒ Diameter of circle = 2r
Area of semi-circle (with AO as diameter)
2
= 40 cm
1 22 Ê 7 ˆ
= x Diameter must be same as diagonal of the
2 7 ÁË 2 ˜¯ square
11 49
= x ⇒ Diagonal of square = 40 cm
7 4
So, area of rhombus
77
= cm² 1
4 = x 40 x 40
2
Area of semi-circle (with AB as diameter)
= 800 cm²
1 22
= x x7x7
2 7 27. Radius of long hand = 60 cm
= 77 cm²
Distance travelled by long hand in 1 round
So, area of shaded region = 2p (6)
77 = 12p
= + 77
4
Number of rounds made by long hand
77 + 308
= In 24 hours = 24
4
308 So, Total distance travelled by long hand in
= cm²
4 24 hours = 24 x 12p
= 96.25 cm² = 288p
Radius of short hand = 4 cm
26.
Distance travelled by short hand in 1 round
= 2p (4)
= 8p
Number of rounds made by short hand
In 24 hours = 2
So, Total distance travelled by short hand in
As all the vertices of a rhombus lie on a circle 24 hours = 8p x 2
∴ it must be a square = 16p
ADDITIONAL
R

PR ACTICE MATHEMATICS - 10 223


So, Sum of distances = 228p + 16p ∠OPA = ∠OPB = 90° (By construction)
= 304p ∴ ∆OPA ≅ ∆OPB (RHS)
= 304 x 3.14 ⇒ AP = BP (CPCT)
= 954.56 cm ⇒ AB = 2AP
28.
Area of trapozium 10
1 ⇒ AP = = 5 cm
= (Sum of parallel sides) x Distance 2
2 In ∆OPA,
between the parallel sides
1 OA² = OP² + AP²
= (AB + DC) x 14
2
1 ( 5 2 )2 = OP² + 5²
= (18 + 32) x 14
2 50 – 25 = OP²
= 350 cm²
1 25 = OP²
area of a quadrant = p (7)²
4 ∴ OA² = 5 cm
1 22
= x x 49 1
4 7 So, area of ∆OAB = AB x OP
2
154
= cm² 1
4
= 10 x 5
So, area of shaded region 2
= area of trapezium = 25 cm²
– 4 (area of a quadrant) In ∆AOP,
Ê 154 ˆ AP
= 350 – 4 Á ˜ tan θ =
Ë 4 ¯ OP
= 350 – 154 5
=
5
= 196 cm²
=1

29. ⇒ θ = 45°

O So, ∠AOB = 2 (45°)


5 2 cm
θ = 90°
P
A B ∴ area of sector AOBC
10 cm
90 22
= x x 25 x 2
360 7
C 275
= cm²
Draw OP⊥AB 7
In ∆OPA and ∆OPB So, area of shaded part

OA = OB (radii of same circle) = area of sectior AOBC

OP = OP (common) – area of ∆AOB


224 ADDITIONAL
R

PR ACTICE MATHEMATICS - 10
275 Section-D
= – 25
7 31. A
275 - 175
=
7 I
100
= cm²
7
30. D C
B C
II
P 3
Area of ∆ABC = (side)²
4
1.732
I
Q = (8)²
4
= 27.712 cm²
A B
area of sector 1
Area of square ABCD = 7² 60
= x 3.142 x 4²
= 49 cm² 360
Area of quadrant ABPD 60
= x 3.142 x 16
90 360
= p (7)²
360 = 8.38 cm²
p So, area of shaded part
= (49)
4
= Area of ∆ABC
49 22
= x – 3 (area of sector 1)
4 7
= 27.712 – 3 (8.38)
77
= cm² = 27.712 – 25.14
2
= 38.5 cm² = 2.572 cm²

So, area of part II 32.


II
= 49 – 38.5
= 10.5 cm²
I 14 cm
Similarly, area of part I = 10.5 cm²
∴ Area of the shaded region
28 cm
= area of square ABCD
– area of 1 – area of 11 Area of ractangle piece = 28 x 14

= 49 – 10.5 – 10.5 = 392 cm²

= 49 – 21 1 22
area of part 1 = x x 7²
2 7
= 28 cm²
= 77 cm²
ADDITIONAL
R

PR ACTICE MATHEMATICS - 10 225


1 22 Area of circle with center O1
area of part II = x x (14)²
2 7 22
308 cm² = x 14 x 14
7
So, area of shaded region = 616 cm²
= area of rectangular piece – area of part So, area of shaded part
I + area of part II
= Area of square – 2 (area of sector) +
= 392 – 77 + 308 = 623 cm² (area of circle)
20 m = 784 – 2 (154) + 2 (616)
33.
6m
= 784 – 308 + 1232
= 1708 cm²

35.
B
I
Area of the grassy lawn in which the calf D
can graze initially
90 22
= x x 6x6
360 7 A C
= 28.286 m²
In DBAC
Area of grassy lawn in which the calf can
graze if the length of rope is increased (BC)2 = (AB)2 + (AC)2
by 5.5 m
= (14)2 + (14)2 = 392
90 22
= x x 11.5 x 11.5
360 7 BC = 14 2
= 103.911 m²
Area of semicircle with BC as diameter
∴ Increase in the area of the grassy lawn in 2 2
which the calf can graze 1 14 14
= p 2 2 = 154 cm2
2 Ł2 ł Ł2 ł
= 109.911 – 28.286
Area of DBAC =
= 75.625 cm²
Area of quadrant ACDB
34. Area of square = (28)² = 154 cm2

= 784 cm² Area of region I = area of quadrant area of


D
Area of part of circle inside square
= 154 – 98 = 56 cm2
= Area of sector
Area of shaded region = Area of semicircle –
90 22 Area of region I
= x x 14 x 14
360 7
= 154 cm² = 154 – 56 = 98 cm2
226 ADDITIONAL
R

PR ACTICE MATHEMATICS - 10
36. ∴ Diameter = 2 (radius)
A a cm D
2275
I = m
11
In ∆BCD,
B C
BC² + CD² = BD²
2
Ê 2275 ˆ
To find : area of shaded region x² + x² =   Á ˜
Ë 11 ¯
area of part 1 2
Ê 2275 ˆ
90 22 2x² =  Á ˜
= x x a² Ë 11 ¯
360 7
2
11 1 Ê 2275 ˆ
= a² cm² ⇒   x² =
14 2 ÁË 11 ˜¯
area of square ABCD
Area of square ABCD = x²
(with side a + a = 2a cm)
2
= (2a)² 1 Ê 2275 ˆ
= m²
= 4a² cm² 2 ÁË 11 ˜¯
So, area of shaded region = 21386.88 m²
= area of square
38. (a) Area of square ABCD = (22)²
Ê 11 2 ˆ = 484 cm²
– 4a² – 4 Á a ˜
Ë 14 ¯
484
∴ area of central part = cm²
6
= 4a² – 22 a² = a² 5
7 7 484
⇒  pr² =
5
37. A B
22 484
⇒   r² =
7 5
x
22
484 7
⇒  r² = x
D x C 5 22

Circumference of circle = 650 m 154


=
⇒  2pr = 650 5
22 ⇒   r = 5.55 cm
⇒ 2x r = 650
7
325
650 ¥ 7 Circumference of the central part
⇒   r =
2 ¥ 22 22
= 2 pr² = 2 x x 5.55
2275 7
= m = 34.88 cm
22
ADDITIONAL
R

PR ACTICE MATHEMATICS - 10 227


1 Also, area of rectangle = 20 x 15
(b)  EF =
(Circumference of circle)
4 = 300 cm²
1
= (34.88) 1 22 Ê 15 ˆ
2
4 So, area of semicircle = x xÁ ˜
= 8.72 cm 2 7 Ë2¯
. .
( . BC = AD = 15 cm)
11
E H 1 22 225
x = x
2 7 4
O
2475
F = cm²
G 28
∴ Area of shaded region
BF = AE
2475
= AO – OE = 246 +
28
= AO – 5.55 9363
=
28
= 11 2 – 5.55 = 334.393 cm²
= 15.554 – 5.55 = 10.004 cm
40. Area of semicircle (with diameter CD)
ÈIn DAOB, –AOB = 90 ˘
Í ˙ 1 22
Í\ AB² = AO² + OB² ˙ = x x 7²
Í( 22 )² = AO² + BO² ˙ 2 7
Í ˙ = 77 cm²
ÍÎfi AO = BO =11 2 cm ˙˚
area of rectangle ABCD
So, Perimeter of part ABEF
= AB x BC
= 22 + 10.004 + 10.004 + 8.72
= 14 x 7
= 50.728 cm²
= 98 cm²
39. Area of rectangle ABCD = 20 x AD area of semi-circle (with diameter BC)
In ∆ADE, ∠AED = 90°, 2
1 22 Ê 7 ˆ
= x xÁ ˜
AE² + DE² = AD² 2 7 Ë2¯
9² + 12² = AD² 1 22 49
= x x
2 7 4
81 + 144 = AD² = 19.25 cm²
  225 = AD Similarly, area of semicircle (with
∴   AD = 15 cm diameterAD)

1 = Area of rectangle ABCD


So, area of n ∆ADE = x 9 x 12
2 – area of semicircle (with diameter CD)
= 54 cm² + area of semicircle (with diameter BC)
228 ADDITIONAL
R

PR ACTICE MATHEMATICS - 10
+ area of semicircle (with diameter AD) 9
Radius of circle D = cm
= 98 – 77 + 19.25 + 19.25 4
= 59.5 cm² Area of circle D = π h2
2
9
CASE STUDY-1 = π
Ł4 ł

3 81
(i) (a) Radius of semicircle a = cm = π cm2
2 16
3
Radius of semicircle b and c = cm 9
2 (iii) (b) Radius of semicircle E = cm
2
2
3
p r2
Ł2 ł Area of semicircle E = π
Area of semicircle A = 2
2
2
9 9
p× π
4 Ł2 ł
= = cm2
2 2

= cm2 81
8 p
= 4 cm2
9p 2
Area of semicircle B and C = cm2
8
81
= p cm2
9π 9π 9π 27π 8
Total area = + + = cm2
8 8 8 8
(iv) (d) Area of shaded region = Area of semicircle
(ii) (b) Radius of semicircle E = Daimeter of circle E – (Area of
D circle D + Area
of semicircles A
Daimeter of semicircle E = Daimeter of and C)
semicircle A + Area of semicircle A = Area of semicircle
Daimeter of 9p
semicircle B + C= cm2
8
Daimeter of
semicircle C
Area of shaded region
= 3 cm + 3 cm + 3 81  81 18p 
= p − p +
cm 8 16 8 
= 9 cm
9 cm2
Radius of semicircle E = cm 162 117
2 = p- p
16 16
9
Daimeter of circle D = cm 90
2 = p cm2
R 32 229
ADDITIONAL
PR ACTICE MATHEMATICS - 10
81 (iii) (a) As AOB is an equilateral triangle ∴ area of
(v) (b) Area of circle D = π cm2
16 3 2
∆AOB is a
4
81
Area of semicircle E = p cm2 Where 'a' is the side of triangle AOB
8
3
× (10 )
2
Hence area of circle D is 2 times the area Area of ∆AOB =
of semicircle E. 4
= 25 3 cm2
CASE STUDY-2 Area of sector OCD = 6π cm2
θ Area of red region = Area of ∆AOB –
(i) (b) Area of sector = πr
2
Area of sector
360
OCD
For sector OCD, θ = 60°, r = 6 cm
(
= 25 3 - 6p cm2 )
60
π (6 )
2
Area = (iv) (a) Area of minor sector OCD (Yellow
360
region) = 6π cm2
= 6π cm2
Area of major sector OCD (Blue region)
300°  = 30π cm2
p (16 ) 
2
(ii) (c) Area of blue region =
360°  
∴ Area of major sector OCD is 5 times
30 the area of minor sector.
=
36
(36π )
(v) (a) Total area of red + yellow region is the
= 30π cm 2 area of ∆AOB i.e. 25 3 cm2.

230 ADDITIONAL
R

PR ACTICE MATHEMATICS - 10
Chapter

13 Surface Areas and Volumes


Multiple choice Questions

1. (a) Volume of piece of iron = (49 × 33 × 24) cm² 1


(18)² (32) = × (r2)² × 24
3
4 3 4
Volume of sphere = πr 324 × 32 × 3
3 (r2)² =
A T Q; 24 3
(r2)² = (18 × 2)²
Volume of iron = Volume of sphere
r2 = 36 cm
4 3
49 × 33 × 24 = πr
3 4. (a) C. S. A. of cylinder = 264 m²
4 22
49 × 33 × 24 = × × πr3
3 7 Volume of cylinder = 924 m3
3 6 3
49 × 33 × 24 × 3 × 7 C .S. A of cylinder 264
r =
3
=
4 × 22 924
Volume of cylinder
2
2p r h 264
r3 = 9261 =
pr h
2 924
3
r = 9261 2 264
=
r = 21 cm r 924
2 × 924
r =
2. (a) A. T. Q; 264
Volume of cone = Volume of cylinder r = 7 cm (i)
We know;
1
p r ² h1 = p r ² h2 C. S. A of cylinder = 264 cm²
3
1 2 πrh = 264 cm²
h = h2
3 1 22
1 2× × 7 × h = 264
h = 5 7
3 1 24
264 × 7
h1 = 15 cm h = =6m
2 × 22 × 7
2
3. (a) A T Q, 2r 2×7
Ratio of Diameter to height = =
Volume of cylinder = Volume of cone h 6

1 2 7
p r12 h1 = pr h =
3 2 2 3
ADDITIONAL
R

PR ACTICE MATHEMATICS - 10 231


5. (a) In the right circle cone, the cross section 3. Volume of sphere = n × Volume of cones
made by a plane parallel to its base is a circle.
4 1 2
π r3 = n × π r h
3 3
WORKSHEET - 1 4 × (10.5) = n × (3.5)2 (3)
3

SECTION-A 4 × (10.5)3
n= = 126 cones
(3.5)2 × 3
1. A. T. Q;
Radius of cylinder = Radius of sphere 4. Volume of sphere = Volume of cone

Diameter of sphere = 2πr 4 1


π r3 = π r12 (R)
3 3
= 2r
4r3 = r12 (R)

2. Surface area of cube = 6a² 4r 3


= r12
R
Surface area of sphere = 4πr²
2r r
A T Q; r1 =
R
Surface area of cube = Surface area of sphere
πh
5. Volume of frustum = R12 + R2 2 + R1R2 
6a² = 4πr² 3  

3a² = 2πr² 10
R1 = = 5m
2
2
r 3 4
 a  = 2p R2 = =2m
  2
h = 6m
r 3
= p ×6
a 2 p Volume of frustum = 52 + 2 2 + (5)(2 )

3
4 3 = 2π [39]
Volume of sphere pr
3
⇒ = = 245 m3
Volume of cube a3

 r3  6. Total surface area of canvas


4
= π  a3  = Sum of curved surface area and curved
3
surface area of cylinder
2
4  3 
= π   C. S. A of cone = πrl
3  2 π 
  = (π) (105) (40)
4  3 3 
2
= 4200π m²
= π  
3  2 2π π  C. S. A of cylinder = 2πrh

Volume of sphere 6 = (2) (π) (105) (4)


=
Volume of cube π = 840 π m²
232 ADDITIONAL
R

PR ACTICE MATHEMATICS - 10
Total surface area of canvas = 4200 π + 840 π 1
Time = 30 minutes = hr = 0.5 hr.
= 5040 π 2
Area irrigated by 0.08 m
22
= 5040 × = 12 × 30 × 10, 000 × 0.5
7
= 720 × 22 A × 0.08 = 1, 800, 000
= 15840 m² 1,800,000
A =
0.08
7. Surface area of hemisphere = S urface area of A = 22, 500, 000 m²
cone
3πr² = πrl + πr² 10.
r
2p r2 = p rl r

2r = r ² + h² [ l = r ² + h² ] hH
Squaring b/s :-
r
4r² = r² + h²
3r² = h² Given,
r 1 19
= Volume of air = 41 m³
h 3 21
2r = H ...(i)
8. Volume of hemisphere
Total height of the building
= Surface area of hemisphere
= Height of cylinder
2
p r3 = 3π r2 Total height of the building
3
2 = Height of hemisphere
r = 3
3
H = h+r ...(ii)
9
r=
2 From (i) and (ii) :
2r = 9 cm
2r = h + r
Diameter of hemisphere = 9 cm.
h= r
Volume of building = Volume of cylinder
Section-B
+ Volume of hemisphere
9. Width of canal = 30 m
880 2
Depth of canal = 12 m = πr²h + πr³
21 3
Flow velocity = 10 km/hr = 10, 000 m/hr
2
Standing water required = 8 cm = 0.8 m = πr² [ h + r]
3
ADDITIONAL
R

PR ACTICE MATHEMATICS - 10 233


5 13. Total area without dimples = πr² × n
= πr² [ r]
3 22
= (0.2)² × 150
880 5 7
= πr²   = 18. 857 cm²
21 3
Total area of without dimples = 4πr²
880 × 3 × 7 2
r³ = 321 × 5 × 22 = 8 22  4.1
= 4 × ×  
7  2 
h = 2m = 52. 83 cm²
H = h+r Area where there are no dimples
= (52. 83 – 18. 857) cm²
H = 2+2
= 33. 714 cm²
H = 4m Surface area exposed to surroundings
= (33. 973 + 37. 714) cm²
11. Let radius cone Y =r = 71. 687 cm²
So, radius of cone X = 3r
14. Volume of resulting spheres
Let volume of cone Y = V
= Volume of three spheres
So, volume of cone X = 2V
4 4 4 4
Let height of cone X be h1 and πr³ = πr1³ + πr2³ + πr ³
3 3 3 3 3
height of cone Y be h2 4 4
π r³ = π (r1³ + r2³ + r3³)
3 3
1 r³ = (6³ + 8³ + 10³)
p (3r )2 (h1)
Volume of cone X 3
= r³ = (216 + 512 + 1000)
Volume of coneY 1
p (r )2 (h2 ) r³ = 1728
3
r = 12 cm
2V 9 h1
=
V h2 15. Let the height of platform be 'h' metres.
h1 2 Volume of mud dug out from the well
=
h2 9
= Volume of platform
12. l×b = x 770 = 22 × 14 × h
b×h = y
770
h×l = z h=
22 ×14
(l × b) × (b × h) × (h × l) = x y z h = 2. 5 m

(l b h)² = x y z
16. 2πR = 18 cm
lbh = xyz
22
2 × × R = 18 cm
Volume of cuboid = xyz 7

234 ADDITIONAL
R

PR ACTICE MATHEMATICS - 10
18 × 7 = (12 ) ² + ( 4.5) ²
R=
2 × 22
9 = 144 + 20. 25
R= cm
p
= 164. 25
2πr = 6
6 l = 12. 81 cm
r=
2p

r=
3 Section-C
p
19. Capacity of drinking glass
Given;  l = 4 cm
1
Curved surface area of frustum = πh (r1² + r2² + r1 r2)
3
= π (R + r) l 1
= π (14) (2² + 1² + 2 × 1)
9 3 3
= π + (4) 1 22
Łπ πł = × × 14 (7)
3 7
12
= p (4) 308
Łπ ł =
3
= 102. 6 cm³
= 48 cm²
6 cm
17. Given, 20. Radius of sphere = = 3 cm
2
Area of valley = 97280 km² 2 cm
Radius of wire = = 1 cm
Rainfall = 10 cm = 0. 00010 km 2
Volume of rainwater Volume of sphere = Volume of cylinder (wire)

= Area of valley × Rainfall 4


p r 3 = p r ²h
3
= 97280 × 0. 00010 km³
4
= 0. 97280 km³ (3)3 = (1)² h
3
0. 97280 4×9 = h
Volume in 1 day = = 0. 7 km³
14 h = 36 cm
Volume of a river = l × b × h
∴  length of wire = 36 cm
75 3
= (1072 × × ) km³
1000 1000
= 0. 2412 km³ 21. Height of cone = 9 cm
Volume of 3 rivers = 3 x 0. 2412 24
Radius of cone = = 12 cm
= 0. 7236 km³ 2
1
Volume of cone = πr²h
18. h of cone = 12 cm (Given) 3
1
r of cone = 4. 5 cm = π (12)² (9)
3
slant height (l) = h² + r ² = 432 π cm³
ADDITIONAL
R

PR ACTICE MATHEMATICS - 10 235


Height of cylinder = 110 cm H-h 3h - h
⇒ = =2
Radius of cylinder = 12 cm h h
Required ratio = h : (H – h)
Volume of cylinder = πr²h
= 1 : 2
= π (12)² (110)
= 15840 π cm³ 23. Volume of sphere = Volume of cylinder

Volume of irom pole = 432 π + 15840 π 4


p r 3 = p rc2 h (rs = 4.2 cm)
3 s
= 16272 π (rc = 6 cm)
4
= 51120 cm³ (4. 2)³ = (6)² h
3
( 4. 2 )
3
Mass of pole = 408960 g 4
× = h
(6 )
2
3
22.
2. 744 cm = h
A
24. Radius of cylinder = 6 cm (Given)
l h
Height of cylinder = 15 cm
r H
Radius of cone and hemisphere = 3 cm
E F
R L (Given)
B D C Height of cone = 12 cm

Let R, H and L be the radius, height and slant A. T. Q. ;


height of the larger cone. Let r, h and l be the Volume of cylinder = Volume of cone + volume
radius, height and slant height of smaller cone. of hemisphere
Consider ∆ADC and ∆AEF 1 2 
p rcy2 h =  p rc2 h + p rh3  × n
r h l 3 3 
= = ...(i)
R H L 1 2 
C. S. A. of smaller cone = πrl (6)² × 15 =  × (3)² ×12 + (3)³  × n
3 3 
C. S. A. of larger cone = πRL 540 = (36 + 18) × n
8 540
RL – πrl = = n
9 54
1 n = 10 cones.
⇒ πRL = πrl
9
L R
⇒ × = 9 25. Diameter of copper wire = 3 mm or 0.3 cm
r r
H H Number of rounds of copper wire around
× = 9 [From (i)] cylinder
h h
H Height of cylinder 12
= 3 = = = 40 rounds
h Diameter of wire 0.3

236 ADDITIONAL
R

PR ACTICE MATHEMATICS - 10
Wire required in round Inner surface area of the vessel
= 2πr (Circumference of base cylinder) = C S A of cylinder +
C S A of hemisphere
= 2 × π × 5 = 264 + 308
= 10 π cm = 572 cm²
Length required in 40 rounds
= 40 × 10 π = 400 π
Section-D
27.
22
= 400 ×
7
2 cm
= 1257. 14 cm
0.3 3 cm
Radius of wire = = 0. 15 cm
2 12 cm
Volume of wire = Area of wire × Length of wire
= πr² × 1257. 14 3 cm

22 h 2 cm
= × (0. 15)² × 1257. 14
7
= 88. 898 cm³
Mass of wire = Density x Volume Volume of model = 
Volume of cylinder +
Volume of 2 cones
= 8. 88 × 88. 898
Volume of cylinder = πr²h
Mass of wire = 789. 41 g.
r = Radius of cylinder
3
14 = cm
26. Radius of hemisphere = = 7 cm 2
2 h = Height of cylinder
Curved surface area of hemisphere
h = 12 – (2 + 2) = 12 – 4
= 2 π7²
= 8 cm
22
= 2 × × (7)² 2
7 22 3
= 308 cm² Volume of cylinder = ×   × 8
7 2
Height of cylinder = Total height – Height = 56. 57 cm³
of hemisphere 2
1 22 3
= 13 – 7 = 6 cm Volume of cylinder = × ×   ×2
3 7 2
Curved surface area of cylinder 33
= = 4. 7 cm³
= 2 πrh 7
Volume of model containing air
22
= 2 × ×7×6 = 56. 57 + 2 × 4. 71
7
= 264 cm² = 65. 99 cm³
ADDITIONAL
R

PR ACTICE MATHEMATICS - 10 237


28. 10
1. 4 29. Radius of cylindrical tank =
=5m
1.4 2
= 500 cm
5 cm
Height of cylindrical tank = 200 cm = (2 m)
1.4 Volume of cylindrical tank = πr²h
1. 4
= π (500)² (200)
Total Volume of gulab-jamun Volume of cylindrical tan k
Time taken =
Volume of water flowing in1 hr
= Volume of cylinder +Volume of 2 hemispheres
p (500)²(200)
Volume of cylinder = πrc²hc Time taken =
10 × 300000 × p ×10
rc = Radius of cylinder
50 0 ×50 0 × 2 0 0
2. 8 =
10 × 3 0 0 0 0 0 ×10
= = 1. 4 cm
2 50 5
hc = Height of cylinder = = hr
30 3
= 5 – 2 × (1. 4) 5
= × 60
= 5 – 2. 8 = 2. 2 cm 3
Time taken = 100 minutes
22
× (1. 4)² × (2. 2)
Volume of cylinder =
7
30. O
= 13. 55 cm³
Volume of 2 hemispheres 60°
30°
2  r 20 cm
= 2 ×  p r 3h  A Q 2
B
3 
2
= 2 × × π × (1. 4)³
3
r1
2 22 C D
= ×2× × (1. 4)³
3 7
P
= 11. 50 cm³
Given,
Volume of gulab-jamun = (11. 50 + 13. 55) cm³
= 25. 05 cm³ OP = 20 cm

Volume of sugar syrup in 1 gulabjamun QO = QP = 10 cm

30 ∠QOB = 30°

= × 25. 05
100 In ∆DOP,
= 7. 51 cm³
PD
Volume of sugar syrup for 45 gulabjamuns tan θ =
OP
= 45 × 7. 51 = 337. 95
r1
= 338 (approx) tan 30° =
20
238 ADDITIONAL
R

PR ACTICE MATHEMATICS - 10
20 Speed of water = 10 m/ min
r1 =
3 = 1000 cm/ min
In ∆BOQ Volume of water flowing in 1 minute
BQ
tan θ = = πr²h
OQ
r2 2
tan 30° = 22 1
10 = ×   × 1000
7 4
10
r2 = cm 1375
3 = cm³
Volume of frustum ADBC 7
40
ph Radius of cone = = 20 cm
= (r1² + r2² + r1 r2) 2
3
Depth of cone = 24 cm
π ×10   20   10  20 10 
2 2

=   +   + +  1
3   3   3  3 3  Capacity of conical vessel = πr²h
3
10p  400 100 200 
=  3 + 3 + 3  1 22
3 = × × (20)² × 24
3 7
10p 700
= × 70400
3 3 = cm³
7
7000p
= cm³ Time required
9
Volume of wire = πrw²h Capacity of vessel
=
  Volume of water flowing in1 min ute
2
1  1 1 
= π   × h =rw = cm 70400
 32   16 32  7 256
 2  = =
1375 5
πh 7
=
32 × 32 = 51 min 12 sec
Volume of frustum = Volume of wire
32. Volume of water left = Volume of cylinder
7000 p πh – (Volume of cone + Volume of hemisphere)
=
9 32 × 32 Volume of cylinder = πrc2hc
7000 × 32 × 32
h= cm rc = 5 cm
9
h = 796444. 44 cm hc = 10. 5 cm
5 Volume of cylinder = π (5)² (10. 5)
31. Radius of cylindrical pipe = mm
2
= 262. 5 π cm³
5 1
= × cm
2 10 1
Volume of cone = π (3. 5)² (4)
1 3
=
4 = 16. 33 π cm³
ADDITIONAL
R

PR ACTICE MATHEMATICS - 10 239


2 h= 4.6191
Volume of hemisphere = πrh³
3 h = 2. 149 m
rh³ = 3. 5 cm
C. S. A. of cone = πrl
2 22
π (3. 5)³
Volume of hemisphere = = × 2. 15 × 3. 04
3 7
= 28. 58 π cm³ = 20. 5417 m²
Volume of water left = 262. 5 π T. S. A. of building = C. S. A. of cylinder
– (16.33 π + 28. 58 π) + C. S. A. of cone

= π (217. 59) cm³ = 51. 3543 + 20. 5417


= 71. 896 m²
22
= × 217. 59 cm³ Volume of building = Volume of cone
7
= 683.579 cm³ + Volume of cylinder
1
πr²hVolume of cone =
4.3 3
33. Radius of cylinder = = 2. 15 m
2 1 22
= × × (2. 15)² × 2. 15
Height of cylinder = 3. 8 m 3 7
= 10 .4116 m³
C. S. A. of cylinder = 2 πrh
Volume of cylinder = πr²h
22
= 2 × × 2. 15 × 3. 8
7 22
= × (2. 15)² × 3. 8
359. 48 7
= m² = 55. 2059 m³
7
= 51. 3543 m² Total Volume of building
As vertical angle of cone is a right angle. = 10. 4116 + 55. 2059
Let ABC be a triangle = 65. 6175 m³
l = AB = AC
34. A
l² + l² = BC²
15 cm
P
2 l² = 4. 3² O1 x O
l² = 9. 245
l = 3. 04 m 20 cm

We know,
B
l² = r² + h²
Let DAOB is the right triangle with AB as
(3. 04) = (2. 15)² + h² hypotenuse.
9. 2416 = 4. 6225 + h² Let OA = 15 cm
h² = 9. 2416 – 4. 6225 OB = 20 cm
240 ADDITIONAL
R

PR ACTICE MATHEMATICS - 10
In DAOB Surface area of cone AOO' = πrl
= π (PO) (AO)
AB = OA2 + OB 2
= π (12) (15) = 1
152 + 20 2 = 180 π
=
Surface area of cone BOO' = πrl
= 25 cm
= π (12) (20) = 240 π
Let OP = x and AP = y
Net surface area = 100π + 240π = 320π cm2
Area of DAOB = Area of DAPO + Area of DBPO
1 1 1
× 15 × 20 = (y) (x) + (x) (25 – y) WoRKSHeet - 2
2 2 2
300 = xy + 25x – xy Section-A
x = 12 1. Volume of cone are in the ratio 1 : 4 diameters
In DAPO are in the ratio 4 : 5
So, radius are also in the ratio 4 : 5.
AO2 = AP2 + PO2
Volume of cone1 1
225 = y2 + 144 =
Volume of cone 2 4
y = 9 cm 1
π r12 h1
3 1
AP = 9 cm =
1 4
BP = 25 – 9 π r22 h2
3
= 16 cm  r1 
2
h 1
  × 1 =
Volume of cone AOO' =
1 2
πr h  r2  h2 4
3 2
h1 1
4
1  5  × =
π (PO)2 (AP)
= h2 4
3
16 h1 1
1 × =
= π (144) (9) 25 h2 4
3
Volume of cone BOO' = 1357.17 cm3 h1 25
=
h2 64
1 2
= πr h
3 2. 4πr² = 616
1
= π (PO)2 (BP) 22
3 4× × r² = 616
7
1 616 × 7
= π (144) (16) r² =
3 22 × 4
= 2412.74 cm3
4312
r² =
Net volume of cone = Volume of BOO' + 88
Volume of AOO' r² = 49
= 2412.74 + 1357.17 = 3770 cm 3
r = 7 cm
ADDITIONAL
R

PR ACTICE MATHEMATICS - 10 241


3. Radius of hemisphere = Radius of cone = r Surface area of sphere = C. S. A. of cylinder
Also, Height of cone = Height of Hemisphere = r 4 p rs2 = 2 p rh
l (Slant height) = h² + r ² = r² + r² = r 2 2rs2 = (12) (6)
rs2 = (6) (6)
Curved surface area of hemisphere 2p r 2
=
Curved surface area of cone p rl rs = 6 cm
2p r ²
=
p r 2r 7. Radius of greatest sphere inside the log
= 2 : 2
= radius of the log
= 2 :1 4
So;Volume of sphere = πr³
3
4. The total thickness of the plates become the
height of cylinder. 4
= π (1)³
3
1 4 22
Net thickness = cm × 50 = 25 cm = ×
2 3 7
radius = 7 cm
88
Total surface area of cylinder = 2pr (r + h) = = 4. 19 cm³
21
= 2p (7 cm) (7 cm + 25 cm)
8. In the hemisphere,
= 1408 cm 2
Height of hemisphere = Radius of hemisphere
Volume of cylinder = pr2h
h=r
= p (7 cm) (25 cm) 2
For the volume of cone,
= 3850 cm 3
Radius of cone = Radius of hemisphere

5. Volume of water = Volume of conical flask R=r

1 1
= πr²h πR²h
Volume of cone =
3 3
As the water is poured into the cylindrical flask, 1
= πr²h [R=r]
3
So,Volume of cylinder = Volume of water
1
= πr³ [h=r]
p ( mr )²H = 1 p r ²h 3
3
h 9. After r = 7 cm
H=
3 m² 4 3
πr
Volume of sphere =
6. Height of cylinder (h) = 12 cm 3
12 4
Radius of cylinder (r) = = 6 cm = π (7)3
2 3
A. T. Q., = 1436.75 cm3
242 ADDITIONAL
R

PR ACTICE MATHEMATICS - 10
10. Slant height (l) = 5 cm l = 625
R = r1 – r2 = 4 cm = 25 cm
l² = R² + h² Total surface area of toy
5² = R² + h² = Curved surface area of cone
+ Curved surface area of cylinder
5² = 4² + h²
+ Area of bottom part of cylinder
h² = 5² – 4²
= πrl + 2 πrchc + πrc²
h² = 25 – 16
= π [ rl + 2 πrchc + rc² ]
h² = 9
= π [ (20) (25) + (2) (20) (21) + (20)² ]
h = 3 cm
= π [ 500 + 840 + 400 ]
22
Section-B = × 1740
7
11. = 22 × 248. 57
4c
m 4 cm = 5468. 54 cm²

13. Height of cylinder (h) = 10 cm


4 cm 4 cm
8 cm Radius of cylinder (r) = 3. 5 cm
The length of the cuboid becomes 8 cm. Breadth Radius of hemisphere (R) = Radius of cylinder
and height will remain 4 cm.The surface area of
cuboid = 2(lb + bh + hl) = 3. 5 cm

= 2 [(8 × 4) + (4 × 4) + (4 × 8)] Total surface area of article


= Curved surface area of cylinder
= 2 [80 cm2] + Curved surface area of 2 hemispheres
= 160 cm2 Total surface area of article = 2 πrh + 2 (2 πr²)
12. = 2 π [ rh + 2R² ]
15 cm 22
=2× [ 3. 5 × 10 + 2 (3. 5)² ]
7
20 cm 22
=2× [ 35 + 24. 5 ]
7
21cm
22
=2× × 59.5
7
= 374 cm²
20 cm

Slant height of cone = r ² + h² 14. Height = 18

l = (20)² + (15)² Radius = R = 18 cm and r = 12 cm

l = 400 + 225 l = (R - r )² + h²
ADDITIONAL
PR ACTICE
R

MATHEMATICS - 10 243
l = (18 −12)² + 8² 1
Time = hr = 30 min = 1800 seconds
2
l = 6² + 8² We know,
l = 100 Volume of cylindrical tank
l = 10 cm = Area of cross section
Total surface area = π (R + r) l + π (R² + r²) × speed of flowing water
× time
= π [ (R + r) l + (R² + r²) ]
= π [ (18 + 12) (10) + (18² + 12²) ] p r ²h = p R² × Rate of flowing water × time

= π [ 300 + 468 ] (0.4)² h= (0. 01)² × 7 × 1800

22 0. 16h = 1. 26
= π [ 768 ] = × 768 = 2413. 71 cm²
7 1.26
h=
0.16
15. Total surface area of remaining solid
h = 7. 875 m
= C. S. A. of cylinder + Area of upper part
+ Curved surface area of cone 17. Let the radius be r and height be h.
= 2 πrh + πr² + (πrch) ATQ
Radius of cylinder (r) = 6 cm h = 6r
Height of cylinder (h) = 20 cm
Total surface area is 2pr (r + h)
Radius of cone (rc) = 6 cm
2pr (r + h) = 2pr (r + 6r)
Slant height of cone (l) = rc2 + h
= 14pr2
= 6² + 8²
60
Cost of painting = 14p r ×
2
=
100 100
l = 10 cm 60
14p r 2 × = 237.60
100
Total surface area of remaining solid
r = 3 dm
22 22  22  Volume of cylinder = pr2h
=2× × 6 × 20 + × 6² +  × 6 ×10 
7 7 7 
= p (32) (6 × 3)
22
= [ 240 + 36 + 60 ]
7 = 509 dm3
22
= × 336 18. Radius of well (r) = 5 m
7
= 1056 cm² Depth of well = 14 m
Volume of Earth taken out
16. Water is flowing at 7 m/s
22
Radius of pipe (r) = 1 cm = 0. 01 m = × 5² × 14
7
Radius of tank (R) = 40 cm = 0. 4 m = 1100 m³
244 ADDITIONAL
R

PR ACTICE MATHEMATICS - 10
As the Earth is spread around the embankment; 2
× 18³ = n × 3² × 6
Inner radius (r1) = 5 m 3
2 18³
Outer radius (r2) = (5 + 5) m = 10 m × =n
3 3² × 6
Height = h 11664
=n
162
Volume of Earth taken out = π (r2 – r1²) h2
n = 72 bottles.
= 1100
22 Section-C
= (10² – 5²) h
7 21. Let the radius of base of cylinder be 'r'
1100 × 7
=h A. T. Q.,
22 × (75)
h = 4. 67 m Volume of cylinder = Volume of two cones
1 1
6 πr²h = π r ²h + πr2²h
19. Radius of sphere (r) = = 3 cm 3 3
2
12 1 1
Radius of cylindrical vessel (R) = = 6 cm r² = r1² + r2²
2 3 3
Let water be raised by height 'h' r12 + r22
r² =
3
A. T. Q.,
r12 + r22
Volume of water raised = Volume of sphere r=
3
4
π R ²h = p r³
3 1
22. Volume of bucket = πh (R² + r² + Rr)
4 3
R²h = r³
3 1
5390 = π (15) (196 + r² + 14r)
4 3
36 × h = × (3)³
3 22
4 × 27 108 5390 × 3 = × 15 (196 + r² + 14r)
h= = 7
36 × 3 108
5390 × 3 × 7
= 1 cm = 196 + r² + 14r
22 ×15
343 = 196 + r² + 14r
36
20. Radius of hemisphere = = 18 cm r² + 14r + 196 – 343 = 0
2
Radius of cylindrical bottle (R) = 3 cm r² + 14r – 147 = 0
Height of cylindrical bottle (h) = 6 cm
r² – 7r + 21r – 147 = 0
A. T. Q.,
r (r – 7) +21 (r – 7) = 0
Volume of hemisphere = n × Volume of cylinder
r = – 21 or r = 7
2
p r ³ = n × p R ²h As Radius can't be negative, So r = 7 cm
3
ADDITIONAL
R

PR ACTICE MATHEMATICS - 10 245


23. r1 = 18 cm r² = 110 . 25
r2 = 12 cm r = 10 .5 cm
4
h = 8 cm Volume of sphere = πr³
3
Total surface area of frustum = p[(r1 + r2) l + r12 4 22
+ r22] = × × (10. 5)³
3 7
= p [(18 + 12) l + 182 + 122] 4 22
= × × 10. 5 × 10. 5 × 10. 5
3 7
l = h2 + ( r - r ) = 4851 cm³
2
1 2

A. T. Q.,
8 + (18 −12 )
2 2
=
Volume of sphere = Volume of wire (cylinder)
= 64 + 36 4851 = πr²h
= 10 cm 22
4851 = × r² × 31. 5 × 100
Total surface area = p [(30) (10) + 468] 7
[ length of wire = 31. 5 m ]
= 2413.71 cm2
4851× 7
= r²
24. Volume of water in tank = l × b × h 22 × 31.5 ×100
r² = 0. 49
21
= 50 × 44 ×
100 r = 0. 7 cm
= 462 m³
∴ Diameter of wire = 0. 7 × 2 = 1. 4 cm
For cylindrical pipe,
26. Sum of radius and height of cylinder = (r + h)
14 = 37 cm
r= = 7 cm = 0. 07 cm
2
Total surface area of cylinder = 2 πr (r + h)
Water is flowing at the rate of 15 km/hour.
22
Volume of cylindrical pipe = πr²h 1628 = 2 × × r (37)
7
22 1628 × 7
462 = × (0. 07)² × h = r
7 2 × 22 × 37
462 × 7 We know,
= h r= 7
22 × (0.07)²
r + h = 37
h = 30,000
7 + h = 37
30000
Time = = 2 hours h = 30 cm
15000
Volume of cylinder = πr²h
25. Surface area of sphere = 4 πr²
22
1386 = 4 πr² = × 7² × 30
7
1386 × 7 22

4 × 22
= r² = × 49 × 30 = 4620 cm3
7
246 ADDITIONAL
R

PR ACTICE MATHEMATICS - 10
27. For cylindrical tub, 28. Radius of hemisphere = Radius of cone
12 = r = 3. 5 cm
r= = 6 cm
2
Volume of total wood used
h = 15 cm 5 1001
= 166 = cm³
Volume of cylindrical tube = πr²h 6 6
Volume of wood used in toy
= π × 6² × 15
= Volume of hemisphere + Volume of cone
= π × 36 × 15
1001 2 1
= 540 π = πr³ + πr²h
6 3 3
For ice-cream, 1001 1
= πr² (2 r + h)
6 3
We know,
1001 22
h = 2 × diameter = 2 × 2r = 4r = × 3. 5 × 3. 5 (2 × 3. 5 + h)
6 7
1001 × 7
Radius of cone = Radius of hemisphere = R =7+h
2 × 22 × 3.5 × 3.5
Volume of ice- cream cone
13 = 7 + h
= Volume of cone + Volume of hemisphere
h = 13 – 7
1 2
= πR²h + πR³ Height of cone = h = 6 cm
3 3
1 Height of toy = Height of cone + Height of
= πR² (h + 2R)
3 hemispher
1 = 6 + 3. 5
= πR² (4R+ 2R)
3
1 = 9. 5 cm
= π 6R³
3 Curved surface area of hemisphere = 2 πr²
A. T. Q.,
22
Volume of cylindrical tub = 2 × × (3. 5)²
7
= n × Volume of ice - cream cones = 77 cm²

1 ∴ Cost of painting the hemispherical part


540 p = p 6R³ × 10
3 = 77 x 10
540 × 3 = 6R³ × 10 = ` 770.
54 × 3 = 6R³
3.5
27 = R³ 29. Radius of cone (r) = cm
2
R = 3 cm Height of cone (h) = 3 cm

∴ Diameter of ice - cream cones A. T. Q.,

= 3 × 2 = 6 cm Volume of 504 cones = Volume of sphere


ADDITIONAL
R

PR ACTICE MATHEMATICS - 10 247


1 4 22
504 × × π × r² × h = πR³ = × 2² × 14
3 3 7
504  3.5  4
2
= 176 m³
×   × 3 = R³ A. T. Q.,
3  2  3

504 × 3.5 × 3.5 × 3 × 3 Volume of well = Volume of embankment


= R³
3×2× 4 ×2 176 = π (r1² – r2²) h
1157. 625 = R³ [ r² = inner radius = 2 m ]
R = 10. 5 cm 22 40
176 = (r1² – 4) ×
Total surface area of sphere = 4 πR² 7 100
140 = r1² – 4
22
= 4 × × 10. 5 × 10. 5 144 = r1²
7
= 1386 cm² r1 = 12 m
Width of embankment = (12 – 2) m
30. Height of cone (h) = 60 cm
= 10 m
Radius of cone (r) = 30 cm
Height of cylinder (H)= 180 cm 32. Radius of the water tank = 40 cm

Radius of cylinder (R)= 60 cm Increase in water level = 3. 15 m = 315 cm

Volume water left = Volume of cylinder Volume of water flowing in the tank in half on
– Volume of cone hour = πr²h
22
1 = × 40 × 40 × 315
Volume of water left = πR²H – πr²h 7
3 = 1584000 cm³
1
= π [ (60)² (180) – Rate of water flow = 2. 52 km/hr
(30)² (60) ] 3
1
= π [ 648000 – 18000 ] Water flow in half an hour = 2. 52 ×
2
22 = 1. 26 km
= × 630000
7 = 126000 cm
= 1980000 cm³
Let internal diameter be 'd'
= 1. 98 m³
Water that flows in half an hour through pipe

Section-D d 
2

=   (126000)
4 2
31. Radius of well = =2m We know,
2
Depth = 14 m Water flowing through pipe in half an hour
Volume of well = πr²h = Volume of water flowing in half an hour

248 ADDITIONAL
PR ACTICE
R

MATHEMATICS - 10
22 d² 4
× × 126000 = 1584000 = π (4. 2)³
7 4 3
d² = 16 = 98. 8 π cm³

d= 16 Volume of scooping metal cylinder

d = 4 cm = 176. 4 π – 98. 8 π
= 77. 6 π cm³
36
33. Radius of hemisphere bowl = = 18 cm For wire,
2
1.4
2 Radius of wire = = 0. 7 cm
Volume of liquid in the bowl = πr³ 2
3
A. T. Q.,
2
= πr³ Volume of wire = Volume of scooping metal
3
cylinder
2
= (18)³ π
3 p r²h = 77 .6 p
= 3888 π cm³ 77 .6 = r ²h
Diameter of the bottle = 6 cm 77. 6 = (0. 7)²h
6
Radius of the bottle (r1) = = 3 cm 77.6
2 h=
0.49
Volume of each bottle = πr1²h
h = 158. 36 cm
= π (3)² (h)
∴ Length of the wire is 158. 36 cm
= 9 πh
A. T. Q., 35. Volume of water = Volume of cone
90 % of volume of liquid in bowl 1
= 72 × Volume of liquid in each bottle = πr²h
3
90 1 22
× 3888 p = 72 × 9 p h = × × 5² × 8
100 3 7
3499. 2 = 648 h 1 22
= × × 25 × 8
3 7
3499.2
h= = 209 .5 cm³
648
h = 5. 4 cm A. T. Q.,

34. Volume of solid metal cylinder before scooping Volume of 100 spherical balls
out = πr²h 1
= of volume of water
4
r = 4. 2 cm, h = 10 cm 4 1
100 × πR³ = × 209. 5
πr²h = π × (4. 2)² × 10 3 4

= 176. 4 π cm³ 209.5 × 3 × 7


R³= = 0. 125
4 × 22 ×100 × 4
4
Volume of scooped part = πr³ R = 3 0.125 = 0. 5 cm
3
ADDITIONAL
R

PR ACTICE MATHEMATICS - 10 249


36. A  400 100 200 
10p
=  3 + 3 + 3 
3  
h 30°
B 20 cm 10π  700 
r =  3 
G C 3  
L
7000π
= cm³
9
60°
R
F D E Volume of cylinder = πr²h

Consider ∆ABC & ∆ADE; 1 1


[r = = cm
12 24
h r l 2
= =
H R L 2
 1 
h r Volume of cylinder = π   h
=  24 
20 R
ph
In ∆ADB =
24 × 24
∠ADE = 90° & ∠DEA = 60° ph
= cm³
576
20
tan 60 = As Volume of frustum must be equal to volume
DE of cylinder.
3 DE = 20
7000 π πh
20 =
DE = 9 576
3 7000 × 576
20 h=
R= cm 9
3 h = 448000 cm
In ∆ABC
or
r
tan 30 = (Length of wire) h = 4480 m.
AB
1 r 37. Length of water flows in 40 minute
=
3 AB 20 km
= × 40 min
From (i) 60 mm

10 40
r= = km
3 3
ph 40,000 m
Volume of frustum = [ R² + r² + Rr ] =
3 3
Volume of water flow = (length of water flow
p ×10   20   10   20   10  
2 2 2 2

=   +   +      in 40 minutes) × (width of canal) × depth of


3   3   3   3   3   canal)
250 ADDITIONAL
R

PR ACTICE MATHEMATICS - 10
40,000 200,000 3 1 22
= × 5 ×1 = m = × × 24 (343)
3 3 3 7
Let the area irrigated = xm2 = 8624 cm³
10
Height of standing water = 10 cm = m
100 ② Slant height (l) = h² + (R – r )²
x ×10 200,000
∴ =
100 3 = 24² + (14 – 7)²
2000,000 2
x= m
3 = 576 + 49
Since 10,000 m2 = 1 hectare
= 625 = 25 cm
200
\ Area irrigated =
hectare
3 Area of metal sheet = C. S. A of frustum
38. h = 24 cm + Area of base
Lower end = r = 8 cm
= πl (R + r) + πr²
Upper end = R = 20 cm
= π [ 25(14 +7) + 7² ]
ph
Volume = (R² + r² + Rr)
3
22
24π = [ 25 (21) +49 ]
= [ (20)² + (8)² + (20) (8) ] 7
3
= 8 π [ 400 + 64 + 160 ]
22
= [ 525 +49 ]
22 7
= 8 π [ 624 ] = 8 × × 624
7
= 15689. 142 cm² 22
= [ 574 ]
7
= 15. 689142 l

Cost of milk per liter = ` 27 = 22 × 82

Total cost = 15. 689142 × 27 = 1804 cm²


= ` 423. 606
40. A
39. ① Volume of bucket = Volume of frustum
1
= πh (R² + r² + Rr)
3
H cm
D E
1 22
= × × 24 (14² + 7² + 14 × 7)
3 7
1 22
= × × 24 (196 + 49 + 98) B C
3 7 10 cm
ADDITIONAL
R

PR ACTICE MATHEMATICS - 10 251


h CASE STUDY-1
Given that AD = DB = cm
2
(i) (c) Area of hemisphere = 2πr2
Consider ∆ADE and ∆ABC
= 2π (2.5 cm)2
∠ADE = ∠ABC = 90°
= 12.5π cm2
∠BAC = ∠DAE = (Common) (ii) (b) Area of 1 hemispherical dome

By Angle - Angle similarity [ ∆ADE ~ ∆ABC ] = 2πr2

= 2π (1 cm)2
AD DE r
So, = = (r is radius of cone) = 2π cm2
DB BC 10
Area of 7 hemispherical dome = 14π cm2
H r
= (iii) (a) Curved surface area of 1 cylinder = 2πrh
2 10
H
Curved surface area of 2 cylinder = 4πrh
1 r = 4π (1.4 cm) (7 cm)
= =
2 10
= 123.15 cm2
r = 5 cm
≈ 123.2 cm2
⇒ Volume of frustum = Volume of cone ABC – (iv) (d) Inner surface area of big dome = 12.5π cm2
Volume of cone ADE
Inner surface area of 1 small dome = 2π cm2
2
1 1 R H
= πR²H – π   Area of big dome 12.5π
3 3 4 2 =
Area of small dome 2π
 R 10 
Here, = = 5 cm 25
2 2  =
4
Volume of frustum of cone
(v) (d) Curved surface area of cylinder is 2πrh.
1 1 H 175 π H
= π (100) × H – π (25) = CASE STUDY-2
3 3 2 6

Volume of cone (i) (a) Length of water in canal is 30 min = 5 km


= 5000 m
1 H 25 p H
ADE = × π × 5² × = Volume of water flowing = (1.5 m) (6 m)
3 2 6
(5000 m)
= 45000 m3
25 p H 1
6
Ratio of Volume = = (ii) (b) The volume of tank is given by πr2h, where
H 7
6 "r" is the radius and "h" is height of cylinder.
252 ADDITIONAL
R

PR ACTICE MATHEMATICS - 10
Volume of water collected = Volume of tank (iv) (d) The volume of water collected is equal to
2 the volume of the frustum
10 
= p   (2 ) p
2 Volume of frustum =
3
(r 1
2
)
+ r2 2 + r1 r2 h
= 50p m3
p
(iii) (b) Let the area irrigated is 'x' m =
3
(14 2
)
+ 7 2 +19 × 7 15

Width of canal = 6 m
= 5390 cm3
Depth of canal = 1.5 m

h2 + ( r1 r2 )
2
Length of water in canal in 30 minutes (v) (a) l =
= 5 km = 5000 m
= h2 + 4 2
Area irrigated by 8 cm = 6 × 1.5 × 5000
25 = h2 + 16
8
6 × 1.5 × 5000 = .x h2 = 9
100
x = 562500 m2 h = 3 cm

ADDITIONAL
R

PR ACTICE MATHEMATICS - 10 253


Chapter

14 Statistics
Multiple choice Questions

1. (c) Mode is the most frequent value. But median is 25

 xi ∴ 25 = 2x + 1
2. (c) X =
n 24 = 2x
1 + 2 + 3 +… + n
15 = x = 24
n
n(n +1)
15n = 3. h is the class size
2
2 x 15=n + 1
 fx
n =29 4. Mean = Âf
3. (b) Median 4 p + 63
7 =
4. (a) Median 17
4 p + 63
5. (c) Mode = 3 median – 2 mean ⇒ 119 = 4p + 63
17
⇒ 119 – 63 = 4p
WoRKSHeet - 1
⇒ 56 = 4p
Section-A ⇒ 14 = p

1. Mode of the data is 8 5. We know that

if x = 8 because then 8 has highest frequency i.e 4. Mode = 3 median – 2 mean


15 = 3 median – 2 (30)
2. Here, number of terms is 10 i.e. even
th th 75
Ênˆ Ên ˆ = median
Á 2 ˜ obs + Á 2 +1˜ obs 3
Ë ¯ Ë ¯ 25 = median
So, Median =
2
5 obs + 6 obs
th th

= 6. Mode is 2 because it has the highest frequency.


2
2 x - 8 + 2 x +10
= 7. First five odd natural number 5 are 1, 3, 5, 7, 11
2
4x +2 1 + 3 + 5 + 7 +11
= So, mean =
2 5
= 2x + 1 = 5.4
254 ADDITIONAL
R

PR ACTICE MATHEMATICS - 10
 fx 10. We know that
8. Mean =
Âf mode = 3 median – 2 mean
3p + 36
3 = 21.4 = 3 (21.2) – 2 mean
15
45 = 3p + 36 2 mean = 63.6 – 21.4
45 – 36 = 3p = 42.2
9 = 3p ∴ mean = 21.1
p = 3
11. Marks Number of CF
Section-B students
0 – 10 5 5
9. Class Frequency
10 – 30 15 20
3–6 2
30 – 60 30 50
6–9 5
60 – 80 8 58
9 – 12 10 f0
80 – 100 2 60
12 – 15 23 f1
60
15 – 18 21 f2
18 – 21 12 n 60
= = 30
2 2
21 – 24 3
So, median class = 30 – 60
Here, Modal class is 12 – 15
∴ l = 30
Ê f1 - f 0 ˆ
Mode = l + Á ˜h f = 30
Ë 2f1 - f 0 - f 2 ¯
cf = 20
l = 12
f0 = 10 h = 30

f1 = 23 Ên ˆ
Á 2 - CF ˜
f2 = 21 Median = l + Á ˜h
ÁÁ f ˜˜
h =3 Ë ¯
Ê 23 - 10 ˆ
So, Mode = 12 + Á ˜3 Ê 30 - 20 ˆ
Ë 46 - 10 - 21¯ = 30 + Á ˜
Ë 30 ¯
Ê 13 ˆ
= 12 + Á ˜ 3
Ë 15 ¯ 10
= 30 + (30)
13 30
= 12 +
5
= 30 + 10
= 12 + 2.6
= 14.6 = 40

ADDITIONAL
R

PR ACTICE MATHEMATICS - 10 255


12. 15.
Classes Frequency (fi) xi Fixi Number of Number of fn
0 – 10 3 5 15 branches (n) plants (f)
10 – 20 5 15 75 2 49 98

20 – 30 9 25 225 3 43 129

30 – 40 5 35 175 4 57 228

40 – 50 3 45 135 5 38 190
25 625 6 13 78
 fi xi 200 723
Mean =
 fi Average Number of branches per plant
625
= Â fn
25 =
Âf
= 25
723
13. =
200
Class Frequency Cumulative
interval frequency = 3.615

0 –10 5 5 16.
10 – 20 3 8 Weekly Number x di = fidi
wages of xi – A
20 – 30 10 18 workers
30 – 40 6 24 (f)
40 – 50 4 28 40 – 43 31 41.5 –6 –186
50 – 60 2 30 43 – 46 58 44.5 –3 –174

14. 46 – 49 60 47.5 0 0
xi fi fixi 49 – 52 m 50.5 3 3m
10 5 50 52 – 55 27 53.5 6 162
176 + m –198 + 3m
15 10 150
p 7 7p  f i di
25 8 200 Mean = A +
 fi
30 2 60 - 198 + 3m
32 460 + 7p 47.7 = 47.5 +
176 + m
 fi xi - 198 + 3m
Mean = – 0.3 =
 fi 176 + m
460 + 7 p ⇒ – 52.8 – 0.3 m = – 198 + 3m
18.75 =
32
600 – 460 = 7p 145.2
⇒ 145.2 =
140 = 7p 3.3
20 = 7p = 44
256 ADDITIONAL
R

PR ACTICE MATHEMATICS - 10
Section-C 18.
Mark Number of x1 f1 u1 f1u1
17. Students
Class xi fi ui = fi ui (f1)
intervals x i - 50 1 – 10 20 5 – 20 – 2 – 40
20 10 – 20 24 15 – 10 – 1 – 24
0 – 20 10 17 –2 – 34
20 – 30 40 25 0 0 0
20 – 40 30 fi –1 – fi
30 – 40 36 35 10 1 36
40 – 60 50 32 0 0
40 – 50 20 45 20 2 40
60 – 80 70 f2 1 f2     140 12
80 – 100 90 19 2 38
4 fi + f2  ∑f u 
Mean = a +  i i h
 ∑ fi 
∑fi = 120
10
⇒ 17 + fi + 32 + f2 + 19 = 120  12 
= 25 + 
140 
⇒ 68 + fi + f2 = 120
12
⇒ fi + f2 = 52 ...(i) = 25 +
14
Also, mean = 50
6
Ê Âf u ˆ = 25 +
⇒ a + Á i i ˜h = 50 7
Ë Â fi ¯
181
=
Ê 4 - fi + f2 ˆ 7
⇒ 50 + 20 Á ˜ = 50
Ë 120 ¯ = 25. 86

1 19.
⇒ (4 – f1 + f2 ) =0 Class cf fi xi fi xi
6
Intervals
⇒ f1 – f2 =4 ...(ii) 20 – 30 100 100 25 2500

Solving eq. (i) and (ii), we get 30 – 40 220 120 35 4200


40 – 50 350 130 45 5850
f i + f 2 = 52
50 – 60 750 400 55 22000
fi - f2 = 4
60 – 70 950 200 65 13000
2f i = 56
70 – 80 1000 50 75 3750
f i = 28
1000 51300
From (i), f2 = 52 – 28
∑f u 
Mean =  i i 
= 24  ∑ fi 

ADDITIONAL
R

PR ACTICE MATHEMATICS - 10 257


51300 ⇒ 12 + 30 + f i + 65 + f 2 + 25 + 18 = 229
=
1000 ⇒ 150 + f i + f 2 = 229

= 51. 3 ⇒ fi + f2 = 79
⇒ 33.5 + f 2 = 79
20.
⇒ f 2 = 79 – 33.5
Variable Frequency cf
10 – 20 12 12 = 45.5

20 – 30 30 42
fi 21.
30 – 40 42 + fi
107 + fi Class Frequency cf
40 – 50 65 Interval
50 – 60 52 107 + fi + f2 0–6 4 4
60 – 70 25 132 + fi + f2 6 – 12 x 4+x
70 – 80 18 150 + fi + f2 12 – 18 5 9+x
229 18 – 24 y 9+x+y
Median is 46 which lies in interval 40 – 50 24 – 30 1 10 + x + y
10 + x + y
∴ Median class 40 – 50
l = 40 As Median is 14.4 which lies in interval 12 – 18,
So, Median class is 12 – 18
f = 65
l = 12
cf = 42 + fi
f = 5
h = 10
cf = 4+ x
Median = 46
h = 6
n 
−cf Now, ∑fi = 20
2 
l+   h = 46
f ⇒ 10 + x + y = 20
 
 
⇒ x + y = 10 (i)
 229  Also, ∑fi = 14..4
− 42 − f1
 2 
40 +   10 = 46 n 
65 −cf
  2 
 
l +   h = 14.4
f
 77.5 − f1   
 
 65  10 = 6
10 - 4 - x
72.5 – fi = 39 ⇒ 12 + 6 = 14.4
Ł 5 ł
fi = 33.5
6
Also, ∑fi = 229 ⇒ (6 - x) = 2.4
5
258 ADDITIONAL
R

PR ACTICE MATHEMATICS - 10
2.4 x 5
⇒ 6–x = = 2 Y
6
50
⇒ x =6 –2=4 45

Cumulative Frequency
40
Form (i), y = 10 – x
35
= 10 – 4 30
25
= 6 20
15
10
22. 5
Age Number c f 0 100 120 140 160 180 200
(in year) of Daily Income X
persons
– 0.5 – 9.5 5 5 less than 9.5
9.5 – 19.5 15 20 less than 19.5 24.
19.5 – 29.5 20 40 less than 29.5 Monthly Number Cumulative
29.5 – 39.5 23 63 less than 39.5 Consumption of Frequency
of electricity Consumer (cf)
39.5 – 49.5 17 80 less than 49.5
65 – 85 4 4
49.5 – 59.5 11 91 less than 59.5
85 – 105 5 9
59.5 – 69.5 9 100 less than 69.5
105 – 125 13 22
125 – 145 20 42
145 – 165 14 56
165 – 185 8 63
185 – 205 4 67

Median
n 68
Here = = 34
2 2
23. Median class is 125 – 145
Daily Number Cumulative l = 125
Income of workers frequency cf = 22
less than 120 12 12 f = 20
less than 140 14 26 h = 20
less than 160 8 34
n 
less than 180 6 40 Median = l +  2 −cf  h
 f 
less than 200 10 50  
 
ADDITIONAL
R

PR ACTICE MATHEMATICS - 10 259


 34 − 22  Modal class is 150 – 160 as this class has
= 125 +  20 highest frequency
 20 
l = 150
= 125 + 12
f0 = 74
= 137 f1 = 163
Mode f2 = 135
Modal class is 125 – 145 as it has highest h = 10
frequency.  f −f 
l = 125 Mode = l +  1 0  h
 2f1 − f 0 − f 2 
h = 20
f0 = 13  163 − 74 
= 150 +  
f1 = 20  326 − 74 −135 
f2 = 14
 89 
= 150 + 
 f1 − f 0  117 
Mode = l+  h
 2f1 − f 0 − f 2 
17550 + 890
= = 157.61
 20 −13  117
= 125 +  20
 40 −13 −14 
26.
7 Rainfall Rainfall
= 125 +   20
13  (in cm) cf (in cm) cf
(less than) (more than)
140
= 125 + less than 10 22 more than 0 66
13
less than 20 32 more than 10 44
= 135.8
less than 30 40 more than 20 34
25. less than 40 55 more than 30 26
Height Number of less than 50 60 more than 40 11
x1 f1 x1
(in cm) student (f1) less than 60 66 more than 50 6
140 – 150 74 145 10730
Y
150 – 160 163 155 25265 70
65
160 – 170 135 165 22275 60
55
170 – 180 28 175 4900 50
45
400 63170
Cumulative Frequency

40
35
30
f1x1 25
Mean = 20
f1 15
10
63170 5
= = 157.925 0
10 20 30 40 50 60 x
400 Rainfall (in cm)

260 ADDITIONAL
R

PR ACTICE MATHEMATICS - 10
27.  12 − 8 
= 40 + 
Amount Number of Cumulative  24 − 8 −11
students frequency
(f1) 4
0 – 20 5 = 40 +  
5 5
20 – 40 8 13 = 40 + 16
40 – 60 12 25 = 56
60 – 80 11 36
80 – 100 4 40 28.
Number Number x1 d1 f1d1
40
of of
n
=
40
= 20 Mangoes boxes f1
2 2
50 – 52 15 51 –6 – 90
Median Class is 40 – 60
53 – 55 110 54 –3 – 330
l = 40
56 – 58 135 57 0 0
cf = 13
59 – 61 115 60 3 345
f = 12
62 – 64 25 63 6 150
h = 20
400 75
n 
 −cf  Mean = a + ∑ f1d1
Median = l+ 2  h ∑ f1
 f 
  75
= 57 +
 20 −13  400
= 40 +  20
 12 
= 57.19
7 We use assumed mean method as values of
= 40 + (20) x1 and f1 are large.
12

= 51.7
29.
Modal Class is 40 – 60 Class Frequency x1 f1
l = 40 Interval (f1)

f0 =8 0 – 20 17 10 170
20 – 40 f1 30 30 f1
f1 = 12
40 – 60 f2 = 4x 50 200x
f2 = 11
60 – 80 f3 = 3x 70 210x
h = 20 80 – 100 19 90 1710
 f1 − f 0  120 1880 + 30 f1
Mode =  h + 410x
 2f1 − f 0 − f 2 
ADDITIONAL
R

PR ACTICE MATHEMATICS - 10 261


f2 = f3 = 4 :3  f1 − f 0 
Mode = l+  h
Let f2 = 4x and f3 = 3x  2f1 − f 0 − f 2 
∑f1 = 120
⇒ 17 + f1 + f2 + f3 + 19 = 120  12 − f1 
65 = 60 +  20
 24 − f1 − 6 
⇒ 17 + f1 + 4x + 3x + 19 = 120
⇒ f1 + 7x = 84 (i)
12 − f1 
65 = 60 +  20
Also, mean = 50 18 − f1 
∑ f1x1
= 50
∑ f1 5 12 - f1
=
20 18 - f1
1880 + 30f1 + 410 x
= 50
120
1 12 − f1
1880 + 30f1 + 410x = 6000 =
4 18 − f1
30f1 + 410x = 4120
18 – f1 = 48 – 4f1
⇒ 15f1 + 205x = 2060 (ii)
Solving (i) and (ii), we get 3 f1 = 30
f1 = 28 f1 = 10
x = 8
∴ f2 = 4x = 32
f3 = 3x = 24 WORKSHEET - 2

30. SECTION-A
Class Frequency (f1) 1. Mean = 10. 5
0 – 20 6 Median = 9. 6
20 – 40 8
We know that
40 – 60 f1
3 Median = mode + 2 mean
60 – 80 12
80 – 100 6 3 (9.6) = mode + 2 (10. 5)
100 – 120 5 28.8 = mode + 21
Modal Class is 60 – 80 as this class has the Mode = 28. 8 – 21
highest frequency
l = 60 = 7. 8
f0 = f 1
∑f 
f1 = 12 2. Mean = a +  1  h
 ∑f 
f2 = 6

262 ADDITIONAL
R

PR ACTICE MATHEMATICS - 10
 −3  ∑ x1
= 55 +  10 6. Mean =
100  N

-3 ∑ x1
= 55 – 18 =
10 50

= 54. 7 ∑x1 = 900


3. New ∑x1 = 900 + 200
Marks Number Marks Number = 1100
Obtained of Obtained of
1100
student (Class student So, New mean =
50
(cf) intervals) (f)
Less than 20 8 10 – 20 8 = 22
Less than 30 13 20 – 30 5
Less than 40 19 30 – 40 6 7. Median is equal to the x – coordinate of point
of intersection of less than ogive and more than
Less than 50 24 40 – 50 5 ogive
Here, point of intersection is (18, 20)
4.
So, Median = 18
Classes frequency (f) Cf
0 – 10 4 4 8. We get less than ogive when upper limits are
10 – 20 4 8 taken along x-axis and cumulative frequency
20 – 30 8 16 along y-axis.
30 – 40 10 26
∑ x1
40 – 50 12 38 9. Mean =
n
50 – 60 8 46
60 – 70 4 50
∑ x1
50 40 =
5
n 50
= = 25 ∑x1 = 200
2 2
If 88 is excluded,
Cf of class 30 – 40 is greatest than 25
New ∑x1 = 200 – 88
So, Median class is 30 – 40
= 112

5. Modal class is 40 – 50 as this class has highest New n =5–1=4


frequency.
112
So, lower limit is 40 ∴ New Mean = = 28
4

ADDITIONAL
R

PR ACTICE MATHEMATICS - 10 263


10(12) + 20(9) 3620
10. Mean of 30 number = =
30 140
120 + 180 = 25. 86
=
30
300 13.
=
30 Frequency
Age in
= 10 x1 f1x1
years (f1 )
Section-B 20 – 30 4 25 100
30 – 40 5 35 175
11.
40 – 50 8 45 360
Class Interval Frequency
50 – 60 3 55 165
100 – 110 6
60 – 70 6 65 390
110 – 120 35
26 1190
120 – 130 72
130 – 140 48 ∑ f1x1
Mean =
∑ f1
Modal class is 120 – 130
 f −f 
Mode = l +  1 0  h = 1190
 2f1 − f 0 − f 2  26

= 45. 77
 72 − 35 
= 120 +  10 14.
144 − 35 − 48 
x1 f fx
5 6 30
 37 
= 120 +   10 15 4 60
 61 
25 3 75
= 126. 07 35 k 35k
45 2 90
12. 15 + k 255 + 35k
Marks f1 x1 f1x1
0 – 10 20 5 100 Mean = ∑ fx
∑f
10 – 20 24 15 360
20 – 30 40 25 1000 255 + 35k
30 – 40 36 35 1260 = 15 + k
40 – 50 20 45 900 322.5 + 21.5k = 225 + 35k
140 3620 67.5 = 13.5k
∑ f1x1 67.5
mean = k = =5
∑ f1 13.5
264 ADDITIONAL
R

PR ACTICE MATHEMATICS - 10
15. Number of observations = 10 (even) Mean = 20

n n 
th th
fx
= 20
So, Median =  2  obs +  2 +1 obs f
2
295 + 5p (20 + p)
th 2 = 20
5 obs + 6 obs 15 + 5p
=
2
x+2 + x+4 59 + p (20 + p)
= ⇒ = 20
2 3+ p

= x+3 ⇒ 59 + 20p + p2 = 60 + 20p


Median = 24 ⇒ p2 = 1
∴ x+3 = 24 ⇒ p = 1
x = 24 – 3
= 21 18. x1 = 5 + 7 = 12
16. Number of observations = 10 (even) x1 + x2 = 18

n
th
n 
th ⇒ 12 + x2 = 18
So, Median =  2  obs +  2 +1 obs x2 = 18 – 12
2 = 6
th th
5 obs + 6 obs 18 + 5 = x3
=
2 23 = x3
48 + 35
= x3 + x4 = 30
2
85 23 + x4 = 30
=
2 x4 = 30 – 23
= 42.5 = 7
If 25 is replaced by 52 and 19 by 29, median 19.
remains same.
Class Frequency ( f1) Cf
Median will get affected by 5th and 6th
observation only. 40 – 50 5 5
50 – 60 x 5+x
17.
60 – 70 15 20 + x
x1 f fx
70 – 80 12 32 + x
15 2 30
80 – 90 7 39 + x
17 3 51
As mode is 67 which is in class 60 – 70,
19 4 76
20 + p 5p 5p (20 + p) So, Modal class is 60 – 70

23 6 138 l = 60
15 + 5p 295 + 5p (20 + p) f0 =x
ADDITIONAL
R

PR ACTICE MATHEMATICS - 10 265


f1 = 15 ⇒ Total score of boys = 71x
f2 =12 Average score of girls = 73
 f −f  ⇒ Total score of girls = 73y
Mode = l+  1 0  h
 2f1 − f 0 − f 2  Average score of school = 71.8
Total score of school = 71.8 (x + y)
 15 − x 
67 = 60 +   10 So,
 30 − x −12 
71x + 73y = 71.8 (x + y)
15 − x  71x + 73y = 71.8x + 71.8y
7 = 10 18 − x 
1.2y = 0.8x
7 (18 – x) = 150 – 10x 3y = 2x
126 – 7x = 150 – 10x x 3
∴ =
3x = 24 y 2

x = 8 22.
20. Class Frequency Cumulative
Life time Frequency x1 f1x1 f1 Frequency
(in hours) f1 Cf
0 – 20 15 10 150 Less than 20 13 13
20 – 40 10 30 300 Less than 25 18 31
40 – 60 35 50 1750 Less than 30 31 62
60 – 80 50 70 3500 Less than 35 25 87
80 – 100 40 90 3600 Less than 40 15 102
150 9300 Less than 45 5 107
∑ f1x1
Mean = ∑ f1 Y
110

100
9300
= 90
150 80

70
= 62
Cumulative Frequency

60

50
Section-C 40

21. Let number of boys be x and number of girls 30

be y. 20

10
∴ Total number of students = x + y
0 15 20 25 30 35 40 45

Average score of boys = 71 Class X

266 ADDITIONAL
PR ACTICE
R

MATHEMATICS - 10
23. Median is 35 which lies in class 30 – 40.
Age in year Number of Cf
patients (f1) l = 30

0–8 10 10 f = 10
8 – 16 12 22
cf = 6
16 – 24 8 30
24 – 32 25 55 h = 5+x
32 – 40 15 70
Median = 35
40 – 48 11 81
48 – 56 21 102 n 
−cf
56 – 64 30 132 2 
l+ 
f  h = 35
64 – 72 22 154  
 
154
n 154  21 + x − 5 − x 
= = 77 30 +   10 = 35
2 2  6
Median class is 40 – 48
l = 40
f = 11 10 +  21 + x −10 − 2 x  =5
cf = 70 6  6 
h = 8
n  11 – x = 6
−cf
2 
Median = l+   h
f x=5
 
 
 77 − 70  25.
= 40 +  8
 11  Y 70

64
56
= 40 + = 45.1 56
11
48
24.
Number of Teacher

40
Class Frequency Cf
interval f1 32

24
0 – 10 2 2
16
10 – 20 3 5
8
20 – 30 x 5+x
0 45 50 55 60 65 70 75 80 85
30 – 40 6 11 + x Weight (in kg) x
40 – 50 5 16 + x
50 – 60 3 19 + x
60 – 70 2 21 + x
21 + x

ADDITIONAL
R

PR ACTICE MATHEMATICS - 10 267


26. 27.
Literacy Number x1 f1x1 Class Frequency x1 f1x1
rates of cities f1
(f1) 10 – 30 5 20 100
35 – 40 1 37.5 37.5
30 – 50 8 40 320
40 – 45 2 42.5 85
50 – 70 f1 60 60 f1
45 – 50 3 47.5 142.5
50 – 55 x 52.2 52.5x 70 – 90 20 80 1600

55 – 60 y 57.5 57.5y 90 – 110 f2 100 100 f2


60 – 65 6 62.5 375 110 – 130 2 120 240
65 – 70 8 67.5 540 50 2260
70 – 75 4 72.5 290 + 60 f1
75 – 80 2 77.5 155 + 100 f2
80 – 85 3 82.5 247.5
85 – 90 2 87.5 175 ∑f1 = 50
40 2047.5 ⇒ 35 + f1 + f2 = 50
+ 52.5x
⇒ f1 + f2 = 15 ...(i)
+ 57.5y
∑f1 = 40 Mean = 65.6

∴ 31 + x + y = 40 ∑ f1x1
⇒ = 65.5
x+y =9 (i) ∑ f1

Mean = 63.5
2260 + 60 f1 + 100 f 2
∑ f1x1 ⇒
= 63.5 50
∑ f1
= 65.5
2047.5 + 52.5 x + 57.5y
= 63.5 ⇒ 60 f1 + 100 f2 = 1020
40

52.5x + 57.5y = 493.5 ⇒ 15 f1 + 25 f2 = 255

⇒  525x + 575y = 4925 ⇒ 3 f1 + 5 f2 = 51 ...(ii)


⇒ 105x + 115y = 985 On solving (i) and (ii), we get
⇒ 21x + 23y = 197 (ii)
f1 = 12
Solving (i) and (ii), we get
f2 = 3
x =5
y =4

268 ADDITIONAL
R

PR ACTICE MATHEMATICS - 10
Section-D 29.
Marks Number of
28. Cf
obtained students
Weight Number of 25 – 35 7 7
(in kg) Students 35 – 45 31 38
38 – 40 3 45 – 55 33 71
40 – 42 2 55 – 65 17 88
42 – 44 4 65 – 75 11 99
44 – 46 5 75 – 80 1 100
46 – 48 14 100
48 – 50 4
Mode
50 – 52 3
Modal class is 45 – 55 as this class has the
highest frequency
Weight Weight l = 45
Cf Cf
(in kg) (in kg) f0 = 31
lass than 40 3 more than 38 35 f1 = 33
lass than 42 5 more than 40 32 f2 =17
lass than 44 9 more than 42 30
h = 5
lass than 46 14 more than 44 26
 f1 − f 0 
lass than 48 28 more than 46 21 Mode =  2f − f − f  h
1 0 2
lass than 50 32 more than 48 7
lass than 52 35 more than 50 3  33 − 31 
= 45 +  66 − 31 −17  5

Y 2
35 = 45+   5
18
30

25 5
= 45 +
Cumulative Frequency

20 9
15
= 45.6
10

5
Median
46.5
0 38 40 42 44 46 48 50 52 n 100
= = 50
Weight (in kg) X
2 2
∴ Median = 46.5cs
Median class is 45 – 55

ADDITIONAL
PR ACTICE
R

MATHEMATICS - 10 269
n  31.
Median = l +  2 −cf  h
 f  Height Number Class f
 
  (in cm) of girls Intervals
 50 − 38  (Cf)
= 45 +  5
 33 
less than 140 4 135 – 140 4
12  less than 145 11 140 – 145 7
= 45 +  33  5
less than 150 29 145 – 150 18
= 46.82
less than 155 40 150 – 155 11
30. Mean = x less than 160 46 155 – 160 6
x1 less than 165 51 160 – 165 5
(a) ⇒ =x
n
n 51
Here = = 25.5
⇒ ∑x1 =nx 2 2
New ∑x1 Median class is 145 – 150
= (x1 + 1) + (x2 + 1) + ____+(xn+ n) Median = l +  n −cf  h
2 
= ∑x1 + (1 + 2 + 3 + ....+ n)  f 
 
 
n (n + 1)
= ∑x1 +
2
= 145 +  25.5 −11 5
 18 
n (n + 1)
= nx +
2 = 145 + 4. 03
n (n + 1)
nx + = 149. 03
∴ New mean = 2
2
CASE STUDY-1
 n +1
= x +  2  (i) (b) Maximum number of students have marks
in the range 35–45. Hence the modal class
(b) Mode = 7. 88 is 35–45.
Mean = 8. 32 (ii) (b) Lower limit is 25.
(iii) (d) Maximum number of students is 24, hence
We know that
it is the maximum frequency.
3 Median = Mode + 2 Mean 45 + 55
(iv) (c) Class mark =
⇒ 3 Median = 7. 88 + 2 (8. 32) 2
= 7. 88 + 16. 64 = 50
⇒ Median = 8. 17 (v) (a) Class size of each class is 10.

270 ADDITIONAL
PR ACTICE
R

MATHEMATICS - 10
CASE STUDY-2   50 
  2 − 24  
(i) (a) No. of No. of Cumulative Median = 60 +   × 20
 12 
apples boxes frequency  
 
(frequency)
0–20 6 6 1
= 60 + × 20 5
12 3
20–40 8 14
185
40–60 10 24 = = 61.6
3
60–80 12 36
(iii) (b) The maximum frequency is 12, which lies
80–100 6 42 in the class internal 60–80. Hence the
modal class is 60–80.
100–120 5 47
 f −f 
120–140 3 (iv) (d) Mode = l +  1 0  × h
50
 2f1 − f 0 − f 2 
N = 50 Where l = lower limit of modal class
N f1 = frequency of modal class
= 25
2 f2 = frequency of class following
Cumulative frequency more than or equal modal class
to 25 is 36 which belongs to class interval f0 = frequency of class precedding
60–80. Thus Median class is 60–80. modal class
h = width of modal class
N 
  2 − cf    12 −10 
(ii) (b) Median = l +  ×h Mode = 60 +   × 20
 f   2 (12 ) −10 − 6 
 
 
 2 × 20 
l = lower limit of median class = 60 +  
 8 
N
= half of sum of cumulative frequencies = 65
2
3 Median - Mode
Cf = Cumulative frequency of class (v) (a) Mean =
preceding the Median class 2

f = frequency of Median class


3 (61.6 ) − 65
h = class height =
2
l = 60, N = 50, Cf = 24, f = 12, h = 20 = 59.9

ADDITIONAL
R

PR ACTICE MATHEMATICS - 10 271


Chapter

15 Probability
Multiple choice Questions
P (Getting even number on both dices)
1. (c) When two dices are thrown together, total
number of outcomes are: Number of favourable outcomes
=
Total number of outcomes
(1, 1), (1, 2), (1, 3), (1, 4), (1, 5), (1, 6)
Number of favourable outcomes
(2, 1), (2, 2), (2, 3), (2, 4), (2, 5), (2, 6)
= (2, 2), (2, 4), (2, 6), (4, 2)
(3, 1), (3, 2), (3, 3), (3, 4), (3, 5), (3, 6)
(4, 1), (4, 2), (4, 3), (4, 4), (4, 5), (4, 6) (4, 6), (6, 2), (6, 4), (4, 6)
(5, 1), (5, 2), (5, 3), (5, 4), (5, 5), (5, 6) P (Getting even number on both dices)
(6, 1), (6, 2), (6, 3), (6, 4), (6, 5), (6, 6) 9 1
= =
P (Getting the same number) 36 4
Number of favourable outcomes
= 4. (c) Number from 1 to 15 that are multiple of
Total number of outcomes
4 = 4, 8 , 12
6 1 1
= = 3
36 6 P (Multiple of 4) = =
15 5

2. (d) Total Number of cards = 52 5. (c) Prime number from 1 to 30


Cards that are not ace = 48
= 2, 3, 5, 7, 11, 13, 17, 19, 23, 29
P (card not an arc)
P (Prime number between 1 and 30)
Number of favourable outcomes
= 10 1
Total number of outcomes = =
30 3
48 12
= =
52 13
WoRKSHeet - 1
3. (d) When two dices are rolled together, total
number of outcomes are:
Section-A
(1, 1), (1, 2), (1, 3), (1, 4), (1, 5), (1, 6) 1. If an event cannot occur, then its probability
(2, 1), (2, 2), (2, 3), (2, 4), (2, 5), (2, 6) is 0.
(3, 1), (3, 2), (3, 3), (3, 4), (3, 5), (3, 6) 2. Total number of face cards = 12 cards
(4, 1), (4, 2), (4, 3), (4, 4), (4, 5), (4, 6) Total number of red face cards = 6 cards
(5, 1), (5, 2), (5, 3), (5, 4), (5, 5), (5, 6) 6 3
P (red face cards) = =
(6, 1), (6, 2), (6, 3), (6, 4), (6, 5), (6, 6) 52 26
272 ADDITIONAL
R

PR ACTICE MATHEMATICS - 10
3. Total number of outcomes when a die is Section-B
thrown = 1, 2, 3, 4, 5, 6
9. When two coins are tossed together, total
Odd number less than 3 = 1
number of outcomes are (H, H), (H, T), (T,
1 H) (T, T)
P (odd number less than 3) =
6 Outcomes for at least 1 head and 1 tail
4. If three coins are tossed simultaneously, total
number of outcomes are (HHH), (HHT), = (H, T), (T, H)
(HTH), (THH), (HTT), (THT), (TTT), (TTH) 2 1
P (at least 1 head and 1 tail) = =
Outcomes for at least two heads 4 2
= (HHH), (HHT), (HTH), (THH)
10. Tickets are numbered from 1 to 20
4 1
P (at least two heads) = = Multiples of 2 between 1 and 20
8 2
= 2, 4, 6, 8, 10, 12, 14, 16, 18, 20
5. A non-leap year has 365 days
Multiples of 7 between 1 and 20 = 7, 14
For 364 days, there are 52 weeks i.e. 52
Sundays Outcomes which are multiple of 2 or 7
For the remaining 1 day, only one Sunday = 2, 4, 6, 7, 8, 10, 12, 14, 16, 18, 20
can exist.
11
So, P (multiples of 2 or 7) =
20
P (Getting 53 Sundays in non - leap year) =
1
11. Number of red marbles = 3
7
Number of blue marbles = 2
6. Number of aces in a deck of cards = 4
Total number of marbles = 5
4 1
P (ace) = = 2
52 13 P (blue marble) =
5
12 a) When a die is thrown, total number of
7. Given number = 3, 5, 5, 7, 7, 9, 9, 9, 9
outcomes are 6 namely 1, 2, 3, 4, 5 and 6
Average of the given number
Outcomes which are multiple of 3 = 3, 6
3+ 5+ 5+ 7 + 7 + 9+ 9+ 9+ 9
= 2 1
9 P (multiple of 3) = =
6 3
63
= =7 b) Outcomes which are even number or a
9 multiple of 3 = 2, 3, 4, 6
So, 7 comes two times in these numbers
P (even number or multiple of 3)
2
Thus, P (selecting their average) = 4 2
9 = =
8. In a single throw of dice, total number of 6 3
outcomes are 6 namely 1, 2, 3, 4, 5, and 6
13. Total number of children = 3
Perfect squares = 1, 4
2 1 0
P (Getting perfect square) = = P (number girl) = =0
6 3 3
ADDITIONAL
PR ACTICE
R

MATHEMATICS - 10 273
1 18. (1, 1), (1, 2), (1, 3), (1, 4), (1, 5), (1, 6)
P (one girl) = (2, 1), (2, 2), (2, 3), (2, 4), (2, 5), (2, 6)
3
2 (3, 1), (3, 2), (3, 3), (3, 4), (3, 5), (3, 6)
P (two girls) =
3 (4, 1), (4, 2), (4, 3), (4, 4), (4, 5), (4, 6)
3 (5, 1), (5, 2), (5, 3), (5, 4), (5, 5), (5, 6)
P (three girls) = =1
3
(6, 1), (6, 2), (6, 3), (6, 4), (6, 5), (6, 6)
1
So, the probability of each cannot be Sum of number greater than 10 = (5, 6),
4
(6, 5), (6, 6)
∴ the given statement is incorrect.
3 1
P (sum greater than 10) = =
26 12
14. No, the given statement is false and we do
want a higher chance of getting tail in the 19. A leap year has 366 days in which there are
4 th because every coin toss has an equal 52 weeks and 2 days
probability of getting head and tail which is
1 These 2 days can be filled as :
2 { Monday, Tuesday }
∴ There are equal chances of getting head { Tuesday, Wednesday }
and tail in the 4th toss.
{ Wednesday, Thursday }

15. Prizes available in 1000 tickets = 5 { Thursday, Friday }


{ Friday, Saturday }
5 1
P (winning a prize) = = { Saturday, Sunday }
1000 200
1
16. Given number = – 2, – 1, 0, 1, 2 P (53 Sundays and 53 Mondays) =
7
Number when x² < 2 = – 2, – 1, 0, 1
20. Total number of marbles = 225
4
P (x² < 2) = Let 'x' marbles be green
5
2
Probability of green marbles =
Section-C 3
2
P (green) =
17. Word 'Assassination' has 6 vowels and 7 3
consonants x 2
=
225 3
6 Vowels = { A, A, I, A, I, O }
225 × 2
7 Consonants = { S, S, S, S, N, T, N } x =
3
x = 75 × 2
6
i) P (vowels) =
13 x = 150 green marbles
7 Number of blue marbles = 225 – 150
ii) P (consonants) =
13
= 75 blue marbles

274 ADDITIONAL
R

PR ACTICE MATHEMATICS - 10
21. a) Total number of cards = (60 – 13) + 1 a) Heart,
= 48 cards Number of cards (Heart) = 13
Cards divisible by 5 = 15, 20, 25, 30, 35, 40, 13
P (heart) =
45, 50, 55, 60 49
b) Queen,
10 5
P (disible by 5) = = Number of cards (queen) = 3
48 24
b) Cards which are perfect square = 16, 25, 36, 3
P (queen) =
49 49
c) Clubs
4 1
P (perfect square) = =
48 12 Number of cards (clubs) = 10
10
22. a) When two dices are thrown, the total P (clubs) =
49
number of outcomes are : 24. Given card number are from 1 to 20
(1, 1), (1, 2), (1, 3), (1, 4), (1, 5), (1, 6)
a) Number divisible by 2 or 3
(2, 1), (2, 2), (2, 3), (2, 4), (2, 5), (2, 6)
= 2, 3, 4, 6, 8, 9, 10, 12, 14, 15, 16, 18, 20
(3, 1), (3, 2), (3, 3), (3, 4), (3, 5), (3, 6)
(4, 1), (4, 2), (4, 3), (4, 4), (4, 5), (4, 6) 13
P (divisivle by 2 or 3) =
20
(5, 1), (5, 2), (5, 3), (5, 4), (5, 5), (5, 6)
b) Prime number between 1 and 20
(6, 1), (6, 2), (6, 3), (6, 4), (6, 5), (6, 6)
= 2, 3, 5, 7, 11, 13, 17, 19
Getting a number greater than 3 on each
dice 8 2
P (prime numbers) = =
20 5
= (4, 4) (4, 5), (4, 6)
(5, 4) (5, 5), (5, 6) Section-D
(6, 4) (6, 5), (6, 6) 25. Total number of red face cards = 6
9 1 When red face cards are removed, the
P (greater than 3 on each dice) = = total number of cards now becomes 46.
36 9
b) Getting a total of 6 or 7 a) A red card
= (1, 5), (1, 6), (2, 4), (2, 5), Total number of remaining red cards = 20
(3, 3), (3, 4), (4, 2), (4, 3),
20 10
P (red card) = =
(5, 1), (5, 2), (6, 1) 46 23
b) A face card
11
P (total of 6 or 7) = Total number of remaining face cards
36
= 12 – 6 = 6
23. As king, queen, jack of clubs are removed
from the deck of cards, total number of 6 3
cards becomes 49. P (face card) = =
46 23
ADDITIONAL
R

PR ACTICE MATHEMATICS - 10 275


c) A red card Prime numbered cards = 3, 5, 7, 11, 13, 17,
17, 23, 29, 31, 37.
Total number of remaining clubs cards = 13
11
13 P (prime numbers) =
P (clubs) = 19
46
29. Total number of red balls = 5

26 When a dice is thrown two times, total Total number of white balls = 3
number of outcomes are :
Total number of black balls = 7
(1, 1), (1, 2), (1, 3), (1, 4), (1, 5), (1, 6) Total number of balls = 5 + 3 + 7 = 15
(2, 1), (2, 2), (2, 3), (2, 4), (2, 5), (2, 6) a) Red or white

(3, 1), (3, 2), (3, 3), (3, 4), (3, 5), (3, 6) Red number of red and white balls = 5 + 3 =
8
(4, 1), (4, 2), (4, 3), (4, 4), (4, 5), (4, 6) 8
P (red or white) =
(5, 1), (5, 2), (5, 3), (5, 4), (5, 5), (5, 6) 15
b) Not black = Total number of red and white
(6, 1), (6, 2), (6, 3), (6, 4), (6, 5), (6, 6) balls
25 8
a) P (5 will not come either time) = P (red or white) =
36 15
c) Neither white nor black = Red balls
10 5 P (Neither white nor black) = P (Red)
b) P (5 will come exactly once) = =
36 18
5 1
= =
15 3
27. Total number of cards = (45 – 5) + 1 = 41
a) Odd number cards = 5 , 7, 9, 11, 13, 15, 17, 1
30. a) P (queen) =
19, 21, 23, 25, 27, 29, 5
31, 33, 35, 37, 39, 41, Number of aces 1
43, 45 b) i) P (Ace) = =
Total number of cards 4
21 ii) P (king)
P (odd number) =
41 Number of Kings in second draw
b) Perfect square numbers = 9, 16, 25, 36 =
Total number of cards in second draw
4
P (perfect square) = 0
41 = =0
c) Multiples of 5 = 5, 10, 15, 20, 25, 30, 35, 40, 4
45
31. Number of red balls = 4
9
P (multiple of 5) = Number of black balls = 5
41
d) 0, as 2 is the only even prime number and Number of white balls = 6
cards are numbered from 5 to 45.
Total number of balls = 4 + 5 + 6 = 15
28. Cards are numbered as 3, 5, 7, _ _ _ _ 37. 6 2
a) P (white) = =
Total number of cards = 19 15 5
276 ADDITIONAL
R

PR ACTICE MATHEMATICS - 10
4 (5, 1), (5, 2), (5, 3), (5, 4), (5, 5), (5, 6)
b) P (red) = (6, 1), (6, 2), (6, 3), (6, 4), (6, 5), (6, 6)
15
c) P (not black) = P (red and white) 9 1
P (even number on both dice) = =
4+6 10 2 36 4
= = =
15 15 3
2 3. Let B be Boy and G be Girl
4+6 10
d) P (red or white) = = = Total number of outcomes = GGG, GGB,
15 15 3
GBB, GBG, BBB, BBG, BGB, BGG
32. a) Total number of cards = 52 Favourable outcomes for at least 1 boy = 7
Number of black kings = 2 Total number of outcomes = 8

2 1 7
P (black king) = = P (at least 1 boy) =
52 26 8
b) Cards which are neither red nor queen = 24 4. Cards are numbered from 1 to 25
24 6 Total number of outcomes = 25
P (neither red nor queen) = =
52 13 Cards divisible by both 2 and 3 = 6, 12, 18, 24
c) Cards which are neither king nor queen = 44 4
P (divisible by both 2 and 5) =
44 11 25
P (neither king nor queen) = =
52 13 5. Total numbers = – 3, – 2, – 1, 0, 1, 2, 3
d) Cards which are either black or a king = 28
Number of total outcomes = 7
28 7
P (either a black card or a king) = = Number less than 2 = – 3, – 2, – 1, 0, 1
52 13
5
P (x < 2) =
6
WORKSHEET - 2
6. Total number of cards = 52
SECTION-A Total number = 4
Total number of jack = 4
1. Total number of discs = 90
Total number of cards which are neither ace
Prime number less than 23 = 2, 3, 4, 5, 7, 11, nor jack = 52 – (4 + 4)
13, 17, 19
= 52 – 8
8 4
P (prime number less than 23) = = = 44
90 45
44 11
P (neither ace nor jack) = =
2. If two dice are thrown together, the total 52 13
number of outcomes are :
7. Number of red balls = 5
(1, 1), (1, 2), (1, 3), (1, 4), (1, 5), (1, 6)
Number of green balls = 8
(2, 1), (2, 2), (2, 3), (2, 4), (2, 5), (2, 6)
(3, 1), (3, 2), (3, 3), (3, 4), (3, 5), (3, 6) Number of white balls = 7
(4, 1), (4, 2), (4, 3), (4, 4), (4, 5), (4, 6) Total of white balls = 5 + 8 + 7 = 20
ADDITIONAL
R

PR ACTICE MATHEMATICS - 10 277


8+7 3
P (getting a white balls or green balls) = P (at least one tail) =
20 4
15 3
= = 13. When two dice are tossed together, total
20 4
number of outcomes are :
8. When a dice is thrown once, the total (1, 1), (1, 2), (1, 3), (1, 4), (1, 5), (1, 6)
number of outcomes is 6.
(2, 1), (2, 2), (2, 3), (2, 4), (2, 5), (2, 6)
2 1 (3, 1), (3, 2), (3, 3), (3, 4), (3, 5), (3, 6)
P (number less than 3) = =
6 3 (4, 1), (4, 2), (4, 3), (4, 4), (4, 5), (4, 6)
(5, 1), (5, 2), (5, 3), (5, 4), (5, 5), (5, 6)
9. Total number of alphabets = 26
(6, 1), (6, 2), (6, 3), (6, 4), (6, 5), (6, 6)
Number of consonants = 21
a) Outcomes for number on each dice
Number of vowels = 5
= { 2, 2 }, { 2, 4 }, { 2, 6 },
21 { 4, 2 }, { 4, 4 }, { 4, 6 },
P (consonants) =
26 { 6, 2 }, { 6, 4 }, { 6, 6 }

10. Probability of two students not having the 9 1


P (both numbers are even) = =
same birthday = P (B') = 0. 992 36 4
Probability of two students having the same b) Outcomes for sum on two dices is 5
birthday = P (B') = 1 – P (B')
4 1
= 1 – 0. 992 P (sum on two dices is 5) = =
36 9
= 0. 008
14. When two dices are rolled simultaneously,
total number of outcomes are :
Section-B
(1, 1), (1, 2), (1, 3), (1, 4), (1, 5), (1, 6)
11. Total number = – 3, – 2, – 1, 0, 1, 2, 3 (2, 1), (2, 2), (2, 3), (2, 4), (2, 5), (2, 6)
Number whose square is less than or equal (3, 1), (3, 2), (3, 3), (3, 4), (3, 5), (3, 6)
to 1
(4, 1), (4, 2), (4, 3), (4, 4), (4, 5), (4, 6)
i) (–1)² = 1 (5, 1), (5, 2), (5, 3), (5, 4), (5, 5), (5, 6)
ii) (0)² = 0 (6, 1), (6, 2), (6, 3), (6, 4), (6, 5), (6, 6)
iii) (1)² = 1 Outcomes for sum on the two dices is 10
3 = (4, 6), (5, 5), (6, 4)
P (square is less than or equal to 1) =
7
3 1
12. When two coins are tossed, total number of P (sum on the two dice is 10) = =
outcomes are { H, H }, { H,T }, { T, H }, { T, T } 36 12

Outcomes for at least one tail 15. Total number of jacks = 4


= { H,T }, { T, H }, { T, T } Total number of ace = 4
278 ADDITIONAL
R

PR ACTICE MATHEMATICS - 10
Number of cards is neither ace nor jack 4+4 8 2
= 52 – (4 + 4) = 44 P (queen or jack) = = =
52 52 13
44 11
P (neither jack nor ace) = = 20. Total number of red cards = 100
52 13
16. When three coins are tossed simultaneously, Total number of yellow cards = 200
the total outcomes are : Total number of blue cards = 50
(H H H), (H H T), (H T H), (T H H),
Total number of cards = 100 + 200 + 50 =
(T T T), (T T H), (T H T), (H T T) 350
Outcomes for exactly 2 heads 50 1
a) P (blue card) = =
= (H H T), (H T H), (T H H) 350 7
b) P (not a yellow card) = P (red and blue
3 card)
P (exactly 2 heads) =
8
100 + 50 150 3
= = =
17. Total number of cards = 52 350 350 7
c) P (neither yellow nor blue card) = P (red
Total number of spades = 13 card)
After losing 3 spades, number of spades left 100 2
= =
= 13 – 3 = 10 350 7

Total number of black cards after losing 3


spades
Section-C

= 26 – 3 = 23 21. When two dices are thrown together, the


total number of outcomes are :
23
P (black colour card) = (1, 1), (1, 2), (1, 3), (1, 4), (1, 5), (1, 6)
52
(2, 1), (2, 2), (2, 3), (2, 4), (2, 5), (2, 6)
18. Cards are numbered from 1 to 20
(3, 1), (3, 2), (3, 3), (3, 4), (3, 5), (3, 6)
Number which are multiples of 3 or 7
(4, 1), (4, 2), (4, 3), (4, 4), (4, 5), (4, 6)
= 3, 6, 7, 9, 12, 14, 15, 18
(5, 1), (5, 2), (5, 3), (5, 4), (5, 5), (5, 6)
8 2
P (multiple of 3 or 7) = = (6, 1), (6, 2), (6, 3), (6, 4), (6, 5), (6, 6)
20 5
a) Outcomes for prime number on each dice
19. a) Total number of cards = 52
= (3, 1), (3, 5) (5, 2)
Total number of red king = 2
(5, 2), (5, 5), (3, 3)
2 1
P (red king) = =
52 26 (2, 2), (2, 3), (2, 5)
b) Total number of queen = 4
9 1
P (prime number on each dice) = =
Total number of jack = 4 36 4

ADDITIONAL
R

PR ACTICE MATHEMATICS - 10 279


b) Outcomes for total of 9 b) P (Kewal buys the selected shirt)
= (3, 6) (4, 5), (5, 4), (6, 3) Number of good shirts
Outcomes for total of 11 = (5, 6) (6, 5) + shirts with min or defects
=
Total number of shirts
4+2 6 1 88 + 8
P (total of 9 or 11) = = =
36 36 6 =
100
96 24
22. When two dices are thrown together, the = =
total number of outcomes are : 100 25

(1, 1), (1, 2), (1, 3), (1, 4), (1, 5), (1, 6) 24. When three coins are tossed together, the
(2, 1), (2, 2), (2, 3), (2, 4), (2, 5), (2, 6) total number of outcomes are :

(3, 1), (3, 2), (3, 3), (3, 4), (3, 5), (3, 6) (H H H), (H H T), (H T H), (T H H),
(4, 1), (4, 2), (4, 3), (4, 4), (4, 5), (4, 6) (T T T), (T T H), (T H T), (H T T)
(5, 1), (5, 2), (5, 3), (5, 4), (5, 5), (5, 6)
a) Outcomes for exactly two heads
(6, 1), (6, 2), (6, 3), (6, 4), (6, 5), (6, 6)
= (H H T), (H T H), (T H H)
a) Outcomes for a number greater than 3 on
each dice = (4, 4), (4, 5), (4, 6) 3
P (at least two heads) =
8
(5, 4), (5, 5), (5, 6)
b) Outcomes for at least two heads
(6, 4), (6, 5), (6, 6)
= (H H H), (H H T), (H T H), (T H H)
P (number greater than 3 on each dice)
4 1
9 1 P (at least to heads) = =
= = 8 2
36 4
c) Outcomes for at least two tails
b) Outcomes for getting a total of 6 on both
dice = (T T T), (T T H), (T H T), (H T T)

= (1, 6), (2, 5), (3, 4), (4, 3), (5, 2), (6, 1) 4 1
P (at least two tails) = =
8 2
P (getting a total of 6 or 7 on both dice)

5+6 11 25. Total number of cards = 52


= =
36 36 Total number of jack, king = 2 + 2 + 2

23. Total number of shirts = 100 And queen of red colour = 6


Shirts which are good = 88 After removing these 6 cards, the total
Shirts with minor defects = 8 number of cards become 52 – 2 = 46
Shirts with major defects = 4 a) A black king
a) P (Ramesh buys the selected shirt) Total number of black kings = 2
Number of good shirts 88 22 2 1
= = = P (black king) = =
Total number of shirts 100 25 46 23

280 ADDITIONAL
R

PR ACTICE MATHEMATICS - 10
b) A card of red colour a) An odd number
Number remaining red cards = 26 – 6 = 20 Outcomes for odd number = 1, 3, 5, 7

20 10 4 1
P (red cards) = = P (odd number) = =
46 23 8 2
c) A card of black colour b) A number greater than 3

Number of black cards = 26 Outcomes for number greater than 3

26 13 = 4, 5, 6, 7, 8
P (black king) = =
46 23 5
P (number greater than 3) =
8
26. Cards are numbered from 1 to 100 c) A number less than 9
Number divisible by 9 and is a perfect Outcomes for number less than 9
square
= 1, 2, 3, 4, 5, 6, 7, 8,
= 36, 81
8
a) P (divisible by 9 and a perfect square) P (number less than 9) = =1
8
2 1
= = 29. A number ' x ' can be selected from 1, 2, 3, and 4
100 50
b) Prime number greater than 80 = 83, 89, 97 A number ' y ' can be selected from 1, 3, 9 and 16
3 Total number of Outcomes = 4 × 4 = 16
P (prime number greater than 80) =
100
Cases for product ' x y ' to be less than 16 :
27. When a coin tossed 3 times, the total 1) (1, 1) = 1 × 1 = 1
number of outcomes are :
(H H H), (H H T), (H T H), (T H H), 2) (1, 3) = 1 × 3 = 3

(T T T), (T T H), (T H T), (H T T) 3) (1, 9) = 1 × 9 = 9

Ramesh wins if all the tosses show same 4) (2, 1) = 2 × 1 = 2


result
5) (2, 3) = 2 × 3 = 6
= P (A) = (H H H), (T T T)
6) (3, 1) = 3 × 1 = 3
2
P (A) = 7) (3, 3) = 3 × 3 = 9
8
P (Ramesh lossing the game) = 1 – P (A) 8) (4, 1) = 4 × 1 = 4

2 6 3 9) (4, 3) = 4 × 3 = 12
=1– = =
8 8 4 9
P ( product of x and y less than 16) =
16
28. Eight equal parts of the game are numbered
as
30. When three coins are tossed together, the
= 1, 2, 3, 4, 5, 6, 7, 8, total number of outcomes are :

ADDITIONAL
PR ACTICE
R

MATHEMATICS - 10 281
(H H H), (H H T), (H T H), (T H H), 1
(T T T), (T T H), (T H T), (H T T) P (B) =
3
a) Outcomes for at least 2 heads 1
× 24 = 8 balls which are blue in colour
= (H H H), (H H T), (H T H), (T H H) 3
∴ 8 blue balls are present in the jar.
4 1
P (at least 2 heads) = =
8 2 32. Total number of balls in a bag = 18 balls
b) Outcomes for at most 2 heads
Total number of red balls = x
= (H H T), (H T H), (T H H), (T T H),
Total number of balls which are not red
(T H T), (H T T), (T T T)
= 18 – x
7
P (at most 2 heads) = 18 – x
8 a) P (ball is not red) =
18
Section-D x
b) P (ball is red) =
18
1 As 2 red balls are further added in the bag,
31. Probability of selecting red balls P (R) =
4
Total number of red balls = x + 2
1
Probability of selecting blue balls P (B) = Total number of balls in the bag = 18 + 2 = 20
3
Probability of selecting orange balls P (O) x+2
P (red ball) =
20
=1–
1 1
+ 9 x ( x - 2)
A. T. Q., = × =
Ł4 3ł 8 18 20
7 x x+2
=1– =
12 16 20
12 – 7 20 x = 16 x + 32
=1–
12
4 x = 32
5
=1–
12 x = 8 balls
5 ∴ Initial number of red balls = 8
P (O) =
12
Let there be ' n ' balls in a jar. 33. Cards are numbered from 1 to 25
So, a) Outcomes for number divisible by 3 or 5
5 = 3, 5, 6, 9, 10, 12, 15, 18, 20, 21, 24, 25
P (O) =
12 12
P (numbers divisible by 3 or 5) =
5 25
× n = 10
12 b) Outcomes for a perfect square number
10 ×12 = 1, 4, 9, 16, 25
n =
5 5 1
P (perfect square number) = =
n = 24 balls 25 25
282 ADDITIONAL
R

PR ACTICE MATHEMATICS - 10
34. a) Total number of cards = 52 9
P (multiple of 5) =
Total number of spades = 13 49
c) Outcomes for perfect square number cards
Total number of aces excluding spades = 3
= 1, 4, 9, 16, 25, 36, 49
13 + 3 16 4
1
P ( spade or ace) = = = 7
52 52 13 P (perfect square) = =
49 7
2 1
b) P (black king) = = d) Even prime numbered cards = 0
52 26
c) Total number of jack = 4 0
p (even prime number) =
49
Total number of king = 4
=0
4+4
P (Either jack or king) = P (J) =
52 36. The sample space is
8 2
= = {(1,1), (1,2), (1,3), (1,4), (1,5), (1,6),
52 13
P (neither jack noe king) = 1 – P (J) (2,1), (2,2), (2,3), (2,4), (2,5), (2,6),
(3,1), (3,2), (3,3), (3,4), (3,5), (3,6),
2
=1– (4,1), (4,2), (4,3), (4,4), (4,5), (4,6),
13
(5,1), (5,2), (5,3), (5,4), (5,5), (5,6),
13 - 2
= (6,1), (6,2), (6,3), (6,4), (6,5), (6,6)}
13
11 25
= a) P (5 will not come either time) =
13 36
d) Number of king = 4 5
b) P (5 will come up exactly one time) =
Number of queen = 4 36

4+4 37. Total number of persons = 12


p (Either a king or a queen) =
52 Number of persons who are extremely
8 2 patient = 3
= =
52 13
Number of persons who are extremely
35. Cards are numbered from 1 to 49 honest = 6
Total number of outcomes = 49 Number of persons who are extremely
Outcomes for odd number cards = 1, 3, 5, 7, kind
9, 11, 13, 15, 17, 19, 21, 23, 25, 27, 29, 31, 33, = 12 – (3 + 6)
35, 37, 39, 41, 43, 45, 47, 49
= 12 – 9
a) Total odd number cards = 25
=3
25
P (odd number) = 3
49
a) P (person who is extremely patient) =
b) Outcomes for multiple of 5 12
1
= 5, 10, 15, 20, 25, 30, 35, 40, 45 =
4
ADDITIONAL
R

PR ACTICE MATHEMATICS - 10 283


b) P (persons who are extremely kind or (v) (d) Probability both numbers are prime
honest) 9 1
= =
3+ 6 36 4
=
12
cASe StUDY-2
9
=
12 (i) (c) Let A: getting a King
3 Let B: getting a club
=
4 P (either A or B) = P (A ∪ B)
P (A ∪ B) = P(A) + P(B) – P(A.B)
cASe StUDY-1 4 13 1
= + -
(i) (b) Sample space when pair of dice is thrown 52 52 52
{(1,1), (1,2), (1,3), (1,4), (1,5), (1,6), 4
=
(2,1), (2,2), (2,3), (2,4), (2,5), (2,6), 13
(3,1), (3,2), (3,3), (3,4), (3,5), (3,6), (ii) (a) Number of black Queens = 2
(4,1), (4,2), (4,3), (4,4), (4,5), (4,6),
P (Black Queen) = 2 = 1
(5,1), (5,2), (5,3), (5,4), (5,5), (5,6), 52 26
(6,1), (6,2), (6,3), (6,4), (6,5), (6,6)} (iii) (c) There are 8 cards which are either ace or
Probability that 4 will come either of them jack
11 8 2
= P (Ace or jack) = =
36 52 13
11 2 1
(ii) (b) P (5 will come at least once) = (iv) (b) Probability of red king = =
36 52 26
(iii) (a) (5, 5) is the only outcome in which 5 is (v) (a) No. of King and Queen = 4 + 4 = 8
1 No. of card that don't have King and
coming on both dice P (5,5 ) =
  36 Queen = 44
9 1 44 11
(iv) (c) Probabily both number are odd = = P (neither King nor Queen) = =
36 4 52 13

284 ADDITIONAL
R

PR ACTICE MATHEMATICS - 10

You might also like